Você está na página 1de 356

AOÇÕES

DE
MATEMÁTICA
z,=1 cisO°
z2=1 cis60°
z3=1 cis120°
Z4
z4=1 cis180°
z5=1 cis240°
z6=1 cis300°

VOLUME 7 Aref Antar Neto


José Luiz Pereira Sampaio
Números Complexos Nilton Lapa
Polinômios Sidney Luiz Cavallantte
Arei Antar neto
José Luís Pereira Sampaio
NÜton Lapa
Sidney Luiz Cavallantte

NOÇÕES DE MATEMÁTICA
Números Complexos Polínômios e Equações
Algébricas

Volume 7.

Editora Vestseller
Fortaleza - CE
1a Edição 2011
Capa
Rafael Feitosa Parente

CIP — Brasil. Catalogação-na-Fonte.


Câmara Brasileira do Livro, SP

Números complexos, polinômios, equações


N916 algébricas : 2o grau / Aref Antar Neto.
(et al.) Fortaleza: Ed. Vestseller, 2010.
(Noções de matemática; v.7)

1. Equações 2. Matemática(2° grau) 3. Números


complexos 4. Polinômios I. Antar Neto, Aref, 1949 -
II. Série

17. CDD-510.07
95-0032 18. -510.7
17. -512 2
18. -512.94
17. -512.21
18. -512.942
17. -512.81
18. -512.7

índices para catálogo sistemático:


1. Equações algébricas 512.2 (17.) 512.94 (18.)
2. Matemática : Estudo e ensino 510.07 (17.) 510.7(18.)
3. Números complexos : Álgebra 512.81 (17.) 512.7 (18.)
4 Polinômios : Álgebra 512.21 (17.) 512.942 (18.)

• Z__ _
Iry>F Editora

V V C4Í

www.VestSeller.com.br
índice
Parte 1

Capítulo 1.0 conjunto dos números complexos

1.1 —Introdução ... 11


1.2 — Números complexos ___ 17
1.3 — O conjunto dos números complexos, 21

Capitulo 2.Forma algébrica dos números complexos ... 29

2.1 —Introdução 29
2.2 — Descobrindo que números reais são complexos 29
2.3 — Unidade imaginária 30
2.4 — Forma algébrica ............. 31
2.5 — As potências naturais de i .............. 39
2.6 “ Conjugado de um número complexo 43
2.7 — Divisão 44

Capítulo 3.A geometria dos números complexos 51

3.1 —0 piano de Argand-Gauss ........................ 51


3.2 — Módulo de um número complexo 54
3.3 — Propriedades imediatas do módulo 55
3.4 — Outra propriedade do módulo: a desigualdade triangular 61

Capítulo 4.A trigonometría dos números complexos .,,.65

4.1 Argumento de um número complexo . ..,05


4.2 — A forma trigonométrica dos números complexos ...67

Capítulo 5.Operações na forma trigonométrica ... 71

5.1 —Introdução 71
5.2 —Multiplicação e Divisão . 71
5.3 —Potenciação 73
5 4 — Radiciação ... 7B
“ Exercícios Suplementares.... ... 06
Parte II

Capítulo 6 0 conceito de polinómio. igualdade..

6.1 — O conceito...................... 91
6.2 — Valor numérico de um polinòmic 91
6.3 — Definição — Polinómio nulo ..... 92
64 — Quando um polinómio é nulo — Teorema 92
65 — Definição — Polinômios Iguais ............... ....... 93
6.6 — Quando polinômios são iguais — Teorema ...... 93

Capitulo 7.Operações com os polinômios. Grau ...97

7.1 — adição de polinômios 97


7.2 — Multiplicação de polinômios 99
7.3 — Grau de um polinómio................. 102

Sapítulo 8 A divisão de polinômios ..113

8.1 — Divisão euclidiana ....113


8.2 — Método de Descartes para divisão de polinômios ...110

Capitulo 9.A divisão de polinômios em que o divisor é de grau 1 129

9.1 — Teorema do resto .,


9.2 — O dispositivo de Briot-Ruffini . 130
9.3 — Observações sobre o dispositiva 132
9.4 — A dívísibílidade pelo produto — Teorema ...... ,133
9.5 — A dívísibílidade por (x-c)m —Teorema . ,134

Capitulo 10 Outros temas importantes.... 115

10.1 — Polinómio derivado 145


10.2 — Máximo divisor comum 152
10.3 — Mínimo múltiplo comum...... 159
10.4 — Uma observação importante 160
Exercícios Suplementares... 161
Parte lll

Capítulo 11.Equações algébricas .... 167

11.1 — Introdução .......................................................... 167


11.2 — Equação algébrica do primeiro grau 168
11.3 — Equação algébrica do segundo grau ............... .... ........ 169
11.4 — Teorema Fundamental da Álgebra 170
11.5 —Tearema da decomposição.......................................... 170
11.6 — Demonstração do teorema da decomposição 172
11.7 — alguns artifícios....................... „ ............. ....... 182
11.8 — Polinômios de mesmas raizes 190

Capítulo 12 Raízes múltiplas 193


12 1 — Raizes múltiplas e derivadas — Teorema ,...193
12.2 — Demonstração do teorema
.... 195

Capítula 13 Raízes imaginárias 203

13.1 —Tearema das raízes conjugadas. 203


13.2 —Consequência 205
Capítulo 14 Relações de Girard 213

14.1 — introdução . .21


14.2 — Equação do segundo grau .213
14.3 — Equação do terceiro grau ... .214
14.4 — Equação do quarto grau,... 215
14.5 — Caso geral .216

Capítulo 15.Raízes racionais ... . 241

15.1 — Tearema das raízes racionais....


15.2 — Demonstração do teorema ,243
15.3 — Consequências .. 244
15.4— Propriedades ........... .251

Capítulo 16.Equações recíprocas .... 253

1 S.1 — Definição 253


16.2 — Reconhecimento de uma equação reciproca.... ... 254
16.3 — Resolução de um equação reciproca 256
16.4 —Resolução de um equação reciproca normal.... ... 257
Capitulo 17.Raizes comuns 263

17.1 — Introdução....................... 263


17.2 — Raizes comuns e MDC . 263
17.3 — Raizes múltiplas e MDC 265

Capitulo 18. Raizes reais 267

17.1 — Introdução............................... 267


17.2 — Gráfico da função polinomial 267
17.3 — Teorema de Bolzano............. 275
Exercícios Suplementares... 279

Testes de vestibulares................................. 281


Respostas dos exercícios propostos........ 323
Respostas dos exercícios suplementares 349
Respostas dos testes de vestibulares...... 353
Tabela de razões trigonométricas............. 354
PARTE I

Capítulo 1 - O conjunto dos números complexos


Capítulo 2 - Forma algébrica dos números
complexos
Capítuío 3 - A geometria dos números complexos
Capítulo 4 - A trigonometria dos números
complexos
Capítulo 5 - Operações na forma trigonométrica
O conjunto dos números
complexos

1.1 - Introdução
A criação dos números complexos teve estímulo na necessidade sentida pelos
matemáticos de ampliar o campo numérico de seu trabalho, dando significado,
principalmente, às raizes quadradas de números negativos
Por que razão, quando se trabalha com números reais, algumas equações do
segundo grau tém duas raizes, enquanto que outras nào tem nenhuma?
Como sabemos, isso acontece porque não existe, entre os reais, um número que
elevado ao quadrado dê resultado negativo:

(?)2 = -25
(?)’=-!
y

2
e isso se confirma quando a equação do segundo grau ax + bx + c = 0 é escrita
na forma:

x+—
bf b2 -4ac
2a ) 4 a2

onde vemos que não existe valor real de x capaz de satisfazer a equação no caso
onde o discriminante A = b2 - 4ac é negativo.
Ora, como todo número negativo pode ser escrito como produto de seu simétrico

-25 = 25K-1) = 52 ><?)2


2 2 J2
X?)2
9 9 3

a questão de ampliar o campo numérico reside, portanto, em criar algum ente que
verifique a igualdade:

(?)2 = -1

11
Dois problemas históricos

Observa-se. assim, a necessidade de ampliação do universo numérico.


Essa necessidade não ê nova: Diofante de Alexandria (cerca de 250 d.C.) estevs
às voltas com o seguinte problema:
Determinar os iados de um tnanguto retângulo de perímetro 12 e área 7
A montagem desse problema nos traz a equação:

6xJ-43x + 34 = 0

onde x é a medida de um dos catetos


O discrimmante dessa equação é negativo (A = -167), o que nos leva a concluir
que não existe o triângulo procurado, Não te ri a Diofante despertado sua atenção
para equações que, como essa, tém A < 0 ?
Em 1545, o médica italiano irolamo Cardano (1501-1576) propôs e resolveu o
problema seguinte:
Dividir 10 em duas partes tais que seu produto seja 40.
A equação representativa desse problema é:

x3-10x + 40 = 0

anda Cardano encontrado as respostas 5 + 7-15 e 5-7-15 e, snhre isso, teria


dito que o resultado é tão sutil quanta inútil, no que, ao menos quanto à utilidade,
estava inequivocadamente enganada já que hoje, a teoria dos números complexos
tem larga aplicação na Física e na Engenharia.
Ele próprio, sem saber, leve uma mostra de que o fato de se dar uma interpretação
ã raiz quadrada de números negativas seria útil, pois, ao estudar a solução de
equações da forma:
x3 = ax + b

encontrou resultados que, em certos casos incluíam tais raízes quadradas e,


nesses mesmas casos, ele sabia existir uma solução real. Par exemplo, o
resultado que se obtém para a equação x3 =15x + 4 , utilizando o método de
Cardano, é dado pela expressão:

X= + 7^121 + $2 -\tl21

Uma verificação direta mostra-nos que x=4 ê raiz dessa equação, o que levaria a
supor, num raciocínio simples, que:

^2 + -121 + ^2 - -121 = 4

Ora, se não existe 7-121 ccmo podería tal expressão apresertar um valor real?

12
Inventando um novo número

Suponha-se então que, convencidos da necessidade, resolvamos inventar um


número cujo quadrado seja -1,
Imediatamente surge a questão: como operar com esse novo número?
É claro que essa criação nos será útil apenas se pudermos usar os mesmos
processos operacionais a que estamos acostumados.
Assim, sem ter, por enquanto, a preocupação de formalizar uma estrutura numérica
nova, procurando apenas ganhar intimidade com a idéia, consideremos um novo
número que, por falta de outro melhor, representaremos pelo símbolo i. e
formulemos duas hipóteses.

1a) i2 = -1
2a) i obedece ás principais propriedades operacionais
da Álgebra,'

Com isso, podemos já complelar as igualdades questionadas no inicio deste item:1

-25 = 25><-1)=5z x2
r rz” 2
2 2 .. V22 M2
— = — X-1)= —
9 9 3

e também podemos encontrar soluções para equações antes impossiveis


Optamos por exemplificar esse fato por meio de exercícios para que, durante a
Leitura, as eventuais dúvidas que o leitor tenha sobre a consistência das hipóteses
feitas (e o modo de operar), sirvam de estimulo para o estudo do próximo item,
onde daremos tratamento mais formal e rigoroso aos números complexos.

Exercícios Resolvidos

11) Resolva a equaçao x +1 — 0 .

Solução

Este ê um exercício típica de equação cujo conjunto-verdade, no campo dos


números reais, é vazio.
Agora, com base nas hipóteses adotadas, podemos escrever:

Cúmutatíva (da adição e multiplicação), associativa (da adiçao e da multiplicação] e


dislributiva.
13
x2 + 1 = 0 <=> x2 -(-1) = 0 w
« x2 - i2 = 0 <=>
« (X + i)(x - i) = 0 <=>
»x + i = 0oux-i = 0w
« x = -i ou x = i

Logo: S = {-i; i).

1.2) Resolva a equação x2 4x + 13 = 0.


Solução
O discriminante dessa equação é:
A = 16-52 = -36
Baseados mas hipóteses adotadas, escrevemos:

4 + ^36 4±./36(-1) 4 ± V36i2 4 ± 6i


Xi 2 - 2 “ 2
2 2

Logo. S = {2 + 3I; 2-3i}.

O leitor pode verificar que, realmente, qualquer um dos números obtidos


satisfaz a equação; por exemplo, substituindo x por 2 + 3i em x2-4x + 13,
temos:

(2 +3i)2-4(2 +3i) + 13 = (4 + 12i + 9Ú2))-(8 + 12i) + 13 =

= 4 + 12i-9-8-12Í +13
=0

1.3) Resolva a equação x3 + 8 = 0 .

Solução
Trabalhando no campo dos números reais, faríamos simplesmente:

x3 + 8 = 0«x3=-8 <=> x = -2

No entanto, com as hipóteses adotadas, podemos verificar a existência de


duas outras soluções, lembrando a fatoração:

a3 + b3 = (a + b)(a2 -ab-b2)

escrevemos:

x3 + 8 = 0 <=> (x + 2)(x2 -2x + 4) = 0 «

14
x+2=0 X =-2
ou ou
x2 -2x + 4 = 0 2 + 7^12 2±7l2i2
x =------------- = i±r7ã
2 2
Logo: S = {-2; 1 + ijã; 1 i7s}.

1.4) Verifique que o número i é raiz da equação x3 - 2x2 + x - 2 = 0.

Solução
Substituindo, x por í no primeiro membro da equação vem:

i3 - 2i2 +í- 2 =(í>- 2$> í —2 = -i + 2+ í- 2 := 0

Logo: í é raiz,

15) Mostre, com base nas hipóteses adotadas, que o número í não é positivo,
não é negativo e não é igual a zero, isto é, que as relações i > 0, i < 0 e
i = 0 não têm significado.
Solução
Lembrando que:
1°)a <0=-a2 >0
2°)a> 0=?a3 > 0
3°)a = □ =? a2 =0

mostremos que qualquer uma das hipóteses: i > 0, i < 0 ou i = 0 nos conduz
a um absurdo.
I) Supondo i > 0, vem;

i > 0 => í2 >0=>-1>0 (!!!)

II) Supondo i < 0, vem:

[<0 =>i2 >0^1>0 (!!!)

III) Supondo i = 0. vem:

i = Ü^Í2=0^>-1 = 0 (!!!)

Assim, nao valem as relações í > 0, i < 0 ou i = 0.

15
1.6) Talvez o fato de termos inventado um número (i), e trabalhado com relativo
sucesso apenas supondo que ele obedece às principais propriedades
operacionais, leve-nos a querer usar esse procedimento em outras si­
tuações.
Por exemplo, sabemos que não se define divisão por zero. Teriamos
sucesso se inventássemos um resultado nesse caso?

Suponha, então, um novo número, representado pelo símbolo QQ , tal que:

•>5-m
II) DO satisfaz as principais propriedades operacionais da Álgebra (veja nota
de rodapé anterior).

Mostre que I e II conduzem a absurdos como a afirmação:

Números diferentes são iguais.


Solução

Sejam x e y números tais que x * y.

Sabemos que 0 xx = 0 xy. Multiplicando ambos os membros por QQ •

□□><0x<) = mXOxy)
Como estamos supondo válida a propriedade associativa, podemos escre­
ver:

(□>)>« = (E2D>°)xy (1)


Como, de I e II temos = QQ => l_ll_l >0 = 1, a igualdade (1) fica:

1 xx = 1 xy
Donde: x = y (!!!)

Exemplificando numericamente:

0x75 = 0 >8
□EHoxjs) = rruoxB)
(üü>O)xV5 = (ITI>0)>fi

1x75 = 1>6
75 = S (!!!)

16
Isso nos mostra que não basta criar uma nova proposta numérica apenas na
esperança de que as regras sejam obedecidas, É preciso montar uma estrutura
consistente e compatível com o rigor matemático.
Exercí closPro postos

1.7) Resolva as equações

a) x3 + 9 s □
b) x2 + 5 = 0
c) x4 -1 = 0
d) x* - 255 = 0
(Sugestão: fatore as expressões usando diferença de quadrados nos ilens c e d)

1.8) Resolva as equações:

a) x3 -2x+2= 0
b) 4x2 -16x + 17 = 0
c) x2-6x+17 =0
d) x2-10x + 40 = 0

1.9) Resolva a equação x3 + 1 = 0,

1.10} Verifique se o número tü é raiz da equação (E) nos seguintes casos:

3 2x2 + x-2 =0
a) <ü = -i; (E):x
b) w=2i, (E) x4 2x3 + 8X“16 = 0

c) u=2 + i: (E):x3- Sx2 + 9x-5 = 0

1.2 - Números Complexas


Há, na matemática, mais de uma maneira de se conceituar. formalmente, número
complexo. Preferimos aqui adotar a seguinte definição
Chama-se número complexo a qualquer par ordenado (x; y) de números reais.

É usual indicar um número complexo pela letra z, Assim, zt = (-2; 3), Zj = (0; -1),

z3 =f-;o] (\/2; it) são exemplos de números complexos.

17
Já com vistas a introduzir uma estrutura operatória aos números complexas,
estabelecemos, também, as seguintes definições, onde a, b, c e d sãc números
reais:

I) Igualdade de números complexos

(a: b) = (c: d) t=? a = c e b=d

II) Adição de números complexos

(a; b) + (c; d) <=> a + c e b + d

lil) Multiplicação de números complexos

(a; b}?<c; d) í=j (ac - bd; ad + bc)

Exemples

a) Se (x; y) = (-2; 3), então x = -2 e y = 3

b) Se z,-(2;3) e z? = (4: 5), então:

z,+zj = (2; 3)+ (4; 5) = (2+ 4; 3 + 5) = (6, 6) e

z, xzz = (2b 3) ><4; 5) = (2x4-3 *5; 2x5+ 3x4) = (-7; 22)

c) Sendo z um número complexo qualquer, temos:

z2 =zxz
( 1 \
Assim, se z= -2 , então;
'. 2 J
z2 = zxz =
1
=
2

12*21 - [-2] H-2]; A X-2) + [-2] J l = 15;- 2


2 2 J v 4 ’
4

Para quem está tomando contato pela primeira vez com esta teoria, é intrigante
observar que as definições de igualdade e adição dadas sáo, digamos, intuitivas,
enquanto que a da multiplicação foge completam ente a essa intuição: porque não
definir (a; b)>(c; d) por (a>c, b >d)?
A Isso respondemos com a lembrança de que estamos procurando criar um campo
onde existam números cujo quadrado seja negativo e, como veremos mais
adiante, será justamente esse modo de multiplicar, estabelecido na definição (lll),
que nos fornecerá tais resultados

18
Exercícios Resolvidos

1.11) Determine os reais x e y para que (x - 3y; x + y) = (1: - 4).

Solução
Da igualdade de números complexos vem:

íx-3y~1
(x + y = ^4

Resolvendo este sistema, obtemos


11 5
X-— « y = ~7
4 4

1.12) Determine o número complexo z tal que (-2; â) + z = (2: Ú)xz.

Solução
Seja z = (x; y); então:

(-2;8Mx;y) = {2; 0)X«: y) =>


(-2 + x; 8 + x) = (2 xx-0 xy; 2xy + üxx)=>
. . í-2 + x = 2x
=> (-2 + x; 8 + y) = (2x; 2y) => 4
| S + y = 2y

Logo, x = -2 e y = 8 e daí z = (-2.8).

1.13) Resolva a equaçao z2 =(0; 1).

Solução

Seja z = (x; y); então:


z2 = (0; 1) => zxz = (0; 1)=> (x; y)>(x; y) = (0J) =>
oíxw - yxy; xxy + yxx) = (0:1)
x2 - y2 = 0
=-(x2- y2;2xy) = (0; 1)
2xy 1

19
Resolvendo este sistema (lembrando que x e y são números reais),
obtemos:

72 72)
x = — e y = — ou
f x =------
72 e y=-
72
2 2j l 22 2

Logo, z = í^.^Ou zJ — “ 2 , e o conjunto-solução da equação


l2 2J t 2
é:

S
^2'2 2 2 ,

Exercícios Propostos

1.14) Dados z, = (1,4), z2 = (-2; - 2) e z3=(0;3), calcule:

a) z, + z2
b) z, + z2 + z 3
c) z, xz2
d) (z, xz2)xz3
e) z2 x(z, + z3)

g) (Zi + z2)2

1.15) Determine os reais x e y para que se tenha:

a) (x + 2, y-3) = ^2,

b) (2x-3y; 3x + 2y)

4 21
1.16) Dados z,=(5;-5) e z2 = —, — j, determine z tal que:

a) z + z, = (0: 0)
b) zxz2 = (1; 0)
c) z + z2 = Zt
d) zxz, = z2

.2
1.17) Resolva a equaçao z= (0;-8).

20
1.3-0 Conjunto dos Números Complexos

Com a definição de número complexo, seguida dos conceitos de igualdade, adiçao


e multiplicação dados no item 1.2, podemos agora definir conjunto dos números
complexos como sendo o conjunto C de todos os pares ordenados de
números reais:

C = {z = (x; y) [ x& R e y e

É importante notar que, desta forma, estamos definindo □ conjunto dos números
complexos como o produto cartesiano do conjunto dos números reais por ele
mesmo, isto ê:

C-3 x S = R!

Estando, então, definidas em € a igualdade (I), a adição (II) e a multiplicação (III),


conforme o item anterior, é bastante simples provar que, nesse novo conjunto, são
verdadeiras as nove propriedades operacionais da Álgebra, a saber

1. Propriedade comutativa da adição

= z2 + zt, Vz^ <= C, Vz2 e

Sendo z1 = (a;b) e z2=(c;d), temos:

z, + z, = (a; b)-r(c, d) = (a + c; b + d) =
= (c +a; d + b) = (c, d) + (a; b) = z3 + z,

2. Propriedade associativa da adição

^+[2, + ^]-^+2.]+ 2 3- Vz1 e C, Vz? e ¥z3eC

Sendo z, = (a: b), z2 = (c; d) e z, = (p;q), temos:

Z1 + [z2 + Zj] = (a; b) + [(c; d) + (p + q)] = (a; b) + (c + p; d + q) =


= (a + [c + p]; b +{d + q]) = ([a + cbp: [b + d] + q) =
= (a + c; b + d) + (p; q) = [(a: b) + (c: d)] + (p: q) -
= (z1 + z2] + za

21
3. Existência do etemento neutro da adição

3T!aE C I Z + na = Z?ze C

Sendo z = (a, b), vamos mostrar que existe rja = (x; y) tal que Z + r> - 2■

(a;b) + (x; y) = (a; b) «=>

ía + x = a
Da igualdade, vem .

donde x = 0 e y = 0 Existe, portanto, o elemento neutro tia = (0;0)que somado


a qualquer número complexo z dá como resultado 0 prõprio z,

4. Existência do elemento simétrico

3z’e C|z4z’ = nH, Vzé C

Sendo z = (a: b), vamos mostrar que existe z' = (x; y) tal que z + z’ = qa Como
qa = (0, 0), escrevemos:

(a;b) + (x;y) = (0; 0) » (a + x, b + y) = (0; 0)

|a + x = 0
i»1
jja igualdade, vem
(b +■ y = 0

donde x = -a e y = -b. Existe, portanto, 0 elemento simétrico z' = ( — a, -b).

Observação: Indicando zf por -z, temos z = (-a; -b) = -z = -(a; b).

Com isso, sendo z, = (m, n) e z2 = (p, q), ao efetuarmos a adição z, + Zj’


escrevemos:

z, + 2/ = z, + (-zz)= (m: n) + (-p; - q) = (m - p; n -q)

dando assim um significado para o que chamamos de subtraçao de numeras


complexos. Por exemplo:

(5, 8)-(9; 3) = (5-9; 8-3) = (—4, 5)


(-13; 5)-(2;-4) = (-15; 9)

5. Propriedade comutativa da multiplicação

z, *z2 = z2 xz^ Vz, e 't\ Vz2 e C

22
Sendo z1 = (a; b) e z2 = (c, d), temos:
Zt xzj = (a; b) ><c; d) = (ac - bd; ad+bc)
zixz< = (c;d)Xa;b) = (ca-db;da + cb)

Logo, zA >z2 = Zj x*!-

6. Propriedade associativa da multiplicação

z1 >^z2 xz3] = [z1 xz2] xz3, Vz1 e C, Vz2 e e ~

Sendo Z1 = (a; b), z2 (c;d) e z3 = (p;q), temos.

z, >(z2 xz3] = (a; b)[(c; djxfp; q)] = <a; b)>(cp -dq: cq + dq) =
= (a[cp - dq] - b[cq + dpj; a(cq + dp] + b{cp - dq]) =
= (acp - adq -bcq -bdp; acq + adp + bcp - bdq)

[z, xzj xz3 = [(a,b)(c; d)]x(p; q) = (ac -bd; ad+ bc) x(p xq) =
= ([ac-bd]p-[ad+bc]q; [ac-bd]q+]ad-F*bc]p) =
= (acp-bdp-adq-bcq: acq + bdq + adp -bcp)

Lago: zy >{z2 xzj^z, xz2]xz3

7, Existência do elemento neutro da multiplicação

Sq^e C| ZXi]m = Z, Vze C

Sendo z - (a; b), vamos mostrar que existe nm = (x; y) tal que Zxrjm = z:

(a: b) >(x; y) = (a; b) <=> (ax -by; ay +bx) = (a: b)

í ax - by = a
Da igualdade, vem
[ay r bx - b

A resolução do sistema fornece x=1 ey = 0 Existe, portanto, o elemento neutro


nm = (to)
que, multiplicado por qualquer número complexo z, dá como resultado o próprio z
Exemplificando numericamente:
{5.3)x(1;0) = (5xi-3xO;5>ÍJ + 3xl) = (5; 3)

(t □) M3; - 72) = (1 x3) - 0 ><-72]; 1 xf-72 ax3) - (3. - 72)

23
8. Existência do elemento inverso

3z e C | z>z"' = r|m,Vze C*

Seja z = (a, b) £ (0: 0), isto é, ze C-{(0; 0)} = C *. Vamos mostrar que existe o
número complexo (x; y), indicado por z'1 tal que z > z-1 = r,m :

(a; b) x(x; y) = (1; 0) <=> (ax - by; ay + bx) = (1; 0)


|ax-by = 1
Da igualdade, vem
|ay+ bx = 0
a -b
A resolução do sistema fornece x = y = “õ---- 7 Existe, portanto, o
-----7 ee y-
a2+b2 a2+b2
elemento inverso de z = (a; b) (também chamado inverso multiplicativo):

a -b
z
a2 + b2
Exemplos
10) O inverso de í—; - 'l é:
15 5)

4 2
5 ___ . 5
z7 =
J5.
4

2") O inverso de z2 = (-3; 0) é:


U' íIWJ
|2 ’

-3 0
z2 =
[-3]2 + O2 ’ [-3]2+ o2
l;o
3

3°) O inverso de z3 = (0; - 4) é:

0 - -H
Z3 =
0z+[-4] 2' 02+[-4]\

9. Propriedade distributiva
z1 x|z2 + z3] = z, xz2 +z1 xz3, Vz, € C, Vz2 g C, Vz3 e C

24
Sendo z1 = (a; b). z2-(c‘^) e z3 = (p; q), temos:

+ z3] = (a;b)>£(c; d)+(p; q)] =


= (a. b)x(c + p; d + q) =
= (a[c + p]-b[d + q|; a[d + q| + b|c + p]) =
=■ (ac + ap-bd-bq; ad + aq + bc-bp)

z, xz2 + Zi xz3 = (a; b)x(c; d) + (a; b)x(p; q) =


= (ac-bd; ad + bc)-i-(ap-bq, aq + bp) =
= ([ac-bd] +[ap - bq], [ad + bc] - [aq - bp]) =
- (ac-bd + ap-bq; ad + bc +aq + bp)
Logo: z1 x[z2 * z 3 ] = z^xz2+z1 xz 3-

Exercício Resolvido

1,13) Divisão de números complexos — A existência do elemento inverso


(propriedade 3) permite-nos definir divisão de dois números complexos
como sendo o produto do primeiro pelo inverso do segundo. Assim,
sendoz1 e z3 com dois elementos de C, a divisão de z^ por z3 é indicada
e definida por:
£L = 2^2
Z2

Nessas condições, efetue:

(2:-3j
a)
(3,2)

(-t1)
b)
(t-1)

Solução
a) O inverso de (3; 2) é:
3 -2 2
(3;2)-q =
3=72=; 3? + 2: 13

Então:

P:~3) = (2;-3)x(3:2)-,=(2:-3)J—=
(3:2) \ IO i O/

<2 _3 2 1 _ f 2 ’
[-3]x — ; 2>
13 13 13

25
b) O inverso de (1; —1) ê:

1 -ir-i] i_. 1
í+í-u^+h2 2’2

Então

<-i; i) = (-1;1)x(1;-1) f1 1 ’)

= f [-1] xl -1 xl; [-1] xj+1 = (-1; 0)

Exercícios Propostos

1,19) Determine os inversas multiplicativos dos números complexos:

a) b) (5; 0) c) £0; 3)
U3 ’ 13 J
1,20) Efetue as divisões1

(1.1) (Q:-5)
a) b) c)
12. 5 > (5;0) (0;3)
13 13 J

1.21) Seja z = (a;b) e C*. Se o inverso multiplicativo dez b z (c;d), mostre que-

[a2 + b?] Mc2 + d2] = 1


1.22) Potências de expoentes inteiros - Tendo como fundo de consistência
a propriedade associativa, para todo z e C e para todo ne N, n > 2, defi­
nimos:

zfl = ZXZXZX..Z
n fatores

Por exemplo, se z = £2; 1) e n = 3, temos:

z3 = (2; 1)3 - (2; 1) x£2; 1) x(2; 1) = (3; 4) x(2; 1) = (2; 11)


Adotando, também, as definições:

z°= nm= (to)


z1 = z
z = (z’1)" = (zn)’\ Vze C‘, Vne N

26
ficam completamente caracterizadas as potências de expoentes inteiros
dos números complexos, valendo para elas as propriedades usuais, como,
por exemplo:

z"' e (z"1)"7 = zn,*"J

Assim sendo, calcule z tal que:

(t 3)* (-3; 5)s


z= + (to)7
(t t3.-5)s

27
Capítulo
Forma algébrica dos
números complexos

2.1 - Introdução

Desde que nos habituemos à regra de multiplicação, o trabalho algébrico com


números complexos não apresenta maiores dificuldades. No entanto, não podemos
dizer que seja um trabalho confortável, principalmente quando com-parado ao que
usualmente temos ao operar com números reais.
Por isso, visando a agilizar o modo de operar com números z e C, vamos introduzir
algumas notações novas.

2.2 - Descobrindo que os números reais são complexos


Comecemos por considerar o subconjunto Cr de C, formado pelos com-plexos da
forma (x; y), isto é:
Cr = {(x; y)| xe R e y = 0]

Assim, (0; 0), (1; 0), (—1; 0), I — —; 0 I e (>?2; o) são exemplos de elementos de Cr.

Efetuando a adição e a multiplicação de dois elementos, (a: 0) e (c: 0), de Cr:


(a; 0) + (c, 0) = (a + c; 0 + 0) = (a + c; 0)
(a; 0) x(c; 0) = (a xc - 0 xO; a xO + 0 xc) = (a xc; 0)

notamos que as segundas componentes (y) dos resultados continuam sendo


zero (portanto esses resultados pertencem a Cr), e as primeiras componentes
são obtidas como se tivéssemos usado a adição e a multiplicação dos números
reais.

Exemplos

Em Cr Em 5
(2; 0) + (5; 0) = (7; 0) 2+5 =7

(12; 0>-(5;0)=(y:0) 1
12-—«—
23
2 2
(2; 0)x(5; 0) = (10; 0) 2 >5 = 10
(-12)x| = -6
12;O)^;oj = (-6;O) :

29
Vemos então que os elementos de Cr, obedecendo às definições de igualdade,
adição e multiplicação de complexos, têm comportamento idêntico ao dos
números reais, (num ramo da Matemática, chamado Álgebra Moderna, os
conjuntos Cr e R são ditos isomorfos).
Ora, já que podemos operar com os complexo da forma (x; 0), do mesmo modo
que operamos com o real x, vamos adotar a notação:
(x; 0) = x
para todo x real. Podemos, portanto, escrever:
(0;0) = 0, (1;0) = 1, (-1;0) = -1, (>/2;0) = V2

Admitida a igualdade (x; 0) = x, é evidente que Cr = R; passamos, então, a


considerar o conjunto dos números reais como subconjunto dos números com­
plexos: ___________

R c C

podendo, por isso, afirmar que todo número real é complexo.

2.3 - Unidade imaginária

A igualdade (1; 0) = 1 define, em C, o número complexo (1; 0) como unidade real.


Definimos como unidade imaginária o número complexo (0; 1), que passamos a
indicar pelo símbolo i.

i = (0; -I)

Era nossa intenção conseguir um número cujo quadrado fosse negativo. Agora, já
o temos! Observando que:
i2 = i xj = (0; 1) x(0; 1) = (0 xO-1 xl; 0 xl + 1 x0) = (-1; 0)

E utilizando a notação adotada no item 2.2, obtemos a propriedade fundamental


da unidade imaginária:

Imaginários puros - Todos os números complexos da forma (0; y), com


y * 0, são denominados imaginários puros.
Notemos que
(y, 0) x(0; 1) = (y x0 - 0 xl; y + 0 xO) = (0; y)
Assim, por exemplo:
(0.3) = (3;0)x(0;1)
(0; -5) = (-5; □) x(0; 1)

30
Como (y; 0) = y e (0; 1) = i, podemos escrever
(0; y) = (y; 0)x(0; 1) = y x
ou seja, para todo imaginário puro vale a notação;
(0; y) = yi
Portanto, (0; 3) = 3r, (0; -5) = -5i, (0; -1) = -i sâo exemplos de imaginários puros.

2.4 — Forma algébrica


Com as novas notações e definições vistas, podemos representar um número
complexo qualquer z = (x; y) numa outra forma que, como veremos, tornará hem
mais práticas e simples as operações
Notando que sempre podemos escrever:
z = (x;y) = (x;0) + (0; y)
Como (x; 0} = x e (0; y) = yi, temos:

Z = (x; y) = x + yí

que é chamada forma algébrica de z.

Observação: Se z = (x; y) = x + yi apresenta y * 0, z é chamado número


imaginário.
Exemplos

a) (2;7) = 2 + 7Í
b) (-3; 5) =-3+ 51

c) i2._n.2_r
U' 2) 3 2
d) (-10; 0) = -10+Oi =-10
e) (0; 7) = 0 + 7Í = 7i
f) (0.0) = 0 + Oi = 0

Os números reais xe y são. respectiva mente, denominados parte real de z e


parte imaginária de z E usual representa-lcs pelos símbolos:

X Ke(í) e y = 1m(z)

Operações na forma algébrica - Examinando alguns exemplos, vejamos como se


aplicam com a forma x + yi a igualdade, a adição e a multiplicação de complexos.

31
Exemplos

(x, y) = (-2; 1 3) <=> x = —2 e y = 13


1.0)
x + yi = -2 + 13i<=> x = -2 e y = 13

Vemos aqui que, para igualarmos dois complexos na forma algébrica, basta
identificar as partes reais e as parte imaginárias, isto é:

a + bi = c 4- di c=> a = c e b = d (a, b, c e d reais)

(2;13)+(5;1) = (7;14)
2o)
2 + 13i + 5 + i = 7 + 14i

Para somarmos dois complexos na forma algébrica, somamos as partes reais


e as partes imaginárias, ou seja:

[a + bi] + [c + di] - [a + c] + [b + d]i (a, b, c e d reais)

(7; 3)(5; 5) = (7 >6 - 3 >6; 7 >6 + 3 >6) = (17; 57)


3°) [7‘73ÍM}'+6i] = 35 4-42i +15i +1 17 + 57i

Para multiplicarmos dois complexos na forma algébrica, aplicamos a


propriedade distributiva, como se estivéssemos trabalhando com
expressões reais da forma a + bx, mas lembrando sempre que i2 = -1. Em
termos gerais:

l[a + bi] >{c + di] - ac + adi + bci + bci2 - [ac - bd] 4- [ad + bc] i

(a, b, c e d reais)
Resumo da nomenclatura - O quadro a seguir nos dá uma visão
da nomenclatura introduzida neste capítulo. Nele acrescentamos algumas
denominações que não foram citadas anteriormente.

Símbolo ____________ Denominação__________


O; o) = 1 unidade real________________________
(Q; 1) = í unidade imaginária_________________ _
z = (x: y) forma cartesiana de z________________
z -- x. ~ yi forma algébrica de z_________________
Re (Z) parte real de z______________________
(z) parte imaginária de z_________________
Se y = 0, então z = x + yi = xé real,_______
Se y * 0, então z = x 4- yi é imaginário.______
Se x = 0 e y * 0, então z = x 4- yj = yi é imaginário puro.

32
Exercícios Resolvidos

2.1) Sendo z1 = 1 + 4i, z2 = 3-3i e z3=5i, efetue:

a) z^ + z2 -z3
b) z, :«z2 + z3
c) (z, + z2)2
d) (z,-z2)2-z|

Solução

Lembrando que as operações com números complexos na forma algébrica


se processam de modo análogo ao que utilizamos com expressões
algébricas, e que i2 =-1, temos:

a) zn + z2 — z3 = (1+4Í) +(3 — 3i) — (Ei) = 1 + 4i + 3 — 3i + 5i — 4 — 4i

b) 2. xz2 - z3 = (1 + 4ÍJX3 - 3i) + 5i = (3 - 3i + 12i-12Í2) + 5i =


- 3-3i + 12i+12 +5i = 15 + 14i
c) (2, + z2)2 = (1 + 4Í + 3-3l)2 = (4-í)2 = 16-8Í + ÍZ =16-8i-1 = 15-8i

d) (Zj-Zí) 2-zj = (1 + 4i-3 + 3i)2 - (Si)3 - (-2+ 7i)2 -125i3 =


= (4 - 2Si + 49i2) -125i2 xi = 4 - 28Í - 49 + 125i = -45 + 97i

2.2) Calcule (1-i)6

Solução

É claro que poderiamos calcular (1-i)6 efetuando o produto de 6 fatores


(1 — í) (1 - i) ... (1 — i) ou utilizando a formula de desenvolvimento do binômio
de Newton (x- a)n.‘ Porém, neste caso, podemos fazer

(1-i)6 = í2]3=[1-2í-1]3 =[-2i]3 = -8Í3 = -8i2 xi = 8i

2.3) Determine c real x para que o número complexo z = 2 + (x-4i)><2 +xi) seja:

a) real b) imaginário c) puro

Solução
Vamos, ínicialmente, escrever z na forma algébrica a + bi:
2 =; 2 + (x-4i)(2 + xÍ) = 2 + 2x + x2i-8i - 4xi2 = 2 + 2x + xai-8i+ 4x
Assim: z = (2 + 5x) + (x2-8)1.

Nc capitulo S estudaremos um processo geral para o cálculo de potências na forma


(a + bi)n.
33
a) Para que z seja real é necessário que sua parte imaginária não seja nula:

1m(z) = 0 « x2-8 = 0 « x = ±2^2

b) Para que z seja imaginário é necessário que sua parte imaginária não
seja nula:

lm(z) * 0 « x2-8 * 0 <=> (x * 2V2 e x*-2\/2)

c) Para que z seja imaginário puro é necessário que sua parte real seja nula
e sua parte imaginária não seja nula:

Re(z) = 0«2 + 6x = 0<=>x =

(Note que, se x = Re(z) = 0 e 1m(z)*0).

2.4) Sendo, z1 = 3x - 2yi, z2 = x + 5yi e z3 = 6 - 6i, determine os reais


x e y para que z1 + z2 = z3.

Solução
+ z2 = z3 <=> 3x - 2yi + x + 5yi = 6 - 6i <=>
4x = 6
<=> 4x + 3yi = 6 - 6i «
3y = -6
3
Logo, x = - e y = -2

2.5) Determine os reais x e y para que se tenha (2x - 3yi) x(2 + i) = -30i.

Solução

Efetuando o produto indicado no primeiro membro da igualdade, temos:

4x + 2xi-6yi-3yi2 =-30i
4x + 2xi-6yi + 3y = -30i

É importante que, para igualarmos dois números complexos, escrevamos


em destaque a parte real e a parte imaginária de cada um. Assim, a última
igualdade acima se descreve:

(4x + 3y) + (2x-6y) i = 0-30I

4x + 3y = 0
e dela tiramos o sistema
2x-6y = -30

que resolvido fornece x = -3 e y = 4

34
2.6) Determine z - C tal que z3 = 0.

Solução

Para que possamos utilizar a igualdade de complexos, fazemos z = x + yi


(x e y reais). Então, z3 = B se escreve:

(x + yi)3 = 8
x3+ 3x2yi +3xy2í2+ y3i3 = 8
x3 + 3x2yi-3xy2 + y3i2 x = 0

Destacando a parte real e a parte imaginária de cada membro, temos:

(x3-3xy2) + (3x2y-y3)i = 8 + 0i

x3-3xy2 = 8(1)
e dar o sistema
3x2y-y3 = 0 (II)

Fatorando a equação (II):

y{3x2-y2) = D

obtemos y = D ou y2 3x2.
Fazendo em (I). y = 0. vem x = 2.
Substituindo, em (I), y2 por 3x2. vem x = - 1 e, portanto, y = ±73.
Logo, os valores de z = x + yi são:
(x = 2;y = 0) « z - 2
(x = -1;y = 73) <=> z = -1+ 7ãi
(x = -1;y = -73) « 2 = -1- 73 i

2.7) Determine ze C para que se tenha z2 + 5 = 2x

Solução

Fazendo a substituição z2 + 5 = 2z. (x e y reais), temos:

z2 + 5 = 2z « (x + yi)2 + 5 = 2(x + yi) «


« (x2 + 2xyi + y2i2 + 5 = 2x + 2yi w
«(x2-y2 + S) + 2xyi = 2x + 2yi
Desta igualdade tiramos o sistema:
jx2-y2 + 5 = 2x (I)
[2xy = 2y (||)

35
Da equação (II), y(x-1) = 0, obtemos y = 0 ou x=1.
Para y = 0, a equação (I) não apresenta soluções reais para x.
Para x = 1, a equação (I) fornece y = ± 2.
Logo, o número z = x + yi é z = 1 + 2i ou z = 1 - 2i.
Outro modo - A equação z2 + 5 = 2z pode ser escrita:
z2-2z + 5 = 0
Trata-se, então, de uma equação do 2° grau de coeficientes reais,
podendo, por isso, ser resolvida como segue:
-b + VÃ 2±V^Í6 2±Vl6i2 2±4i
z =-----------=------------- =------------- = 1±2i
2a 2 2 2
Assim, z = 1 + 2i ou z = 1 - 2i.
O leitor deve entender que este processo só foi utilizado no momento para
facilidade de calcularmos a raiz quadrada do discriminante A, que, neste
exemplo, é um número real (A = — 16).
No caso em que A resultasse um número imaginário - e isso pode ocorrer
quando os coeficientes da equação não são reais - teríamos alguma
dificuldade em calcular a raiz quadrada de A. No capítulo 5, exercício 5.12,
mostraremos como enfrentar o problema.

2.8) Sabendo que a equação em x:


x2 + (a + bi)x + c + di = 0
(onde a, b, c e d são reais não nulos), admite uma raiz real, mostre que
abd = d2 + b2c.

Solução
Seja x = a a raiz real da equação. Então:
a2 +(a + bi)a + c + di = 0 ou
a2+ aa + bai + c + di = O ou
(a2 + aa + c) + (ba + d)i = 0 + Oi
Da igualdade de complexos, tiramos;
í 7
Ia + aa + c = 0 (D
|ba + d = 0 (II)
d
De (II) vem a - —; substituindo em (I), obtemos:
b
í
l bj +aIik —bJ: + c = 0
d2 ad
-y- —+ c = 0
b2 b
d2 -abd + b2c = 0
Logo: d2 + b2c = abd.
36
2.9) Considere os complexos z, - sen a + i cos a e z2 = cos a-i sen a, on­
de a ê um real qualquer. Mostre que, se z = z, >z?, então -1 á RB (z) í 1 e
-1 í |m(z)£ 1.

Solução

z = z, xz2 ~ (sen 12 + 'cosa) (oos « — i sen a)


z = sena cosa - i senza + icos2 a -i2 senacosa
z = (2 sena cas a) + (cos2 a- sen2a) i

2 senacosa = sen2a As fórmulas trig ono métricas do quadro ao


■5 7
cos a - sen a = cos 2a lado permitem escrever:

z = sen 2a + i cos 2a
LW

Como R#(z) = sen2cr. e -1ssen2a£l, temos -l£Re(z)s1.


Como Im(z) = sen2ot e -1 í cos 2a <1, temos-1 s lm(z)s 1.

2.10} Determine a lugar geométrico dos pontos P(x; y) do plano cartesiano para os
quais o produto de números complexos (x + yi)(y + xr) seja 4L
SclLiçaa
(x + yi)(y + xi) = 4i
2- -2
xy + x i+y i + xyi - 4i
xy + x2i + y2i- xy =4i
0 4-(x2 + y2) i = 0 + 4i
21 2
Temos, então, que x + y =4.
Da Geometria Analítica, sabemos que uma equação da forma x2 + y2 = r2
representa uma circunferência com centro na origem (0; 0) e raio |r|.
Portanto, o lugar geométrico procurado é a circunferência de centro O (0; 0)
e raio 2.

-2 0 '2

-2

37
Exercícios Propostos

2.11) Sendo z, = 3-2i,z 2 = 2 + Si e z3-1+i, calcule:

a) Zl + z2+za
b) z1-3z2 + iz3
c) z,xz2xz3
d) iz, + z3 xz3
e) (zl + z2)2 + z22
f) z?-zl + zl

2.12) Calcule:

a) (-31) s
b) (1 + i/
,6
c) (-1’i)
d) (1-i)10

2.13) Determine o real x de modo que o complexo z = (13i - 6) + (2x+i)>(3 - 2xi)


seja:

a) real
b) imaginário
c) imaginário puro
14) A que condição devem obedecer os reais x e y para que z= (x + yi) >(y + xi)
seja um imaginário puro?

2.15) Seja z = ai (a - 2i) - (10 + 9i), ae R. Determine a para que se tenha:

3)Re(z)iO tOlrn(z) < 0 C)Re(z)<0 elJz):>0

2.16) Sendo z1 = 2x + yih zz = y-2xi e z3=1-i, determine os reais x e y tais


que:

a) Zt + z2 = 23z3 - 2

b) z1>22 = z3+(1 + i)

2.17) Determine o real x para que:

a) x(2 + xi)-6i2 =9i

b) [x - 5i)(2 + xi) = 14

38
2.18) Determine Ke C, tal que:

a)z2 =3 + 4i

b)z3 = -27

2.19} Determine Z e C, tal que:

a)z2 = 2z-3

b) z2 + 14z + 50 = 0

2 20} Sejam os reais a, b, ced não nulos. Sabendo que a equação em x


X2 + (a + bi)x 4-c + dr =
admite um número imaginário puro como raiz, mostre que abd = d'i 2 ““ 3 C ■

2.21) Escreva na forma algébrica a + bi os seguintes números complexos

rr n l2
a) cos — + »sen— :
12 12J

/
,._i n n i
b)2 sen— +1cos— I
l 8 8}
2.22) Sendo a e b números reais e a + bi = (sen a + i cos a)(cos a - i sen a), mos­
tre que a2 + b2 = 1, Vote R.

n
2.23) Seja a e R - J - + k7t, k e z k Se z = (1 + i tg «)(1 - í tg u), mostre que Re(z) à 1
2
e lm(z) = 0.

2.5-As potências naturais de i

Consideremos as potências do tipo in, onde n é natural. Vejamos alguns


exemplos:
i° = 1 j< =p>i2 = (-1)(-1) = 1
i1=i js sp« = 1>$ = i
i2 = -1 i5 = i4xj2 = 1xf-1) = -1
j3 =j2M = (-1)X = -i j7 = >? = 1 X(-1) = -i
Começamos então a perceber que, à medida que n cresce, os resultados de
inh vão-se repetindo periodicamente, assumindo sempre um dos valores da
sequência:
1, i

39
sendo, pois, de 4 unidades o período de repetição; isto nos sugere que, para
calcular o valor de in, basta elevar i ao resto da divisão euclidiana de n por 4.

De fato, se dividindo n por 4 encontramos quociente q e resto r, temos:

n « n = 4q + r e ir < 4
r q

Então:
jn =j4q+r =j4q>jr =(j4)q^r =(i)q>jr = f

e, portanto:

in =ir

Exemplos

a) Calculemos i67.

67
27
k_
16
(2)
Como r = 3, temos: i67 = -j

b) Calculemos i726.

726
32
k181
06
(D
Como r = 2, temos: j725 =j®=-i

40
Quando o expoente n é maior que 99, podemos facilitar a regra acima utilizando
um resultado da Aritmética que nos diz:

O resto da divisão euclidiana de um número natural n(n z 100)


por 4 pode ser obtido dividindo por 4 apenas o número formado
pelos dois últimos algarismos de n,

Assim, no caso do exemplo b acima, podemos fazer:

728 4 ou, simplesmente, 4


32 1B1 ______ @ 8
06
0>
, — , , .B3S 7S3
c) Calculemos i
Como o número formado pelos dois últimos algarismos do expoente é 53,
fazemos:

53 4
&36753_ j®_ j
13 13
O

Em resumo, temos as seguintes igualdades, para todo ke N:

i4k =1 :4k+2 ^k+3


=i

Observação: Na realidade, as quatro igualdades acima são válidas para todo


k e Z, Basta notar que, índependentemente de k ser positivo ou
não, temos:

(i4)fc = 1k = 1
14k+1 = i')k XÍ = 1 X = 1
j4k+2 _ j4k M2 =1 =
j4k+3 _ j4k XÍ3 =14-1) = _j

41
Exercícios Resolvidos

2.24) Quantos valores distintos assume a expressão A = in + i ne N

Solução
_ jH | 1 _ i20 +1 _ (i2)n +1 (~1)n + 1
A = in+i
in in in in

(-1)n=1
Se n é par, temos
in = ±1
e portanto:

A = —= ±2
±1

Se n é impar, temos (-1)n = -1 e, portanto, A = =0


in
Logo, A assume três valores distintos: -1,0,1.

2.25) Determine a relação entre os naturais m e n para que im = in.

Solução

Dividindo a igualdade dada por in, temos:

im in
in in
ou seja, im-n =1.

Como 1 = r4k para todo k e Z, vem:

m-n = 4k (m - n é múltiplo de 4)

Exercícios Propostos

2.26) Calcule:

36
a) i
b) i507
c) i134S
d) ,307533

e) i32 -i5 + i7 -I i«
28
f) (1 + i)

42
2 27) Sendo n e N. calcule:

j" . . jn jin
a) A=-^ b) A = ín + in+1 c)A = ^j-

2,28) Quantos valores distintos assume a expressão A = I 3n-2 _j


jne N?

2.29) Determine a relação entre os naturais m e n para que se tenha

a) im =i b) rm = r3

2.6 - Conjugado de um número complexo


Seja z = a + bi (a e b reais) um número complexo. Chama-se conjugado de z o
número complexo:

z = a- bi

Exemplos

a}Se z = 5 +11r seu conjugado é z - 5 -11L


b)Se z = 1-6i, seu conjugado ê z = 1 + 6i
o) 2 = 3i « z = -3i
d) z = 5cv z=5
e) -7+5Í =-7 - 5i
f) 2 = 3 + 4i w z = 3-4i w (z) = 3 + 4i
É imediato que;

1o) Se z = 2 entáo z é real (veja exemplo d acima).

2o) (z) = z (Veja exemplo f acima)

Um resultado muito importante é dado pelo produto de um número complexo pelo


seu conjugado:

z*z - (a +bí)(a - bi) = a2 — (bi)2 = a2 -b=i= = a2+b2

ou seja, o produto zxz é sempre um número real não negativo:

(a + bi)(a- bi) = a 2 + b2

43
Exemplos
g) (3 + 4i)(3-4i) = 32 + 42 =9 + 16 = 25

h) (2-5i)(2 +5i) = 22 + (-5)2 =4 + 25 = 29

i) (V2+i)(V2-i) = (V2)2+12 =2 + 1 = 3

2.7-Divisão

No exercício 1.18, aprendemos a dividir dois números complexos multiplicando o


primeiro pelo inverso do segundo. Agora, utilizando o resultado do produto z>z
visto o item anterior, podemos efetuar a divisão de forma bem mais prática.
Acompanhemos o exemplo:

2+i 2 + i 5-3i 10-6i + 5i - 3i2 13-i 13 1


=------- x-------
5 + 3i 5 + 3Í 5-3i 5z + 32 34 34 34

Assim, temos que. para se efetuar a divisão —, basta multiplicar nume-


z?
rador e denominador pelo conjugado do denominador.

Exercício Resolvidos

3i
30) Calcule —
2-i

Solução

_3i 3i 2 + i 6i + 3i2 -3 + 6Í 3 6.
“ 2-i X2 + 1 “ 22 +(-1)2 ■ + —I
2-i I5 5 5

2.31) Determine o inverso de z nos seguintes casos:

v < 1-
a) z = 1+—i b) z = i

Solução

1 1 —— i 1--Í 1-li
t . 1 ■ 1 1
a z = 1 + -1 => z
1 = 1 + -Í
1 X
1
1
2
5
í-£i
2 z 1~1| 5 5
r2 2 2 4

1 1 1 -i -i
b) z = i => z = - x— = — = —i
z T i -i 1

44
2x + i
2.32) Determine xe 5 para que z= : seja:

a) real b) imaginário não puro

Solução
Vamos determinar as partes real e imaginária de z, efetuando a divisão:

2x + í 2x + i 3-xi 6x + 2x2i + 3i - xi2 7x 3_- 2_X2


z =-------
3 + xí 3 + xr 3 - xi 9 + x2 9 + x2 9 + x3

7x 3-2x2
Assim, Re(z) = e lm(z) =
9 + x2 9 + x2

a) Para que z seja real, devemos ter Lm(z) = 0. Então:

3-2x2‘ Ve
= 0=>3-2x2 = 0 = x=±J~3 = ± —
9 + x2 2 2

b) Para que z seja imaginário não puro, devemos ter Re(z) *0 e lm(z) 41 0 :

7x
Re(z) * 0 => íO => 7x^0 X 51 0
9+x2
3-2x 2 n0 Vã V6
=> X # —— e x *-----
Im(z) * 0
9 + x2 2 2

Vb
Então: x at 0 e x * — e x *-------
Ve
2 2

2,33) Seja z = , onde a, b, c e d são reais e c + di ü. Mostre que, se z e ■ü,


c + di
então bc = ad.

Solução
a + bi (a + bi)(c- di) <ac -adi + bci + bd ac + bd bc - ad.
2 =---------
c + di (c + dí)(c-di) C2+d2 c +d ' c +d
~
. , . „ bc-ad
Ze R lm(z) - => —- = 0 =■ bc - ad = 0 —’ bc = ad
c* + d'

45
2.34) Determine z e C tal que iz + 2z = -3 - 3i.

Solução
Fazendo z = x + yi (x e y reais), a equação fica:
i(x + yi) + 2(x-yi) = -3-3i
xi + yi2 + 2x - 2yi = -3 - 3i
(2x-y) + (x-2y)i =-3-3i

J2x-y = -3
Então
[x-2y = -3
Resolvendo este sistema, encontramos x = -1 e y=1.

Portanto, z = x + yi = -1 + i.

2.35) Determine ze C tal que (1 + i)z + 2 — 3i = 3 + 1.

Solução

É evidente que podemos resolver esta equação fazendo, como no exercício


anterior, z = x + yi. Vamos, no entanto, cuidar deste caso apenas isolando z:
(1 + i)z + 2-3i = 3 + i
(1 + i)z = 3 + i-2 + 3i
(1 + i)z = 1 + 4i
1 + 4i
Z - —’—
1+i
Efetuando a divisão:
; (1 + 4i)(1-i) 1-Í + 4Í + 4 5 + 3i
’ (1 + Í)(1-0 12+12 2

5 3.
Logo: z = -+-i
2 2

2.36) Seja ze Ctal que zxz = 1. Prove que —— = z, com z*-1.

Solução
Tomando z = a + bi, a e b reais, temos que:

46
zxz = 1=>a2+ b2 = 1 e
1+Z 1 + a + bi (1 + a + bi) (1 + a + bi) (1+a + bi)2 [d^ai + bi]2
1+2 1 + a-bi (1 + a “bi) (1 + a+bi) (1 + a)2 +b2 1 + 2a + a2 -rb 2
1
(1 + a)2 + 2(1 + a)bi - b2 1 + 2a + a2+ 2(1 + a)bi - b2
1 + 2a +1 2(1 +a)
1 + 2a +a2 -b2 + ^Ibi = 1 + 2a + a2 - (1 - a2) + bi =
2(1 +a) 2(1 +a) 2(1+a)
3
2a + 2a 2a(1 + a)
- + bi + bi - a + bi = 2
2(1 +a) " 2(1 +a)

Exercícios Propostos

2.37) Efetue as divisões:

1 + 2i 1 + 3i
3 3 + 2i d)
1 + 3l 3 - 4i

7 + 7i

2.38) Determine o inverso do complexo z nos seguintes casos:


a) z = -3i

b) z = 1+i
43
C) Z = í + i26
d) z = cosx + i sen x, xe®

30 X + 4i
2.39) Determine xe Kde forma que □ complexo seja:
X^+4 2 + Xi
a) rea! b) imaginário náo puro

a + bi
2,40) A que condições devem obedecer os reais a. b, c e d para que z =
c + di
seja um imaginário puro? (c + di * 0)

47
4 + wi
2.41) Determine o imaginário puro w para o qual Z = é real
w +1+i
Z2 +1
2.42) Mostreque.se z = cos a + i sen a, ae R, então é real.
z

2.43) Sendo z = sen a + i cosa, prove que —- = z, z *-1

2.44) Determine ze C tal que:

a) z-2>z = 4-3i
b) 2z-iz = 15i
c) z-(z)2
d) z = ixz-i>z

2.45) Determine z e C tal que:

a) iz + 3 - i = 4 + 3i b) (2-3i)Z + 5-i = 4

c)(1 + i)z + 1-3i = -2iz


2.46) Efetue:

a) (1 + 7i) + (3 - 5i)

b)(1 + 7i) + (3-5t)

c)(1 + 2i)>(3-i)

d)(1 + 2i)><3 -i)

1-i
e)
1+i

f) =
1+i
2.47) Propriedades dos conjugados - Sendo z-i e z2 números complexos
quaisquer, prove que:

1o) Zt + Zj = z1 + z2

2o) Z, xz2 = Z1 XZz

3o)
VZ2 Z2

48
2,43) Mostre que, para todo n natural, (zr) = (z)n.

2 49) Sabendo que zé C e 2z3 + 3z2 = a +bi a e b reais, mostre

2(z)3 + 3(z)2 = a-br

2 50) Considere a equação (E): az3 + px + y = 0, onde cx. (3 e y são reaís (a # 0).
Mostre que, se número imaginário z é raiz de (E)> então z também o é.

49
Capítulo
A geometria dos
3 números complexos

3.1 - O Plano de Argand-Gauss

Sabemos que os números reais podem ser associados aos pontos de uma reta,
isto é, a cada número real corresponde um ponto da reta e a cada ponto da reta
corresponde um número real:

_3 7
-n 2 2 2'
-------- H---------------- 1---------+1-------- . - h
-3-2-1 0 1 2 3 4
Como representar graficamente um número complexo? As próprias defi-nições
dadas:

— um número complexo z é um par ordenado de números reais a e b

Z = (a; b) = a + bi
—C = R x R

são, por si só, suficientes para percebermos a possibilidade de representação


através de um ponto do plano cartesiano.

Vamos, então, a cada número complexo z = a + bi, associar o ponto P(a; b) do


plano cartesiano. Desta maneira:

A cada número complexo corresponde um único ponto do plano cartesiano, e


a cada ponto corresponde um único número complexo.

Chamaremos o ponto P de afixo ou de imagem geométrica do complexo z.


yi
p
b 3

P(a;b) z = a - D!
-►
0 a x

51
Exemplos

Consideremos os pontos assinalados na figura:


YA
2 M
■ç>

N R «
—1
Q
—C------ F 4- -+
-2-1 0 1 12 3 x

-2

-3 -* P

a) O ponto M é a representação gráfica do complexo 3 + 2i.


b) O ponto N é o afixo do complexo - 3 + i.
c) O ponto P(2; - 3) é a imagem geométrica do complexo 2- 3i.
d) O afixo do complexo - 2 + 0i=-2éo ponto Q
e) A unidade imaginária i tem por afixo o ponto R(0; 1)
f) O afixo do imaginário puro - 2i é o ponto S(0; - 2)

A representação gráfica dos números complexos foi introduzida através de estudos


de Caspar Wessel (1745-1818), publicados em 1798 na Revista da Academia
Dinamarquesa. No entanto, a ideia só começou a ser reconhecida a partir de
1806, quando Jean Robert Argand (1768-1822) publicou sua exposição. A
incorporação definitiva da representação gráfica à Matemática só se deu quando
da divulgação dos trabalhos de Carl Friedrich Gauss (1777-1855), durante a
segunda década do século XIX.
Por isso, quando o plano cartesiano é utilizado para representar números
complexos, é costume chamá-lo de Plano de Argand-Gauss.

Observemos que:

todos os números reais têm seus afixos no eixo Ox; por isso Ox é chamado
eixo real;
os números imaginários têm seus afixos fora do eixo Ox: em particular, os
imaginários puros são representados por pontos do eixo Oy, que, por isso,
é chamado eixo imaginário.

Exercício Resolvido

3.1) Represente, no Plano de Argand-Gauss, os afixos dos números complexos


z tais que:

a) Re(z) = 2 b) lm(z) i 1 c)-2 < Re(z) £ 1


d)Re(z)-lm(z) = 0
52
Solução

Façamos z = x + yi, com x e y reais. Entào:

a) Re(z)=2cs>x = 2 y*
No plano cartesíano. a equação x = 2
representa uma reta vertical traçada
pelo ponto (2; 0). ■+ 2L
0 1 2 x

b) IM{Z)^1 ~ y >1 y*
Esta desigualdade representa a região
do plane situada acima da reta
1
horizontal da equação y = 1.
(incluem-se os pontos dessa rela).
o

C) < Re(z) < 1 » -2 < x í 1 y


Esta desigualdade representa a faixa
vertical do plano delimitada pelas re­
tas, x = — 2 e x = 1, excluídos os
-2; 0 x
pontos da primeira.

d) Re(Z)-lm(z) = 0 «■ x-y = 0
yf
Da Geometria Analítica, sabemos que
x + y = 0 é a equação da bissetriz dos
quadranles ímpares.
0 x

Exercícios Propostos

3.2) Represente, no Plano de Argand-Gauss, os afixes dos números complexos


z tais que;
a) lÉlm(z)<3
b) DíRe(z)í3 e -2 í^zJíO

c) £z) = g

53
3.3) Determine o lugar geométrico dos afixos dos números complexos z para os
quais o quociente é um imaginário puro.

3.4) Represente graficamente os números complexos 2 = (z; y) para os quais:


(x-2yí)(2y-xi) =-16i

y*
3.2 - Módulo de um Número Complexo

Considerado um número complexo 2 = a + bi, o b


ponto p (a; b)é o seu afixo. A distância p, de P
até a origem O ê facilmente obtida pelo teorema p.
de Pitágoras:
0 a x
p - a2 + b2 => p = \' a2 + b2

Pois bem: esse número p (real e nao negativo) é chamado módulo do número
complexo z. podendo também ser representado por |z|.
Temos, então, duas maneiras de entender módulo de um número com*
plexo z:

’a) (algebricamente) módulo do número complexo z = a 4- bi é o número real


não negativo dado por:

izl = P = 7a2 + b2
2a) (geometricamente) módulo do número complexo zéa distância do afixo de
z até a ongenn:

|z| - p = ÕQp

Exemplos y •

a) Sendo z = 2 + 3i. temos: 3


|z| = |2 + 3i| = h2 - 32 2

Logo, iz| - p = 7Í3 1


+■
0 í 2 X

54
h) Sendo z = - 4 - 3i, temos: y-
jz| = |-4-3í| = J(-4)2 + (-3)2 0
—I----- 1--------- 1-------
Logo, [z| = p^5. -3 -2 -V X
- '1
- -2

-3
P

c) Sendo z = -3 = -3*0i, temos: y±


|zf = |-3| = J(-3)2 + Q2
Logo, |z| = p = 3.
p 3
-3 C x

Note, neste exemplo, que z é real e a definição de módulo de um número complexo


está de acordo com a definição de módulo de um número real, que já conhecíamos
do volume 1 desta coleção

djSejam Pi o afixo de z = 3+ 2i e Pj o afixo y*


de seu conjugado z - 3 - 2i. 2
Repare que:
9 i
1a) Pi e P z sao simétricos em relação ao
eixo O),. 01 X
2’) |z| = |z| = >/Í3
-2 : p

3.3 - Propriedades Imediatas do Módulo

Sendo z e w dois números complexos, valem as propriedades:

1?)ÍZ] = |Z|
2.a)| zxw | = [ z|*J W I

3?)l— = i£l (w * 0)
w 1 wl
Suas demonstrações são imediatas, podendo ficar a cargo do leitor,

55
Exemplos
|z| = |12 + 5í| = VÍ22 + 52 =13
a) Sendo z = 12 + 5i, temos z = 12 - 5i e
|z| = |l2-5i| = 7l22+(-5)2 =13

b) | (2 + 3í)>(1 - 4i) | = | 2 + 3i i x| 1-4i | =


= V22 + 32 xjl2+(-4)2 = >/í3 xTÍ7 = V22Í

c) 1~3ij - l1~3ll- x^-H-3»2 -


C) -z—
_ /õ
2+i ’ M x/22 4-12 ’ V5 ”

Exercícios Resolvidos

3.5) Qual o lugar geométrico dos afixos dos números complexos z tais que
|z| = 3?

Solução

Podemos resolver este problema de dois modos: por um simples raciocínio


geométrico ou analiticamente.

1o modo- Como sabemos, o módulo


de um número complexo é a distância y
de seu afixo até a origem; estamos, 3
então, procurando o conjunto dos
pontos cuja distância até a origem
seja 3,
3 *x
Ora, sabemos que o lugar geométrico
dos pontos do plano, cuja distância a
um ponto fixo deste plano é -3
constante, é uma circunferência de
centro na origem e raio 3.

2°modo - Fazendo z = x + yi, temos:

z = 3 => v'x2 +y2 = 3 => x2 + y 2 = 9 (!)

Na Geometria analítica (vol. 6 desta coleção) aprendemos que a equação (I)


representa uma circunferência de centro 0(0; 0) e raio r = 3.

Assim, o lugar geométrico procurado é a circunferência da figura anterior.

3.6) Qual o lugar geométrico dos afixos dos números complexos z tais que
|Z|S3? '

56
Solução
De modo análogo ao exercício anterior, concluímos que a região do plano
correspondente à condição dada é o círculo de centro 0(0: 0) e raio 3 (sua
relação x2 + y2 <9.
y+
3

í
-3 73 x

-3
3-7) Represente graficamente os números complexo z que satisfazem a equaçao
I z |2 xi = z2.
Solução
Fazendo z = x +yi, temos:

| z |2 xi = z2 => (Jx2 + y2)2 M = {x - yi).2 (x2 + y2) i = x2 - y2 - 2xyi

Desta igualdade de números complexos vem o sistema-


Í0 = x2-y2
|x2 + y2 = -2xy

que também pode ser assim escrito:


j(x + y)x(x-y) = 0
[x + y = 0
É fácil, então, perceber que essas duas equações são simultaneamente
satisfeitas apenas pelos pares (x; y), que obedecem á relação:

x+y = 0
y-
que, conforme a Geometria Analítica,
representa a bissetriz dos quadrardes
pares.
x

3.8) Sendo w = 1 - i, represente graficamente os números complexos z tais que:


|z + w| >|z-wi|

Solução
Fazendo z = x + yi, temos:

57
• |z + w| = |x + yi + 1-i| = |(x + 1) + (y-1)i| = 7(x + 1)2 +(y~1)'>2

• |z-wi| = |x + yi-(1-i)i| = |(x-1) + (y-1)i| = V(x“1)2 + (Y ~1)2

Como jz + w| > |z - wi| podemos escrever:

J(x+i)2+(y-i)2 s V(x-1)2+(y-1)2
(x + 1)2 + (y-1)2 5(x-1)2+(y-1)2
x2 +2x + 12 x2 ~2x + 1
4x5 0
X50

Portanto, a representação gráfica dos Y4


complexos z tais que |z + w| > |z + wi| é
)
o semi-plano da figura ao lado.
0 x

3.9) Determine a área do polígono cujos vértices são os afixos dos números
complexos z tais que |z| = 1 e Re(z2) = 0.

Solução
Fazendo z = x + yi, temos:
1o) |zj 1 <=> 7x2 - y2 1 <=> x2 + y,22 = 1 (I)
2o) z2 = (x + yi)2 = (x2-y2) + 2xyi.
Como Re(z2)-0, vem x2 - y2 = 0. (II)
(!) é a equação de circunferência de centro na origem e raio 1.
(II) é a equação do par de bissetrizes (x + y = 0 ou x - y = 0).

58
Assim, os afixos dos complexos que satisfazem simultaneamente as duas
condições são os vértices de um quadrado cuja diagonal tem medida 2
(diâmetro da circunferência).
Logo, a área pedida é:
diagonal'2 4
S = (lado)2 =
72 J "2“
3,10) Sendoz e w, z * w, dois números complexos tais que:

n |z|=H=i
2o) zxw é imaginário puro

|z + w|
determine -------- .
k-w|

Solução

Fazendo z = a + bi e w = c +di. temos:

• |z| = 1 => 7a2+b.2: = 1 => a2 +b2 = 1

. |w| = 1 7c2 + d2 = 1 c2+d2=1

• zxw = (a + bi)(c-di) = ac-adi + bci + bd = (ac+bd) + (b-ad)i

Como z xw é imaginário punor ac + bd = 0 e bc — ad # D.


Então:

z+w I | z +w | | (a + c) + (b + d)J
z-w] ]z-w| | (a-c) + (b-d)i |

7(a + c)2 +(b + d)


_)2 Ia3 + 2ac + c2 + b2 + 2bd + d2
7(a-c)2 + (b-d)I2 v a2 -2ac + c2 + b2 -2bd + d2

Como a2 + b2 = 1 e ca + d2 = 1, essa última expressão pode ser escrita:

2. + w ■1 + 1-»- 2ac + 2bd í2 + 2(ac+bd) [2


z-w 11 + 1-2ac-2bd Í2-2(ac + bd) " \2 ~
nr
3.11) Prove, utilizando o Princípio da Indução Matemática . que |zn^-^í|n para to­

do natural n & 1.

Veja volume 2 desta coleção, p. 35.


59
Solução

Teorema 1 - A igualdade é válida para n = 1:

|z1|=z = |z|1

Teorema 2 - Vamos provar que, se a igualdade é válida para n = k, entáo o


ê para n = k + 1.
Hipótese: Jzkj - Jz|k
Tese: izk+il=kr1
Vamos partir do primeiro membro da tese e utilizar a propriedade:
|zxw| = |zMw|,
Hipótese
””’A

|z^|=p x?i'| = |zk | >|zf = |z|k >|z| - jz|k+1 - (2° membro da tese)
Aplicação: Vamos calcular - i)3

r /------------
J3-Í|a = VÍ3)3 + Í-1)2 =[VÍ]S =2® = 256

Exercidos Propostos

3.12) Calcule'
a) |7-24i|
b) |(2 + 3i)xí1-i)|
|(-3-4i)(2 + 2i)
c) .------- -----------
l 1+1
(s/2 + 2i)a
d)
(-1 + 2j2t)d

3.13) Represente graficamente os números complexos z tais que:


a)|z[=2 b)|z[32 c)1<|z|í2

3.14) Determine o lugar geométrico dos afixos dos números complexos z para os
quais:
a)|z| + z = 2 + 4i b)|z-3i|=|z + 2|
c)|z-1 + 2i| = 2 d)fz-1 + 2ijí2

60
3.15) Represente, no Plano de Argand-Gauss, qs complexas z tais que:

a) |z + 2[ + |z-2| = 6
b) |z + 2| + |z - 2| í 6
3.16) Sendo w - cos a + i sen ct, a real, determine a módulo do número complexo
Z2
z para o qual tem-se — = S.
w

3.17) Senda z e w númeras complexos tais que-


1°} |z| =|w[ = 1

z +w
2o) - J10
j zw

determine R_í — I

3.4 - Outra Propriedade do Módulo: a desigualdade triangular

|z + w| Ú |z| +|wj

Sendo z e w números complexos quaisquer, tem-se que


Examinemos um exemplo. Senda z = 1 + 3i e w = 2 + i, temos:

- |z + w[ = |3 + 4i[ = -Ja2 + 4 2 =5

• |z| = |1 + 31| = 7l2 + 32 =710 = 3,16


♦ |w[ = j2 + í] = V22 +12 = 75 = 2,23

□ e fato, temos 5 < 3,16 ■+■ 2,23; portanto, neste exemplo vemos que o módulo
da soma é menor que a soma dos módulos,

4 ■
Pelo gráíica abaixo, a situação torna-se
/ +
visível: a figura mostra um paraieloqramo,
//
onde os lados têm medidas | z | e | w | e
uma diagonal mede |z + w|.
3
7

1
I 7Í
! * z
|z|/V
/|z|

0 i 2 3 4 x

61
Assim, observamos que nos triângulos em que o paralelogramo está dividido, os
lados têm medidas|z|, jw| e |z + w|;da Geometria Plana sabemos que (num
triângulo) cada lado é menor do que a soma dos outros dois. Daí a propriedade:

]z + w| < |z| + ]w|


ser conhecida como desigualdade triangular
Fica evidente que, quando |z| e |w| estiverem representados por segmentos
contidos numa mesma reta que passa peía origem, ocorre 0 caso em que
|z+w| = l2H*L
Por exemplo, se z = 1 + 2i e w = 2 + 4i y+
(note os pontos (1; 2) e (2; 4) 6
pertencem â reta da equação y = 2x),
temos: 5
-]z + w| = i3 + 6i| = J9 + 36 = 3^5
4 ■
- p + 2ij = 7^+4 = Vb
• |w| = |2 + 4i| = 747Í6 = 2^5 3 |£4W|

(375 = 75+2^5) 2-

1-
*7/^
0 1 2 3

Demonstração da propriedade
Sejam z = a + bi e w - c + di, a, b, c e d reais. Assim, temos
z + w = (a + c) + (b + d) i,
A desigualdade |z + w| < |z| + ]w| pode, entáq, ser escrita:

7(a + c)2 +(d + d)2 < a2+b2 + ÍW


Elevando ambos os membrcs ao quadrado, ela é verdadeira se:
(a+c)2 +(b + d)2 s a2 + b2 + 27(a2+b2)(c2 + d2) + c2 + d2

que simplificada fica: ac + bd í + b2) (c2 + d|22)


Quadrando novamente ambos as membros,a desigualdade é verdadei­
ra se:
a2c2 + 2abcd+b2d2 5 a2c2 + a2d2 + b2c2 + b2d2
Isto é, se:
2abcdí a2b2 +b2c2
2
Esta última relação é equivalente a: (ad -bc) 5 0
que é indiscutivelmente verdadeira, sendo a, b, c e d reais.
Portanto, partindo desta desigualdade, revertendo o processo pas­
sagem por passagem (e cada uma delas é reversível), provamos a pro­
priedade_______________________________________ __________ _____ .
62
Exercicios Resolvidos

3.18) Sendo z um número complexo tal que | z + 3 — 4i | = 12. determine o valor


minimo de | z |.

Solução

Pela desigualdade triangular, temo que:


|z + 3-4i|Ê|z| + |3-4i|

Como a |z + 3 —4i[ = 12 e ]3-4i]= a/32+ (-4)2 = 5, desigualdade se

escreve:
12 <|z| + 5
donde obtemos’
|z|57

Assim, o valor mínimo de é | z | é 7.

3.19) Os números complexos z = a + bi e w = c + di são tais que a>b>c>d*0 e


|z + w| = | z| + | w|.

Determine a relação entre a, b, c e d.

Solução

Sabemos que, quando o módulo da soma é igual ã soma dos módulos, os


afixos dos complexos pertencem a uma mesma reta (r) que passa pela
origem.

Seja, então, y = mx a equação dessa


reta (r). d
Se (a; b) e r, temos b = ma. (I)
Se (c; d) e r, temos d = mc. (II)
De (!) e (II) temos: b
b d
m = - e m= —
a c ■>

b d
a c x
Portanto, — = — ou ad = bc. r
a c

Exercicios Propostos

3.20) Mostre que, sendo z e w números complexos quaisquer, então


|z-w|<|z| + |w|

3.21) Mostre que, sendo z-f, zj e 23 números complexos quaisquer, então:

|zi + z2 + z3|<|zi| + |z2| + |z3|

63
Capítulo
A trigonometria dos
4 números complexos

4.1 - Argumento de um Número


Complexo

Sejam, z = a + bi um número complexo b


não nulo e P o ponto que o representa.

A medida 0 do ângulo formado pelo


semieixo positivo Ox e pelo segmento OP
(tomada no sentido anti-horário) é
chamada argumento principal do número
0 a x
complexo z, e é indicada por arg(z):

6 = arg(z)

No caso em que b = 0 e a > 0, isto é, quando P está no semieixo positivo Ox,


adotamos 0 = 0. percebemos, então, que:

0 < arg(z) <

Nota: Damos o nome de argumento principal a 0 pelo fato de também serem


considerados como argumentos do número complexo z = a + bi todos os
côngruos de 0, ou seja, os ângulos de medidas:

0k - 0 + 2kre

onde keZ. Assim, se 0 =— é o argumento principal de um complexo z, também


2
são argumentos de z:

5n 9n 13n 3n 7n
— , etc.
~2 '~2 '~2~ '~~2 ' 2

No entanto, é frequente que nos refiramos ao argumento principal 0 simplesmente


como argumento de z.

65
Exemplos y+
a) Seja z = - 5.
Temos, então, o ponto P(- 5: 0) no semi-
eixo negativo Ox. Portanto: 0
p
0 = rc *x
-5 p-5 0


b) Seja z = - 3i,
Temas, então, o ponto P(0; - 3] no semieixo
negativo Oy. Portanto:
a
>
9=^ 0 X

2 p=3

-3' ■P

c) Seja z = 1 + 75 i. y|
Calculando | z |, temos:
3 p
|zl = p = Ji2+(75)2 =77 = :2
»V(
No triângulo retângulo OAP, temos:
sen cateto oposto _ 75 4/ 3

hipotenusa 2
Como 0 é agudo, ê imediato que;
1

d) Seja 2 = 275-21 YA

Calculando ] z|. temos:


0
|z| = p = J(275)3 + (-2)2 =4 A
0 o. X
Para determinarmos 6, vamos primeira­
mente determinar □ ângulo a no triângulo 2
OAP da figura, já que é imediato que p=4
0 = 2n - a :
-2 P
2 1
sen a = - -
4 2

66
Temos, então, a = — e,
6
portanto:

e = 2^--
6 6

e) Seja z - - 3 + 4i.
Calculando | z |, temos: y*
P — 4
| ZI=p= 4.4 2 =5

Para determinarmos 9. vamos primeira­ p = 5


mente determinar o ângulo ct no triãn-gulo
OAP da figura, já que ê imediato que 4
9 = 180’ -çt.
4
sena = — = 0,8000
5
a 'P*
-3
Consultando a tabela do final do livro,
temos a = 53°.

Logo. 9 = 180°-53’ =127’

4.2 - A Forma Trigonométrica dos Números Complexos

As definições de módulo e argumento de 2 nos permitem escrevê-lo numa nova


forma, além das já utilizadas: (a; b) (cartesiana) e a + bi [algébrica),

Se lembrarmos que os sinais das coordenadas (a; b) de um ponto do plano


cartesiano são, em todos os quadrantes, os mesmo que os do cosseno e do seno,
respectiva mente, è fácil verificar que para todo número complexo z = a + bi * 0.
cujo módulo é p e cujo argumente é 9. valem as relações:

Yl
b a b P
,sen9 = e cos9 = —
fc____ P _al
P
Õ
p a x

67
Dessas igualdades, tiramos:

b = p sen 6 e a = pcosB

Podemos, então, escrever:

z = a + bí = pcosB + (p sen 6)i

isto é:

z = p(cos 0 + i sen 9)

que ê a forma trigonomêtrica do complexo z,

Exemplos

a) z = -5temp = 3 e 9 = h veja exemplo a, do item 4.1)

Podemos, então, escrever: z = = 5 £cosn + i sen jt)

3*
b) z = -3Hemp-3 e 9 = — (veja exemplo b, do item 4.1)
2

- _J 3n: 3n
Então: z - -3i cos— + ix—
2 2

c) z = 1 + V3 temp = 2 e 6 = - (veja exemplo c, do item 4.1)


3

Então: 2 = 1 + J3i = 2| cos— + i sen —


l 3 3,

d) z = 2-^3-2i tem p = 4 1U (veja exemplo d, do item 4.1)


e 0-
5

Então: Z = 2\/3-2i = 4| cos^-^ + isen-^-^


l 6 6
Observação: Como, da Trigonometria, temos que dois arcos (ângulos) cõngnjQs
têm senos iguais e cossenos iguais, é evidente que a forma trigo-
nométrica de z pode ser escrita com qualquer dos argumentos de z
dados por 6k = 0 + 2k!t,ke Z, isto é:
z = p (cose + i sen 0) = p(cos +i sen )
Por exemplo, como n e 3rr são còngnjos, podemos escrever:

z = —5 = 5 (cos jt+i sen rr) = 5 (cos3tt + i sen 3rc)

68
Exercícios Propostos

4.1] Escreva na forma trigonométrica os números complexos:


a) z - 3

c) Z » 5i

d) z = -3+ 3J3i

e) z = -1
f) z =-1-i
g) z = -4i
h)z = 3-3i

4.2) Escreva na forma algébrica os números complexos'

n n
a) z = 6 cos- + isen­
3 3
3rc 3jt
ta) z — cos— 4- i sen —
4 4
7n 7n
c) z = 2|cos— + i sen —
6 6

2
4.3) Mostre que se o número complexo z tem argumento 0. então z tem
argumento 29.
4,4) Sendo Gi e 02, com 0, + 02 = 2tt, os argumentos de dois complexos de
mesmo módulo, mostre que tais complexos são conjugados.

69
Capítulo
Operações na forma
5 trigonométríca

5.1 - Introdução

Quando introduzimos a forma algébrica dos números complexos, notamos que


certas operações, como a multiplicação e a divisão, tornaram-se bem mais simples
de se executar.
A forma trigonométríca. por sua vez. tem a vantagem de simplificar o trabalho de
potencíação e de radíciação de números complexos, como veremos a seguir.

5.2 - Multiplicação e Divisão


Sejam os número complexos (não nulos):
z, = p^cose, +isen 9j
z. = pz(cos02 +i sen02)

Vamos calcular o produto 21 xz2

Zj>za = p1(cosÔ1 + i sen >ç2(cos fi2 + iseng2) =


= p, p2(cos01 cose, + icos61senG2 + Ísen01cos62 + r senG1 senü2) =
= p, p2 [(cosâ1 cosG2 - senG1 senG2) + UsenS^osQ^ + senS2 ccsSJl
Como, da Trigonometría, temos que:
casS, cos02 - senQ1 senG2 - cos(01 + 02)
e
senS, cose2 + sen&2 cosBt = sen(61 + 02)
escrevemos:
zr<z:2 - Pi PjcosfÔ! + 02) + l senfQj + G2)]

onde lemos que, para multiplicar dois números complexos na forma trigonométríca.
basta multiplicar seus módulos ê somar seus argumentos.
É fácil verificar que esse procedimento pode ser generalizado para um número
qualquer de fatores:

Zi xz2xz3 x..xzn = p1 yp2 *p3 x. xpn[cose1+02+e3 + ... + en) +


+ I sen (0, + S2 + e3 + ... + 0^)]

71
Vamos agora calcular o quociente —:
Z2

z, p, (cos G, + isenG,) p^cosB, + isenGJ Jcos02 - isen02) _


Z2 p2(cos02 + isen02) p2(cos92 + isen02) (cos02 - isenfJJ

p1^CO501CO502 - icosG, sen()2 + ísen91COSB2- i^senf^seriSj

p2(ccs2 92 - ii2?sen
sen 282)

p, I(cqs0,COS02 + senfi,sen02) + i(senG)Cos92 senGj gosGJ]


cos202 + sen?02
P!

Como, da Trigonomelna. temos:

OOSB, COS02 + 3000,56002 = 003(0, - 9j)


sen0,cos02 + senB2 cose, = sen(fl, - fl2)

cos202 + sen262 = 1

escrevemos:

— = “>{003(9) - 02) + isenfO, - 0J]


z2 p2

onde lemos que. para dividir dois números complexos na forma trigonomêtrica,
basta dividir seus módulos e e subtrair seus argumentos.

Exemplo

„ . „/ rt k 3n . 3?t’l
Sejam z, = S^cos- +■ isen— e z2 = cos— + i sen— L
4j 2 2J
Para calcularmos z, xz2, fazemos:
P)>p2 = 6>2 =12
„ n rt 3rr 7k
0, + 0 2 = ■— -I- ■—
l 4 2 4
e escrevemos:
7r 7n
- 12| cos— + isen —
4 4
Para calcularmos -- fazemos

= - = 12
p2 2
3u 5 71
e,1 - e22 = -4
T T
72
e escrevemos:

5n
— = 3 cos
4
+ i sen —4 Jli
Z2 L
Note-se que, neste resultado, o argumento obtido não é o principal 0. e sim o seu

congruo-------.Caso desejemos a resposta em função de 6. devemos calcular a


4
• 5rt
primeira determinação positiva dos arcos da mesma extremidade que------
4

_ _ 5n 3n
0 = 2n------
4 ~4

podendo, depois, escrever:

Zi _f 3n 3 rr
— = 3 cos- - + i sen —
z2 l 4 4

5.3 - Potenciação

Com base na multiplicação, na forma trigonométnca, vamos verificar como se


processa o cálculo de potências da forma zn = (a + bi)n. n e 7.‘, sem que
precisemos recorrer a métodos exaustivos como, por exemplo, o binômio de
Newton.

Consideremos o número complexo z = p(cos 0 + i sen 0) e o número inteiro n,


ambos não nulos e calculemos zn.

Se n > 0, temos:

zn = ZXZX..Z
n fatores

zn * p >p x.. >p [cos(0 + 0 + ...+ 0 + isen(0 + 0 +... + 0)]


n fatores 'w X
x n parcelas 7

Veja volume 3 desta coleção, p. 39.


73
Portanto:
zn = pn(cosn0 + isennfl) (I)

Se n < 0, temas - n > 0, podendo por isso, ser usado □ resultado (I) para
- n, Então, fazemos:
__ ______ 1
z° = -
2 ' p "ri[cos(-n0) + i sen(-nâ)]

cas(-nâ) = cos(n6)
Da Trigonometria:
sen(-n0) - -sen(n9)

1 p" pr casnfl + I senn9


zr = — cos nB - i sen n9 cos nâ - isennâ cos nS + I sennâ
z

pn(ccs nB -r i sen n0)


cos^nfi T sen3 nâ
“T”

Vemos, então, repetido o resultado (I).

Como, para n = 0 este resultado também se verifica [z° = pc (cos 0 + i sen 0) = 1],
temos que, para iodo inteira n:

z" = pn(cos n6) + i sen nSJ


nu seja, para elevarmos um complexo z * 0 a um expoente inteiro n qualquer,
basta elevarmos o seu módulo a n e multiplicarmos o seu argumenta por n.

A fórmula acima ê conhecida como fórmula de De Moivre,

Exemplas

a) Seja 2 = <5í ces — + isen — . Vamos calcular z a.


i 4 4J

18 , 13ir
19,1 18it
18:1
2 8 = (V2) ! cos— + i sen-----
4 4
9rr
zie = 2qf(, cos — Sjt
+ isen —
2 2
18 n9Í 0jt
ajT ■ 9,1
z =2 cos— + i sen —
2 2

„ 9tk . * , n
Cama — econgruode -:

21S = 512; cas — + i sen —


\ 2 2

74
ti ■ 5 rr
b) Seja z = cos—+isen—. vamos calcular z (note que p = 1 j.
3 3

"6 fcos{-6 x— ) +1 sen(


z = (1) -6 x— ,
L l 3J t 3 J
z‘6 = 1 >£cos(-1 DxJ + j sen(-IOrc)]

Como—107T é côngruo de zero:

z-5 = cosO + i sen 0 = 1

Exercícios Resolvidos

5J) Calcule (-2 + 2i)s

Solução Xy
P A
Vamos escrever —2 + 2i = z na forma í2
tngonomètrica:
P=2vr
p = 7(-2f + 2' = Va =272
H
Na figura, OAPB é um quadrado;
b;
portanto,
-2 0 x
n Jt 3 71
e 0 = k - a - n-—
4 4 T
3n 3n
Logo:-2 + 2i = 27s| cos— + i sen —
4 4
5 >3it 5x3n
zs = (-2 + 2Í)5 = (272)5 COS--------- + i sen------ - l =
4 4 J

15n 7n 72
COS---------- COS-----=
4 4 2
Como
15k 7rt
sen----- = sen — -
72
4 4 2

Z5 = 12S72Í— - i —
= 1B2-12SÍ
1 2 2

75
5.2) Deduza as fórmulas de sen 20 e cos 20 em função de sen0 e cos0 uti­
lizando a fórmula de De Moivre.

Solução

Sendo z = p (cos 0 + i sen 0), a fórmula de De Moivre nos dá:

zn = pn(cosn0 + isennB)

Para obtermos cos 0 e sen 20, façamos n = 2:

z2 = p2(cos20 + i sen 20)


Mas. como z = p(cos 0 + isen0):

z2 = p2(cos 0 + i sen 0)2 = p2(cos20 + 2icos0sen0 + i2 sen2 0)

isto é:
z2 = p2(cos20 - sen20 + i>2 sen0 cos0)

De (I) e (II). temos'

p2(cos20 + i sen 20) - p2(cos2 0 - sen20 + i>2senOcos0)

Portanto:

cos20 = cos20 - sen20 e sen 20 = 2sen0>cos0

5.3) Mostre que se (75 + i)2n, ne Z,é real, então n ê múltiplo de 3.

Solução y* senO = -j
Vamos escrever 73+i = z na forma tri-
P
gonométrica; temos: 1

P = \/(73) 3 + 12=2
>
0=^ 0 X
3
6

Portanto, Z-J3 + i = 2fcos- + isen-j


6 6

z2n = 22n 2mt 2mt


cos—- + isen-----
6 6

z2n n?r . nrr


= 4n cos— + i sen —
3 3 .

76
n 2n nn nn
Para que z seja real, é necessário que sen— = 0, isto é, o arco deve
3 3
ter extremidade em A ou em A’ (figura abaixo).

Logo, — = kn, k e Z, e daí, n = 3k = múltiplo de 3.


3
5.4) Determine o menor natural n para o qual (Vã +i)n seja um imaginário puro
de coeficiente positivo.
Solução
Sendo z = V3+i, temos: |z| = p = 2 e arg(z) = 8 = -.
6
Portanto:
z=2^. rt n
2 cos— + i sen — e
l 6 6
nn nn
zn = (V3 +i)n = 2n cos — + isen—
6 6

Como 2n > 0 para que zn seja imaginário puro de coeficiente positivo.


devemos ter:

nn nn
cos— = 0 e sen— > 0
6 6

isto é, o arco — deve ter extremidade no ponto B (figura acima}.


6
nn n
Logo, — = — + 2kn
6 2
e dai, n = 3 + 12k, k inteiro.

Finalmente, o menor natural que satisfaz a condição acima é n = 3, para


k = 0.

77
Exercícios Propostos

5.5) Calcule:
60
a)(-Vã+i)ls b) - —i c)(-2-2if1S
I2 2 '

5.6} Calcule
72 72
------- 1-------
72 72 Y
2 2 2 2 'I ’
5.7) Deduza as fórmulas de cos(nS) e sen(n0) em função de cosâ e sen6,
utilizando a fórmula de De Moivre e o binômio de Newton nos seguintes
casos:

a) n = 3
b) n = 4

5 6) Sendo n um número natural, mostre que se (1 + i)'1 ê imaginário puro, então


n é um termo qualquer de uma progressão aritmética em que a razão é
r = 4.

5.9} Determine o menor inteira positivo n para que (1 + i73}r seja;

a) real positivo
b) real negativo

5.10) Determine o menor natural n para que (-73 + i)n seja;

a) imaginário puro de coeficiente positivo


b) imaginário puro de coeficiente negativo

5.4 - Radiciação

Definição - Dado um número complexo z e um número natural n * 0,


chamamos raiz n-ésima de z a todo número complexo tu que satisfaz a
relação <çn = z.
A raiz assume um nome especial para cada valor de n, Assim, se n = 4,
dizemos raiz quarta de z Assim, se n = 7, dizemos raiz sétima de i, etc.
No caso de n = 2, costuma-se dizer raiz quadrada e, para n = 3, raiz cúbica.
Conforme veremos todo número complexo não nulo admite n raizes
n-êsimas. Por exemplo, o número 1 admite 4 raizes quartas (1; -1; i; _i)>
pois;
14 = 1; <-!)* = 1, i4 = 1 e (-Í)4 -1

78
Vejamos como determinar as raízes de um número complexa
Dado z = p cos 6 + I sen 8, seja:

ti) r (cgs ct + i sen a] uma raiz n-ésima de z,

Então:
ü/1 = z
rn(cos na + isenna) = p(cos8 + i senft)

donde tiramos:

rn = P, cos na = COS 8 e sen n« senfl

1* conclusão: <=P r^]

□u seja, o módulo da raiz n-ésima de um complexo z ê igual a raiz n-ésima do


módulo de z.
Assim, se por exemplo z tem módulo 2, suas raizes quadradas têm modula
72;suas raizes cúbicas têm modulo suas raizes quartas têm módulo Vi, etc.

cos n<t = cosô


2* conclusão: - na = 6 + 2krt
sen na = sen8

donde:
9
a = - +----- {k e Z)
n n

Lembremos que nt é argumento de ue notemos que, se atribuirmos a k os valores:

0. 1, 2, 3..... n- 1
obteremos n valores distintos e nào cóngruos para ct e, para qualquer outro valor
de k, o valor resultante de a será cõngruo de um dos jã obtidos.

Por exemplo, se z tem argumenta 6 = n, suas raizes quartas (n = 4) têm


argumentos dados por:
ir 2krt
a = — +---- , k e Z
4 4
onde:
n
para k = 0, temos a,= —
4

para k = 1, temos a
2=T
79
„ £ 5x
para k - 2, temos a3 = —

~ 771
para k = n — 1 =3, temos =—
4

e, caso continuemos, começaremos a encontrar côngruos:

9rr
para k = 4, a = — (cÓngrua de cti)
4

11n
para k = 5, Ct =----- (congruo de új)
A

Por tudo isso, concluímos que, com o valor de r já determinada e cada um dos n
valores distintos e não congruentes de a, pcdemas formar n números complexos íu
= r (cos a + i sen a), todas eles raizes n-ésimas de z. dados por:

2kn 6 2kx
i sen — 4------- (ke Z) (5.4)
n n n

Exemplo

Vamos calcular as raizes cúbicas de z = 0 + Si


Temos, na forma trigonometnca:
n( Jt Jl)
z = 8) cas— + i sen —
< 2 2j
onde p = 8 e 8 = -. Como n = 3, as raízes cúbicas de z são dadas por:
2
x n
(.Ü - à'8 cos 1+ + i sen 1+
3 3 3 3

ou seja:
F (x 2kxA (x 2kx
uj = 2 cos — + ----- + i sen| — + -—
l U 3 )
<6 3

Atribuindo a k os valores D, 1 e 2 = n - 1. temes:

k ■= D => (o, = 2 cos— + isen — = yã+i


1 l 6 6j
= 2|'úús— + isen^
k 1 =■ w3 = - V3 + i
\ 6 6
í 3ít 3n i
k = 2 => lüj = 2 cos— + isen— = -2i
l 2 2

80
Assim, J3 + i. - 73 -i- i e -2i são as três raizes cúbicas de 8i.

Interpretação geométrica - Observemos 0 exemplo dado: como a três raizes


cúbicas têm o mesmo módulo r = 2, seus afixos estão sobre uma circunferência de
n 2kn
centro na ongem e raio 2; a expressão — + —— nos diz que os argumentos a
6 3
determinam, sobre esta circunferência, 3 pontos distintos e separados entre si de
um arco de medida — .
3

(¥) 0>2

í +■
0 2 x

013 -2

Vemos, então, que os afixos das raizes cúbicas de z = 8i são vértices de um


triângulo equilátero inscrito numa circunferência com centro na origem e raio 2.
Analisando, agora a expressão das raízes n-ésimas de z:

n/ r Í9 2kn>| (0 2kn
cü = Wp , cosl — + ----- + 1sen[ - + -----
V L <n n ) \.n n

percebemos que:

1°) as n raízes n-ésimas de z têm o mesmo módulo r = yp. estando, por isso,
seus afixos sobre uma mesma circunferência com centro na origem e
raio r = ^p.

0 2kn
2o) da Trigonometria, sabemos que arcos da forma - +----- representam n pon-
n n
tos distintos no ciclo, distribuídos de modo a dividir a circunferência em n
partes iguais.
Portanto, os afixos das raizes n-ésimas de z são:

-se n = 2, extremidades de um diâmetro da circunferência de centro (0;0) e


raio r = ^/p.

81
- se n 2 3, vértices de um polígono regular de n lados, inscrita na
circunferência de centro (0;0) e raio ^p.

Exercícios Resolvidos

S.11) Sendo w — ^ + \ uma das raízes quartas de um complexo z, determine as


demais.

Solução

Vamos resolver o problema de dois modos.

1Ú modo - temos que | w0 | = r = Vl + 1 = V2 e arg(üJ0) = a = 7


4

As raizes quartas de z são dadas por:

r- F ( 9 kn \ f 9 kn 1
tü=f'p cos —+ — +isen -4—
L 2 ,J ^4 2/
.keZ

Como tiJo é uma dessas raizes, devemos ter:

tfp = |üjoi = &


e
9 krr x 0 x kit
- = a - - — ------- => e = w-2kn
4 2 4 4 4 2
Como 9 representa o argumento principal de z, tomemos 9 - ir.
Portanto, as raízes quartas de z são dadas por:

rx f n kit (ir kn^i


d) = J2 cos — + — + isen —+ —
U 2 U 2 ,1
e dai:
f * ■ n i
para k - 0, tü0 i cos—+ l sen — I = 1 + í
l 4 4)
„ n:( 3ir 3rr
parak = 1, tp, = v2^cos— +■ isen—- j = -1 + i

, _ çrf 5* , Srt^
parak = 2, ld, = ^/2icos------- f i sen—-i = -1 - i
2 l 4 4 J

para k = 3, w3 = 2icos— + i sen — - 1


\ 4 4
Assim, as demais raizes de z são;
~1 + i. -1-i, 1-i

02
2° modo - Sabemos que as 4 raizes quartas de z têm seus afrxos como
vértices de um quadrado inscrito na circunferência de raio r - [ | = ^2
Como ü)0 é um desses vértices, temos:

V2
WiZ- m0 = 1 + j
1 TT

4
>
2 -b 1 x

Ci>2 «3

-2

Então:
co4 é simétrico de too em relação ao eixo y; logo, = - 1 + i.
W2 é conjugado de to,, logo, = — 1 - i.
é conjugado de logo, üj2 = 1 - í.

5.12) Resolva, em C, a equação binômia x5 + a = 0.

Solução
Temos x3 + 8 = 0 x3 = -8
Portanto, os valores de x que satisfazem a equação dada são as raizes
cúbicas de z = - 8.
Na forma trigonométrica, temos:
z = -8 = 8 (cos ir + i sen rc)
Como n = 3, p = 8 e 0 = ti, os valores de x são dados por1
rc 2k7i’'i . (jt 2kir j .
x = í/8 cos - +----- l + isenl- +------ , k e
3 3 ) 13 3 J
Então:
_( n n1 . /r
para k = 0, x. = 2icos— + isen—i = 1 iv3
13 3J
para k = 1, x 2 = 2(cos7t + i senrr) = - 2

para k = 2, x 3 = 2lcos — -t- i sen — '1; = 1 - ij3


13 3 J

83
3
Logo, o conjunto-solução de x + 8 = 0 é:

s = {-2; 1 + i7ã: 1 - iTã}


5.13) Resolva, em C, a equação binômia x8-15xJ-16 = 0.

Solução
Fazendo a mudança de variável a equação dada se escreve:
y2-15y-16 = 0

Como suas raizes são y = -1 ou y= 16, vem:


x4 =-1 ou x< =16

Portanto, os valores de x que satisfazem a equação dada sao as raizes


quartas de -1 reunidas ás raízes quartas de 16,
— As raizes quartas de -1 - 1 (cosn + i sen n) são dadas por:

tffcosí^ 2kn |* 2kir


x=
u 4
+ i sem — +
U 4
, k e Z

— As raizes quartas de 16 = 16(cos0 + isenO) são dadas por:

(2kx í2kjrYI
x = ti 16 cos— + i sen ----- ; i k e Z
k 4 k 4 JJ
calores 0, 1, 2 e 3, obtemos, dessas duas últimas
Atribuindo a k os valores
expressões, o conjunto-solução da equação proposta:

8=
v2 72. 72
V2 72.
+ —t -*7- + Vt
72 72,h —
72 - —
7ã.i; 2;„ J
2i; -2; -2i
2 2 2 2 2 2 2 2 í

5.14) A fórmula de Baskara para determinação das raizes da equsçao do segundo


grau ax' + bx + c = 0, a * 0, no caso em que os coeficientes a, b e c são
números complexos, pode ser escrita:

-b + fq
*1.2 =
2a

onde o símbolo rq representa as raizes quadradas do discriminante


A = b2 - 4ac.

Utilizando esta fórmula, resolva a equação x5 -(1 + i)x— (3 + 2r) = 0,


4

Solução
1
Tendo a = 1, b = - (1 + í) e c = —(3 4-2i), calculamos o discrinninante:
4

ó = b2-4ac =[-(1 + i)]2-4 >dx ~(3 + 2i)

ô = 3^4i

84
Vamos, agora, calcular rq, isto é, as raizes quadradas de A = 3 + 4i.
É claro que, para isso, poderiamos utilizar a expressão (5.4) para n = 2:

â 2krt 2kn:
<0 = rQ = cos
2 T- -t- i sen
rl
onde p e R são, respectiva mente, o modulo e o argumento de ,x = 3 + 4i,
No entanto, como se trata do caso simples de determinação das raizes
quadradas, parece-nos conveniente utilizara definição de raiz: determinar
rfl = a + pi (a e 0 reais) tal que:

A
Temos, então:
(a + pi)2 = 3 + 4i
(a2-p2) + 2ctpi = 3 + 4i
donde
Íc£2-P2 = 3
|2«p = 4
Resolvendo este sistema (lembrando que cr e p são reais), obtemos:

(a =2 e (3=1) ou (a = -2 e P = -1)
Portanto, as raizes quadradas rq = a + £i são

■ 2 * t r„ = -2-i
Finalmente, com a fórmula fornecida, obtemos:

v ,_-b + rgi _ (1 + Í) + (2 + Í) _ 3
1 2 2 2
*2
-_ (i + |)-r(-2--Í) 2
2 2 2
Assim, o conjunto-soluçáo da equação dada é:

_ Í3 .11
S =<- + r, - - >
12 2]
Exercícios Propostos

5.15) Determine:
a) as raizes quadradas de 4 + 41^3
b) as raizes cúbicas de - 27i
c) as raízes quartas de - 16

5.16) Resolva, em C, as equações:


a) x3 + 8i = 0
b) xe-1 = 0

85
5.17) Resolva, em c, as equações:
a) x6 - 19x3 - 216 = 0
9
b) x3 - 2ix - — - 31 = 0 {Utilize a fórmula fornecida no exercício 5 14.)

5.18) Uma das raízes cúbicas de um número complexo z é 8. Determine as outras


raizes cúbicas de z.

5.19) O número io=3.cos— + issn — é uma das raizes quartas de um número


^33
complexo z. Determine as demais raizes quartas de z.

5n
2.2D) Uma das raízes sextas de z tem modulo 2 e argumento —.Represente
6
graficamente as seis raizes sextas de z.

Exercícios Suplementares

1-1). Determine a soma dos reais x e y para os quais x 2 + y2+5t-13 + xyí

I2) Sendo z = (tg a + i)(cotg a + i), determine Re(z) e lm(z).

1.3) Determine o número complexo z tal que z2 = iz.

1.4) Sejam a.b.c e d reais não nulos.


Mostre que a equação x3 + (a + bi)x + c + di - 0 não admite um número real
e um imaginário puro simultaneamente como raízes.
1.5) Prove, utilizando o Princípio da Indução Matemática, que:

P = i, tfn e N

I.6) Sendo z e w números complexos tais que z,2 co2 = 6 e z + w=1-i, deter-
mine z-ui

IV) Os números complexos distintos b + ai e a + bí, com a xb # 0, tém por


afixos os pontos A e B, respectivamente. Determine o complexo cujo afixo é
a extremidade do vetor que se obtém fazendo o vetor AB girar de 27Q15 em
torno de A, no sentido trigonométrico positivo.

I 8) Represente graficamente os números complexos z tal que

1 < | z| í 2
yí arg(z) <
4 4

86
1.9) Sendo cü=1-2í, represente graficamente os números complexos i tais que
□ produto m.xz seja

a) real
b) rea! positivo
c) imaginário puro
d) imaginário puro de coeficiente negativo

1.10) a) Determine o complexo z tal que Í2 + 2z+3 + 3i = 0.


b) Determine o módulo e o argumento de z.
c) Determine a potência do expoente 16 desse complexo.
d) Represente graficamente as raízes oitavas do complexo 17 + z - i.

87
PARTE II

Capítulo 6 - O conceito de polinômio.


Igualdade
Capítulo 7 - Operações com os polinõmios.
Grau
Capitulo 8 - A divisão de polinõmios
Capitulo 9 — A divisão de polinõmios em que o
divisor é de grau 1
Capítulo 10 — Outros temas importantes
f
Capítulo
O conceito de polinômio
Igualdade

6.1 - O Conceito
Um polinômio na variável ou indeterminada x é uma expressão da forma:

p(x) = a0 + a^ + a2x2 +... >- anxn

onde os coeficientes a0, a^, a2x anxn são números complexos; às parcelas
aQ, a^. a2x2, anxn, chamamos termos do polinômio p(x); e a0 é o seu termo
independente.
Exemplos

1°)Seja P(x) = 4 + x/2x + ix2; temos aQ = 4, a1 = -Jz e a2 = L

2o) No polinômio f(x) = 3x + 5x3, temos a0 = 0, a1 = 3, a2 = 0 e a3 = 5.

6.2 - Valor Numérico de um Polinômio


Sejam o polinômio:
p(x) = a0 + a^ + a2x2+.,. + anxn
e o número complexo a.
Chama-se valor numérico do polinômio p(x) em a ao número p(ot) que se obtém
substituindo-se. em p(x), x por a, e efetuando-se as operações indicadas, isto é:

p(cc) = a0 + a^ + a2a2 + ... + anan

Em particular, se para o número complexo a tem-se p(ct) = 0, diz-se que a ê uma


raiz ou um zero do polinômio p(x).
Exemplos

Seja g(x) = 2 - 3x + x2; temos:

g(1) = 2 - 3X1) + d)2= 0


g(0) = 2 - 3><0) + (O)2 = 2
g(-1) = 2 - 3><-1) + (-1)2 = 6

91
Observe que o número 1 é uma raiz de g(x), pois g(1) - 0.
6.3 - Definição - Polinômio Nulo
Denominamos nulo (ou identicamente nulo) um polinômio p(x) que assume valor
nulo para todo ce, nt e C. e indicamos:

p(x)= D ou p(x) = Q
Podemos, então, escrever:

I p(x) = 0 <=j p(a) - 0, para todo a, cí e C

6,4 - Quando um Polinomio é Nulo - Teorema

O polinômio p(x) = a0 4- + a2x2 + ... + anxrt é nulo se e


somente se todos os seus coeficientes forem iguais a zero:

p(x) = 0 <=> (a0 « = a2 = ... = an= 0)

Demonstração
V parte

Hipótese: a0 - a, - az =... - an - 0
Tese: p(x) = 0
=or hipótese, temos:
p(x) = 0 + Ox + Ox2 +, ,. + 0xrt
2, para todo n, o. e C;
p(ct) = 0+ Oa + Oa2 + + 0o°= 0
o que demonstra a 1a parte.
2a parte
Hipótese: p(x) = 0
Tese. aQ - aí = a? = a,, = 0
Se p(x) é nulo o seu valor numérico é zero em todo a, a e C.
Sejam, então, «o, op. a2, an, n + 1 números complexos, distintos dois a dois,
temos
p(a0)=a0 + a1a0 + a2ot^+.„+anaS = 0
p(«i) = aD -i-ala1 + a2o'.2 + ,,. + ana^ = 0
p(a2) “ a0 + a$a2 4-a2úÍ2 +... + anaS " 0

p(afl) = a0 + + a2a2 +... + ana" = 0

92
As equações anteriores constituem um sistema linear homogêneo, formado por
n + 1 equações e n + 1 incógnitas: a0, alR a2 an. A matriz incompleta do
sistema é:
‘1
aü cíq... a3
1 a2 ...a"
A= 1 «2 at..a;

1 «í-aí
Note que A é uma matriz de Vandermonde e det A * 0, pois os elementos de base
da matriz são dois a dois distintos (Veja volume 4 desta coleção, p.115). Daí
podemos concluir que a única solução do sistema ê a trivial:
a □ - al - a2 - - an = 0

o que demonstra a 2a parte (Veja volume 4 desta coleção, p.157)

6.5 - Definição - Polinomios Iguais


Os polinomios p(x) e q(x) dizem-se iguats (ou idênticos) quando assumem o
mesmo valor numérico para qualquer ot, a e C. Indica-se:

p(x) = q(x) ou p(x)sq(x)

Podemos, então, escrever:

p(x) = q(x) «=> p(a) = q(a), para todo a, ceeC

6.6 - Quando Polinomios São iguais —Teorema

Os polinomios:
p(x) = a0 + a1x + a2x3 +.. +aflxri

q(x) = b0 H-b^ + bjX2 +-... + bnxn


são iguais se e somente se os seus coeficientes forem
ordenadamente iguais".
p(x) = q(x) c=> (aD = b03l = bi, a 2 an =b<i)

Demonstração

1 * pa rte
Hipótese. a0 = b0. a-, = bi, a2 = b2, . an _ bn
Tese. p(x) = q(x)
Para todo a, ct c C. temos:

p(a) = a0 + a1ct + a2az +...-anan = bQ + b^a + b^cr2 + ,.. + br,an = q(a)

93
2* parte
Hipótese: p(x) = q(x)
Tese: — bo, ai — bi. aj = b2,a^ = b^
Se p(x) = q(x), por hipótese, a definição 6.5 permite-nos escrever para todo ot, ae
C:
p(ct) = q(a)
Então.
aa + a^cn- a2a2 + ,.. + anan - b0 -i-b^ + bja2 + + b^a0
e dai:
(a0-bo) + (a1-b1)a + (a2-b2)a2 + ... + (3n -bja" =0. Va
A igualdade acima, válida para todo a, garante-nos escrever que o poli-
ncmio:
h{x) - (a0-b0) + (a1-b1)x+(a2-b2}x2+... + (an-bn)xíl
é nulo. (Veja definição 6.3)
Então pelo teorema 6.4, temos:
—bo = O, a1 — bf — 0, a2-b2=0, 3n-br —0
E dai a tesa:
a0=b ai “ ^1, a2 ” b2, .... an = bn
Exercícios Resolvidas

6.1) Seja o polinõmío f(x) = 1+x + x3; determine f(-2), f(0)t f[f(3)J, í(x - 1) e
f(3x)
Solução
f(^2) = 1 + (-2) + (-2)3 = ^9
f(0) = 1 + 0 + 03 =1
f(1) = 1 + 1 + 1a = 3
f(3) = 1 + 3 + 33 =31, e dai, f [f(3)| = f(31) = 1 + 31 + 313 - 29 023

f(x-1) = 1 + (X-1) + (x-1)3 = -1 + 4x-3x3 + X3


f(3x) = 1 + (3x) + (3x)3 =1 + 3x + 27x3

6.2) Consideremos o polinõmío P(x) = aD + a-,x + a^x2 + ... + a^x" O que repre­
sentam P(1) e P(0) em relação aos coeficientes?
Solução
Temos:
n
P(1) = a0 + a1>íl + a2'Zl2 + ... + aflx1n = aC]+a1 + a2+.+an =z> i-0
Então, P(1) é a soma dos coeficientes de P(x).
P(0) = aB + a, >0 + +... + an >0n
Então, P(0)é □ termo independente de P(x).
94
6.3) Dado o polinõmio na variável x: P(x) = pq + qx + px2 + x3, determine p e q
para que se tenha P( 1) =: 2>P(—1) “ 12.
Solução

P(1) -12 = pq + q><T) + q><1)= + (1)3 - 12 = pq + q + p + 1 = 12 (I)


2 >P(-1) -12 = P(-1) = 6 = pq +■ q(-1) + p(-1)2 + (-1)3 = 6 = pq-q + p-1 = 6 (II)

Subtraindo membro a membro, (I) - (II), obtemos:


2q + 2 = 6
Logo: q = 2
Substituindo em (II): 2p-2 + p-1=6, obtemos p = 3.

3
6 4) C polinõmio na indeterminada x, f(x) = — (2c — 3) + (b + 2)x + (a — 1)x , é
identicamente nulo. Determine a, b e c.

Solução
Cs coeficientes de f(x) devem ser nulos, então:
- (2c-3) = 0
b-r2 = 0
a-1 = 0
3
Dai, temos a = 1, b = - 2 e c= —
2
Observe que o termo em x2 foi omitido, pois o seu coeficiente é tgua! a zero.

6 5) Sejam os polinomios:
A(x) = 3-bx + 2x2+(a-1)x3
B(x) = 3 + cx -s-bx2

Determine a, b e c para que se tenha A(x) = B(x).

Solução
Os coeficientes de (A)x e de B(x) devem ser dois a dois iguais:
-b = c
2 =c
0 = a-1
Então, temos a = 1, b = 2 e c = - 2.

95
Exercícios Propostos

6.6) Dado o polinõmio f(x) = 1d-8x+6x2-4x3 + X5, calcule f{2).

6 7) Dado o polinõmio f(x) = -1+ 5ix-4x2-31x3 + x'1, calcule f(1 + 2i).


2
6 8) Verifique se o número 2 é uma raiz do polinõmio: p(x) =-8 +4x - 2x +
+ 7x3 -5x4 + xs

6.9) O polinõmio abaixo, na variável x. é identicamente nulo Determine a, h e c


f(x) = (a + 2b c - 3) + (2a - b + 3c + 4)x2 + (3a - 2b + 3c + 1)x3

6.10) Determine ri e p, sabendo-se que a + px + x2 = p-([3 + 1)x + x2

|100^
6 11) Qual a soma dos coeficientes do polinõmio p(x) - (-1 + x)

6 12) Seja o polinõmio f(x) = b + ax + x3, Determine a e b, sabendo que 1 e


são raizes f(x)

b x2 + a ij , onde a e b são números reais e a


6.13) Seja o polinõmio f(x) = — 0.
—x
2 3

Se Xi e x3 sâo números reais distintos, tais que f(x1) + f(x2) =

, . -b
demonstre que x1 + x2 =—
a

96
Capitulo
Operações com os
Polinõmios Grau

7.1 - Adição de Polinõmios

Definição
Sejam os polinõmios na variável x:

p(x) = a0 + a^ + ajx2 + ... + anxn


q(x) = b0 + biX + b2x2 + ... + bnxn
Denomina-se soma de p(x) e q(x) ao polinômio:

p(x) + q(x) = (a0+b0) + (a1 + b1)x + (a2+b2)x2 + ... + (an + bn)xn|


Exemplo
Sejam os polinõmios:

p(x) = 2 + x-x2
q(x) = 3 + x2 + 2x3
Temos:

p(x) = 2+x-x2 + 0x2


q(x) = 3 + Ox + x2 + 2x2
Então:

p(x)+q(x) = (2+ 3) + (1 +0)x +(~1 + 1)x2 + (0 + 2)x3


p(x) + q(x) = 5 + x + 2x3

Propriedades da adição de polinõmios


No conjunto dos polinõmios com uma indeterminada x, de coeficientes complexos,
valem as propriedades;

97
1a) A adição de polinônnios é comutativa.

Para os polinòmíos p(x) e q(x) temos:

p(x) + q(X) = q(x) + p(X)

Demonstração

Sejam os polinõmios:

p(x) = a0 + apr + azx* +... + ap,*"

q(x) = b0 + tqx + b2x2 +... + bnx"

n n
Observe que p(x) + q(x) = (a,+bj)xx' e q(x)+ p(x) = £(b, + a,)x'.
i=0 1=0

Então.
n n
p(x) + q(x) = L(ai + b^x' = S(bi + ai)!í' -Q(x) + p(x)
iO i=0
2a) A adição de polinòmíos ê associativa.

Para os polinòmíos p(xj, q(x) e h(x) temos:

[p(x) + q(x)+h(x) = p (x) + [q(x) + h(x)J


Demonstração
n n
Sejam os polinòmíos p(x) = ^a.x’, q(x) = ^bjX' e h(x) = £C[x'.
i-0 i=0 i=0

(p(x) + q(x)]-rh(xj - Y(a(+bi]x'+ ^2c,x“ = ^[(a. + bil + c.) x* -


Í=D i-0 l=0
= Dai + <bi + Ci)] x' = ^aix' + ^(b( + Gi) X1 =

í=0 i-0 1=0


= P(x)+[q(x) + h(xJ]
31} Existe o elemento neutro.

Dado □ polinõmio p(x), existe um polinõmio e(x) tal que:

p(x) + e(x) = p(x)


n n
De fato, sejam p(x) = e e(x) = e/1; então, da igualdade p(x) + e(x) = p(x)
i=0 i=0
obtemos, para todo i, 0 í i £ n:
a, + »i = a,
isto é. e, = 0 para i = 0. 1, 2 n.
Concluirmos que o elemento neutro na adição de poünómios é o polinõmio nulo.

98
4”) Existe o polinòmio oposto

Dado o polinòmio p(x), existe um polinòmio p(xj tal que:

|p(x) + p(x) = e(x)|


De fato, sejam X^oa'x e p(x) = yn oaix'; então, da igualdade p(x) + p(x) = e(x)
obtemos para todo i, 0 < i £ n:
Sj + a, = 0
isto é, a, = -a, para r = 0, 1,2,..,, n.

Então, dado o polinòmio

p{x) =an + a1x + a2x2 + ,.. + anx,n

o polinòmio oposto de p(x] é:

p(x) 5= (-aD) + (-a1)x 4-(-a2)x2 + ... + (-an)x"

O polinòmio oposto de p(x) indica-se com:

-P(x)

Note-se que - (- p£x)) = p(x).


Exemplo

Seja p(x) = 3 + 4x2 -3x3; então, -p(x) =-3 - 4x2 + 3x\

Definição
Sejam os polinõmios p(x) e q(x). A diferença p(x) — q(x) define-se por:

p(x)-q(x) = p(x)4-(-q(x))

Observe que sep(x) = a0 + a1x + a2x2 + ... + anxrteq(x)-b(j s-b^ + bjX2 + ... + brxn.
então;
p(x)-q(x) = (a0 - b0)+ ía.~ bjx t[a2 -b2)x2 +■-<■ +(an -bn)xn

7.2 - Multiplicação de Polinõmios


Definição
Nos cursos elementares de Álgebra, o processo para se multiplicar dois polinõmios
é constituído essencialmente por duas etapas.
Inicialmente, todos os pares de termos, um em cada polinòmio, são multiplicados,
utilizando-se a regra:
(ax^)>(bxn) = (ab)xmrn

99
Em seguida, os coeficientes de iguais potências de x são somados:
axp 4-bxp =(a + b)xp
Por exemplo, para multiplicarmos os polinõmios:
p(x) = 2 + x-2x1 + x 3

e, q(x) = 3’2x4-xa
usamos o processo familiar:
2 + x-2x2 4-X3
3-2 + x2
3>p(X) 6 + 3x-6xs + 3xl
(-2x)>p(x) -4X-2X* +4xJ -2x*
(x?)>p(x) ______ 2xi + Xi-2x* + Xi>
6-x-6xz + 8x3 -4x' + x*

ou usamos o seguinte processo:


(2 + x-2x2 + x3)><3-2x + x2) =

= 2 ><3 - 2x + xs) + x(3 - 2x 4- x3)-2xJ ><3 -2x4-x3)4-xa(3-2x + xJ) =


= 6 - 4x + 2x! + 3x- 2X1 + xJ - 6x’ + 4x’ -2x' + 3xa - 2x" 4- xs =
= 6-x-6x2 + Sxa-4x*4-5xi
Não é conveniente definirmos a multiplicação de polinamio usando a des­
crição das processos acima. Haremos uma definição em termos gerais, mas, nas
prática, utilizaremos um dos métodos anteriores.

Sejam os políncmios:
p(x) = a0 4-a1x + a2x2'i.-. + a^x™
e
q(x) = bD 4-b^ 4-b2x2 + .<.4-brtxn
na variável x com coeficientes em C.

O produto de p(x) e q(x) ê o polinòmio:


p(x)>q(x) = £aoba) + [a^ + a^Jx
4- (a gb2 4" adb, 4- a2 bg Jx +...

H^bn)xm+n

O coeficiente x1 no produto p(x)>q(x) é o número:

a^ + a^ + „ 4-ai_1b1 + ajb0 = Xa)bi_í


>0

quando a, = 0, se j > m. e bk = 0. se k > n.

100
Propriedades da multiplicação de polinõmios

No conjunto dos polinõmios com uma indeterminada x, de coeficientes complexos,


valem as propriedades

1a) A multiplicação de polinõmios é comutativa.


Para os polinõmios p(x] e q(x) temos:
p(X)>q(x) - q(x) xp(x)
Demonstração
Sejam os polinõmios:

p(X) = aD + a,X -r a2x2 + .. + a


q(x) = b0 + b-jX + b2x2
+ -- + twm
O coeficiente de x no produto p(x)x?(x) é:

a^. + a^ + ,,. + al_1b1 + 3,00

E o coeficiente de x1 no produto q(x) >p(x) é:

b oa. + biai-i+ ■■■ + bi. ai+biag

Mas. em C, a adição e a multiplicação sao comutativas: dar podemos concluir que


os coeficientes calculados são iguais para todo t. Então, p(x)>q(x) = q(x) >p(x).

2a) A multiplicação dos polinõmios é associativa.


Para os polinõmios p(x), q(x] e h(x) temos:

[p(x} xq(x)] >h(x) = p(x) X)

Demonstração
Veja o exercício 7.25.

3a] Existe o elemento neutro


Dado o polinõmio p(x), existe um polinomíú e(x) tal que:

pÇx) xe(x) = p(x)

101
Tomemos e(x) = en + e,x + e2x2 + ... + enxn, no qual aD = 1 e e, = D para
i 2 1. Verifica-se imediatamente que-

p(X)xe(x) = p(X)

para todo polinómio p(x)

4a) Distributividade

Para os polinômios p(x). q(X) e h(x) temos:

^p(x) + q(x)] >h(X) = p(X)>h(X) + q(X)>ti(X)

Demonstração

Veja o exercício 7.26.

(,3 - Grau de um Polinómio

Seja p(x) = a0 + a,x + a3x2 +...-i-anxn um polinómio com coeficiente em C.


Suponha que p(x) não é um polinómio nulo.
C grau de p(x} é o maior inteiro não negativo n, tal que an * 0.
O coeficiente a^ é chamado coeficiente dominante de p(x).

Exemplos

O grau do polinómio 3 + Ox é zero.


O grau do polinómio 2+ (—3x) é 1.
O grau do polinómio 3 + 2x - 4xJ é 3,
Os polinômios de grau zero são polinômios constantes, não nulos; os polinômios
de grau 1 são os polinômios da forma a + bx, com b # 0: os polinomioE de grau 2
são os polinômios da forma a + bx + cx2, com etc. Não se define grau para
o polinómio nulo
É comum indicarmos o grau de um polinómio não nulo p(x) por:

gr[p(x)] ou ôp(x)

Por exemplo:

gr [3 + 2x-4xa +0x3 + 0x<] = 3

gr [x’°]=10

gr - =ü

102
Se o polinõmio p(x) = ao + a,x + a2x? + ... + a^x" e se gr[p(x)] = n. então ê óbvio
escrever:

p(x) = a^x" 4-a

onde an # 0.

Por outro lado, se p(x) = a^x" + a^x +.., + a,X *a0, com an? 0. então gr(p(x)] =
n.

T eorema

Sejam p{x) e q(x) polinõmios na variável x, não nulos, com coeficientes em C.


então:
a) gr[p(x) :<q(x)l = gr[p(x)J + gr[q(x)J
b) se p(x)+q(x) A 0. então gr [p(x) + q(x)]smax. {gr [p(xj]; gr [q(x)]}
c) se gr [p{x)] * gr [q{x)], então gr [p(x) + q(x)] = mãx. {gr [p(x)j; gr (q(x)]}

Demonstração

Sejam os polinõmios:

p{x) = arixn + an_1xn-1 + ... + a0


q(x) = bmxm + bm_ixm bg

onde an * 0 e * 0, isto è:
gr (p(x)J = n e gr[q(x)] = m
Assim.
pfx)xq(x) = (a„bm )xrt+m + (an^m + i)x +... + a0b0

Em C, se an * 0e bn* 0, então art >bm * 0; dai;

gr [p(x)>q(x)l = n + m - gr [p(xj] + [q(x]j

Para provarmos b e c, suponhamos inicialmente que n > m; podemos escrever:


q(x) = 0xn + Ox"-1 +... + bmxm +bm_, xm-1 + ... + ba
Daí:
p(x) + q(x) = anxn + a
a n1 x""1+ + (a, + bw]xm +

+(am-i + bm_1)x' (a0 + M

Então; gr [p(x) + q(x)J - n = mãx. (gr (p(x)]; gr [q(x)]J

De forma análoga, se n < m, concluímos que:

gr (p(x) + q(x)] = m = máx. {gr[p(x)]b gr[q(x)]}

103
Observe que fica provado c.
Para completarmos a demonstração de b, elevemos examinar o caso n = m; assim:
p(x)+ q(x) = (an + bjxn + (an_i + bn_1)x°"1 +... + (aD +b0)

Se p(x) + q(x) * 0, então ak + 0, para algum k, 0 > k > n.

Portanto, fica claro que:

gr [p(x) + q(x)] < n = mãx {gr(p(x)J; gr [q(x)]}

Exemplos

Sejam os polinòmios:

p(x) = 2x3 + x+ 2
q(x) = x2 +2x+ 2

fl[p(x) = 3, í)[q(x)] = 2;í[p(x) + p(x)] = 3 = máx.{a[p(X)]; rl[q(x)]]]

r)|p(x)>q(x)] = 3 + 2 = 5

Exercicios ResoIvidos

7-D Determine os números reais A, S é C para que se tenha:


2x2 -x + 1 = A(x-1)2 + B(x-1) + C.

Solução

Temos, sucessivamente.
2x2 - x +1 = A(x-1)(x -1) + B(x -1) + C

2x2 — x + 1 = A(xz -2x + 1) + B(x-1) + C

2 x2 - x +1 = Ax2 - 2Ax + A + Bx-B + C

2xz - x+1 = Ax2+(B-2A)x +A-B + C

Os coeficientes de polinômios iguais (ou idênticos) são ordenadamente


iguais:
A -2
R-2A = -1
A-B+C= 1

Resolvendo o sistema acima, obtemos: A = 2, 0 = 3 e C 2.

7-2) No conjunto de todos os polinômios de coeficientes reais, resolva o pro­


blema. "determinar a e b para que 0 polinômio f(x) = x* 6x3 + ax2 + bx + 1
seja um quadrado perfeito”.

104
Solução
Se f(x) é quadrado perfeito, existe um polinòmio g(x) tal que:

f(x)-ígCx)]3

Observe que se f(x) tem grau 4, então g(x) é um polinòmio de grau 2. Isto é,
g(x) è da forma:
J.
g(x) = px^ + qx + r
Então:

x4 -5x3 + ax2 +bx + l s (px2 + qx+r)2


x4 -6x3 + ax2 + bx +1 = p2x4 + 2pqx3 + (q 2 2pr)x2 + 2qrx tr 2

Dai:

P2=1
2pq = -6
a = q3 H-2pr
b = 2qr
r2 = 1

Resolvendo o sistema acima encotramos: a = 11, b = -6 ou a = 7, b = 6.

7.3) Determine um polinòmio f(x), do segundo grau, para o qual tem-se:

íf(x)-f(x-1)s x
V(0)=0
Em seguida, deduza que a soma dos n primeiros inteiros positivos é igual a
n(n+1)
2
Solução
O polinòmio f(x), do 2° grau, é da forma
f(x)~ax3 -r bx + c, a * 0
De f(0) = 0, obtemos c - 0; então, temos, sucessivamente:
f(x)-f(x-1) < X

(ax2 + bx) - [a( x -1)2 + b(x -1}] - X


f(x—1] f(x], * è
aubslluidc por x-1

105
2a = 1
Dai:
b-a = 0
1
e, portanto: a = b = —

1 ? 1
O polinòmio f(x) está determinado: f(x) = - x + — X.
2
O polinòmia f(x) - f(x - 1) s x, substituindo x por 1, 2 ,3, .... n, obtemos.
sucessivamente
f(1}-f(0) = 1
f(2)-f(1) - 2
f(3)-f(2) = 3

f(n)~f(n~1) = n
E, somando-se membro a membro as igualdades acima:
f(n) - f(0] = 1+ 2 + 3 + . . + n
1 ■? , 1 n _ n(n +1)
Sendo f(n) = — n e f{0) = 0, temos:
" 2 2
n(n + 1)
1 + 2 + 3 +„, + n =
2
7.4] No conjunto dos poiinõmios de coeficientes reais, prove que o polinòmio do
segundo, grau f(x) = ax2 + bx + c, ê um quadrado perfeito se e somente se
a > 0 e A = b2 - 4ac ~ 0.

Solução
1a parte
Hipótese' a > 0 e A = O.
Tese. f(x) é um quadrada perfeito.
Para o polinòmio f(x), temos, sucessivamente:

b c
f(x) = a x2 + —x + —
a a
te b2 _ b2 c
f(x) = a x2 + — x
a 4a2 4a2 a

b b2 b2 -4ac
f(x)-a x + -x +
a 4a2 4a2

x +
2a J

f(X) = a
+ -f 2a J
A
4az
(Veja volume 1 desta coleção p. 113)

106
Sendo a > 0, existe vã em i, e como A = fl, podemos escrever:

2
f{x) = (7ã)3/x + ^-l
l, 2a J
e dai:

b
f{x)-[v^>| x + —
2a

o que demonstra que f(x) ê um quadrado perfeito


2* parte
Hipótese: f(x) ê um quadrado perfeito
Tese.' a > 0 e A - 0,
Se f(x) é quadrado perfeito, existe um polinómio do primeiro grau g(x) tal
que:
f(x)=[g(x)]2
E, sendo g(x) da forma px + q. p * 0. temos:

ax! -i-bx + c = (px + q)2


ax! + bx + c a p2x2 + 2pqx + q?

e daí:

a -p2
b = 2pq
c = q2
Da igualdade a = p2, p * 0. concluímos que a > 0.
A = b7 -4ac = (2pq)7 -4>p2>q2 = 4p2q2 - 4p2q? =■ 0
75) Determine o grau do polinómio na variável x;
p(x) = (a3 - 3a + 2)x3 + (a - 1 )x + a - 2

Solução

Se a2-3a + 2#0, isto é. a*1 e a * 2 : d[p(x)] = 3.


Se a2-3a + 2 = 0, isto é, a=1 ou a = 2, temos:
Para a = 1; p(x) = -3 e r)[p(x)J - 0
Para a = 2: p(x)=x e 3(0)] = 1

7.6) Definimos uma fraçao racional na variável x como o quaciente formal

p(x) aQ + + a2x2 +... + aRxn


p(x) bfj + b^ + b^x2+... + bmx|T’

107
onde a,sC, bte t? e q(x) nao é polinômio nulo.

Para as frações racionais ——- e , definimos:


q(*) â(x)
^77 = ^7 » p(x)>g(x)»f(x)>q(x)
= —— P(x)>g(x) = f(x)>q(x)
q(X) g(x)
p(x) + 2(x) p(x)>g(x) + Í(x)>q(x)
q(x) g(x) q(x)>g(x)
P(x) ?/(x) = p(x)xf(x)
q(x) g(x) q(x)>g(x)

Problema' demonstre que a fração racional, não nula:

ax 2 + bx + c
ax2 + b'x3 + c‘

b
é independente de x se e somente — - = — (supondo a' * 0, b‘ * 0,
a' b' c'
c * 0).

Solução

V parte

ax2 4-bx + c
Hjpófese: a fração é independente de x, isto é, a fraçao ê
ax2 4-b'x + c'
igual (idèntica)a um polinômio constante de grau zero:

ax2 +bx + x
—7^.----------- —-k
a x4 + b'x + c‘

t a b c
Tese: — = —
a' b' c’

Da hipótese temos:

ax2 +bx + c - ka x2 4-kb x + kc1


ê daí:
a = ka'
b = kb‘
c = kc'
Então;
a b c
ã7 b‘ c’

108
2a parte
3
Hipótese: — ■ £ =k
a1 b* c‘

a? + bx + c
Tese/ -k
a-x2 + b'x + c1

Da hipótese temos
a = ka1
b = kb‘
c - kc‘
e dai:
axg +bx + c ka1 xz + kb' x + kc k(a1 x3 + b'x + c‘)
a'xz+b' a" x2 -r-b" x + c‘ a' X3 -r b ' X -r c'

7.7) Determine A e B para que se tenha:


3x-1 A B
== ------- +
xz-5x + 6 x-2 X-3
Solução
Temos, sucessivamente:
3x-1 A(x-3) + B(x - 2)
x2 - 5x + 6 (x-2)(x-3)
3X-1 (A + B)x- 3A -2B
x2-5x +6 X2 -Sx + 6

3x-1 = (A + B)x-3A-2B

E, então:
fA + B= 3
í-3A-28 = -1

Obtemos: A = — 5 e B = 8.

Exercidos propostos

7.3) Dados os púlrnõmios:


A(x) = x
3
B(x) = x + x
C(x) = x + x3 + xs
~ 3J + 2x
P(x) = 3xs -6x
determine os números a, b e a para que se tenha
p(x) = a>A(x) + b>B(x) + c>C(x)

109
7.9) Efetue as multiplicações:
a) (2x4 - x3 + x2 + x 4-1)(x2 -3x +1) b) (x3 -b x2 - x-1)(x£-2x-l)

7.10) Um polinòmio P(x) é tal que


P(x) + x >P(2 - x) = x2 + 3
a) Determine P(0), P(1) e P(2)
b) Verifique que o grau de P(x) ê 1

7.11) O grau dos polinòmios f(x), g(x) e h(x) é 3, O grau do polinòmio, nao nulo,
f(x)4g(x) + h(x)] ê n Quais são os possíveis valores de n?

3
7,12} Determine b c e d para que se tenha: (x + c} + b (x + d) = x + 6X2 +
+ 15x + 14.

7.13) Seja n polinòmio na indeterminada x:


f(x) - (x-a)2(b-c) + (x-b)2(c-a) + (x-c)2(a-b) + (b-c)(c-aMa-b)
Verifique que f(x) s 0.

7.14) No conjunto de todos os polinòmios de coeficientes reais^ resolva o pro­


blema: "determinar a e b a para que o polinòmio f(x) = x + 4x + ax + b
seja um cubo perfeito".

7.15) No conjunto de todos os polinòmios de coeficientes reais, resolva os


problemas:
ITdar a condição para que o polinòmio (a + bx)2 + (a‘ b'x)2, onde
aa bbV 0. seja um quadrado perfeito "',
2°) "mostrar que se (a + bx)2 + (a1 + bx)3 e (a + cx}2 + (a‘ + c x)2 sác
quadrados perfeitos, então, (t + cx)2 + (b’ + c'x)2 também o êH.

, n*2
7.16) Qual e a progressão aritmética na qual a soma dos n primeiros termos ê
2
para lodo n?

7,17) Determine um polinòmio de grau 3, para o qual se tem:


g(x)-g(x-1) = X2 e g(0) = 0
Em seguida, deduza que a soma dos quadrados dos n primeiras inteiras
v i n(n + 1)(2n + 1)
positivos e igual a —------ .

110
7.19) Determine um polinõmio f(x), de grau 3, taf que.

P(x) s P(x-1) +x(X’1) e P(0) = 0

n(n? -1)
Em seguida, deduza a igualdade 1x2 + 2x3 3x4 + ... + (n - 1) *=
3

7.19) Mo conjunto de todos os polinòmios de coeficientes reais, determine todos


os polinòmios f(x), de grau 2, tal que:

f(x2) = k >í(x)>f(-x), ke R'

7.20) Determine p e q para que se tenha: x2z+px +q = (x - p)(x - q).

7.21) Determine A, B, C e D para que se tenha

x3 = A(x-1)[x-2)(x-3) + B{x-1)(x-2) + C(x-1) + D

7.22) Determine A e B para que se tenha:

7.23) Determine A e B para que se tenha:

1 A B
x(x +1) + {x + 1)(x + 2)
x(x + 1)(x + 2)

Em seguida, deduza a igualdade:

1 1 1 1 . 1_ i
+ 2 x3 x4 2|_1x2 n(n + 1)(n + 2)J
1x2x3 n(n + 1)(n + 2)

7.24) Mostre que se p(x) e q(x) nao sao nulos, então o produto p(x)>q(x) * 0
também nâa é nula, isto ê:

(p(x) * 0 e q(x) * 0) =r> p(x)>q(x) f 0

7.25) Demonstre que a multiplicação de polinòmios é associativa.

7.26) Demonstre que para os polinòmios p(x), q(x) e h(x) temos:

[p(x) + q(x)|>h(x) = p(x)>h(x) + q(x)xh(x)

7.27) Seja a igualdade entre polinòmios:

(1 + x + x2)r = a0 + a1x + a2x2+._. + a 2nx

Determine a soma aQ + a2 + a 4 +"'’ra2rT

111
f
Capítulo

A divisão de poiinômios

8.1 - Divisão Euclidiana

Teorema
Dados um polinômio qualquer A(x) e um polinômio B(x). não nulo, existe um e
somente um par de poiinômios, Q(x) e R(x), tal que

I) A(x)sB(xj>a;x) + R(x)
II) gr[R(x)] < gr[B(x)] ou R(x) = D

Efetuar a divisão de A(x) por B(x) é determinar o par de poiinômios Q(x) e R(x)
satisfazendo as condições acima; A(x) é o dividendo, B(x) é o divisor, Q(x) é o
quociente e R(x) é o resto na divisão.

Demonstração (opcional)
1* partetex/sténcía de uma solução
Inicialmente observe que se A(x) = 0 ou se gr[A(x)j < gr(B(x)] existe, visivelmente, a
solução Q(x) = 0 e R(x) = A(x), satisfazendo as condições I e II.
SuponhamoSí então, A£x) não nulo e gr[A(x)] > g[B(x)].
Sejam:

A(x) = aílxn + a, xn-1 +.,. + a1x + aD. an # 0


B(x) = bmxíf1 +b, xm-1 + ...-t-b1x + bü. bm * 0

onde n S m.
Construamos o polinômio:

R1(X)=A(X) - ±LXn-m (1)


hm

113
onde:

9[Rn(x)] gr[A(x)], pois hã □ cancelamento de pelo menos anxri.

Se srLR^xXI c gr[B(x)], a existência está estabelecida; o par Qi(x) e

R^x) respondem à questão.

Se grfR^x)] £ gr[B(x)], vamos escrever:

R^x) = cpxp + cp_1xp^1 + ... + c1x + xD onde cp A 0 e pím

Construamos:

R^xJ^R/x) - ^xP-n>S(x) (2)


bm

onde gr[R2(x)]<gr[R1(x)]r

Se gr|R2(x)j< gr[B(x)], a existência está estabelecida; □ par Qi(x) + Clií*). on^e

Q2(x)^-£-x p rn, e Rí(x} respondem à questão.

Se gr[R2(x)] > gr[B(x)], vamos escrever:

R2(x) = dqxq + dq_1x + ... + d1q + dD, onde dq * 0 e q > m


Construamos:

R3(x) = R2(x)----- 2-xq‘m>B(x) (3)

onde gr[R3(x)] < gr[R2(x)].

Se gr[R3(x)) < gr[B(x)], a existência está estabelecida; c par Qi(x) + QjM +


d
+ Q3(x), onde Q7(x) = — x’ e Ra(x) respondem à questão:
bn>

Se gr[R3(x)] > gr[B(x)] o processo continua:

R<(x) = R3 - Q4[x) xB(x) (4)

Rr:x).RM:xj-Qn.x)€(x': (n)
Os graus gr [Rj (x)] constituem uma sequência decrescente em 1:J;
grfR^x)] > gr[R2(x)] >. > gr[Rn(x)] > ...

114
Podemos concluir que existe um natural n tal que:
9ÍRn(x)]<gr[B(x)]

Então, somando membro a membro as igualdades de (1) a (n), obtemos:

Rn(x) = A(x)-B(x)[Q1(x)+Q2(x)+ + Qn(x)J


C par Q(x) = Q,(x) + Q?(x)+... + Qn(x) e R(x) = Rft{x) respondem ã ques-
tão proposta.
Observe que se pode ter, na aplicação do processo, algum Rj(x) = 0. Neste caso,
o par que responde à questão è Q(x) = Q1(x) + Q2(x) + .,+ Q,(x) e R(x) = R,(x).

2* parte: untddade
Demonstremos que o par Q{x) e R(x) obtido é único. Suponhamos que existe um
outro par Q-Jx) e R,(x) que responde ã questão.

Então-
jA(x} = B(x)>Q(x) + R(x) A(x) = B(x)X2'(x) + R'(x)
[gr[R(x)] < gr(B(x)] ou R(x} = □ gr[R'(x)| < gr[B(x)| ou R'(x) = 0

Tem-se:
B(x)>Q(x) + R(x)eB(x)jQ’(x) + R’(x)
e dai:
B(x)x(Q(x)-Q'(x)] = R’(x)-R(x) (I)
Se dois membros da igualdade acima não são nuios podemos escrever:
gr{B(x) x[Q(x) - Q (x)]} = gr[R (x) - R(x)]
Note que:
gr{B(x) MQ(x)- Q'(x)]} = gr[B(x)J gr[Q(x) Q'(x)] i gr[B{x)J (1)
gr[R'(x) -R(x)l í máx. {gr[R‘(x}]; gr[R(x)]} < gr[S(x)J (2)
As relações (1) e (2) são incompatíveis. Então, na igualdade (I), os dois membros
devem ser nulos, daí, como B(x) não ê nulo, obtemos R(x) = R'(x) e
Q(x) = Q’(x),
Está estabelecida a unicidade.

O ivisibilidade
Se na divisão de A(x) por B(x) obtêm-se R(x) = 0, di2-se que A(x) ê divisível por
B{x) ou que B(x) ê um divisor de A(x).

Se B(x) é um divisor de A(x), indica-se:

0OÕ~I A(x)

115
Na prática, □ quociente e o resto podem ser obtidos como descrevemos na
demonstração da existência do par, dispondo as operações de forma conveniente,
é o método da chave para se efetuar a divisão.

Exemplos
2
1o) Vamos efetuar ao divisão de A(x) = 3x5 + 4x2 + 1 por B(x) = X + 2X + 1
Adotamos a seguinte disposição prática:

A(x) 3x5 ^4x2 +1 | x2 + 2x + 3 — ~ EM


► -3x5- Bx^+Qx3 |3x3 —6x2 + 3x + 16 B(X)
-Qi >B(x)----
R,fx)----------- -6x4 - 9x3 + 4x2 +1 t
Gi(»l
t
OaÍKl
Q2ÍK1
t
QjM
t x
-Q?(x) >Btx) - + 12X3+18X2
Rztx) -------- —*-3x3 + 22x3+
-QjÍx) -»--3x2 - 6x<^x
W-------- —9x + 1
-Q,(x) >B(x) >-<16x2 - 32x -48
R<(x)R(x) - ——> -41 x - 47 / I

obtém-se "dividindo" obtêm-se ' dividindo'1


o termo de maior o termo de maior
grau de A(x) pelo grau de Ri(x) pelo
termo de maior grau termo de maior grau
|de B(x] de 8(x)

_______ r_____ u_
obtém-se "dividindo" obtém-se "dividindo
o termo de maior o termo de maior
grau de R;(x) pelo grau de Ri(x) pelo
termo de maior grau termo de maior grau
de B(x) de B(x)

Tem-se, então:

3x5 + 4x? + 1 s (x?+2x + 3) (3x3-6x2 + 3x + 6) -41X-47


A(x) B(x) “ Q(xj

O grau da resto, - 41x - 47. é menor que o grau de B(x),

116
2°) Vamos efetuar a divisão de A(x) = 2x4 + 5x2 - 10x + 1 por:
B(x) = 2x2 - 5x + 5.

A(x) 2x4 +5x2 -10x4-1 2xz -5x4-5 - B(x)


-2x4 4-5x3-5x2 2 5 25
X2 4-— X 4------ - Q(X)
2 4
5x3 -10x + 1

-5x3 + 25 x 2 25
:r2 - ------- x
2 2
25 2 45
-----x -------X 4- 1
2 2
-^ + -----125 125
x
2 4--- 4
R(x) -> ^y-x 121
4 4
Tem-se, então:
5 25 35 121
2x4 +5x2-10x + 1 = (2x2- lx2 + -x +— 4------X----------
k 2 4 4 4

35 121 .
Observe que o grau do resto, —x------- , e menor que o grau de B(x).
4 4
6 ,s 4 ,3 2
3o) Vamos efetuar a divisão A(x) = 6x - 12x + 5x + 14x - 20x + 13x - 3 por:
B(x) = 2x2-4x + 3.

6x5 - 12x5 4-5x4 4-14x3 - 20x2 4-13x - 3 2x2-4x + 3


-6x6 4-12x5-9x4 3x4 -2x’ + 3x-1
-4x3 4-14x3-20x24-13x-3
4x4 -8x3 +6x2
6x3-14x2 +13x-3
-6x3 +12x2 -9x
-2x2 + 4x-3
2x2-4x + 3
0

R(x) = 0
a divisão é exata:
A(x) é divisível por B(x)

117
Tem-se, então:

6x6-12x5 + 5x'’ +14x3 -20x2 + 13x-3 =


= (2x2-4x + 3)(3x4-2x2+3x-1) e R(x)-0.

8.2— Método de Descartes para a Divisão de Polinômios


Na divisão de A(x) por B(x), suponhamos que:
gr[A(x)] 2 gr[B(x)]
Sejam Q(x) e R(x), respectivamente. □ quociente e o resto procurados. Da
identidade:

A(x) = B(x)*Q(x) + R(x)

obtemos: gr[A(x)] = gr [S(x)>Q(x) + R(x)]

E, como gr[R(x)] < gr[B(x)) (ou R(x) = 0), podemos escrever:

gr[A(x)] = gr[B(x)xQ(x)]

ou ainda: gr[A(x)] = gr[B(x)]+gr[Q(x)j]

Para os polinômios Q(x) e R(x) procurados temos:

1) gr[a(x))=gr[A(x)]-gr[B(x)]
2) gr[R(x)] < gr[B(x)J (ou R(x) = 0)
O método de Descartes aplica-se da seguinte forma:
1o) determinamos o grau de Q(x) e o máximo grau que R(x) pode assumir, a partir
das relações 1 e 2, acima;
2o) escrevemos os polinômios Q(x) e R(x), onde os coeficientes são incógnitas a
serem determinadas:

3o) escrevemos a identidade A(x) = B(x)xQ(x) + R(x) e daí determinamos os


coeficientes incógnitos de Q(x) e de R(x).

Exemplos
1fl) Efetuemos a divisão de A(x) = õx3 - 6x + 4 por B(x) = x2 - 2x pelo método de
Descartes.

gr[Q(x)] = gr[A(x)J - gr[B(x)] =3-2=1

Portanto, Q(x) é da forma ax * b:

gr[R(x)J<cgr[B(x))=2

113
Então, o máximo valor para é gr{R(x)] ê 1 (ou R(x) s* 0) A forma de R(x} ê
cx + d.
Da identidade A(x) = B(x) O(x) + R(x), obtemos;

0x3 - 5x + 4 = (x2 - 2x)(ax + b) + {cx + d)


0x3 - 6x + 4 s ax3 + (-2a + b)x2 + (c - 2b)x + d

Os coeficientes são ordenadamente iguais.


a-S
-2a+b = 0
c - 2b = -6
d=a
Resolvendo o sistema, obtemos a = 8, b = 16, c = 26 e d = 4, assim:
Q(x) = 8x + 16 e R(x) = 26x + 4

2°) Vamos dividir o polinômio A(x) = 12xs - 8x4 - 2x3 +4x2 - 8x pelo polr-
nõmio 3x - 2x +1,

gr[Q( x)] = gr[ A(x» - g r [ B( X)] = 5-2 = 3

Portanto, Q(x) ê da forma ax3 + bx2 + cx + d :


gr[R(x))<gr[B(x)] = 2

Então, o máximo valor para gr[R(x)] ê 1 (ou R(x) = 0). A forma de R(x) é px + q.
Podemos escrever
12xs -0x4 -2x3 + 4x2 -8x = (3x2 - 2x + 1)(ax3 + bx2 + cx + d) + (px + q)
12x5 - Bx4 - 2x3 + 4x2 - 0x = 3axs + (3b - 2a)x4 + (a - 2b)x3 +
+(b -2c + 3d)x2 + (c-2d + p)x + d + q
Dai:
3a = 12
3b-2a = -0
a — 2b 4- 3c = —2
b-2c + 3d = 4
c-2d + p = -6
d+ q = 0

Resolvendo o sistema, obtemos a - 4, b = 0, c - — 2, d = Q, p = - 6 e q = 0;


assim:
Q(x) = 4x3 -2x
R(x) = -6x

119
Exercícios Resolvidos
8.1) Determine os números reais a e b para que o polinõmio A(x) ■ 2x.33 + ax +
+ bx - 3 seja divisível por x2 - 5x +1,

Solução
Efetuando a divisãc pelo método da chave:

2x3 + ax2 + bx - 3 x2 - 5x -t- J


-2x3 +10x2 - 2x 2x + (10 + a)

(10 + a)x2 4 (b-2)x - 3


-(10 + a)x2 +(50 + 5a)x - (1 + a)
(48 + 5a + b)x — (1 3 + a)

Queremos que a divisão seja exata: assim, o resto deve ser nulo, isto é:
Í48 + 5a + b = Q
^-(13 + a) = 0

Resolvendo o sistema, obtemos a = - 13 e b = 17,


Vamos resolver o problema utilizando o método de Descartes.
Se a divisão ê exata, temos R(x) «0, ec quociente Q(x) ê da forma mx + n,
pois gr[(x)] =3-2=1

Então:
2xa + ax2 + bx-3 = (x2 -5x+1)(rnx + n)
2x3 +ax2 + bx-3 - mx3 + (n-5m)x2 + (m-5n)x + n
Daí:
m=2
n - 5m = a
m - 5n - b
n = -3

Resolvendo o sistema, obtemos: a = — 1 3 e b = 17.


8.2) Numa divisão de polinõmios em que o dividendo é de grau p e o quociente de
grau q, qual é o grau máximo que o resto pode ter?
Solução
Na divisão de A(x) por B(x), sejam Q(x) e R(x) o quociente e o reslo.
respectivamente. Temos:
gr[A(x)] = gr[B(x)] + gr[Q(x)J
gr[R(x)]<gr[B(x)]

120
e, então:

p = gíB(x)] + q
isto é:
gr[B(x)] = p-q
Concluímos que:
gr[R(x)] <p-q

Então, o grau máximo que o resto pode ter ê p - q — 1.


8.3) Determine os números reais a, b, c e d, sabendo que o polmómio
p(x) = x'’-x3+ ax2+bx+c dividido por x2 + d dá resto rt(x) = x, e dividido

por x2-d dá resto r2(x) = -x.

Solução
Vamos efetuar a divisão de p(x) por x2 + d:

x'’ - x 3 + ax2 i-bx + c x2 + d________


-?... -dx2 x2 - x + (a - d)

-x3 + (a - d)x2 + bx + c
x3 +dx

(a - d)x2 + (b + d)x + c
-(a-d)x2 -d(a-d)
r,(x) - (b + d)x + (c + d2 - ad)

E sendo r^x) = x, obtemos:

íb + d = 1 (1)
|c + d2 - ad = 0 (2)
Analogamente, na divisão de p(x) por x2 - d, obtemos:

r2(x) - (b - d)x2 + (c + d2 + ad)

Ê, sendo r2(x) = -x, temos:

íb-d = -1 (3)
[c + d2+ad
+ ad = C (4)

O sistema formado pelas equações (1), (2). (3) e (4) nos dá a = 0, b = 0,


c = “ 1 e d = 1.

121
8.4) Verifique que se o polinõmio xm — an é divisível por xp — aip, então m ê
divisível por p, m > p e a * 0.

Solução

Vamos efetuar a divisão de xm -a" por xp -ap:

x171 -am xp - ap
_xm
-apxm^ X,T|_p
+ ap1 xmJ?p
apxm-p -am
aPx^-P
a2pym-2p
-am
1

Podemos concluir, então, que o quociente na divisão ê na forma:

Q(x) = x"1-*1 + apxrn"2p +■ ..+ a (S-1)P XX

e o resto é da forma:

R(x) = a6px,fl-fip -am, se pr

Devemos ter R(x) = 0, isto é;

asp xxm’sP
-am =0

Corno a 0, para se ter a identidade acima, necessariamente:

m - sp = 0

dai: m = sp, o que demonstra a propriedade.

8.51 Os poünómios f(x) e g(x) sao divisíveis por h(x): então, demonstre que o
resto r(x). da divisão de f(x) por g(x), também é divisível por h(x).

Solução

Se os polinòmios f(x) e g(x), são divisíveis por h(x), existem os polinõmios


qjx] e q?(x) tais que:

f(x) = h(x)xq1(x)
g(x) = h(x)xq3(x)

Na divisão de f(x) por g(x) temos:

f£x) = g(x) xq(x) + r(x)

122
Temos, sucessivamente:
h(x)xí1(x) = h(x)>q?(x)>q(x) + r(x)
rtx) = h(x) ><cr,(x) - q2(x)>q(x)I

A igualdade acima demonstra a propriedade.

8 6) Na divisão de um polinõmio p(x) por a(x), o resto é Ti(x); na divisão de p(x)


por b(x) o resto é r2(xj, na divisão de p(x) por a(x) >b(x), é r(x).
Demonstre que se r(x) è dividido por a(x), o resto é n(x); e se r(x) é
divididopor b(x). o resto é r2(x).

Solução

,£(x) a(x) PÍXL b(x) P(x) a(x]>b(x}


q,M r3(x) r(x) q(x)

P(k) *■ atxj MjjtxJ + r,!*) (1) p(x) = bfx)?q2(x) + r2(x) (2) P(xh [a(x3>b(x)]>q[x)-tr(x) (3J
ígr[r,(x)] c gr |a(x)J ígr[r2(x)J < flr [b(x)J jgr[r(x)] < gr|a(x)>±j{x}|
[ou f|(x) = 0 |ou .JxjbO Lou rfx) b □

a hipótese temas.
Substituindo (3) em (1), obtemos:

|a(x)>b(x)]>q(x) + r(x) = a(x)x|1(x) + r,(x)


r(x) = a(x)>{q1(x)-b(x)x](x)] + r,(x) (4)
- -______ *— '
quociente resto

E sendo grfr^x)] < gr[a(x}] ou r(x] = 0, a igualdade (4) mostra que se


dividirmos r(x) por a(x) o resto ê ri(x),
Analogicamente, se substituirmos (3) em (2), concluímos que o resto da
divisão de r(x) por b(x) ê ra(x).
2
3.7) Um polinõmio p(x) dividido por x + x + 1 dã resto -x + 1 e dividido por
x -x + 1 dã resto 3x + 5.Qual o resto da divisão de p(x) por x +x +1?
Solução
Inicial mente, observe que:
(x2 + x + 1)(x2 -x + 1) = x*1 4- X + 1
4 2
e que c resto procurado, de p(x) por x + x + 1, é da forma:
3 2
r(x) = ax + bx + cx <- d

123
T emas:

PÍ.x.h............ a(x) = xa + x + 1
r,(x)= -x + 1

PCX)_________ bÇx) ~ xz — x 4-1


rjx) = 3x + S

a(x)>b(x)

Pt*)
r(x) = ax} + bxz + cx + d

Do exercício anterior, podemos concluir que se dividirmos r(x) por


a(x) = x2 + x + 1 c resto ê r, = -x +1 r

ax 3 + bx2 + cx + d x2 + x + 1
-ax3 - ax2 - ax ax + (b - a)

(b - a)x2 + (c - a)x + d
-(b - a)x 2 - (b - a)x - (b - a)

r^x)^ (c-b)x+(d-b + a)

edaic-b = -1 (1) e d-b+a=1 (2).


Analogicamente, se dividirmos r(x) por b(x) = x2 — x 1 o resto é
r3(x) = 3x + 5 :

ax3 +bx2 + cx + d x2 - Xjr 1


-ax3 + ax2 - ax ax + (b + a)

(b + a)x3 + (c - a)x + d
-(b + a)x3 + (b + a)x -(b + a)

r2(x) = (brc)x + (d - b - a)

e dai: b * c = 3 (3) e d - b - a = 5 (4)

Resolvendo o sistema constituído pelas equações (1), (2). (3) e (4) obtemos:
a = -2, b = 2. C - 1 e d = 5; então:
r(x) = —2x3 + 2x2 + x + 5

124
8.8) O polmõmic f(x) dividido por x“ + x2 + 1 dá resto x3 + 2x2 + 3x + 4. Quaf ê □
resto da divisão de f(x) por x2 + x + 1?

Solução

Inicialmente, observe que:


x4 + x2 +1 = (x2 + x + 1)(x2 - x +1)
Temos:

(x’ + x i 1)(xi - x M)
a(x) - x? + x + 1 f(x) Ç2Txr7F''
r,(x) rí(x) = x’ + 2xs + 3x + 4

Do exercício 8.6 concluímos que r,(x) pode ser obtido dividindo-se r(x) por
a(x):

x3 + 2x2 + 3k + 4 X2 + X + 1
2
-x1 X2 -x x + 1

x2 +2x + 4
—x z- x-1
x+3

Então, o resto procurado ê t<i(x) = x + 3.

Exercícios Propostos

8.9) Divida o polinõmio A(x) pelo polinõmio B(x). nos seguintes casos

a) A(x)= x*-4x3 + 5x-6 e B(x)=xa + 2

b) A(x) = x5 +2x3-6x + 4 e B(x) = x2 + x-2

c) A(x) = 2xJ’-3xa+4xz-5x + 6 e 8(x)=xz-3x+1

d) A(x)= x5-2x4-6x3 -Bx2 + Sx-6 e B(x) = x3 -3x2 -x + 3

8.10) Determine a e b para que o resto da divisão do polinõmio x5 - 2x4 + ax3


- 7x2 + 3x + b pelo polinõmio x2 ■+ 4 seja 3x + 2.

3 2
8.11) Determine a e b para que o polinõmio 2x 4- ax -1 Dx + b seja divisível por
x2 -3x4-1.

125
8.12) Na divisão de um polinòmio de grau 5 por um outro de grau m,
encontrou-se quociente de grau q e resto de grau 4. Determine m e q.

8.13) Seja o polinomio:


ffx) = xJ -(3 + 1)x3 - (a2 -4)x2 -f-bx + o
a) Determine □ quociente da divisão de f(x) por x2 - x - 2.
b) Determine b e c, em função de a, para que a divisão seja exata.

8.14) Sejam os polinõmios:


f(x) = x3 + 3bx2 + 3gx + d
g(x) = x2 4 2bx *c
Se f(x) é divisível por g(x), verifique que f(x) é um cubo perfeito e g(x) é um
quadrado perfeito.

8.15) O polinòmio a(x) dividido pelo polinòmio b(x) dá quociente q(x) e resto r(x);
a(x) dividido por x xb(x) dá □ mesmo resto r(x). Verifique que x|q(x).

8.16) Na divisão de um polinòmio A(x) por B(x) o quociente é Q(x) e o resto é


R(x). Qual é o quociente e qual é o resto na divisão de A(x) por 2>B(x)?

8.17) Determine m. n e p de modo que o resto da divisão do polinòmio


6x4 + mx2+nx + p pelo polinòmio 2x3— 4xz + 3 seja x2 + 3.

J.18) Um polinòmio f(x), de grau 3. dividido por xz-1 dã resto 6x+2 e quando
dividido por x3+1 dá resto 2x + 8. Determine f(x).

8 19) O polinòmio f(x) = x4 + 4mxa + 5ax2 + 4bx + c é divisível pelo polinòmio


g(x)= x3 + 3mx2 -t 3ax+b. Verifique que: a = m22, k
b_= m3 e c _ md,

8.20) Determine os números reais p e q, sabendo que x< +1 é divisível por


x2 + px + q.

8.21) Sejam os polinõmios:


A(x) = px3 + (p! + q)x2 +(2pq + r)x + qIa: +s
B(x) = px3 + (p1 - q)x3 + rx- q’ + s
C(x) = xz 4-pX-f-q
Mostre que se C(x) | A(x), então C(x) I B(x).

8.22) Um polinòmio P(x) dividido por x + 1 dá resto 4; dividido por xz +1 da resto


2x + 3. Qual é o resto da divisão de P(x) por (x + 1)(x2 +1)?

126
9 23) Um polinómio f(x) dividido por x + 1 dá resto 2, dividido por x2 -x + 1 dã
resto x-6 e dividido por x3 + 1 dá qucciente x+ 2. Determine f(x).

8.24) Determine os reais positivos a, b e c, sabendo que o polinòmio


ax'* + bx3+G dividido por x2 + 1 dá resto r^x), dividido por x3 + 1 dá resto
r?(x), e que r^xjwjtx) = 2(x-1)(x-5).

127
Capítulo
A divisão de polinômios
em que o divisor
é de grau 1

9.1 - Teorema do Resto

□ resto da divisão de um polinõmio p(x) pelo polinòmio x - c é o valor


numérico de p(x) em c, isto é, o resto é p(c).

Demonstração

Na divisão de p(x) por x - c temos:


|p(x)= (x-c)q(x) + r(x)
|gr[r(x)]<gr[x-c]-1 ou r(x) = 0

A segunda relação garante que r(x) é constante' tem grau zero ou é nulo. Para
calcular essa constante, substituímos x por c na primeira relação, temos,
sucessivamente:
p(x) = (x-c)q(x) + r
i
T
constante
p(c) = (c-c)q(c) + r
p(c) = r

o que demonstra a propriedade.

Do teorema do resto, resulta o teorema de d'Aiembert:

Um polinòmio p(x) ê divisível por x - c se e somente se p(c) = 0, isto é,


c é uma raiz de p(x).

É imediato que o teorema do resto pode ser ampliado para os divisores


x + c, ax - b e ax + b, onde a * 0.

129
Temos, então, o resultado.

dividendo divisor resto

P(X) X- c P(c)

p(x) X * c P(- C)

p(x) ax — b

b
P(x) ax + b P
a

Exemplos

1°) O resto da divisão de p(x) = x33 - 2x2 + 3 por x - 1 é:

p(1>1-2 + 3=2

2Ú) O resto da divisão de p(x) = x 3 +3x2 - x —3 por x + 3 é

p(-3) = (-3)1 + 3(-3)I-(-3H3-0

Então, p(x) é divisível por x + 3, ou ainda: — 3 é a raiz de p(x).

3o) O reslo da divisão de p(x) = 8x3 + 2x +1 por 2x - 1 é:

Pfn=8jiy|I3 + 1
2x- + 1 = 3
I2J AsJ 2
1°) O resto da divisão de p{x) = x2 + 3x -1 por 3x + 1 é:
2
1
1=1
3 s
9.2 — O Dispositivo de Briot-Ruffini
Na divisão de um poliromio p(x) pelo polinômln x-c, o quociente e o resto podem
ser obtidos através de um procedimento muito conveniente, conhecido como
dispositivo de Briot-Ruffini.
A descrição desse dispositivo vem a seguir.
Na divisão do polinõmio:
p(x) = anxn + ah_1xr'-1 + afl_2xí1’*2 + ... + a2x2 + a1x + aQ1 ar * 0 e n>1

pelo polinõmio x - c. o quociente q(x), de grau n — 1, é da forma:


q(X)-qn_1 xn-’ + qn^Xn-2 + qn_3xn-3 + .,.+ q2x2 +q,X + q0

130
Vamos aplicar o método de Descartes para efetuarmos a divisão de p(x) por x - c:

Qn-lX + qn-2x + q„-3x"-3 + ...4-q,x + q0


0
_______ X - c
qn-ixn + qn-2x +qn-3* + ... + q1x2 + q0x

- cQn-1x ^ -—2-2
-cqn_2xn-2-...-cq -
x -cq^-cqg

qn-i^+íqn-a-cqn-i)* + (Qn-3 -CQn-jíx" 2 + - + (qi-cq2)x2*(q0 -cq,)x-cq0

constante
T
A identidade p(x) = (x - c)q(x) + r nos dá:
qn-1 ~ an

Qn-2 ~ CA =a
qn-3 - Cqn-2 = an-2

q-i - cct2 — a 2
q0 - cqi= ®i

r~cq0 ~o
E, então:
Çn-1 “ an

qn_2 =an_i + cqn_1

°ln-3 = an-2 + cqn-2

q, =a2 +cq2
q0 = a^cq,
r = a0 +cq0

O cálculo de qn_,, qn_2 q0 e r, através das igualdades acima, tem natureza


recorrente e, na prática, pode ser efetuado com a utilização do dispositivo de
Briot-Ruffini:

a.. an-1___ an-2 a1 ao


an— 1 cqn_2 + an_2 ... cq, + a! I cqQ + a0

n-2 -3

131
Exemplo
Vamos dividir o polinômio x3-7x-6 por x-3.

1 0 -7 -6
13 4-0 3-3-7 2 3-6,
«
3 2 Ò

O quociente é q(x) = 1 xx2 + 3x - 2 eo resto é r = 0.

9.3 - Observações Sobre o Dispositivo

1o) Quando o divisor ê da forma x + c, escrevemos x + c = x-(-c)e aplicamos o


dispositivo; por exemplo, vamos dividir o polinômio p(x) = x4+6x2-3x-6 por
x + 1:
___ I—l 0__________ 6__________ -3 -6
-1 I 1 1 • (-1) 4- 0 H) (~1) + 6 7-(-1)4- (-3), (-10) ■ (-1) 4- (-6)z

-1 7 -10 4

O quociente é q(x) = 1 xx3 1>x2 + 7xx -10 e o resto é r=4.

2o) Quando o divisor é da forma ax - b, onde a * 0, o dispositivo de Briot—Ruffini


pode ser usado com pequenas modificações. Temos:

p(x) = (ax-b)xq(x) + r
e daí

>aq(x) + r
q'(x)

Dividimos, então, p(x) por X- —, obtendo o quociente q’(x) = aq(x) e o resto r,


a
Observe que, para obtermos q(x), fazemos q(x) = - xq'(x) e que o resto

desejado é o resto r obtido.


Por exemplo, vamos dividir o polinômio p(x) = 2x3 + 4x2 -6x + 8 por 2x - 1.

132
, 1
Dividimos, então, p(x) porx- —

2 4 -6 6
_1_ 2 1
2 5 - + (-6) \ 2 ) T‘5
~T~ 7 25
2 4
25
O resto encontrada, r = —, é o resto procurado: os coeficientes do quo-
4
ciente encontrado, q'(x). devem ser divididos por a = 2. para se obter q(x):
7
q(x) = |x3 5 ~2
-t- — X 4-
2 2
5 7
q(x) = x?4 — x —
2 4
Vamos, dividir a palinómio p(x) = 32xs 243 por 2x + 3.
Dividimos, então, p(x) por x+ |:

32 0 ____ 0 c___ 243

32 321 (--2-).O (-46: 72 (-t) + 0 -108


H)- 162 ' A'
r 2j
+ 2^3

-48 72 -103 1G2 zero


O resta encontrado ê o resto procurado: r = 0; os coeficientes do quociente
encontrado, q’(x), devem ser divididas por a = 2. para se obter q(x)
q(x)^|xq'(x) = 16x4-24x3 + 36x2 -54x+81

9.4 — A Divisibilidade pelo Produto — Teorema

Se um polinômio p(x) é divisível (separadamente) por x-a e por x - b,


a * b. então p(x) é divisível pelo produto (x-a)4x-b)

Demonstração

Iniclalmente, observe que p(a) = C e que p(b) = 0, pois p(x) è divisível por
x - a e por x - b.
Na divisão de p(x) por (x - a)x(x - b) o divisor ê de grau 2; então, o resto é da
forma r(x) = mx + n. Temos:
r(x)
p(x) = [(x-a)(x-b)]>q(x) + mx + n

133
Substituindo x por a e par b, obtemos:
p(a) = [(a - a) (a -b)]q(a) + ma + n
p(b) = [(a-b) (b-b)]q(a) + mb + n
e sendo p(a) = p(b) = 0:
Resolvendo o sistema acima, vem m = n = 0. e daí r(x) « 0.

Observações
1a) É fàcíl verificar que a recíproca desse teorema é verdadeira, isto ê, se
p(x) ê divisivel por (x - a] (x — b], então p(x) è divisível par x - a e ê
divisível por x - b
2a) O teorema pode ser generalizado para um número finito de fatores
(x-b), (x - c),... (x—f), onde a, b, c £ são distintos dois a dois.

9.5 — A Divísibllidade por (x - b)”- Teorema

2
Um polinõmio p(x) é divisível por (x — c) se e somente se:

1 °) p(x) é divisível por x - c;


2’) o quociente q(x) da divisão de p(x) por x — c também é divisivel
por x - c.

Demonstração

1* parte

2
Hipótese: p(x) é divisível por (x— c) .
Tese: p(x) é divisivel por x — c e q(x) é divisívet por x — C.
2
Se p(x] é divisivel por (x - c) , podemos escrever:

p(x) = (x-c)3q(x)
p(x) = (X —c)[(X - c)q'(X))
q(*)
, identidade acima mostra que p(x) é divisível por x - c e que □ quociente da
divisão de píx) por x — c:
q(x) - {x-c)q'(x)
também é divisível por x - c.

2* parte

Hipótese; p(x) ê divisível por x — c e q(x) é divisivel por x — c.


Tese; p[x) ê divisivel por (x - c]2
Se p(x) é divisivel por x - c, podemos escrever:
p(x) = (X —0) xq(x)
e se, q(x] è divisível por x - c:
q(x) = (x-c) xq'(x)

134
Daí:
p(x) = (x -c)[(x - c)q (x)]
p(x) = (x-c)2 xq'(x)
o que garante que p(x) é divisível por (x - o)2
Observação: Da mesma forma, pode-se demonstrar que “um polinõmio p(x) ê
divisível por (x — c)J se e somente se:

1a) p(x) é divisível por x “ c;


2fl) o quociente q(x) da divisão de p(x) por x - c é divisível por x - c;
3a) o quociente q (x) da divisão de q(x) por x - c é divisível por x - c".
O teorema pode ser generalizado para um divisor da forma (x - c)m

Exercícios Resolvidos

9.1) Determine k. k e ÍR, para que o polinõmio p(x) = x4 -5x3 + kx2 2x + 6 seja
divisível por x + 2.

Solução
Devemos ter p(-2) - (teorema do resto):

(-2)4-5(-2)3 + kX-2)z-2(-2)+6 = 0
4k + 66 = 0
K.-«
2

9.2) Determine os valores de a, a s Êí, sabendo que o resto da divisão de


p(x) = x3 + x2 -5x-a3 + 6 por x-a é positivo.

Solução
Devemos ter p(a) > 0
a 3 + a2 - 5a - a3 + fi > □
a2 - 5a + 6 >□
e dai: a < 2 ou a > 3.

9 3) Determine a e b, no poíinõmio f(x) = x3 + 2x2 + ax + b. de modo que f(x) + 1


seja divisível por x + 1 e f(x) - 1 seja divisível por x - 1.

Solução

Se o polinõmio f(x)-i-1 = x3 + 2x2 +ax + Ó4-1 é divisível por x + 1, tem-se:


(-1)3 + 2(-1)? + a(-1) + b + 1 = 0
Se o polinõmio f(x)-1 = x3 + 2x2 + ax + b-1 ê divisível por x - 1. tem-se:
13 + 2 >42 + axl +b -1 - 0

135
Dai. temos o sistema:
—a -t- b = -2
a + b - -2
Então, a = 0 e b = - 2 .

9.4) Qual deve ser o valor do coeficiente c para que os restos das divisões de
p(x) = xlO+ax'4+bx2+cx + d púr x+12 epor x -1 2 sejam iguais?

Solução
Deve-se ter: p(“12) = p(12).
Então:
(-12)10 a(-12/ + b(-1 2}z + c(-12) + d = 1210 + a 2)J +
e dai: — 12c = 12c e c - 0.

9.5) Determine m para que o pclinomio p(x) = xJ1 -5x7+4x-m seja divisível
por 2x +1. Qual □ quociente na divisão?

Solução
1 0. -5 4 -m
1_ 1 1 _J9 51
2 2 4 B
zero
O problema pede o quociente na divisão. Neste caso, a melhor soíução é
aplicarmos □ dispositivo de Briot-Ruffini:
-51 -51
A divisão é exata:--------- m = Q;edaí: m =------ .
16 16
3 1 2 19
O quociente e: q(x) = — x ----- X
------ x-------- x + —.
2 4 8 16

9.6) Um polinômio p{x) foi dividido pelo polinómio x — a. Usando-se a dis-


positivo de Briot-Ruffini, obteve-se o quadro abaixo:

Determine p(x).
3 —4 5 d____ e
a b -10 G 24 | 40
Solução
Observe que no dispositivo temos sueess iva mente:

3=b
b >a 4 =-!□
-1 0 xa + 5 = c
c + d - 24
24 >a +■ e = 40

136
Dai: a = -2, b = 3, c = 25. d = 74, e = 58.
Então p(x) = 3x4-4x3+ 5x2+74X + SS.

9.7) Efetue a divisão de xn -an por x - a, n > 1 e a 4 0.

Solução

Aplicando □ dispositivo de Briot-Ruffini:


(n -1) zeros

j 1 0 0 0 0___ 0 -a11
a I 1 a a2 a3 grt-2 a1 0

Então: q(x) = xn + ax" a2x"'3 +.,, + g X + a' ' e r - 0-


9.6) Efetue a divisão de x n + a por x + a. n > 1 e a 0.

Solução
Inicialmente observe que o resto é:
r=(-a)n + an
se n ê par: r = 2a11; se n ê impar: r = 0.
Aplicando o dispositivo de Briot-Ruffini;

1a caso: n ê par
j 1 0 0 0 ... 0 art
-a | 1 -a a2 -a3 ... -an-i | 2an

-ax^2 + a2xn‘a ...-a"-2 x + a n


Então: q(x) = x e r = 2a

2° caso: n é ímpar

I 1 0 0 O o an
-a I 1 -a a2 -a3 a11 I 0
Então: q(x) = xn“1 -ax"-2 + a2xn-3 ...-an-2 x + a*1-1 e r = 0

9.9) Determine os valores de m e n de modo que o polinõmio.


P(x) = x4 + (m + n-2)x3 + 5x2 - 3mx + 6
seja divisível por (x - 1) >(x - 2),

Solução
P(x) ê divisível por (x-1) ><x - 2) se e somente se P(x) for divisível por
(x — 1) e por (x-2). isto ê, P(1) = 0e P(2) = 0:
p(1) = 14+ (m + n - 2)X1)‘3 + 3m><1) + 6 = 0

P(2) = 2 4 + (m + n-2)><2)3 + 5>(2)3 + 5^2)2- 3m=<2) + 6=0

137
Dai:
n-m + 10 = 0
8n + 2m + 26 = 0

o que nos dá: n = -4 e m = 3

9.10) Calcule os valores de a e b de modo que o polinõmio P(x) - x



+ ax + b seja divisível por x + 1.
Solução
Observe que:
x2 + 1 - x2 — i2 = (x-i)(x + i)
Assim, para que P(x) seja divisível por x2 + 1 é necessário que P(x) seja
divisível por (x - i) e por (x + i), isto é, P(i) = 0 e (- i) = 0:

p(í) = Í4+P + 3><í)3 + a><i) + b=0


Pt-i) = (-C4+(-i)3 + 3><-Í]S + a^-i) + b = 0
Daí.
íai + b-2-i = 0
ai + b-2 + i = o
o que nos dã a = 1 e b = 2.

9.11) Determine aebde modo que o polinõmio: P(x) = x* - x3 + 2x2 + ax+ b seja

divisível por (x - I)2,

Solução
C teorema 9.S sugere-nos efetuar duas divisões sucessivas pelo dispo­
sitivo de Briqt-Ruffini:

-1 _2_______ a b______
2 £ _0 2 a +2 [ a + 2 + b- 0
1 1 1 3 | 3 + 5=0
O sistema:
a +■ 2 +■ b _ 0
a+5 - 0
nos dá; a = -5eb = 3.

9 12) Um polinõmio P(x) dã resto 4 quando dividido por x — 1 e resto 12 quando


dividido por x - 2, Determine o resto da divisão de P(x) pelo produto
(x-1)(x-2).

Solução
O teorema do resto nos dã: P(1) = 4 e P(2) =12.
Como o produto (x -1) (x - 2) tem grau 2, o resto da divisão de P(x) p°r
(x-1)(x-2) terá, no máximo, grau 1 (ou será nulo); então será da forma
R(x) = ax + b.

138
Temos:
R(x2
Pfx) f(x - 1)(x - 2)]Q(x) + ax + b
P(1) = [(1 —1)(1 - 2)]Q(1) + aXÍ + b = 4
P(2) = [(2-1){2-2)|Q(2) + ax2 + b = 12

O sistema
a+b =4
2a + b = 12
apresenta cama solução a - 8 e b = -4.
Assim, o resto procurado ê R(x) = 0x - 4
2
9.13) O polinõmio f(x) dividido por x - 1 dá resto 2x + 1. O polinõmio g(x) dividido
por x2 - 3x + 2 dá resto x + 2. Qual é o resto da divisão de s(x) = f(x) + g(x)
por x - 1 ? Qual o resto da divisão de p£x) = f(x)*g(x) por x-1?

Solução
T emas:

f(x) =, (x2 - 1)q(x) +(2x +1) (1)


g(x) = (X3 - 3x + 2)q(x) + (X + 2) (2)

C resto da divisão de s(x) por x - 1 ê s(1) = f(i) + g£ 1)


O resto da divisão de p(x) por x-1 p(1) = f(1)>g(1)
Em (1) e (2) substituindo x por 1;
f(D = 3
g(1)= 3
Dal, o resto da divisão da divisão de s(x) por x-1 é 6; o resto da divisão
de p(x) por x-1 é 9.

9 14) f(x) ê um polinõmio tal que 3[f(x)£3. f(x) dividido por x - 1 dá quociente
q,[x) e resto a; q-i(x) dividido por x - 2 dá quociente qz(x) e resto 3;
q2(x) dividido por x - 3 dá quociente q3(x) e resto 4. Determine a,
sabendo que 2f(2) = 3f(3).

Solução

Temas:
f£x) = (x-1)qi(x)+a (1)
q1(x)s(x-2)q2(x) + 3 (2)
q2(x) s (x-3)q3(x) + 4 (3)

133
Substituindo (2) e (3) em (1), sucessiva mente:

f(x)g(x-lH(x-2)c|a(x) + 3] + a
qi(x)
f(x} = (x -1) {(X - 2) [(x-3)g1(x) + 4)+ 3} + a (4)
q2(x)
Em (4). substituindo x por 2 e. em seguida, por 3:

f(2) = (2-1){(2-2)[(3~2)q3(2) + 4] + 3} + a
zero
f(2) = 3 + a
f(3) = (3 -1) {(3-2) [(3-3)q^3) + 4j + 3} + a
zero
f(3) =14 + a
E sendo 2 f(2) = 3 f(3). temos:
6 + 2a = 42+ 3a
E dai: a = — 35.

100 2X51 +■ 1 por x2 - 1 ?


9.15) Qual o resto da divisão do polinõmio f(x) = x

Sol uç ao
O divisor x 2 -1 tem grau 2, então, o resto ê da forma:

R(x) = ax + b
Temos:
J^x)
x 100 -2x51 +1 = (x2 - 1)q(x)+ ax + b

Substituindo x por 1 e sem seguida por — 1, obtemos:

^100 -2x(1)51 + 1 = (12 -1)q(1) + a + b


(-1)10D -2x^1)51

C sistema.

a +b - D
-a + b =4

nos dã a = -2 e b = 2

Então, R(x) = - 2x + 2.

140
9 16) Determine um polinõmio f(x). de grau 3, que é divisível por x - 3 e que
dividida por x + 1, x - 1 e x - 2 dá resto sempre igual a 5

Solução

Podemos escrever.

ftxJ^íx + Dq^xJ + S
f(x)s{x-1)q2(x)+5
f(x) = (x - 2)q3(x} +5

Daí:

f(X)-5=(X+1)q,(X)
f(X)-5 =(X-1)q?(X)
f£x)-5 = (x-2)q3(x)

As identidades anteriores garantem que o polinõmio f(x) - 5 é divisível por


(x + 1). (x - 1) e (x - 2); então, f(x) - 5 é divisível pelo produto (x + 1) (x - 2)
Logo:
f(x) - 5 = (x + 1)(x-1)(x - 2) xq(x)

E, sendo f(x) - 5 um polinõmio de grau 3, q(x) tem grau zero, ou seja, é


constante.
f(x)-5 = (x + 1)(x-1)(x-2) >4< (1)
Também f(3) = 0. pois f(x) é divisível por x - 3,

Substituindo x por 3 na identidade (1):

f(3)- 5 = {3 + 1)(3-1)(3 - 2)>k. com f(3) = 0.

5
daí: k = —; e, voltando em (1):
8

f(x) = --(x + 1)(x-1)(x-2)


o
n . n-1
9.17) Determine os coeficientes aeb para que a polinõmio f(x) = ax + bx + 1
2
seja divisível par (x - 1) Determine o quociente.

Solução

Vamos aplicar o dispositivo de Briot-Ruffini. efetuando duas divisões


sucessivas:

a b 0 0 1
_________ 0_________
2 a a + b a - b a + b - a + b I .1 ’ P i - Ü
1 a 2a + b 3a + 2b 4a + 3tí ~ I na + [n -1' b =

141
O sistema:

Ja + b + 1~ 0
|na + (n-1)b = 0

nos dáa=n-1 e b = - n.
2
Observe que o quociente na divisão de f(x) par (x — 1) é

q(x) = (n-1)xn-z + (n - 2)x 0-3 + . ., + 2x+l

Exercícios Propostas

9.10) Efetue a divisão de A(x) por B(x) nos seguintes casos:

a) A(x) = x'1 — 2x3 +4xz -6x+8 e B(x)= x -1


b) A(x) = 2x5 -5x3 -ôx e B(x) = x + 3
c) A(x) = 4xJ + x2 e B(x) = x + 1+i
d) A(x) = xa - x2 - x e B(x) = x-1+2i
e) A(x) = 4x3 - Sx2 +6X + 10 e B(x) = 2x-6
f ) A(X) = 5xz-7x + S e B(x) = 3x + 4

9.19) Um polinômio f(x), do segundo grau, dividido por x - 1, x - 2 e x - 3 dá


restos 1,8 e 27, respectivamente. Determine f(x).
9.20) Determine o valor de m de modo que o polinômio 5x4 - 6x2 + 4x'.? - mx + 2
seja divisível por x + 2.
9.21) Dividindo-se polinômio P(x) por x — a, usando-se □ dispositivo de
Briot-Rufftni, obteve-se o quadro abaixo:

4 13 c d e
a b 1 ^5 Z "Õ

Determine o polinômio P(x).

9 22) Divida x n -an por x + a. n > 1 e a 0.


n
9.23) Divida x + an por x - a, n > 1 e a

9.24) Determine a e b para que o polinômio ax3 + bx2 - 2Ôx + 1 S seja divisivel
por x + 3 e que, dividido por x - 3, dê resto - 60.

142
9.25) Dê as condições que p e q devem satisfazer para que o polinõmio seja
divisível por x + a (p e q naturais, p > q e a * 0).
9.26) Verifique que:

a) o polinõmio f(x) = (x - 2)1ÚO + (x-1)50 -1 é divisível por x2 - 3x + 2.


b) o polinõmio f(x) = (x + 1)|:3ri- xZn-2x~1 é divisível pelo polinõmio:
x(x + 1) >(2x + 1).
,n
9.27) Se n é ímpar e a * b, verifique que o polinõmio: (a + b + x)' 3 b “ X 6
divisível por (x + a) (x + b).

9.28) Verifique que o polinõmio 2x5-15x3+12x2+ 7x-6 é divisível pelo


polinõmio (x - 1)(x - 2) (x + 3) QuaJ o quociente dessa divisão?
9.29) Os restos das divisões de um polinõmio p(x) por x-1 e por x-2 são 3 e
4, respectivamente. Qual é o resto da divisão do polinõmio P(x) pelo produto
(x-1) (x-2)?
2
9.30) Determine o resto da divisão de um polinõmio A(x) por B(x) - X + 1,
conhecendo-se A(i) e A{-i), onde i é a unidade imaginária.

9.31) Qual é o resto da divisão de x1w+2xw-3xs + 2x+ 5 por x2 + x-2?

9.32) Determine o resto da divisão de um polinõmio A(x) por B(x) - -x3 + 5x - 6,


sabendo que A(2) = 1. A(- 1) - 3 e que o quociente da divisão de A(x) por
B(x) é divisível por x + 1,
9.33) Os restos das divisões de P(x) por x + 1, x — 1 e x - 2 são 5. -1 e
-1, respectiva mente. Qual é o resto da divisão de P(x) pelo produto
(xz - l)(x - 2)?

9.34) Um polinõmio p{x), quando dividido por (x -1) (x + 2). dá resto 2x + 5, Dê os


restos das divisões de p(x) por x - 1 e por x + 2.
9.35) Quando um polinõmio p(x) ê dividido por ax - b e por bx - a, os restos são
iguais e os quocientes são q,(x) e q2(x), respectivamente; a e b são reais
não nulos e [a| / |b|. Mostre que q2(x) é divisível por ax-b e que
q,(x) + q2(x) ê divisível por x- 1,

9.35) O polinõmio f(x), dividido por ax + b, a 0, dá quociente q(x) e resto r. Qual


ê o resto da divisão de:

a) f(x) por x + —
a
b) x xf(x) por ax + b
c) x2>í(x) por ax + b

143
9.3?) O polinõmio f(x), quando dividido por x — 1. dá resto a, dividido por x + 2, dà
resto b. O quociente da divisão de f(x) por x-1 é q(x). qual é o resto da
divisão de q(x) por x + 2?
9.33) Determine o polinõmio f(x), de grau 3, sabendo que f(x) é divisível por x-4
e que, dividido por x - 1, x - 2 e x- 3, dá o mesmo resto 6.

polinômo ax’+ bxa+1 seja divisível por


9.39) Determine a e b para que o polinõmio
(x-1)2.

9.40) O polinõmio A(x) — x.5“ + px + q é divisível por (x - a) . Verifique que


P = -5a4 e Q = 4a5h a * 0 Qual o quociente na divisão?

9 41) Verifique que o polinõmio p(x) - xn'? - xn+14-(n-1)xz - (2n-1)x + n ê


divisível por (x-1)2, mas não é divisível por (x-1)3. Qual □ quocienteda
divisão de p(x) por (x - 1 )2?

9 42) Verifique que o polinõmio p(x)=x(x" -na J + a^n-l) é divisível por


(x — a)2. Qual é □ quociente na divisão?

9 43) Determine os números reais a e b e o maior inteiro m, de tal modo que o


polinõmio p(x) = xS - a/ +■ bx3 - bx2 + 2x - 1 seja divisível por (x -1)m.

9.44) Mostre que o polinõmio


1 x x2 x3
1 1 1 1
P(x) =
1 2 3 4
12 2= 3? 42
é divisível por (x -1)3

9.45) O polinõmio p(x) = 2x4 - ax3 + 1 9x 2 —20x4-12 é divisível por (x-p)?t


onde a e p são inteiros positivos. Determine a e p.
9.46) Determine o polinõmio f(x) de grau 3, sabendo que f(x): é divisível por
X4-2; dividido por x + 1 dá resto 3; e dividido por (x-1)2 dá resto-9.

9.47) Verifique que o polinõmio p(x) = x43 + x4^1 4c+2 + X onde a, b.ced
são inteiros positivos, è divisível por x3 . v2

9.4fl) O polinõmio f(x) = xn(x2 + ax+ b), com n inteiro positivo, dividido pof
(x-2)2 dã resto 2fl(x-2). Determine a e b.

9,49) Prove que o polinõmio f(x) ~ (cos cp+ x sen <p)n — cos(n<p)- x sen (ncp) é
divisível per x2 +1.

144
Capítulo
Outros temas
10 importantes

10.1 — Polinômio Derivado

Definição
Seja o polinômio de coeficientes complexos:

p(x) = aD + a1x + a;x; + a3x3+...+ a(:k xfc-r,rr + anxr',an ?iC e ni1

Chama-se polinômio derivado de p(x) ao poliría mio;

p’(x) = a1 + 2a2x + 3a3x2 +...+ kafcxk 1 + + nanxn

De uma forma abreviada, podemos escrever:


r
1
pw=y, afcxk =-d'íx)^ £kakkx
x
k±±Q t=i

Em particular, se o polinamiD p(x) é uma consfante ou é o polinõmío nu!o. então o


polinômio P’(x) é o polinõmío nu!o.
Exemplos

1fl) Se p(x) = 6+5x+ 4x2, então p‘(x) = S + 2>4x - S + 8x.


2o) Se p(x) = 2x3 +4x2 i-2XtW, então p (x) = 3x2x3 + 2^x + 2 = 6x2+&x + 2.
3°) Se p(x) = 2x + 1, então p (x) = 1x2 xx° = 2.
4°) Se p(x) = 3, então p (x) = 0.

Adição - Propriedade I
Quaisquer que sejam os polinõmios p(x) e q(x). tem-se:

[p(x) + q(x)]’ -p '(x) + q (x)


Demonstração
Se um dos polinõmios è uma constanlet a propriedade é evidente. Caso contrário,
sejam:
m
p(x)=yakx’< e q(X)=£bkX,v
k=ü k=0

145
Suponhamos n £ m: observe então que os coeficientes b^ sao rniios para
m < k < n. E, considerando esses eventuais coeficientes nulos, podemos escrever:

p(x)+q(x)= £(ak+bk)xk
k=O

Por definição, tem-se:

„K-1 . (P(x} + q(x)]' = £k(ak+bk)*'


p'(x) = ^Tka*"1, q'(x)=£kb k*
k»i k-i

Daí:
n n
P’(x) + q-(x)=Xkai + - £(kakx' + kt\x
k=l

«Yk(ak + bjxk-'t = [p(x)=q(x)]'


k=i

Propriedade II
Sejam □ polinómio p(x) e □ número complexo X.. Tem-se:

Demonstração
Se p(xj é constante, a propriedade é evidente. Caso contrária, seja:
Fl
P(X}= £akxk
k=0

Tem-se
n
X->p(x) = £ Xa kx
x
k
k=0

e, por definição:
n
p'(x)-Ykakx k-1 e [Xp(x)]' = £(kXakx k-’)
k=1

Dai:
fl n
?lSç'(x) = Xx]Tka k* k-1 - (X>kakx

146
Multiplicação - Propriedade lll
Quaisquer que sejam os polinômíos p(x) e q(x], tem-se:
[pfxjxtfx)]' = p'(x)>q(x) +■ p(x)><(x)

Demonstração
Veja exercido 10.12
Propriedade IV
Sejam os polinômíos p(x) e q(x), tais que:
p(x) = [q(x)]n; ondene N*
Então
p'(x)= n>[q(x)]

Demonstração
Veja exerclcín 10,12
Exemplos

1°) Se f(x) = (x3 + x + 2) + (x3 + 4x), tem-se:


P(x) q(x)

f'(x) = (2x +1)-i-(3x2 + 4)


P'(x) q'(X)

2°) Se f(x) = 20><x4 +3x2 +4x + 5), tem-se;


Z PW
f’(x) = 20 X4x3 +6X + 4)
76Õ
3a) Se f(x) = (x3 + 2x2 + 5x)>(xs + 6x2 + 2), tem-se:
p(x) q(x)
f’(x) =■ (3x2 + 4x + 5)X*5 + 6x2 + 2) + (x3+2x2-r-5x)><5x^+12x)
P(x) q(x) p(x) q'(x)

4o) Se então f(x) = (xs +13x2)2ao , então P(x) = 200><xs+13X2)199 >(6xs +2Gx),
gZQ0 9 (x)
Derivação sucessiva
Seja p(x) um polinõmio de coeficientes complexos.
Define-se polinõmio derivado de ordem n do polinõmio p(x)b notado com
p(n\x), como segue: ____________ ________

1. ’)P W(x) = p(x)


2. °)p(n+1) (x) = lP(n\x)r

147
Assim, tem-se sucessivamente:

p[C)(x) = p(x)
p‘1>(x) = p’(*)
p[2)(x) = [p'(x)]h. que também se o nota p"(x)

p,3í(x) a= [p"(x)]\ que também se nota p'"(x)

Exemplo

Sendo o polinõmio p(x) = x4 +6x3 + 5x2 + 7x + S, temos, sucessivamente

p(01(x)= p(x) = x4 + 6x3 + 5x2 + 7x + 8

pí1í(x) = p(x)=4x^ + 1âx2 + 10x + 7


p(2)(X] =P“(X) = 12x2 + 36X + 10

pt3)(x) = pl,,(x) = 24X + 36


pH)(x] = pN,,(x) = 24

pf5í(X) = p (x) = 0

Pf6)(x>pí7í(x) = p(Sl (x)=... = 0

Observe que na sequência o grau dos polinòmios diminui de 1, assim, se p(x) tem
grau n, todos os polinòmios derivados de ordem superior a n são nulos.

Exercícios Resolvidos

10.1) No polinõmio p(x) = ax2 + bx + c, a + 0. tem—se p'(a) = O, Verifique que


b2 - 4ac
p[ti) = -
4a

Solução

Temos.

p'(x) = 2bk + b
b
p '(íx)=O => 2 a a + b -■ 0 =>
2 21
2
-b b2 h2 bJ-2bJ+4ac
p<n,-p(-sha{fí) I + b
2a
+ c =
4a

2a
+ c - „-------------- —
4a
b? - 4ac
»------------ —.
4a

148
10.2) Determine um polinõmio na forma p(x) ~ a0 + a,x + a 2x2 + a3x3, sabendo
que:
p(2X) p (X)

Solução
Temos:

p(2x) = aQ + a,(2x) + a2(2x)2 + a3(2x)3


p(2x) = aQ + 2a1x + 4a2x2 + 8a3x3
p'(x) = a-, + 2a2x + 3a3x2
p"(x) = 2a2 + 6a3x

Sendo p(2x) a p'(x):<p"(x), temos:

aQ + 2a i> + 4a jX2 + fla 3x3 = (a.! + 2aax + 3a3x2)(2a2 + 6a 3x)


a0 + 2a,x + 4a2x2 +8a3xJ » 23,32 + (6aia3 -t-4a2 )x + 1Sa2a3x2 +1 Ba3x3
O sistema:
2a,a2 = a0
63,83 + 4a? = 2ai

1 8a2a3 = 4a2
1fla2 = 03j

Apresenta como solução:


l^agsa^ajsa-jsQ e p(x) é nulo
4 4 3
2°) a0 =a, = as =0, a3 =- e p(x) = ~x .

10 3) Um polinõmio p(x) satisfaz a condição p(x) + p'(x) = 2x2 + 5x + 4. Determine


p(x).
Solução
Note que o grau de p(x) é maior que o grau de pr(x); então, o grau de p(x) é
igual ao grau da soma isto ê, p(x) tem grau 2.
Então, p(x) é da forma:

p(x) - ax2+bx + c, a 4 0

Dal:
ax2 + bx + c + 2ax + b = 2x2 + 5x + 4
p(x) p(x)
ax2 + (b + 2a)x + {c + b) 2x2 + 5x + 4

149
Resolvendo o sistema:
ía = 2
lb + 2a = 5
[c + b = 4

obtemos a = 2,b=1ec = 3; então, p(x) = 2x2 + x + 3.

10.4) Os polinômias e g(x) satisfazem a condição:


g(x) = x3 Mf(x)]3
Calcule g'(S), sabendo que f(3) = 2 e f'(3) = 1.
Solução
Usando as propriedades do polinõmio derivado, temos:
propriedade III
g'(X) - 2x>[f(x)]3 + x2íx3[f(3)]I>4,(X)
+ X
propriedade IV

Substituindo x por 3: g'(3) = 2 >3 >{f(3)]3 + 32x3Xf(3)J2>í'(3)


g (3) = 2x3 j-23 + 32>3x22Xl
g (3) = 154

10.5) Para o polinômio p(x)= xkb demonstre as implicações:

1,“) h<k^pí‘:(xi--Akh jock h


1.») h> k => p{h|(x) = 0
Solução
Para a demonstração da primeira implicação, vamos usar o Método da
Indução Matemática. (Veja votume 2 desta coleção, p. 38,)

Teorema 1

A primeira implicação é evidente para h = 0

Teorema 2
Hipótese' suponhamos que a implicação é válida para h < k, isto è:
pth’(x) = Akhxxk'h
Tese: demonstremos que a implicação ê válida para h * 1 S k, isto é:
píh1l(x) = A^k.tui xi?"íh,'l>
De fato:
P <h1,(x) =(p(tll(x)]" = [AKh xx h]r=A
xx kk"-h ’= Ak „ ^xk-r)* = A K.h *(k ~h)xx

= (k-h)xAkh xxk^1l = Ak ht1


^k.^1

15Ü
A primeira Implicação está demonstrada para todo h, h < k. Em particular,
para h = k tem-se:
p(k\x) = k!
Dai, para h > k, como k! é uma constante:
píh)(x) = 0
c que demonstra a segunda implicação.

Exercícios Propostos
10.6) Para cada polinômio p(x) abaixo, determine p'(x);

a) p(x)=5x3-3x! +x-1
L.X X 1 1 3 3 4 13g
b) p(x) = -xJ--x + —x5 -2x6+|x’
J d
c) p(x) = (x2 + X +1) xfx + 4)
d) p(x) = (5x2+7)3
e) p(x) = (1 + 5x-8x3)5
f) p[x) = (a + bx)m, me N* e b*0

10.7) O polinômio f(x). de grau 2, satisfaz as condiçoes: f(1) = 2,, f‘{2) - 1 e


f(0) = 4. Determine-o.

10.fi) Determine a, b e c no polinômio P(x) = 5x3 + ax! + bx + c, Sabendo que

P(x) + kíx-n^V) + (xs-1)P"(x) - o, kejR


3
10.9) Um polinômio f(x), de grau 2, satisfaz a condição f(x)jí'(x) s 2x + 6x3 +
+ 10x + 6. Determrne-o.

10.10) a) Se p(x) = 5x4, determine p (x), p"(x) e p"'(x).


b) Se p(x) = 3x4 + 5x3-4x2+fi, determine p(4)(x).
10.11)Seja o polinômio:
n
P(X) = Vahx,k!
k=ü
Demonstre as Implicações:

k-h
1.“)h<n=.pm(x)=XaKA k n **
k-h

2.“) h > n => (x) = 0

10.12) Demonstre as propriedades ill e IV» referidas anteriormente, neste capitulo.

151
10.2 — Máximo Divisor Comum
Definição
Sejam a(x) e b(x) polinômios não nulos, de coeficientes complexos.
Então, o polinõmio d(x) é um máximo divisor comum (mdc) de a(x) e b(x) se e
somente se:
a) d(x)|â(x) e d(x)^b(x)
b) se c(x)|a(x) e c(x)|b(x), então c(x)|d(x).
I n d i ca-se:
|d(x) = mdc [a(x); b(x)]|
Observe que se d(x) ê um máximo divisor comum de a(x) e b(x), então
À xd(x), X. e -C\ também c é. Dai, um máximo divisor comum de a(x) e b(x) não
é único.
Exemplo

Se a(x) = (x-1)2(x-2)(x-3) e b(x) = (x ~ 1)(x-2)3(x-4), um mdc de a(x) e b(x)


ê d^x) = (x- 1)(x-2); qualquer outro mdc desses polinômios é da forma
d(x) = c>^x-1) (x-2), ondecíO.

□efi níçao
Chamamos normalizado a um polinõmio p(x) não nulo, se c seu coeficiente
dominante é 1; neste caso então, p(x) tem a forma:

xn + an ix + ...+ atx + a0

Se q(x) = bnxn + b„_iX + ..- + bD é um polinõmio nao nulo onde bn *0, exisle um
polinõmio p(x) tal que:
p(X) - —>q(x) = Xn + ,,..|-Éo.
bn bn
Então, para todo polinõmio q(x), não nulo, há um único polinõmio nor-malizado
p(x), que é o produto de q(x) por uma constante c, c * 0; p(x) denomina-se
polinõmio normalizado associado a q(x),
Porexemp1o.se q(x) = 3x3 + 2x2 + 4x + 5, tem-se:

p(x) = ^[3x3 + 2x2 t-4x+5J = x3 + yX2 +^x + ^

Observe que, se d(x) ê um mdc de a(x) e b(x), podemos associar-lhe um


polinõmio d,(*h normalizado, tal que:
d^x) = mdc [a(x); b(x)J
Denomina-se mdc normalizado de a(x) e b(x) ao polinõmio di(x). Assim, para
cada par (a(k); b(x)], esse polinõmio di('x) é único
Agora, vamos construir um processo para a determinação do mdc de polinômios.

152
Teorema

Se a(x) e b(x) sao polinômios não nulos, onde gr(a(x)] > gr[b(x)j, e
r{x) não nulo é o resto da divisão de a(x) por b(x), então:
mdc [a(x); b(x)J = mdc[b(x); r(x)]

Demonstração (opcional)
Temos:
a(x) = b(x)>q(x) + r(x) e, daí, r(x) = a(x)-b(x)>q(x) (1)

Sejam mdc (a(x); b(x)] = d<x) emdc [b(x); r(x)J = d^x); então:

d(x) | a(x)=> a(x) = dtxjxj^x)


d(x) j b(x) => b(x) = d(x)xi2(x)

Substituindo em (1), vem;


r(x) = d{x)xii(x)-d(x)xl2(x)>q(x)
r(x)=d(x)[q1(x)-q2(x)>q(x)]

e dai, podemos concluir que: d(x)|r(x).

Como d(x) | b(x) e d(x) | r(x), concluimos que d(x) i d^x) = mdc(d(x); r(x)j,

Por outro lado, temos:


cUxHbíxJ^bW-d^xJx^íx)
d^x)! r(x)=> r(x)= d1(x)>q4(x)

e, substituindo na identidade a(x) = b(x)>q(x)+ r(x), vem:

a(x) = d.|(x)q3(x)>q(x) + d^xjx^fx) = d^x) Hq3(x] >q(x) + q4(x)]

isto é, d,(x)|a(x). E, como d1(x)|b(x), vem: d^xjjdtx).

Se d(x)| d^x) e d-^xJIdfx), vem a tese: d(x) = d1(x).

O algoritmo de Buclídes
O método de obtenção de um mdc de dois polinômios, que se baseia no teorema
anterior, utiliza o algoritmo da divisão repetidas vezes - esse método denomina-se
a/gorifmo de Eucüdes. Vamos descrevê-lo.

1. Sejam os polinômios a(x) e b(x), com grfa(x) 5 gr[b(x)], e seja ri(x) o resto da
divisão de a(x) por b(x).
Se r,(x) = 0, então b(x) = mdc [a(x); b(x)].

153
2 Se 0, tem-se mdc [a(x); b(x)J = mdc [b(x); r^x)]
Seja Fj(x) o resto da divisão de b(x) por n(x).
Se r2(x) = 0, então ri(x) = mdc[a(x); b(x)],

3. Se r2(x) * 0k tem-se mdc[b(x); r-i(x)] = mdc(ri(x); r?(x)].


Seja r3(x) o resto da divisão de n(x) por r2(x).
Se r3(x) = 0, então r2(x) ~ mdc [a(x); b(x)].

Observe que efetuando as divisões sucessivas temos:


gr[b(x)] > gr[r,(x)] > grfr2(x)] > gr[r3(x)] >...
Então após certo número n de divisões, sempre se atinge uma divisão exata (a
divisão de um poiinõmio por uma constante é certamente exata); isto é, se rn(x) - 0.
temos:
| rn_i(x) - mdc[a(x);b(x)]

Observe a sequência das operações executadas:


=b(x)q1(x) + ri(x)
b(x) = r1(x)q2(x) + r2(x)
r1Cx)=r2(x)q3(x) + r3(x)

rn_2 (x) = r^fx) qn(x)+ rn(x)


mdc 0
O pclinõmio rn_,(x) é um mdc de a(x) e b(x); a ele podemos associar o mdc
normalizado.
Exemplos

1e) Vamos determinar um mdo de a(x) = x6 + 2x5 + x3 + 3x2 + 3x + 2 e


b(x) = x4 + 4x3 + 4x2 “ x - 2.

1 Dividimos a(x) por b(x):


a(x) = x6 + 2x*5 + xX3 + 3x2 + 3x + 2 / + 4x3 + 4xZ ~ X ~ 2 = b{x)
-x6s -4x 5
Av-S
s -4x4 + X3 +2x2 /- 2x + 4

-2x5-4x‘d + 2x3+ 5x2+3x + 2


2xs + 8x4 +8x3-2x2-4x

4x4 + 1Qx3 + 3x2 - x + 2


-4x4 -16x3 -16x2 + 4x + B

r^X) = -6xa -13x3 +3X + 10

154
2. Dividimos b(x) por ri(x)
b(x) = x*1 +4x3 +4x2 -x-2 - 6x3 - 13x'2 + 3x + 10 = rt(x)
4 13 3. 1 2 ’5 ' 1 11
-X--------X J+-XZ + —X — x-----
6 2 3 6 36
11
I I 3 9 J 2
i _
— xJ +-x2 +-X-2
6 2 3
11 3 143 55
----- x ------- x a+ — X + 18
6 36 12
, < 19
r2(x) = —x
~Tâ-
00 12 + 18

3. Dividimos íi por r2(x)


19 2 19 19
— x^ + — x + — = r->(x/
r/x) ~-6x3 -13x2 +3x + 10 36 12 18^
6x3 +18x2 +12x 215 180
--------x +-------
19 19
5x2 +1 5x + 1 0
-5X2 -15x -1 0
r3(x) = O

Então, r2(x) = mdc [a(x); b(x>]; o mdc normalizado é:


d{x) -x2 +3x+ 2
Observe que os cálculos são exaustivos; para facilitá-los, pode-se, por
exemplo, antes de dividir b(x)por r-i(x), multiplicar b(x) por 6

Assim fazendo, r2(x) estará multiplicado por uma cons/ante, o que pouca
importância tem em nosso propósito, pois se d(x) é um mdc de a(x) e b(x),
entáo cxd(x), c * 0, também o é.
2") Vamos determinar um mdc dea(x)- 6x3 -13x2 + 9x -2 e b(x) = 2x2 - 3x + 1,

1. Dividimos af^por b[x):

a(x) = -6x3 -13x2 + 9x-2 2x2 - 3x + 1 = b(x)


6x3 + 9x2 -3x 3x - 2

—4x2 +6x-2
4x2-6x+2

Mx) = o

155
Então. b(x) ê um mdc de a(x) e b(x): o mdc normalizado ê:
3 1
d(x)= x2 — x +—
2 2
3*) Vamos determinar um mdc de a(X) = X4 -x3-7x2 + x + 6 e b(x) = x4 -
2
- 13x + 36.
1, Dividimos a(x) por b(x):
a(x) = x4 - x3 -7x2 + x + 6 x4 - 13X2 + 36 = b(x)

- x4 +13x2-36 1

r^x) = -x3 +6x2 +X-30

2. Dividimos b(x) por n(x):

b(x} = X4 -13x2 + 36 - x3 + 6/ <- x - 30 = r,(x)

x4 + 6x3 2
-30x -x-6

6x3-12x2 — 30x + 36
- 6x3 + 36x2 + 6x~180

r2(X) = 24x2 -24X-144

3.Dividimos r,(x) porr2(x):

riÇx) = -x3 + 6x2 + x - 30 24x2 -24x-144 - r2(xj


‘ x3
-X2 -6x __L +_L
24 24
Sx2 —5x —30
- 5x7 + 5x + 30

r3(x)- 0

Então. r?(x) = mdc [a(x): b(x)); o mdc normalizado é- d(x) - x 2 — X — 6.


Às vezes, e quando isto é possível, dadas as limitações de espaço,
podemos dispor os cálculos da seguinte forma:
b(xk cii(x) q2W
I -1 5
1 "X-6 24
___ 24
a(x) — x4- x3“7x2 + x + 6 x4-l3xa + 36.^ -x3 + 6x2 + x - 3j -24x2- 24X-14

-x3 + 6x2 + x- 30 24x2—24x—144


l
n(x) r2(x)

156
4”) Vamos determinar um mdc de:

a(x) = (x - 2)2(x - 3)3(x +2)


e
b(x) = (x - 2)3 Xx - 3)(X + 4).

Observe que a(x) e b(x) estão fatorados em um produto de potências cujas


bases são polinômios do primeiro grau, dois a dois distintos. Então, para
formarmos um mdc. construímos o produto dos fatores comuns aos
polinômios dados, afetados esses fatores nos menores expoentes com
que aparecem em a(x) e b(x):

d(x) = (x - 2)2{x - 3) = mdc[a(x): b(x)]

Observe que o polinômio d(x) satisfaz as condiçoes da definição.


Definição

Chamamos primos entre si aos polinômios nao nulos a(x) e b(x), quando:
Mdc (a(x); b(x)] = k. k constante, k * 0

Observe que se a(x) e b(x) são primos entre si, o mdc normalizado de a(x) e
b(x) ê 1.
Exemplo

Vamos determinar um mdc de a(x) - 2x4 + 4x2 -2x + 2 e b(x) = 2x2 <• x -1.

1. Diwtiimas a(x) par b(x):

a(x)= 2x4 4 4x3 ~2x + 2 2x2 + x -1 = b(x)


-2x4 -x3 + x 2
x3 22 >+ 114
- x3 + 5x2 - 2x + 2
3 1 1
X +-X 2 -- X
2 2
1 1 2 5
11 _
— x —x + 2
2 " 2
11
------- X
2 11 11
” 4* ' 4
2
, , ~21 19
r/x) = - —x+ —
4 4

157
2. Dividimos b(x) por ri(x):

b(x) = 2x2 + x-1 -T x+T*


4____ ri(x)
4______
........... a 236
- 2X -r ----- X — — x--------
21 21 441
59 7
— x -1
21
59 1_121
------ X +
21 441

r’<x> ’ m
3. A divisão de ri(xj por r2(x) é exata, isto ê, ra(x) = 0.
6 B0
Então, r2(x) =--------- mdc [a(x); b(x)]: e, sendo r2(x) constante, a(x) e b(x), são
441
primos entre si.

Exercícios Propostos

10.13)Dê o mdc normalizado des polinômios a(x) e b(x), nos seguintes casas:
a) a(x) - 2x^-4x3+5xa+2x-3 e b(x) = x£ -x4 + x3 + 4x2 -2x + 3
b) a(x) = 4xà - 20x* +25x3 + 10x2 - 20x - 8 e b(x) = a'(x)
c) a(x) = x3 +3x2-Ôx-24 e b{x) = x3+ 3x2-3x-9
d) a(x) = 2x3+ 7x2+10X + 35 e b(x) = 2x4 + 7xa -2x2 -3X + 14
e) a(x) = x*+12X-5 e b(x) = 2x3 - x2 + 4x + 15

10.14) Determine um mdc dos polinômios:


a) a(x) - (x2 ■■ 1)2(x +1)3
b) b(x) - (x3+ 1)(x-1)

10.15) Determine um mdc dos polinômios:


a) a(x) = 3(X - 1)3X(x + 2)2
b) bÇx) = 4{x-1)3x3(x + 4)

10.16) Determine c, real, para que os polínõmios x2 +(c + 6)x + 4c + 2 e x2 + (c + 2)


x + 2c tenham mdc de grau 1.

10.17)Determine m e n para que os polinômios p(x) = 2x^ ~x3-4x2+mx + n e


q(x) - x2 +2x-1 sejam primos entre si.

15B
10.3 — Mínimo Múltiplo Comum

Definição

Sejam a(x) e b(x) polinõmios não nulos, de coeficientes complexos


Então, o polinómio m(x) é um mínimo múltiplo comum (mmc) de a(x) e b(x) se e
somente se'
a) m(x) ê divisível pora(x) e m(x) é divisível por b(x);
b) se n(x) é divisível por a(x) e n(x) é divisível por b(x), então n(x) é divisível por
m(x).
Indica-se:
I m(x) = mmc [a(x); b(x)J

Observe que, se m(x) é um mínimo múltiplo comum de a(x) e b(x), então


Àxn(x), à e C* também o é. Daí, um mínirrio múltiplo comum de a(x) e b(x) não é
única.

Exemplo

Se a(x) = (x -1)2(x - 2)(x - 3) e b(x) = (x ~1)(x - 2)3(x - 4), um mmc de a(x) e b(x)
é m,(x) = (x — 1)2(x - 2)3(x - 3)(x - 4); qualquer outro mmc é da forma
m(x) = c><x-1)2>(Jt-2)'3(x-3)(x-4)k onde c * 0,
Obseive que, se m(x) é um mmc de a(x) e b(x), podemos associar-lhe um
polinómio m^x), normalizada, tal que:
mt(x) = mmc [a(x): b(x))
Ao polinómio m-i(x) denominamos mmc normalizado de a(x) e b(x). Para cada par
[a(x), b(x)], esse polinómio m((x) è único.
Agora, vamos examinar como podemos determinara mmc de polinâmios.

Teorema

Sejam a(x) e b(x) poíinómíos normalizados; sejam também d(x) e m(x) o mdc
normalizado e □ mmc normalizado de a(x) e b(x), respectivamente; então:

a(x)>b(x)= d(x)xn(x)

Aceitaremos o teorema sem demonstração,

Exemplos

1’) Vamos determinar o mmc de:


a(x) = 4x4 -12x3 + 13x2 -Sx + 1
e
b(x) = 6x3 -13x2 + 9x- 2.
159
Inicialmente. determinamos um mdc de a(x) e b(x) com o algoritmo de
Euclides:
= 2x2 — 3X + 1

Dai, o mdc normalizado é;


d(x) = x1!-|x + ^
Em seguida, multiplicamos o polinõmio normalizado associado a a(x) pelo
polinõmio normalizado associado a b(x); esse produto, dividido por d(x). dá o
polinõmio m(x):
m(x) = 12x5
5 - 44x4 + 63x3 - 44 x 2 + 15x-2

O processa é. ínfelizmente. extremamente cansativo.

2°) Vamos determinar o mmc de:


a(x) - (x-2)2(x-3)3(x + 2)
e
b(x) = (x - 2)3(X — 3)(x + 4).

Observe que a(x) e b(x) estão faforactos em um produto de potências cujas


bases são polinõmios do primeiro grau, dois a dois distintos. Então, para
formarmos um mmc, construimos o produto dos fatores comuns e nao
comuns aos polinõmios dadosT afetados esses fatores nos maiores
expoentes com que aparecem em a(x) e b(x):
m(x) = (x-2)3(x-3)3(x + 2)(x + 4) = mmc [a(x); b(x)]

Observe que o polinõmio m(x) satisfaz as condiçoes da definição.

Exercícios Propostos

W.1 B)Determine o mmc normalizado de a(x) = x4 - x3 - 7x 2 + x + 6 e do poli-


nómio b(x) = x4 - 13x2 + 36.

10.19) Determine o mmc e o mdc, normalizados, dos polinõmios

.12
a(x) = x ” -2xl3 + x + x12 e b(x) = x2-1-

10 4 - Uma Observação Importante

Seja f(x) um polinõmio na indeterminada x:


f(x) = a0 + adx + a2x2 + ... + anxn

com coeficientes em C.
Ao polinõmio f(x), fazemos corresponder uma função f. de C em <C, que associa a

todo a. ti 6 G, a imagem:

f(a) = a 0 + a-jOc + a2a + .,.+ anar*


que è o valor numérico de f(x) em ct.
160
A função f:
ÍC-*C
]f{x) = a0 +a1x + ... + anxri

denomina-se função polinomial,

Por exemplo, ao polinõmio f(x) = x? + 2x + 1. associamos a função quadrãtica f:

C-»C
f(x) = x2 +2x + 1

e, ao polinõmio nulo, associamos a função constante f:

fc
I f(X) = 0

na qual todo a, a e C, tem imagem zero.

Exercícios Suplementares

11.11 Um polinõmio f(x) tem coeficientes inteiros. Sabe-se que


f(A) - A. (A é inteiro positivo)
f(0) = P
onde P é um número primo, maior do que A. Determine A.
11.2) Determine os números reais a e b tais que

1= b
x(x + 1) X x +1

Deduza a expressão da soma:


1
1 ■ 1

1x2" 2>a4 n(n +1)

II.3) Um polinõmio f(x) é tal que:

f(u)+f(v) = f u + v , tfu, tf v, ue 'C e v e D


2 2

Verifique que f(xl é da forma a d+3^.

1G1
11.4) Determine a e b para que:

a b 2xi tr + 3
10’ - 1 1 □" + 2 (10* -1)(10K + 2)

11.5) Determine A. B e 0 tais que'


___________ 1___________ A B C
sen x >cosx(sen x + cosx) sen x CCS X sen x +cosx

11.6) Determine a. b e c para que o polinõmio nana variável n: an3+bn2+cn,


variável n:
represente, para todo n, a soma dos quadrados dos n primeiros números
inteiros positivos.
11.7) Um polinõmio f(x), de grau 3. é tal que f(-2) =-9, f(-1) =-1, f(1) = 3 e
f(2) = 11. Determine f(0).

11.8) Um polinõmio f(x) dividido por x + 3 dá quociente q(x) e resto -5; q(x)
dividido por 2x-1 dã resto 4, Qual é o resto da divisão de f(x) por
2x2 + 5x -3?

11.9) Determine os reais p e q para que o polinõmio A(x) = x2+px+q seja


divisor de A(x2).

11.10) Um polinõmio f(x) de grau 3, quando dividido por x2 - x + 2 e por x2 + x -1


dá restos 5x-7 e 12x-1, respectiva mente. Determine f(x).

11.11) Divida o polinõmio (x-2)n-1 por x - 3 onde n > 1.

11.12) Seja R(x) ~ mx + n o resto da divisão de um polinõmio P(x) pelo polinõmio


T(x) = xz - (a + b)x + ab, em que a e b são constantes distintas.

a) Determine m e n em função de a e b.
b) Determine m e n, no caso particular em que P(x) - x200, a = -1 e b-2.
c) Prove que, no caso ò, cada um dos coeficientes m e n é inteiro.
11.13) Um polinõmio f(x) dividido por (x - b)(x - o), (x - c)(x - a) e (x-a)(x-b) dá
restos 3x-1, x + 1 e 2x + 3 respectivamente.

a) Determine a, b. c.
b) Determine o resto da divisão de f(x) por (x -a)(x - b)(x -c)

11,14) O polinõmio f(x) dividido por x-5 dá quociente g(x) e resto 3; g(x) dividido
por x-3 dá resto 2. Qual o resto da divisão de f(x) por x-3? E por
xz - 8x +15?

II 15) Os polinomios a(x) e b(x) têm uma raiz comum a Seja r(x) o resto da divisão
de a(x) por b(x): verifique que r(x) é divisível por x - a.

162
II,16) Seja f(x)=x3 + x2-2 um polinômio de grau 3 que admite raiz 1 e duas
raizes complexas, não reais, cr e p.

a) Determine um polinômio g(x) = ax3 + bx + c, de grau 2, tal que g{1) = 1,


g(fí>p e g([J) = <x,
b) Determine g[çj(x)j?
c) Qual ê a grau de g[g(x)]2
d) Verifique que h(x) = g[g( x)] - x é divisível por f(x).

11.1?) Verifique que x,3m


‘ ,3n+1 + x3p+2 é divisível por x2 + x +1, onde m. n e p
são naturais.

11.18) Determine m para que o polinômio x2rn + xm + 1 seja divisível por


-
x^ + x + 1, m e W *.

11.19) Calcule o resto da divisão do polinômio:


q o
p(x) = X+ X +x + x27 + x&1 ^x243

a) por X-1
b) por x2 -1
II.2D) Demonstre, usando □ Método da Indução Matematíca, que um polinômio
de grau n, n i 1. admite no máximo n ra ízes.

11.21) Determine p,pg R, sabendo-se que o polinômio:


1 T-
A(x) - x -3x + px + 1
ê divisível por A"(x).

II.22) Seja o polinômio f(x) = x33 + ax + b


a) Determine a e b, sabendo que ft-1)=?4 e f 1) = 0.
b) Determine p para que f[x) s (x - 1)z >(x + p).

II.23) Considere o conjunto dos polinômios, com coeficientes reais, da forma:

P(x)= ax2 + bx + c
Define-se:
<p(P(x)] = P(x) + (X-1)P’(X)
Mostre que;
a) <W + P2(x)] = <dPt(x)] + tp{Pz(x)]
b) <p[kP(x)] = kxp[P(x)], ke Si

II.24) Determine os reais teu para que os polinômios p(x)=x 1 + (t + 1)X2 4-


+2x + 2u e q(x) = x3 + tx2 + u tenham mdc de grau 2.

163
PARTE III

Capítulo 11 — Equações algébricas


Capítulo 12 — Razões múltiplas
Capítulo 13 — Raízes imaginárias
Capítulo 14 - Relações de Girard
Capítulo 15 - Equações recíprocas
Capitulo 16 - Razões comuns
Capítulo 17 - Raízes reais
Capítulo

11 Equações algébricas

11.1 - Introdução

Uma equação algébrica (ou polinomial) é uma equação do tipo

P(x) = 0 (11-1)
onde P(x) é um polinômio. Caso o polinõmio tenha grau n. diremos que a equação
tem grau n.
Exemplos

a) 4x3 + (3-5i)x2+9 = 0 é uma equação algébrica de grau 3. onde a variável é x.


b) (6 + 3i)y2 +7y + 8 = 0 é uma equação algébrica de grau 2. onde a variável é y.
c) 2x + 3 = 0 é uma equação algébrica de grau 1, onde a variável é y.
d) 6x°=0 é uma equação algébrica de grau zero. Observe que esta equação
não tem solução, isto é, seu conjunto-solução é vazio:
S =0
e) Oxx = Q é uma equação para a qual não definimos grau (o polinõmio do "lado
esquerdo" é identicamente nulo). É fácil perceber que qualquer número
complexo é solução dessa equação, isto é, no universo dos complexos, o
conjunto-solução é:
S=C
f) Consideremos a equaçao:
2x3 +9x2 + 1 = 2x3 + 6x2 + 7x - 8 (D
que é equivalente a:
2x3 + 9x2 + 1-2x3-6x2-7x + 8 = 0 (H)
que, por sua vez, é equivalente a:
3x2-7x + 9 = 0 (III)
Observemos que a equação (l), que representa a igualdade entre dois polinômios
de grau 3, é equivalente à equação (III), que é uma equação algébrica de grau 2.
Em principio, poderiamos também dar o nome de equação algébrica a equações
do tipo:
A(x) = B(x)

167
onde A(x) e B(x) são polinómios (como, por exemplo, a equaçao (I) do exemplo /)■
Mo entanto, essa equação sempre é equivalente a uma equação do tipo P(x) = 0:
A(x) - B(x) « A(x)-B(x) = 0 e=>P(x) = 0
P(xj'”

Neste livro, chamaremos de equação algébrica a equações do tipo P(x) = 0.


Conforme veremos, isso facilitará nossas linguagem, pois poderemos, indiferen­
temente, falar nas raizes da equação P(x) = 0 ou nas raizes do polinõmio P(x).

Neste e nos próximos capítulos, faremos um estudo sobre as raízes das equações
algébricas (isto é, sobre as raizes dos polinómios) no universo dos números
complexos, embora possamos dizer que esse estudo já foi iniciada no capitula 9
deste volume, com o teorema de cTAfembert. De inicio, descartamos dois casos
cuja análise é imediata:

1 °) caso em que o grau é nulo (neste caso, teremos sempre S = 0. veja exemplo

2o) caso em que P(x) é identicamente nulo (neste caso teremos sempre S = C>
veja exemplo e).

Portanto, a partir de agora, nos ocuparemos apenas de equações algébricas de


grau n^O, as quais têm a forma:

n (■ a x + ... i-a1x + aD * OI (11.2)

onde;

ne N ‘
^n- , a aoe C
an * 0

Os números a(„an_1l„„a1,a0, do mesmo modo que no casa dos pclinâ-


mios, são chamados coeficientes da equação.

Antes de considerarmos o caso mais geral de equação algébrica, é ccnvenienle


fazermos alguns comentários sobre as equações algébricas de graus 1 e 2.

11.2 — Equaçao Algébrica do Primeiro Grau

Consideremos a equação algébrica de grau 1:

| ax + b = D |
(11.3)

□ nde a e h são números complexos quaisquer, com a * 0. Temos:

b
ax + b = O ax =-b x—
a

169
Assim, concluímos que a equação 11.3 sempre terà uma única raiz, que é o
b
numero complexo —, qual indicaremos por x, :
3

É importante observar (para aplicações que virão a seguir) que:

ax +b = a.Íx + È\a a(x -xn)


aj k a Al
isto é, o polinõmio do primeiro grau, ax + b, pode ser fatorado do seguinte modo:

ax + b = a(x - xd) (1V4)


isto está de acordo com o teorema de d'Alembert: já que xi é raiz do polinõmio
ax + b, então este é divisivel por x - xi (o quociente é igual a a).

11.3 - Equação Algébrica do Segundo Grau

Seja a equação de grau 2:

ax2 + bx + c = 0 (11.5)

onde a, b e c são números complexos quaisquer, com a * 0. Como vimos no


capitulo 2 deste volume (veja exercício 2.7). essa equação sempre tem duas
raizes complexas (que podem ser distintas ou iguais, reais ou imaginárias)
indicadas por Xi e x2que podem ser obtidas pela fórmula:

x=
-b±JÃ , onde A = b2 -4ac
2a
Pode-se demonstrar (faremos isso no item 11.6 deste capitulo) que o polinômio
ax2 -i-bx+c pode ser fatorado do seguinte modo:

ax2 + bx + c = a(x-x1)(x - xz) (11-6)

Já sabemos que a fórmula 11.6 é válida quando os coeficientes a, b e c e as raízes


e x2 são reais (veja capítulo 3 do volume 1 desta coleção). O que falta é
demonstrar sua validade no campo dos complexos.
Quando A = 0, temos xj = x2 e dizemos que a equação 11.5 admite duas raízes
iguais ou, então, que a equação tem uma raiz dupla ou, ainda, que a equação
tem uma raiz de multiplicidade dois Esta linguagem se justifica, pois neste caso
a formula 11.6 fica:
ax2 4-bx + c = a(x - (x - xt) = a(x - x,)2

isto é. o fator x - x1 aparece duas vezes.


Quando A * 0, temos x. xX(. e a equação 11.5 tem duas raízes distintas.

169
11.4 — Teorema Fundamental da Álgebra

Conforme vimos nos dois itens anteriores, as equações algébricas degraus 1 e 2


sempre têm solução. Mas o que acontece com as equações de grau maior que 2 ?
Isto é respondido pelo teorema fundamentai da Álgebra (T.F.A.).'

Toda equação algébrica, de coeficientes complexos e grau rraíor que


zero, admite pelo menos uma raiz complexa.

O que equivale a dizer; todo polinõmio de coeficientes complexos e grau maior que
zero admite pelo menos uma raiz complexa.
Este teorema foi demonstrado pela primeira vez, de modo satisfalório, em 1798,
por Gauss. em sua tese de doutorado [foi o próprio Gauss que o chamou de
teorema fundamental da Álgebra”). Antes de Gauss, outros matemáticos (como,
por exemplo, dAlemberl e Euler) apresentaram demonstrações do teorema, mas
Gauss mostrou que todas elas eram incorretas (apesar disso há livros que chamam
c T.F.A. de ‘teorema de cTAlembert"). Como a demonstração de Gauss usa
conhecimentos que estão acima do rível deste livro, vamos admitir o teorema sem
demonstrá-lo.

Teorema de Abel

O T.F.A. nos diz que toda equação algébrica de grau não nulo tem solução; no
entanto, não nos diz como achar essa solução. Quanto ás equações de primeiro e
segundo graus, já sabemos como resolvê-las. Para as equações de terceiro e
quarto graus, conhecem-se fórmulas desde o século XVI: porém, tais fórmulas em
geral são de aplicação demorada e não serão usadas neste livro (uma fórmula para
a equação do terceiro grau foi mencionada ro capitulo 1 deste volume). Para as
equações de grau maior que 4, o matemático norueguês Niels Henrik Abel
(1802-1629) demonstrou que é impossível estabelecer fórmulas gerais de
esolução envolvendo operações algébricas com os coeficientes. Assim, a nossa
situaçao até o final deste livro é a seguinte: só sabemos resolver, no caso geral,
equações algébricas do primeiro e segundo graus. As equações de grau superior a
2 serão resolvidas em alguns casos particulares, quando for possível usar certos
artifícios, ou forem reconhecidas informações sobre as raizes, aplicando-se
teoremas que apresentaremos adiante.

Dais artifícios usados com frequência serão a fatoração do polinõmio (quando a


conseguirmos) e a mudança ce variável, como fizemos em alguns exemplos do
capitula 5.

11.5 — Teorema da Decomposição

Consideremos um polinõmio P(x), de coeficientes complexas e graun > 1, tal que:


P(x) - anxn + an_1x'1-1 +... + a1x + aD

170
Pode-se derranstrar (faremos isso no item seguinte) que P(x) pode ser
decomposta do seguinte modo:

ân<n + a..J1 1xn 1 + ..,+ a1x + aú =aí1(x-x1)(x - x2)...(x-x J | (11.7)

ande os números complexos x1d x2 xn são as n raízes do polinõmio P(X), isto é.


são as n raízes da equação algébrica P(x) = 0. As fórmulas 11.4 e 11.6 são casos
particulares da fórmula 11.7.
É fácil perceber que a decomposição 11.7 é única (a menos da ordem), isto é. os
números x,, x3,„.. xn são as únicas raizes de P(x). pois nenhum outro número
diferente de xld x2...., xn anularia o "lado direito'1 de 11.7, ao ser colocado no lugar
de x.
Portanto, todo polinõmio de grau n £ 1 tem n (e apenas n) raizes, que podem ser
reais ou imaginárias, não sendo, contudo, necessário que elas sejam todas
distintas: podemos ter algumas (ou todas) iguais. Se na decomposição 11.7 um
certo fator;
x-xt
aparecer m vezes, diremos que o número Xi ê raiz de multiplicidade m. Se um
certo fator x - X[ aparecer apenas uma vez, diremos que x, é uma raiz simples (ou
raiz de multiplicidade 1],

Exemplos

a) Consideremos o polinõmio P(x) = 2x3 - 3x2 + 8x-12 e vamos tentar deter­


minar suas raizes. Neste caso, "por sorte" (Nem sempre teremos essa sorte),
podemos fazer uma fato ração por agrupamento:
2x3 -3x2 + 0x-12 = 0 « x2(2x - 3) + 4(2x - 3) = 0 w
(2x-3)(x2 + 4) - 0 c=> 2x - 3 = 0 cu x2+4=0w
2
» x = — ou X = ±2i
3
3 -
Portanto, as raízes de P(x) sao os números —, 2i e -2i. O polinõmio, sendo

de grau 3 apresenta 3 raízes, as quais (neste caso) são todas distintas. Temos,
então, de acordo com o teorema da decomposição:

2x3 -3x2+ Sx-12 = 2Íx--l(x - 2i)(x + 2i)


r k 2)

b) Consideremos um polinõmio P(x)d que após ser decomposto, fica:


P(x) = 5(x-4)(x-4)(x-2i)
isto é;
P(x) - 5(x-4)2(x - 2i)
171
Se fizermos as multiplicações, obteremos:

P(x) = 5x3 -(40 + 10Í)xz + (80+ 80i)x-l5QÍ = 5(x-4)3(x-2i)

Na decomposição, vemos que o fator x - 4 aparece 2 vezes e, portanto, o


número 4 ê raiz de multiplicidade 2 (raiz dupla), Já □ fator x - 2i aparece
apenas uma vez, assim o número 2i é raiz simples. No total, P(x) tem 3 raizes.
4 (dupla) e 2i (simples).

c) Sendo P(x) = 7(x -9)(x - 3)2(x - 4)s{x + 6)3, temos:


9 é raiz simples
3 é raiz de multiplicidade 2 (raiz dupla)
4 é raiz de multiplicidade 5
- 6 é raiz de multiplicidade 3 (raiz tripla)
P(x) possui 11 raizes; portanto, ê um polinômio de grau 11.

d) Consideremos P(x) = 16x3(x-2)(x + i)4(x - i)5, Observando que;

X3 =(x-ü)3

temos:
0 é raiz de multiplicidade 3 (raiz tripla)
2 é raiz simples
— i é raiz de multiplicidade 4
i é raiz de multiplicidade 2 (raiz dupla)
Assim, p(x) tem raizes 10 raízes; portanto, é um polinômio de grau 10.

11.6 “ Demonstração do Teorema da Decomposição

Consideremos, então, o polinômio P(x), de coeficientes complexos e grau


n ã 1, tal que:

P(x) = anxn + a ■1* + an


De acordo com □ T.F.A., P(x) admite pelo menos uma raiz complexa.
Representando essa raiz por Xi, P(x) deve ser divisível per x — Xi, isto é:
P(x) = (x-x1)>P2(x) (I)
onde Pi(x) é um polinômio de grau n — 1, cujo coeficiente dominante é an.
Supondo n-1 à 1, pelo T.F.A. □ polinômio Pi(x) admite pelo menos uma raiz
complexa x?. Assim, P-i(x) é divisível por x - xz, isto é:
Pi(x) = (x-x2)>P2(x)
ande pj(x) e um polinômio de grau n - 2, cujo coeficiente dominante é an
Substituindo em (I), temos:
P(x) = (x-x1)(x-x2)>P2(x) (lí)

172
Procedendo de modo semelhante, pelo número necessário de vezes,
chegaremos a:
P(x> (x-x1)(x-x2),..(x-xri)>Pn
onde Pfl ê um polinômio de grau n - n, cujo coeficiente dominante é an. Como
n - n - 0, concluímos que Pné o polinômio constante an e. portanto:

P(x) = an(x- x^x- x2) ,.(X-Xn)

Exercícios Resolvidos

11,1) Resolva a equação 2x3 -3x2 - 3x + 2 = 0, sabendo que uma de suas raízes
ê igual a 2.

Solução

Sendo P(x) = 2x3 — 3x2 —3x +2 = 0, o número 2 é uma das raízes de P(x);
portanto. P(x) ê divisível por x - 2:
P(x)-(x-2)>P1Cx);
Efetuando a divisão de P(x) por x- 2, obtemos P»(x);

2 I 2 -3 -3 2 P^x) = 2x2 -r x-1


T 2 1 -1 0

Portanto, 2x3 - 3x2 - 3x + 2 = (x - 2)(2x2 + x -1).

Assim:
2x3 - 3x2 - 3x + 2 - 0 » (x - 2)(2x2 + x -1) - 0 I x-2 = 0
ou
2x2 + x -1 = 0

A equação x — 2 = 0 nos fornece a raiz 2, que já conhecíamos: portanto,


para obtermos as outras raizes, resolvemos a equação 2x2 + x -1 = 0, cujas

raizes são -1 e Então, as raizes da equação dada são 2,-1 e isto

ê, o conjuntc^solução é:

S = {2;-1; ~
I ’ 2J
Para completar, podemos ar a decomposição de P(x) em fatores do primeiro
grau:
1
2x3 -3x2 -3x + 2 = 2(x-2)(x+1) x---- = (x — 2)(x+ 1)(2x - 1)
2

173
11.2) Decomponha a polinâmic P(x) = 3x2+(-1 0 - 3i)x2 + (9+7i)x + (-2-2i) em
fatores do primeiro grau, sabendo que uma de suas raízes é a número 1 +i.

Sc l uçã o

Se 1 + i ê raiz de P(x), este é divisível por X - (1 + í) :

P(x>[x -(1 + ijJP^x) = (x - 1- ij^x)

Efetuando a divisão de P(x) por x—(1 + i) obtemos Pd(x):

1+i I 3 (-10-3i) (9 + 7Í) (-2 - 2i)


3 -7 2 0

P-i(x) = 3x2 - 7x + 2

Para obtermos as raizes que faltam, resolvemos a equação 3xz-7x +

+ 2=0, cujas raizes são 2 e —. Assim, as três raízes de P(x) sãa


3

e y; portanto, a decomposição é:

1
3X3 + (-10 — 3Í)X2 + (9+7i)x+(—2 — 2i) = 3(x - 1 - i)(x - 2) x----
3
= (x - 1-i)(x-2)(3x-1)

11.3) Decomponha o polinómio P(x) = x4-flx3 +17x2 + 2x —24. sabendo que


duas de suas raízes são os números 2 e 3,

Solução

Já que 2 e 3 são raizes de P(x), podemos escrever:

P(x) = (x - 2)(x—3)Q(x)
Dividindo P(x) por x — 2, obtemos Qi(x) e dividindo Q-i(x) por x — 3, obtemos
Q(x):
P(X) (X —3)>Q(x) Q-i(x)
-Q(x)
x-2 Q1(x) x —3

Fazendo sucessivamente as duas divisões


2 1 -8 17 2 -24
7 ^6 T 12 c'
1 -3 □
Q(x) = x2-3x-4

174
2
Para obtermos as raízes que faltam, resolvemos a equação x - 3x - 4 = 0,
cujas raízes são 4e~1. portanto, o polinõmío P(x), de grau 4, possui 4
raízes distintas que são os números 2. 3, —1 e 4, podendo assim ser
decomposto da seguinte modo:
X4 -8x3 + 17x2 + 2x - 24 = (X - 2)(X - 3)(x+1)(X - 4)

114) Decomponha o polinõmio P(x) = x4 + 6x3 + 10x2 + Sx + 9, sabendo que o


número - 3 é ratz dupla.
Solução

Se-3 ê raiz dupla de P(x), podemos escrever P(x) = (x + 3)(x + 3)>Q(x) isto
é, P{x) = (x+3)2 >Q(x).
Para obtermos Q(x), fazemos duas divisões sucessivas por x +■ 3:

-3 1 6 W £ £
-3 T 1 3 □
1 0 1 0

Q(x) = X2 + 1

Para acharmos as outras raízes, resolvemos a equação x 2 + 1 = 0, cujas


raízes são i e - i. Assim, polinõmío P(x), de grau 4. possui 4 raízes (nem
todas distintas), que são os números — 3(raiz dupla), i e — i. Portanto, P(x)
pode ser fator ado do seguinte modo:

x4 + 6xs+ 1 Ox2 + Sx+ 9 = (x + 3)2(x-i)(x + i)

11.5) Sabe-se que uma das raizes do polinômio P(x) = x5 - 5x4 + 7x3 — 2x2 +
+ 4x -8 é o número 2 Verifique sua multiplicidade.

Solução
Vamos lentar fazer várias divisões sucessivas por x — 2, até obtermos uma
divisão não exata:

2 1 -5 7 -2 4 8
2 1 -3 1 0 4 0
2 1 -1 -1 -2 0
2 1 1 1 0
1 3 0
Conseguimos fazer 3 divisões exatas (a quarta divisão nas deu resto 7).
partantc o número 2 é raiz de multiplicidade 3. Podemos, então, escrever:
P(x) = (x-2)3>Q(x)

175
onde O(x) é o quociente da terceira divisão:
Q(x) - x2 4-X + 1

Assim. P(x) = (x-2)3(x24-x + 1)

Se quisermos, poderemos obter as outras raizes resolvendo a equação


x2 + x + 1 = □.

11.6) Determine os valores de a e b de modo que o número — 4 seja raiz dupla


do polinõmio P(x) = x4+ 8x3+17x2+ ax + b, Em seguida, determine as
outras raizes.
Solução
Já que — 4 é raiz dupla de P(x), temos:

P(x) = (x + 4)2 >Q(x)

Isto significa que, fazendo duas divisões sucessivas por x +• 4, essas duas
divisões dever ser exatas (porém, a terceira não pode ser exata):

2 £ 17 a b
-4 1 4 1 (a-4) (b-4a + 16)
1 □ 1 (a-S)
1 17

b-4a + 16 = 0
Devemos ter
a-8 = 0

Resolvendo esse sistema, obtemos a = ô e b - 16. (Observemos que a


terceira divisão não foi exata: o resto foi igual a 17.) Temos, então:
P(x) =(x + 4)2>43(x)
onde Q(x) ê o quociente da segunda divisão:
Q(x) = x2 +1

Para obtermos as raizes que faltam, resolvemos a equação x2 +1 = 0, cujas


raizes são i e - i. Assim, P(x) possui 4 raizes: - 4(dupla), i e - i Podemos,
então, fatorar P(x) do seguinte modo:

P(x) = (x + 4)2(x - i)(x + i)

11.7) Determine os valores de a, b, c e d de modo que o número zero seja raiz


tripla do polinõmio:
P(x) = 2x5 -Sx4 + ax3 + (b-9)x2 +(3c -7)x + d

176
Solução
Se o número D é raiz tripla de P(x), temos:

P(x) = (x - O)3 >Q(x) = x3 xQ(x)

onde Q(x) é um polinõmio que não se pode anular para x = 0.


Poderiamos resolver este problema de modo semelhante ao empregado no
exercício anterior, efetuando divisões sucessivas por x - 0 No entanto,
neste caso, ê mais fácil perceber que devemos colocar x3 em evidência e,
para que isto ocorra, os termos de P(x) que possuem grau inferior a 3
devem ler coeficientes nulos:

b-9 = 0
3c —7 = 0
d=0

Obtemos, então, b = 9, c = — ed=0. Ao mesmo tempo, devemos impor


3
que o coeficiente de x 3 seja diferente de zero, pois, em caso contrário,
poderiamos colocar x^ em evidência; ai o número zero não seria raiz tripla,
mas sim de multiplicidade 4. portanto, a resposta do problema é:
7
a a 0, b = 9, c = -, d - 0
O

11.8) As raizes do polinômio P(x) - X3 -1 2x2+45x - 54 sac a (dupla) e b.


Sabendo que b = 2a, determine os valores de a e b.

Solução
O polinõmio P(x) pode serfatorado do seguinte modo:
P(x)=(x-a)2(x-b)

Efetuando as multiplicações, obtemos:


P(x) = x 3-(2 a+ b)x3 + (2ab + a2)x-a2b
mas temos também:
p(x)= x3-12x2 + 45x- 54
Assim, por identidade de polinâmios, devemos ter:

2a + b = 12
2ab + as = 45
a?b - 54

Resolvemos este sistema, obtemos a - 3 e b - 6.

177
■11.9) Sabe-se que o número 1 é raiz da equação (x3 - 5x? + 7x-3)4 = 0.
Determine todas as raízes dessa equação, com as respectivas multipli-
cidades.

Solução

Seja P(x) = x3 - 5x2+7x-3. O número 1 deve ser raiz de P(x) e, portanto,


este ê divisível por x — 1;

P(x) = (x-1)>Q(x)

Efetuando a divisão, obtemos Q(x):

1 2 -5 7 -3 Q(x) = x?-4x+3
1 -4 3 0
As outras raízes de P(x) são as raizes de Q(x). Resolvendo a equação
x2 -4x + 3 = 0, obtemos as raízes 1 e 3. Assim, as raizes de P(x) sfio ds

números 1 (dupla) e 3;
P(x)= (x-1)2(x-3)
Portanto:

(x3 - 5x2 +7x-3)4 = 0 í(x-1)2(x-S)]4 =0 « (x-1)a(x-3)4 =0

Então, a equaçac dada possui 12 raízes:

1 (multiplicidade 8)
3 (multiplicidade 4)

11.10) Determine m de modo que o número 2 seja raiz da equaçao;


x3 +{2m - 1)x2 + (5 - m)x+ 7 ~

Solução

Substituindo a variável x pelo número 2, temos:


23 + (2m — 1)x2 + (5 - m)x + 7 = 0

Resolvendo esta equação, obteremos m — —

11.11)Sabe-se que o número —2 é raiz da equação x3 + (m+1)x2 +(2m+1)x +


+ 6 = D, onde m e R. Determine m de modo que todas as raízes da equação
sejam reais.

173
Solução
Tentemos inicialmente fazer o que fizemos no exercício anterior, isto ê,
vamos substituir x por — 2:
(-2)3 +(m + 1)(-2)2 + (2m+1)(-2)+6 = 0
Porém, ac simplificarmos esta última equação, obtemos:
0 = 0
Isto significa que, para qualquer valor de m, o número — 2 ê raiz da ecuação
dada. Façamos então:
P(x) = x3 + (m + 1)x2 + (2m + 1)x + 6
Já que “ 2 é raiz de P(x), este ê divisível por x + 2:
P(x) = (x + 2)xJ(x)

Efetuando a divisão, obtemos Q(x):

-2 1 (m + 11 (2m + 1) +6
1 m -1 3 O

Q(x) = x2 + (m-1)x + 3
O polinõmio Q(x) ê do segundo grau e tem coeficientes reais. Portanto, para
que todas as suas raizes sejam reais, devemos ter ASO:
(m-1)2 -4(3) > 0
Resolvendo esta ínequação, vem:
m <1-2^3 ou m>1+2>/3

11.12}Dê um polinõmio de grau 3 cujas raizes sejam as números 2 (simples) e 4


(dupla).

Solução
Seja P(x) o polinõmio procurado:
P(x) = aax3 +a2x2 + a^ + ap
Já que conhecemos suas raizes, podemos escrever:
P(x) = aa(x-2)(x-4)a = a3(x3 -10x2 + 32x - 32)

onde a3 * 0. Podemos dar infinitos valores para aa e, assim, há infinitos


polinõmios que satisfazem as condições dadas no enunciado do problema;
mas. coma a problema pediu apenas um polinõmio, pocemos dar um valor
qualquer para ay Fazendo, Por exemplo, a3 =1, uma resposta para este
problema ê P(x) = x3 - 1 0x2 + 32x - 32. No capítulo 14 (exercício 14.7)
veremos um outro modo de resolver este problema.

179
11.13) Seja P(x) um poiinõmio de grau 3 cujas raízes sao os números 2 [simples) e
4 [dupla). Determine P(x) em cada um dos casos a seguir:
a) P(6) = —SO

b) a soma dos coeficientes de P(x) é igual a 45

Solução
a) Esle problema é semelhante ao anterior, mas agora temos uma condição
adicional' P[6)=-0O. Do mesmo modo que no problema anterior, temos
então:

P(x) = a3(x — 2)(x —4)2

Substituindo x pele número 6, temos:

P(6)= a3(6-2)(6 -4)2 = -00

o que nos dá a3 = —10, Assim:

P(X) = -1 0(x - 2)(x - 4)2 = -1 0x3 + 100x2 -320X + 320


b) Do mesmo modo que no caso anlerior, temos:
P(x) = a3(x - 2)(x - 4)2
A soma dos coeficientes de P(x) deve ser igual a 45. Mas, de acorda
com o que vimos no exercício 6.2, a soma dos coeficientes de P(x) é
igual a P(1). Assim:
P(1) = aa(1-2)(1-4)?=45
donde a3 = -5 e P(X) =-5(X-2)(X-4)2 =-5x3+ 5üxz-160x + 160

Exercícios Propostos

11.14) Decomponha o polinõmio P(x) = (2 + i)x2 + (-3 + i)x + (1- 2i).

11,15) Um polinõmio P(x), após ser decomposto, ficou;


7
3 2Í
P[x) = j3[x-5)(x+6)3(x-V2 + 4)2 X------
4 5
a) Qual é o grau de P(x)?
b) Quantas raizes tem P(x)?
c) Quantas raizes distintas tem P(x)?
d) Diga quais são as raizes de P(x), com as respectivas multíplicidades.
e) Se fizermos todas as multiplicações e reduzirmos os termos semelhan­
tes, qual será o coeficiente do termo em x13 ?

100
11.16)Resalva a equaçao x3-x 2 -14x + 24 = 0, sabendo que uma de suas raizes
é igual a - 4.

11.1?)Decomponha o polinâmio P(x) = 2x3 — 9x3 + 14x —5, sabendo que uma das
1
suas raizes é igual a —.
2

11.10) Decomponha o polinâmio P(x) = x4 - 5x3 + 5x2 + 5x - 6, sabendo que duas


de suas raizes são os números 2 e - 1.

11.19)Decomponha o pollnõmío A(x) = x 4 + 4x3 +1 3x2 + 36x + 36, sabendo que o


número - 2 é raiz dupla.

11.20)Decomponha o polinâmio B(x) = 3x3-7x2 + (5 + 6t)x “(1 + 2i)b sabendo que


uma de suas raizes é o número 2—i.

11 21) Decomponha o polinâmio P(x)-x 3-5x3-Sx+48, sabendo que uma de


suas raizes é igual a 4.

11.22) 0 número-4 é uma das raizes da equaçao x5 +12x4 + 47x3 + 52x2 - 40x
-64 = 0. Verifique sua multiplicidade.
11.23) Determine os valores de p e q de medo que c número 5 seja raiz dupla da
equação:
x4-10x3 + 24x2 +px + q = 0

11.24) Determine os valores de r, s e t, de modo que o número zero seja raiz dupla
da equação:
7x4 -5x3 +(r-6)x2 +(3s-2)x + (t-9) = 0

11.25) Determine k, de modo que o número 3 seja raiz da equaçao


x4 -kx3 + (k-1)x2 -18-0

11.26) Sabe-se que o número 3 é raiz da equação x33 + (k - 5)x2 + (4 — k)x +-


+(6-6k)-0, onde k é real. Determine k, de modo que todas as raizes da
equação sejam reais.
11.27) Seja P(x) um polinâmio de grau 4 cujas raizes são i (dupla), — 2 e zero.
Determine P(x), sabendo que P(-1) =- 10i.

11.2 8) Deter mine o polinómio P(x), de grau 3, cujas raízes são 4, —2 e 3, sabendo
que a soma dus coeficientes é igual a 6.

11 29) Resolva a equação x 3-2x2 - x+ 9 = 3x2 + x — 1 5, sabendo que uma de


suas raizes é o número 4.

181
11,30) Resolva a equação xÈ -3x4 + 26x3 - 24xz = 0, sabendo que uma de suas
raizes ê o número 3.

1 x2 + 3
11 31) Resolva a equaçao ------- = sabendo que uma de suas raizes ê
x-1 5x2 -3x + 3
c número 2.

1 1.32)Resolva a equaçao x 21 — 2x - 4 — 0, sabendo que uma de suas raizes ê o


número 2.

11.7 — Alguns Artifícios

Neste item, examinaremos alguns exercícios que, além da teoria jâ vista,


necessitarão de alguns artifícios para serem resolvidos.

Exercícios Resolvidos

11.33)Obtemos as raizes e faça a fatoraçao do palinâmio:


P(X) = xs -2x4 + 6x3 -12x2 -27x + 54

Solução

Aqui, "por sorte", conseguimos fazer uma fatoração por agrupamento:

P(x) = x£ -2x4 + 6x3 -1 2x2 -27x + 54 =

= x4(x- 2)+ Sx3(x- 2)-27(x- 2) = (x- 2)(x4 + 6x2-27)

Assim:

X-2 = 0 (D
P(X) = 0 (x-2)(x4 + 6x2 -27) = 0 ou
x4 + 6x?-27 = Q (li)

A equação (I) nos dá a raiz 2 e a equação (II) nos dã raizes


V3,-V3, 3í e -31 Portanto, o canjunto-soluçãc da equação P(x) = 0 é:

S ={2;Vã; - Vã; 31; -31}


e a fatoração de P(x) é:
P(x) = (x-2)(x-Vã)( J3)(x- 3i)(x+ 3i)

182
1 *1,34) Decomponha o polinómio P(x) da exercício anterior, no campo real.

Solução
Neste case, não devemos considerar as raízes imaginárias. Assim, ao
resolvermos a equação x4 + 6x2 - 27 = 0, temos:

x2

Isto ê, x2 - 3 ou x2 = —9, Portanto:


x
x44 + = (x
-27 =
6x27 -27
+ 6x (x 2 -3)(x2 + 9) ~(x+ V3)(x- V3)(x 3 + 9)
Temos, então:
P(x) ={x-2)Çx-V3)(x + V3)(x2 +9)
Um outro modo de resolver esse problema é, primeiramente, fazer a
decomposição no campo complexo:
P(x) = (x-2)(x~V3)( V3)(x +3i)(x -3i)
Para, em seguida, fazer a multiplicação dos termos em que aparecem raízes
imaginárias:
(x + 3i)(x—3i) = x3 +9

11.35) Decomponha o polinòrnio P(x)=x4+1 em fatores do primeiro grau.

Solução
1“ modo
Vamos achar as raizes de P(x]:
P(x) = 0 <=> x4 +1 = 0 <=> x4 = -1
Portanto, as raizes de P(x) são as raízes quartas de -1;
Vã . Vã
r.= — +1—- r3 =
Vã -i. —
Vã■
1 2 2 2 2
Vã . V2
r-? =--------- 1-1----- r4 =

—•

-i - -
2 2 2 2 2
Assim, temos:
P(x) - (x -F^íx - r2)(x - r3)(x -r4)
Isto é:

V2 72 .72 ( 72 .72 72 , 72
x------- +1 —
x4 + 1 = x------- X + •-------- 1—~ XH----- + I------
2 2 2 l 2 2 2 2

183
2” modo
Podemos usar o artificio de somar e subtrair um mesmo termo, de modo a
cair em algum caso conhecido de fatoração:
X4 + 1 = x4 + 2x2 + 1-2x2 = (x4 + 2x2 + 1)-(2x2) =
- (x2 +1)2 - (V2x)z = (x2 +1 + 7ãx)(x2 +1- Víx)

Se o problema tivesse pedido a fatoração no campo real, deveriamos parat


aqui. Mas corre foi pedida a fatoraçáo em termos do primeiro grau, deverncs
continuar determinando as raizes de x2+ 1 + V2x e x2 + 1 - V2x. que sãa:

J2 + & J2
i' —
2 2
2 2
e
V? i^ /2 Jz
2 ’ 2 ' 2 2

Estas são as mesmas raizes que obtivemos pelo primeiro moda


obviamente, a resposta será a mesma.

11.36) Resolva a equação x4 + x2z +1 = 0.


Solução
1o modo

Podemos fazer a mudança de variável y = xz, Com isso temos:

X4 + X2 +1=0 c=> y2 + yi1 = 0

Resolvendo esta última equaçao obteremos as raízes yd e y2 Em seguida.


resolvemos as equações:

j x2 = y1r de raízes x1 e xz
[x3 = y2, de raizes x3 e X4

Finalmente teríamos:
P(x) = x4 + x2 +1 = (x^x^fx - x3)(x- x3)(x-x J
No entanto, neste caso, há um modo mais rápido, que veremos em seguida.
2o modo
Temos:

X4 + X2 +1 ~ X44 +x22 + X2 + 1-X2 =(x4 +2x2 + 1)-(x)3 =


= (XZ + 1)2 -(X)2 =(x2 +1+x)(x2+1- X)

184
Assim:

x2 + 1 + x = 0 (I)
x4 x2 + 11= 0 (x2 + 1 + x)(x3 + 1- x) - 0 <=> ou
X2 + 1-X = □ (lí)

1_+ij3 1-iVs
A equação (l) tem raízes — e
2 2
1 + 175 1-i>/3
A equação [II) tem raizes e
2 2
Pnrlanta, o conjunto-soluçáo da equação dada é:

s _ [-1 + 175 .-1-173 .1_+i>/3 .1 -i73 |


2 ' 2 ''2'2

11.37)Resolva a equação x2 + (2+75) 2 J5 = 0.


Solução

A = (2 + 75)2 - 4(275)^4 + 475+ 5-= 4 - a7& + 5 = (2- 75)2


Assim’

X -
(2 + 75 ) ± 7Ã
2

donde: x, = 2 e X^ — i/s .

11.38) Resolva a equaçao x 4 + X3 + X 2 +X + 1 = 0.

Solução

A sequência:
(1; x; x2; x3; x4)

é uma progressão geométrica (PG) de razão igual a x. Assim, usando a


formula da soma dos termos de uma PG (veja capitulo 6 do volume 2 desta
coleção), temos:
3 2 < X5 - 1
x4 + X + X + 1 ----------
X-1
É obvio que a igualdade anterior só ê válida para x * 1: porém, por simples
verificação, concluímos que o número 1 não é raiz da equação dada. Assim:
x5-1 = 0 xfi = 1
x5-1
X* + X 3 2 + x + 1 = 0 <=? e e
x —1
x * 1 x * 1

185
Portanto, as raizes da equação dada sao as raízes quintas do núftie-
rci 1, com exceção do número 1:
6n 6rt
üj1 - cos+1 sen to, = cos— +i sen—
3 5 5
4ir 4it Bk Síi
[Ut ss cos-------hi sen — eu. = cos —+ i sen——
2 5 5 4 5 5
Se quisermos, poderemos indicar essas raízes de modo mais ‘compacto’,
lembrando que:

2it Sir 2rt


CDS------ — COS------- sen— = -sen —
5 5 5 5
4rt Git 4jt 6jt
COS------ = COS — sen—— = — sen—-
5 5 5 5

Assim podemos dizer que as raízes procuradas sacr


2n . 2
2tc
tc 4?r 4k
cos- ± iisen
sen— e cos— ± i sen —
5 55 5 5
n V5-1
No capitulo *1 do volume 3 desta coleção, vimos que sen— — 4 ”
.fl parar
10
2rc 2?c
dai, se quiséssemos, poderiamos obter oos —, sen — sen— e cns —.
5 5 5 a
Poderiamos, também, observar que:
2 3 4
([>2 " CO-j ! = U?i p ÍU w

Fazenda - to = cos-^- + isen^, poderiamos dizer que □ conjunto-solu-

ção da equaçao é:
S - (ül U>2, (O3, C1J4}

Este exercício será resolvido de outro modo no capítulo 16 (exercício 16.3).

11.39) Resolva a equação (z - 1)6

Solução

É fácil verjfícar que o número 1 não é solução da equação (isto é, Z * 1).


Portanto:
6 6
(z-1)6 = (z-3)6 ^~3) = 1 « z-3
=1
(z-1)6 z-1

Sejam ll>.■a ■ ■ ÜJ2, tü3. “4. új5, as raízes sextas do número 1 (corn
7,
<ÜQ = 1, CJ1 = <i), új2 . = tu , = oj4, cü5 — tuÊ6).
= '-' Sendo íDr uma delas,
Z-3
lemos:
z-1

1 86
úJj - 3
Isolando z, obtemos z =
ÚJj -1 ’

Nesta última equação, vemos que * 1 e, portanto, a raiz = 1, não ser­


ve, Sendo, então:
n . te 1 ij'3
tu . (ü = cos — +i sen — = — +------
3 3 2 2
a conjunta -solução é:

ü)- 3 , cn3 -3 új3


___ -3 (f/ — 3 m5 -3
s- ou— 1 ' <ú2 -1 cíi3 -1 a? -1 <n5-1

11.40] Mostre que os pontos representativos das raízes da equação (z - 1) =


= (z - 3)n estão sobre uma reta.

Solução
(2-í)6 = (z-3)6 =3 | z — 1 ] = I z - 3 1

De acordo com o que vimos no capítulo 3 deste volume a equaçao


|z-1| = |z -3| representa a reta de equação x = 2.

Assim, as raízes da equação (z-1)G = (z-3)B6 sao representadas por pon­


tos (apenas alguns) que estão sobre a reta de equação x = 2.

Solução alternativa:

Lembrando que "módulo" pode ser interpretado geometricamente como


"distância", os complexos z que satisfarão a equação |z-11 = ]z-3] são
aqueles cuja distâncias aos complexos 1 e 3 sejam iguais. No plane de
Argand-Gauss, os complexos que estão equidistantes de 1 e 3 certamente
encontram-se sabre a reta mediatriz dos portos (1,0) e (3.0). isto é. a reta
x — 2. Assim, as 5 soluções da equação proposta encontram-se sobre essa
reta,

11 41) Resolva a equaçao x 4-2x3+21x2-32x + 80 = 0, sabendo que uma de


suas raizes é um número imaginário puro.

Solução

Seja ai (com a e TS1') a raiz imaginária pura da equação. Fazendo a subs-


tituiçáo, temos;
(ai)4 - 2(ai)a + 21(ai)2 -32(ai) + 00 = 0
donde:
(a4 -21a2 + 80) + (2a3 -32a) i = 0

187
Ficamos, enlao, com as equações:
|aJ - 21a2 + 90 = 0 (I)
e
2aa -32a = 0 (II)
I a equação (I) nos dá a - ± 4 ou a = ±7õ.
[a equação (II) nos dá a = 0 ou a = ±4.

Para satisfazer simuJtaneamente as duas equaçces, ficamos com a = ±4.


Portanto, a equação dada admite duas raizes imaginárias puras: 4i e -4i.
Dai concluímos que o polinómio P(x) = x4* -2x3 + 21x3-32x + SO pode ser
fatorado do seguinte modo:
P(x) - (x-4i)(x + 4i)>A(x)
xa+16
2 + 16 (ou fazendo uma divisão por x-4i e, em seguida,
Dividindo P(x) por x^
outra por x + 4i), obtemos A(x) - x3 -2x + 5, cujas raizes são 1 + 2i e 1-2i
Assim, o conjuntp-solução da equação dada é:
S = {4r; — 4i; 1-2i; 1-2í}

11 .42) Consideremos o polinómio P(x) = xn —1, com n > 1, cujas raizes sáo
n, í2. rj rn. Sabendo que r-| = 1, calcule □ valor de.
(5-r2)><5’r3)...{x-rn).
Solução

Sendo P(x) - (x-1)(x-r2)(x-r3)...(x-rn). temos:

= (x-r2)(x-r3)...(x-rn)
A(x)
Percebemos, então, que:
(5-r2)(5^r3)...(5^rn) = A(5)
Fazendo a divisão de P(x) por X —1. obtemos:

A(x) = x r-2 + xr>-3 +_ + x2 + x+1

Dai temos:
A(5) = 5*1*1 + 5n“2 + x"-a +... + 52 +5 + 1 (I)
O "lado direito" da igualdade (I) é a soma dos termos de uma progressão
geométrica de n termos e razão igual a 5, Portanto:
5n -1
(S-r^-rJ.. (5-rn) = A(5) =
4
163
11.43] Decomponha o polinõmio P(x) = Xa + (-5 - 2i)x2 + (7 -i-7i)x + (-2 - õi), saben­
do que ele tem uma raiz real.
Solução
Seja a a raiz real de P(x). Devemos ter, então, P(a) = 0, islo é;
a3 +(-5 - 2i)a2 + (7 + 7i)a +(-2 -6i) = 0
Efetuando as mulliplicações e agrupando OS termos de modo que fiquem
separadas as partes real e imaginária, obtemos:
(a3 -5a2 +7a-2)+(-2a2 + 7a -6)i = 0
Tanto a parte real como a parte imaginária devem ser nulas.

a3 -5a3 +7a- 2 = 0 (D
-2a2 +7a-6 = 0 (U)
3
Resolvendo a equaçao (II), obtemos as raizes 2 e — Por substituição,

podemos verificar que, destas raizes, apenas o número 2 satisfaz também a


equação (I). Assim, concluímos que a = 2, isto ê. a raiz real de P(x) ê o
número 2. portanto, P(x) ê divisível por x-2 .

P(x) = (x-2)>Q(x)

Efetuando a divisão, obtemos Q(x):

2 1 (-5-2i) (7 * 71) (-2-6Í)

1 (-3-2Í) (1 + 3i) 0

Q(x) - x2 + (-3-2Í)x+ (1 + 3i)

Podemos, em seguida, obter as raízes de Q(x) que são 1 + i e 2 + i Assim,


as raízes de P(x) são os números 2, 1 + i, 2 + i, e sua decomposição é:
x3 + (-5-2i)X2 + (7+7i)X + (-2-5i) = (x -2)(x - 1 - i)(x - 2 - i)

Exercícios Propostos

11.44)Decomponha o polinõmio P(x) = xs + 3x4 - 5x3 -1 5xz - 36x -1 08.

11.45)Resolva a inequaçao x 5 + 3x 4 -5x3 - 15x2 -36x- 108 < 0, no universo dos


reais.

11 48)Resolva a equaçao x2 +(2 — V3)x - 2^3 = 0.

139
11 47) ReSOlVa a equação X5 -x4 + x3 ^x2 + x-1 = 0.

11 48)Resolva a equação (z-3)5 = z5

11.49) Resolva a equação xfl - 5x3 +10x3 - 20x + 24 = 0. sabendo que uma de
suas raizes é um número imaginário puro.

11,50)Consideremos o polinômio P(X) = Xn-1, com n > 1, cujas raizes são r,, r2,
f3, ... rn. Sabendo que q =1, calcule aS valores de:

a) (3- r2)K3 - r3)x.. x(3 - rft) b) (1-r?)>(1 - - rn)

11.51) Decomponha o polinõmio P(x) — x3 + (~6-t-2i)x2 + (10- 9i)x + (-34-91),


sabendo que ele possui uma raiz real.
11.S2)A equaçao (x - uc)(x - J3)(x - y) + 5 « 0 tem raízes r, s e t Quais as raizes
da equação (x - r)(x - s)(x -1) - 5 = 0 ?

11.53) Consideremos uma equação algébrica de grau 20. Qual o número máximo
de raizes distintas que essa equação pode ter?
11.54) Consideremos dois polinõmios A(x) e B(x), ambos de grau n 2 1. Suponha­
mos que os valores numéricos de A(x) e B(x) coincidam para n + 1 valores
distintos de x: r0, rt, r2 rn. Demonstre que, neste caso, A(x) e B(x) sãn
idênticas,

.1.8 - Polinõmios de Mesmas Raízes

Consideremos dois polinõmios A(x) e B(x), de mesmo grau, n > 0 qua tenham as
mesmas raizes, com as mesmas muttíplícídades:
|A(x) = anxn + an_1x + ... + a,x + aa (0
[B(x)=bnxn + br,_1xn-' + , + b tX + b0 (II)
Sendo as raizes, vem1

fA(x) = an(x-r1)(x-r2)...(x-rn) OH)


|b(x) = bn(x -Qfx -r2)..,(x -r,J (IV)
Observando (III) e (IV), concluímos que:
A{x) ^=k
B(x)
isto é: A(x) = k>B(x) (V)
□ nde k ê uma constante não nula.

190
Substituindo (() e (II) em (V), temos:

arvxn +a •1* . + a.,* +aç = kbnxri + kbn_


+kb-|X + kb a
donde, por identidade de polinômio, concluímos que:
d, - k >bn
an_. =k>tin^

af = k ,
a n = k >b0
Em outras palavras, podemos dizer que os coeficientes correspondentes são
proporcionais.

Exemplo

Consideremos □ polínómio A(x) - 3x2 - 7x + 9 Multiplicando-o pela cons-


tanle k = 10, obtemos o polinômio B(x) = 30x2 -70x 4- 30, oue deve ter as mes­
mas raizes de A(x), com as mesmas multiplicidades.

191
Capitula

12 Raízes múltiplas

12.1 - Raizes Múltiplas e Derivadas — Teorema


Consideremos um pofinõmio P(x) de grau n > 1, que possui uma raiz c de
multiplicidade m > 1. Conforme vimos no capítulo anterior. P(x) poderá ser escrito
na femna:

P(x) = (x-c)m>A(x)

Com A(c) * 0, isto e não sendo c raiz de A(x). Seja P<i:i(x) o polinõmio derivado de
P(x). Vale, então, □ teorema (que será demonstrado no item seguinte):
í ‘
1c é raiz de multiplicidade m-1 de P(1>(x)| (12.1)

isto é, podemos decompor P('!(x) na forma:

P [1’(X) = (X - c) >B(x)
onde B(c) * 0.
Exemplas
a) Consideremos o palinõmio

P(x)= x& -4xJ’ +7x3 -7x33 -7x3 + 4x - 1

que pode ser fatorado do seguinte modo:

P(x) = (x-1)3 (x2 - x + 1)


Ã(x)

A derivada primeira de P(x) é o polinõmio:


P(1,(x) = 5x4 -16x3 +21x 2 -14x + 4

que pode serfatorado da seguinte maneira:

=(x) = (x-1)2X5xí - 6x + 4)
' B(X)

193
Observando que A(1) 0, B(1) 0, podemos dizer que:

1 é raiz de multiplicidade 3 de P(x)


1 é raiz de multiplicidade2 de P|J|’(x)

b) Consideremos novamente o poiinõmio P(x) do exemplo anterior:

P(x) = x5-4x4 + 7x3 - 7x2 + 4x-1

Conforme vimos, o número 1 é raiz tripla de P(x). Determinemos algumas das


derivadas sucessivas de P(x):

pW(x)= 5x4-16x3 + 21x2-14x + 4


Pí2)(x) = 20x3-48x2 +42X-14
Pí3j(x) = 60x?- 96x + 42

Aplicando-se várias vezes o teorema 12.1, podemos dizer que:

1 é raiz de multiplicidade 3 de P(x)


1 é raiz de multiplicidade2 de P^fx)
1 é raiz de multiplicidade 1 de P(!í(x)
1 é raiz de multiplicídadeO de P|Jh(x)

isto é, o número 1 nao é raiz de P(3,(x).

Embora nác tenhamos definido anteriormente, passaremos a usar a frase:


"k é raiz de multiplicidade 0 de P(x)"
com o seguinte significado:
”k não ê uma raiz de P(x)"
O exemplo b anterior ilustra uma Consequência do teorema 12.1:

Se o número c é raiz de multiplicidade m (com m £ 1)


do poiinõmio P(x), então será também raiz dos polinômios:
Pn(x), Pía!(x)J..,1PírtI-1)(x)
(12.2)
com multiplicídades respectivamente iguais a:
m-1, m-2,..., 1
__________ mas não será raizde Pfm)(xJ,

194
12.2- Demonstração do Teorema
Temos, então:

P(x) = (x-c)m xA(k)

Com A(c)^O. usando a regra da derivada do produto, vem:


-m(x - cp-1 >A(x) + (x c)™ >A^(x)-
= (x-c)m_1 >{m>A(x)+(x-c)A(1)(x)] = (x - xr >B(x)
B(x)

Portanto, para demonstrar que c é raiz de multiplicidade m - 1 de Pf1)(x), basta


mostrarmos que B(c) * D. De fato:

b(c) = |m >A(c)] = (c — c)A^(c)] = m xA(c)

Mas, como a(c) Q, concluímos que B(c) # 0.

Teorema
Pode-se demonstrar que:
Se o número c é raiz dos polinõmios :
P(x), P(1í(x), P(2)..... Ptm~1](x)
(12.3)
mas não é raizde P^(x), então será raiz de
multiplicidade m de P(x).

Observação: No capitulo 17 veremos um outro processo para a pesquisa de rai­


zes múltiplas.

Exercícios Resolvidos
12.1) Mostre qtie o número 2 é raiz tripla do polinômio'
P(x) = xs-5x4 + 7x3 -2xa + 4x-8
Solução
Este exercido já foi resolvido no capítulo 11 (exercício 11.5). Agora vamos
resolvê-lo de outro modo. Temos;

P(x) = xs -5x4 + 7x3 -2x2 + 4x-8


Pn){x)= 5x4 — 20x3 + 21x2 - 4x + 4
Pm(x)= 20x3 -60xz +42k-4
Pí3J(x) = 60x2 -120X + 42

195
Podemos verificar que P(2)=0, Pí1)(2) = 0, P(2)(2}=0 e Pt')(2) = 0

Assim, de acordo com o teorema 12,3, podemos afirmar que o número 2 é


raiz de multiplicidade 3 de P(x).

12,2) Resolva a equação x 3 — 7x2 + 15x - 9 = ü, sabendo que admita uma raiz
dupla (multiplicidade 2)r
Solução

Seja P(x)= x3 - 7x2 t 15x - 9. Seo número c é raiz de multiplicidade 2 de


P(x), será raiz de m ultiplicidade 1 de P^(x).

P(1,(x) - 3x2 -14X + 15

Determinemos as raízes de P(1)(x):

14 + TTs 5
3xz-14x + l5 = 0<=* x - ------------- c=» x - — ou X -3
e---------- 3
5
Portanto, c pode ser — ou 3. Fazendo as substituições (ou usando Briob
3
Rufíini), obtemos;

P e P(3) = 0
X''/

Assim, concluímos que c = 3, isto ê. a raiz dupla procurada é o número 3 e,


portanto, P(x) pode ser escrito:

P(x) = (x-3)2 >Q(x)


Determinemos Q(x):

_3 2 —7 15 -9
3 1
1 -1 0

Q(x)= x-1
Q(X) = « x~ 1 = 0 <-> x = 1

Portanto, o conjunto-soluçao da equação dada é;

S = {3;1}

196
12.3) Determine as raizes do polinomio P(x)=;x 3-(4 4-2i)x2 + (4 + 6l)x - 4í, saben-
do que ele tem uma raiz dupla.
Solução

Derivando P(x). obtemos: P(1>(x) = 3x2 - (8 + 2i)x + (4 + 61).

Podemos obter as raizes de Pí11, quesãol + i e


3
Uma delas será a raiz dupla de P(x). Fazendo as verificações obtemos:
Z í- ! \
P(i +-Í) = 0 e PÍ — 1 * 0
< 3 J
Assim, 1 + i é a raiz dupla de P(x) que procuramos:

P(x) =[x-(1 + i)]2 >Q(x)

Fazendo duas divisões sucessivas por x -(1 + i), obteremos Q(x):

1 +■ i 1 (^-2i) (4 + 61) — 4i

1 +1 1 (-3-Í) (2 + 2i) 0

1 -2 0
Q(x) = x —2
Q(x) = 0<-+x-2 = 0 ■*- x = 2

Portanto, as raízes de P(x) são 1+1 (dupla) e 2 (simples).

12.4) Determine as raizes do polinòmio P(x) s= x4 + x3 - 3x2 - 5x - 2r sabendo que


admite uma naiz tripla.
Solução
Temos, então:
p(1>(x) = 4x’+3x2 -6x-5
Pí21(x)= 12x2 + 6x-6

Sendo c a raiz tripla de P(x), deverá também ser raiz dupla de P!1|(x) e raiz

simples de Pí2)(x) As raizes de são -1 e Portanto, c pode ser igual a

-1 ou Fazendo as verificações, obtemos:

P(-1) = 0 ê P
2 *0
2

197
Assim, a raiz tripla procurada é c = —1:

P(x) = (x + 1)3xQ(x)

Fazendo três divisões sucessivas por x + 1, obtemos Q(x):

-1 2 1 3 -5 -2
-1 1 o -3 -2 0
-1 1 -1 -2 0
1 -2 0

Q(x) = x-2
Q(x) = 0 í=? x-2 = D <=> X = 2

Portanto, as raízes de P(x) sao —1 e 2.

12.5) Determine o valor de k de modo que o polinômio P(x)“X:d + x3 -3x2 +


+ kx- 2 admita uma raiz tripla. Depois determine as raizes.

Solução

A raiz tripla c de p(xj deverá ser raiz dupla de P^(x), e raiz simples de
Pt2’(x], porém não poderá ser raiz de P(:i)(x). Temos, então:

Pt1J(x) = 4x3 + 3x2-6x+k


Pf2)(x) = 12x2 + 6x-6
P(3\x) = 24X + 6
1
P<2>(x) = 0 « 12X2 + 6x-6 - 0 <=> x = -1 ou x = —
2

Portanto, os possíveis valores de c são — 1 e —


2
1a possibilidade: c - -1
P(-1) = 0 (-1)4+(-1)3~3(-1)2+k(-1)-2 = 0 wk=-5

P(X) = x44 -i-x33 -3xz — 5x—2


Assim:
Pn)(x) = 4x3-i-3x2-6x-5

Por substituição, verificamos que:


pC>(-1) = 0 e Pí3J(-1)^0
Portanto, uma possibilidade ê teremos:
P(x) = (x + 1)3>£}(x)

1S8
2 1 -3 -5 -2
1 □ -3 -2 0
1 -1 -2 0
1 -2 0

Q(x) = x-2
Q(x) — 0 <=> x = 2
Assim, para k = — 5, as raízes são — 1 e 2.

2* possibilidade: c — —
2
4 2
p[l =0 «
l2
t ? 41
P(X)= x 4 + x3-3x^ + — x-2
Assim: a
P£1l(x) = 4x3 + 3x2 -6x + —
â

Por substituição, verificamos que ^0 e, portanto, a possibilidade

1
g = — nao serve.
2
Em resumo, a resposta do exercício é:
ÍK = -5
/
[raízes ; -1 e 2

12.6] Determine os valores de men de modo que o polinômio:


P(x) - x4 - 5x3 + 6x2 +(m -1)x + (2n - 2)
admita uma raiz tripla.
Solução

A raiz tripla c de P(x] deverá ser raiz dupla de P^(x) e raiz simples de
Pt2)(x). porém não poderá ser raiz de P[3)(x], Temos então:

Pt1)(x)= 4x3 -15x2 +12x-b(m-1)


PÍZ)(X] = 12X2 -30X + 12
P(3)(x)= 24X-30
1
P<!)(x) = 0 « 12xa —30X+12 = 0 » x = 2 ou x = -

199
1
Portanto, os possíveis valores de c são 2 e
2

Observemos que P^(2) * e * 0.

1* possibilidade: c - j

0« + 2n - 2 - 0 «■ 8m + 32n = 25

3 2

p,,,UP °«4(5)1-,s(Í)2*1z(i) + m -1 - Q <=> jti = —


4
7

7 39
Dai temos m = e n = —.
4 32

2a possibilidade: c = 2
ÍP(2) = ü« (2)4 -5(2)3 4-S(2)2 +(m-1)(2) + 2n-2 = 0 m+n 2
|pf1’(2)= 0 <=> 4(2)3 -15(2)2 +12(2) + m-1 = 0 « m = 5

Daí tiramos m = 5 e r - -3,

Portanto, a resposta do problema é:


7 39
m =— e n=—
4 32
ou
m = 5 e r =: —3

Exercícios Propostos

12.7) Resoiva a equaçao x3-i-7x2 + Bk-I'12 = 01 sabendo que admite uma raiz
dupla.

12.0) Resolva a equaçao x3 - £5 - 2i)x2 +(6 - Bi)x + 6i - 0, sabendo que admiie


uma raiz dupla.
■J n
12.9) Resolva a equaçao 2x 4 -7x +9x —5x+1 = 0, sabendo que admite uma
raiz tnpía.

12.10) Determine k de modo que a equaçáo x3 + 9x2 + 24x+k = 0 admita uma raiz
dupla.

200
12.11)Determine □ valor de k de modo que a equação x'1 + 7x3 +1SK2 +KX + 8 = 0
admita uma raiz tripla. Depois resolva a equação

12.12) Determine os valores de k e t de modo que a equaçao x4 + x3 - 3x2 -bkx +


+1 = 0 admita uma raiz tripla

8 3
12 13)Determine o valor de k de modo que a equação x4 + - X + 2x2 +Ôx +k = 0
3
admila uma raiz dupla real e negativa.

12.14] Determine o valor de k de modo que □ polinõmio P(x)= x + (k - 4)x2 +-


+ (4-4k]x + 4k admita a número 2 como raiz dupla.

201
â
Capitulo

Raízes Imaginárias

13.1 - Teoria das Raízes Conjugadas

'Consideremos um polinòmio P(x)de grau n e coeficientes reais.


Seonúmeroimaginário z = a+bi(com ag E. e be R*) ê raiz
de P(x), (13.1)
a) z também é raiz de P(x);
b) z tem a mesma multiplicidade de z.

Demonstração

a) Seja P(x) = a„x° + a^x11"1 + ... + aonde a., an-l........ a0 saa números
reais Êe z é raiz de P(x). temos P(z) = D:

P(z) = anzn+a n-12 + ... + ao (0


Calculemos P(z)/'

F(z) = an(z)n + a^zF + ...+ aa (II) (IO


Já que an, aQ sao reais, temos:

= an

an-l -ã

- ac
Usando esse fato e lembrando que para um número complexo qualquer üj
vale;
(íd)" = {(?)

a igualdade (II) transforma-se em:

P(z) = an(zn)+an_1(zHj„.+aa s=

= (anzn -r ari_1zri"'+...+aü) p(z) = D

203
Assim, se P(2) = o, 2 éraizdeP(x).

b) Temos z = a + bi e z=a-bi, Suponhamos que z seja raiz de multiplicidade m


(úom m > 1). Já que z e z são raizes de P(x), temos:

P(x) = (x - z)(x - z)xA(x) = [x2 -(z + z)x+zz]*A(x) =


= [x2 - 2ax + (a2 + b2)]>A(x)
B(x)
Observando que P(x) e B(x) têm coeficientes reais, concluímos que Afx)
lambem tem coeficientes reais.
Se z for raiz simples de P(x) (isto é, m = 1), então 2 não será raiz de A(x] e.
porlanto, z também não será raiz de A(x}; isto nos leva a concluir que 2 também
serã raiz simples. Em outras palavras, se z é raiz simples, z também é raiz
simples de P(x),
Se m > 1, então z deverá ser raiz de A(x); mas, levandc em conta que A(x)
possui coeficientes reais, z também será raiz de A(x).
Aplicando-se esse raciocínio o número necessária de vezes, chegaremos à
conclusão de que 2 e z têm a mesma multiplicidade.

Exemplos
a) Consideremos o polinõmio P(x) = xa—7x2 + 17x-15.
Suponhamos que alguém nos informe que o número z = 2 + i seja raiz de P(x).
Observando que os coeficientes do polinõmio sãa tados reais, concluímos que
a número z = 2 —i também deve ser raiz de P(x).
3) Consideremos □ polinõmio P(x) - x2-(4 + 2i)x + (2 + 4í).
Observemos que alguns de seus coeficientes são imaginários; disso resulla
que para esse polinõmio não se aplica o teorema 13.1. Oe fato, se
determinarmos as raizes de P(x), obteremos z.^ = 3 + i e z2 = 1-i-i; no entanto,
os números Zi = 3 — j e z2 =1—i não sã□ raizes de P(x):
cj Sabe-se que o número 2 = -3i é raiz dupla do polinõmio:
P(X) = XS-x< + 1ÔX3 -18x2 + 81x-81
Como todos 05 coeficientes de P(x) são reais, o número z = 3i larntém é raiz
dupla de P(x). Donde, podemos escrever:
P(x)= (x + 3r)2(x-3i)2 xA(x)

Observação Nos capítulos 11 e f “ apresentamos teoremas que valem para


12
polinómios de coeficientes complexos; neste capítula, enunciamos
um teorema que vale apenas quando os coeficientes são reais. Nos
próximas teoremas, devemos observar cuidadosamente para que
"tipos" de coeficientes eles valem.

2Q4
13.2 - Consequência

Consideremos um polinõmio P(x) de grau n > 0 e de coeficientes reais. Como


consequência da teorema anterior, temos que o número de raizes imaginárias de
P(x] ê sempre par. Assim, se P(x) for de grau inrpar, terá um número impar de
raizes reais (isto é, pelo menos uma raiz real).

Propriedade
Para facilitar ncsso trabalho em alguns exercícios, chamaremos a atenção para um
detalhe. Consideremos os números imaginários z = a + bie z = a-bi (com a e R
e b e lí*). Sendo P(x) um polinõmio de coeficientes reais, suponha­
mos que z seja raiz de P(x); neste caso, z tambénn é raiz de P(x), portanto
podemos escrever:
P(x) =(x-z)(x-z)*A(x)
Fazenda o pradulo dos dais termos correspondentes às raizes conjugadas, temos:
(x - z](x - z) = [x - (a +bl)[x - (a - bi)] = «x - a) - (bi)][(x - a) + (bi) -
= (x - a)2 - (bi)2 = x 2 - 2ax + (a2 + b 2 )
Observe que -2a é real e a2 +b2 também é real.
O produto de dois termos correspondentes a duas raizes imaginárias cqn-jugadas
apresenta sempre um polinõmio de qrau 2 de coeficientes reais (é este o detalhe
que querismcs ressaltar).

Exerçiçips Resolvidos

13.1) Onal d menor grau possível para um polinõmio P(x) de coeficientes reais e
de grau n > 0 que admite os números 2,5 e 7 + i como raizes?

Solução
Já que P(x) tem coeficientes reais 7 + i ê raiz, conclui mos que 7 — i também
ê raiz. Assim P(x) terã no minimo quatro raizes:
2, 5. 7 + 1 e 7-1
Portanto, P(x) é no minimc de grau 4.

13.2) Dê um polinõmio P(x) de coeficientes reais, de menor grau possivel, que


admita como raizes os números 2 e 1 + i.

Solução

Se 1 + i ê raiz, 1 - i também o é. Assim, P(x) terá nos mínimo trés raizes e


será, no mínimo, de grau 3.

Assim;
P(x) = ar(x - 2)[x-(1 + i)[x - (1 -’)] (0
205
De acordo com o que destacamos nc item 13.3, temos:
[x-(1 + i)J(x-(1-i)] = x2 - 2(1)x + (12 +12) = x2 - 2x + 2
Substituindo em (I), vem:
?{x) = an(x — 2)(x2 — 2x+2) = ajx3-4x2-r 6x-4]

Fazendo a n = 1, obtemos um polinômio qus salisfaz a condição do prcble-


ma: P(x) = x3 - 4x2 + 6x - 4.

13.3) Dê um polinomio P(x) de coeficientes complexos, de menor grau passível,


que admita como raízes os números 2 e 1 + f.
Solução
Este exercício assemelha-se ao anterior. Naquele, pedíamos que o polinó-
mio tivesse coeficientes reais; neste estamos pedindo que os coeficientes
sejam complexos Então, não é necessário que o número 1 - i seja raizdo
polinômio. Temos, então:
P(x) = an(x —2)[x-{1+ i)) = an[x2 - (3 + i)x-=-(2+2i)J

Fazendo an = 1, obtemos um polinômio que satisfaz a condiçío do


exercício: P(x) = x2-(3+i)x+(2 + 2i).

13.4) Dê um |polinômio P(x) de coeficientes reais, de menor grau possível, que


admita como raizes os números 7, 3 — j e 5i, com multiplicidades 1, 3 e 2,
respectivamente.

Solução
Í7 é raiz simples
3 -1 é raiz tripla 3 4- i é raiz tripla
[Si é raiz dupla => -5i é raiz dupla

Assim, P(x) admite no mínimo 11 raizes e é no mínimo de grau 11-


Podemos, então, escrever;
P(x) - aA(x -7)Hx -(3 - i)PX* - (3 + i)j3 >(x -Si)2 ?<x + 5i)2
Onde a„ é um número real qualquer, não nulo.

*a _ -n
13.5) Resolva a equaçao x — 7x'+17x — 15 = 0, sabendo que uma de suas
raizes é o número 2 + í.

Solução
Seja P(x) = xJ - 7x2 + 17x - 15. Como os coeficientes sSo todos reais e o
número 2 + i é raiz, concluímos que o número 2 — í também é raiz. Portanto,
podemos escrever:
P(x) = [x -(2 + i)]Jx - (2 - i)] >A(x) (O
206
Para obtermos A(x). podemos fazer uma divisão por x-(2 + i), e, em segui­
da, outra por x-(2-í):

2+i 1 -7 17
2- i 1 -5 + i 6-3i 0
1 -3 0
A(x) = x—3
A(x) = 0 -> x-3 = 0^->x = 3

Assim, o conjunto-Solução da equaçao dada é:


S = (3;2 + i; 2-i}

No entanto, poderiamos ter obtido A(x) de outro modo. De acordo ccm o que
vimos no item 13.3, temos:

fx^(2 + i)][x-(2-i)] = x3-2(2)x + (25 +12) r= x2 — 4x + 5

Substituindo em (1):

P(x) — (x2 -4x+ 5)xA(x)

Portanto, podemos obter A(x) dividindo P(x) por x?-4x + 5, utilizando o


método da chave:

x1 -7x2 +17x-15 2
x" -4x + 5
- x3 + 4x2 -5x x—3

-3X+12-15
3x-12x + 15 A(x) = x-3 |
0

No capitulo 14 resolveremos este exercício de outro modo (exercício 14.5).

13 E) Determine as raizes do polinômio P(x)-xs -x4 +18x3 -18x2 +Ô1x - 81.


sabendo que o número 3i è raiz dupla.

Scluçào

Os coeficientes de P(x] são reais. Portanto, se 3i é raiz dupla, — 3i também


o é. Logo:

P(x)= (x-3i)2(x + 3i)2>A(x)

207
Para obtermos A(x), fazemos duas divisões sucessivas por x - 3i e duas
divisões per x + 3i:
3i 1 -1 18 -1Ô fl -81

3i 1 -1 + 3i 9-3Í -9 + 27i

—3í 1 -1 +6i -9-6i 9

-3i 1 -1 + 3i -3i 0

1 -1 0

A(x) = x-1
Como a raiz de A(x) é o número 1, concluímos que as raízes de P(x) são: 3i
(dupla), - 3i (dupla) e 1 (simples).
Este exercício será resolvido de outro modo no capítulo 14 (exercício 1<6).
13.7) Determine o número real m de modo que o polinòmic F^x) = 5x3 + 2x"-
- 11x -F-m admita uma raiz imaginária de módulo igual a 1.
Sc I ução
Seja z = a + bi a raiz imaginária mencionada. Já que |z| = 1, lemos
a2 + b2 =1. Como os coeficientes de P(x) são reais, z também é raiz de
P(x); assim, concluímos que P(x) é divisível por (x-z)(x-z). Mas, de
acordo com o que vimos no item 13.3, temos:
(x- z)(x-z)= [x - (a-bi)J[x-(a-bi)] = (a2 + b2)- x2 + kx +1
------ 2) ' ' ‘ A( x)

onde k e ü. As raízes de A(x) são ambas imaginárias e, pcrlanlo, seu


discriminante deve ser negativo:
A = k2 -4 < 0
Assim:
P(x)^A(X)>Q(x)
Façamos a divisão de P(X) por A(x):
5x3 + 2x2 - 11x + m x2 + kx +1_
-5xJ - 5kx2 -5x 5x+(2-5k)

(2 - 5k)x2 ~ 16x + m Q(x)

- (2- 5k)x2-(2k- Sk2)x-(2-5k)

(5x2 — 2k - W)x+(Sk -2 + m)
Como o resto dessa divisão ceve ser identicamente nulo, obtemos o sis­
tema:
5k2^2k-16 = 0 (i)
5k-2 + m = ü (II)
203
g
□3 equação (I) tiramos k = 2 ou k =---- ,
5
Antes de substituirmos na equação (II), devemos verificar se esses valores
de k satisfazem a condição ó < 0;
k=2=>à=k2-4=22-4 = 0 (não satisfaz}
2
k =-^=s ó = k2-4 = 8 * 36 , t .
| -4 = --— (satisfaz)
5 5
g
Substituindo k =---- na equação (II), tiramos m = 10 Portanto, a res-
5
posta do problema ê:
m = 10
No entanto, se quisermos, poderemos obter as raizes de P(x). Temos:
8
-2a - k = -
5
a2 + b2 - 1

4 3 Portanto:
Desse sistema, obtemos
. _a=— ±—
s e b=- 5

4 3 - 4 3.
z = —+ — e z =-------- 1
5 5 5 5
Para obtermos a raiz real, o meio mais rápido è achar a raiz do quociente
Q(4
Q(x) = 5x+ (2-5k) = 5x4-10
Q(x) - 0 <=> x = -2
a t a i 4 3 4 3. *
As raizes de P(x) são, então: —+ —i, ^5' e

13.0} Determine o número real m de modo que o polinõmio P(x) = 5x3 + 2x2
-11x + m admita uma raiz de modulo 1.

Solução
Neste caso, 0 enunciado não esclarece se a raiz de módulo 1 é real ou
imaginária. Temos, então, duas possibilidades:
11 possibilidade: a raizé imaginária
Nes(e caso, lemos um problema idêntico ao anterior e a resposta èm = 10.
2J possibilidade: a raiz ê real.
Neste caso, a raiz real de módulo 1 pode ser 1 ou —1.
ÍP(1)=C « 5(1)3 + 2(1)2 -11(1)+m = 0 » m = 4
lp(-1) = 0 « 5(-1)3 + 2(-1)2 -11(-1) + m= 0 « m= -8
A resposta do exercício é, então, m = 1D oum - 4 ou m = — 8.

209
13.9) Mostre que o polinômio P(x) = x3 - x3 + 3x —7 tem apenas uma raiz real.

Solução
Os coeficientes do polinômio são todos reais e o grau do polinômio é ímpar.
Daí concluímos que P(x) tem pelo menos uma raiz real. Seja K essa raiz
real.
P(x) - (x-k)>A(x)
Vamos dividir P(x) por x - k para obtermos A(xj:
k 1 -1 3 -7

1 (k-1) (ks-k + 3) (k>-k< + 3k-7)

A(x) — xz + (k-l)x + (k2-k + 3)

O discriminanle de A(x] é:

A = (k-1)2 - 4(k2-k + 3)--3k2+2k-11

O discriminante da expressão -3k2+2k-11 é

A' = 22 - 4(—3)(-11) = 1 -132 = -126

Como AcOe o coeficiente de ka ê negativo (— 3), concluímos que a


expressão —3k2 + 2k —11 será negativa para qualquer k e R; assim, lemos
que A(x) não tem raizes reais. Portanto, P(x) tem apenas uma raiz real (que
e o número k).

Exercícios Propostas __

1 3.1 0) Consideremos um polinômio P(xJ de grau n > 0 que admite as raizes 3, 4 e


2 - 61
a) Supondo que cs coeficientes de P(x] sejam reais, qual o menor grau
possivel para esse polinômio?
b) Suponde que os coeficientes de P(x) sejam complexos, qual o menor
grau possivel para esse polinômio?
13.11) Um polinômio de coeficientes reais e não identicamente nula tem o número
6 como raiz dupla, o número 8 como raiz tripla e o número 7 + 5i como raiz
de multiplicidade 4. Quai a menor grau possível para esse polinômio?

13.12) Consideremos um polinômio P(x) de grau n > 0 e de coeficientes reais.


Assinale valor verdadeiro ou fafso para cada uma das sentenças abaixo.
a] Se n for impar, P(x) tem pelo menos uma raiz real.
bj Se n for impar, o número de raízes reais de P(x) também é ímpar.
c) Se n for par, P(x) tem pelo menos 2 raizes reais.
d) Se n for par, pode acontecer que todas as raizes de P(x) sejam
imaginárias.
e) Se n for impar, pode acontecer que todas as raízes de P(x) sejam
imaginárias.
f) Se n for par, o número de raizes reais também é par.

210
13.13)Sabe-se que os números 7,-9 e 2 + 3i são raizes de um polinõmio P(x)
de grau 17 e de coeficientes reais. Assinale verdadeiro ou falso para cada
sentença a seguir:
a) F*t» tem pelo menos 3 raízes reais.
b) P(x) pede ter até 16 raizes reais.
c) P[x) pode ter até 15 raízes imaginárias.
d) P(x) pode ter no máximo 14 raizes imaginárias.

l3.14)Dé o polinõmio P(x) de coeficientes reais, de menor grau possivet que


satisfaz simultaneamente as duas condições a seguir:
1*) os números i, 2 - i e 3 são raízes:
2*) □ termo Independente é igual a 30.

13.15) Dê c palinõmio A(x) de coeficientes reais, de menor grau possível, que


satisfaz simultaneamente as condições:
1') 2l é raiz dupla;

3‘) Q é raiz tripla;


4*) o coeficiente de xs é igual 32.

13.16)Resolva a equação x4-9xS + 24x2-6x-4G = 0. sabendo que uma de


suas raizes é 3 + i.

l3.17)Resolva a equação xB - 3xs-1 8x3 +1 2x2 = 30x3 - Iflx4 - 20x2, sabendo


que uma de suas raizes é o número 4i.

13.18) Resolva a equaçao x 6 -x5-2x3-3x2 — x —2 = 0. sabendo que i é raiz du-


pia.

13.19)Resolva a equação xS-4x4-3x3 -6x2 + ax + b = 0, sabendo que os núme-


ros 1 e 2i são raizes.

13.20) Resolva a equação x7 - 2xs+ 7x5 - 14x4 + Sx3 - 16x2 + ax + b - 0, sabendo


que c número 1 é raiz simples e o número 2i ê raiz dupia

13.21)Sabe-se que o número S + 7i é raiz de um polinõmio P(x), não identi­


camente nuío, cujos coeficientes são complexos. Podemos garantir que o
número 5 - 7i também é raiz de P(x)?

13.22)Sabe-se que o polinõrnio A(x) = Sx3 + 27x2 + 46x + m, ondeme K, admite


uma raiz imaginária de módulo igual a 2. Determine:
a) o vaiar de m;
b) as raizes de A(x).

211
13.23) Determine o número real m de modo que o polinómio A(x) = 5x3 +27x?r
+ 46 + m admita uma raiz complexa de módulo igual a 2.

13.24)Resolva a equação 2x5 + gxd + 4X3 + 54x + 27 = 0, sabendo que ela admite
uma raiz dupla e que o número 1 - 72 m é raiz simples.

1 3.25) Resolva a equação xe - 6xT + 14x6 +14xs-22xs + 25x4 + 8x3-60x2-0.


sabendo que ela admite a raiz 2 + i e uma raiz imaginária pura.

212
Capílulo

14 Relações de Girard

14.1 - Introdução

Exislem importantes relações entre os coeficientes e as raizes de uma equação


algébrica, estabelecidas por Girard (Albert Girard, flamengo, 1590-1633). Porém,
antes de mostraremos como são essas relações no caso geral, estuda­
remos alguns casos particulares.

14.2 - Equaçao do Segundo Grau

Consideremos a equaçao ax2+bx + c = 0, onde a, b e c sao complexos, com a *


0. Sendo x1 e Kj suas raizes, temos:

ax2+bx+c = a(x - -x2) = a[x2 - (x1 + x2)x +


islo é:
2 h C 2 .
X +-X+- = X - (X1 + x2)x + Xi>»2
a a
Por identidade de polinõmios, obtemos as relações:

b
x1 + x2 -
3
(14.1)
C
x1 = —
a

que são as relações de Girard para a equação do segundo grau. Já as havíamos


estudada no campo real (veja capitulo 3 do volume 1 desta coleção), agora,
estudaremos estas relações no campo complexo.

Exemplo
1
Seja a equação Õxz -10x + 3 = 0, cujas raízes são *1 = 7 e *2~2

Neste caso, temos:


a = B, b — “10 e c — 3

213
3 1 3+2
x1 + x3 = - + —
1 2 4 4 4 b
x, + x2 = --
b (-W) = 10 5 a
a 8 8 4
3 1 3
X1 XX2 - 4 X2
8 c
Xi + X2 =-
c 3
a 8

14.3 — Equação do Terceiro Grau

Consideremos a equaçao, onde a, b, c e d são complexos, com a 0. Sendo Xi, X2


e x3 suas raizes, temos:

ax3 + bx2cx + d = a(x-x1)(x-x2)(x-x3)


ou
b c d
x3 + — x 2 + -X + ^ = (x-x1)(x-x2)(x-x3) =
a
= X3 — (X-j + x2 + x3)x2 +
+ (X1X2 + *1*2 + X2X3)X-X1<X2X3

Por identidade de polinômios, obtemos as relações:

. b
1a) x^+x2 + x3 =—
a
c
2a) x1x2 + x-,x3 + x2x3 = — (14.2)
a

3a)XiX2x3 = --
a

que são relações de Girard para a equação do terceiro grau. Observe que na
primeira relação temos as raizes tomadas uma a uma; na segunda relação temos
as raizes tomadas duas a duas; na terceira relação temos as trés raízes.

Exemplos

a) Consideremos a equação 5x3-7x2-9x + (6 + i) = 0. cujas raizes são xr

3'. Temos:
x2 e x3
a = 5, b =-7, c=-9 e d = 6 + i

214
Portanto:
-7 7
5 " 5
-9
X,X2 + X-|X 3 + X 2X j — - =
3- 5
d 6+i
*1*2*3 =--
5

b) Sejam a, b e c as raízes da equaçao. As relações de Girar d para essa equação


são:
9
a + b + c = -y
6
ab + ac + bc = —
7
. 5
abc = —
7

14.4 - Equação do Quarto Grau

Consideremos a equaçao de grau 4:

ax4 + bx3 + cx7 + dx + e - 0

cujas raízes são xv x2, x3 e x4. Por um processo semelhante ao usado nos itens
anteriores, obtemos as relações de Girard para essa equação:

rjXí+Xs+Xj + X^ =-----
a
c
2a) X^2 +XdX3 -rX1X4 + x2x3 + x3x 4 + X3*-4 ~ —
a

3K) x1x2x3 +x1x2x4 +x1x3x4 + x2x3x4 = --


3
0.
4=) x^x3x3x4 --
a

Observe que:

1*) na primeira relação, lemos as raízes tomadas uma a uma;


2a] na segunda relação, temos as combinações das 4 raízes, tomadas duas a
duas; lembrando que C4 2 = 5, temos 6 parcelas;
34) na terceira relação, temos as combinações das 4 raizes, tomadas três a
três; como C4 3 =6, temos 4 parcelas;
4*) na quarta relação, tomamos as 4 raízes

215
Exemplo
Sejam a, b, c ed as raizes da equação 5x4 + 6x3 + 7xz+Sx-r9 = 0.
As relações de Girard para essa equação são:
1‘ . 6
a +b+c+d =—
5

ab + ac + ad + bc + bd + cd = —
5
â
abc + gbd + acd + bcd =

9
abcd = —

14.5 — Caso Geral

Consideremos a equaçao algébrica:


artx° + a„_ vrt-3
a 1i*xn-,+ an-2 n"3 + a n-3x
+ a^,x +... + aQ = D

de grau n > 0, cujas raízes são x1n x2, x3 xn. Por um processe semelhante
ao usada nos três itens anteriores, podemos obter as relações de Girard para
essa equação,

1a} xd + x2 + x3 +. . + xn
an

2a)x1x3 + x1x3+... + xn_lxn


an

34) x1x3x3 + x1x2x4 +...+ Xrl_2Xrl_1Xri = ---LUl

nfl) x,x2X3...Xri ,ao


(-1)"
an

Observemos que
1a) na primeira relação, temos as raizes tomadas uma a uma;
2a) na segunda relação, temos as combinações das n raizes tomadas duas a
duas:
3*) na terceira relação, Lemos as combinações das n raizes tomadas três a
três;

n°) na n^êsima relação, temos todas as n raízes.

Exempla

Vamos escrever as relações de Girard para a equaçao do quinto grau


ax5+bk4 + cx3 + dx? + ex + f = 0

cujas raizes são x1n x2, x3t *4 e xs.

216
b
í*)x, + x2 + if3 4 s=’ã
c
2a)x1x2 + x1xJ + x1x4 K1x5 + x2x5 + x3x 4 +x2x5 +x3x4 + x4x5 =-
a
3’) x,x?x3 + x1x2x4 + XiX2X3 +x1x3x4 + xdx3x5 + x1x4xs + x2x3x4 +x2x3x5 t
d
+ *2X4X5 + x3x4x5 =-----
a
e
4a) X1X2XaX4 + X^XgXç +x1x2x4x& + *1*3 *4*5 + x2x3x4x 5“7

f
5a)xix2x3x4x5 = —
a

Observação: As relações de Girard sõ são úteis na resolução de equações


quando temos mais alguma informação sobre as raizes. Sozinhas,
elas não são suficientes para resolver equações.

Exercícios Resolvidos

14.1) Determine as raizes dos polinômio P(x)= 9x3 -1Sx2 + 11x- 2, sabendo que
uma de suas raízes é igual ã soma das outras duas.

Solução

Sendo a, b e c as raízes do polinômio, as relações de Girard ficam:

, "IS
a + b + c ~--------- = 2 (D
9
, < 11
ab + ac + oc = — (10
9
-2 2
abc —------- (III)
9 9
A condição dada no enunciado pode ser traduzida por,
a=b + c (IV)

Introduzindo (IV) em (I), obtemos:


2a = 2
dcnde:
a= 1
Agora que já temos a raiz a = 1, podemos abandonar as relações de Girard
1 2
e dividir P(x) por x - 1, obtendo o polinômio cujas raizes são - e
3 3
Assim, as raizes de P(x) sao: 1, -i e 2
3

217
14 2) Obtenha a soma e o produto das raízes da equaçao 6x5 + + 9x2 + 4x +
+ 5 = 0

Solução

Observemos que o coeficiente de x* ê nulo, isto é, a equação pode ser


escrita:
6x5 + 0x4 + 7x3 + 9x2 + 4x + 5 = 0
Sendo xr x?u x3, x4 e x5 as raízes da equação, vamos considerar apenas
a primeira c a última relações de Girard:

X-! + X2 + Xj + X

5
*1* 2* 3*4*5 — ~~

14.3) Resolva a equaçao x J + 8x2 + 9x-1 8 = 0, sabendo que uma de suas raízes
é o oobro de outra raiz.

Sol uçao
Sejam a, b e c as raizes. As relações de Girard são:
, 8
a+b+c = —
1
(D
< ab + ac + bc = — = 9 (il)
I abc
u =--- = 1S (111}
I 1
A condição dada no enunciado pode ser traduzida por:

a = 2b (IV)
Tomemos o vaior de a da equação (IV) e substituamos nas equaçces (I) e (II):

Í3h + c = -8 (V)
J
[2b2 + 3bc = 9 (VI)
De (V), tiramos c = -8-3b (VII)

Substituindo em (VI), obtemos a equação 7b2 + 24b + 9 = 0, cujas raizes


3
sâo b =-3 e b" = - —. Substituindo nas equações (IV) e (VII), vem:

b = -3 => a = —6 e c=1

b--’ 6 -47
=> a - ------ c =------
7 7 9

210
A fim de sabermos qual destas duas séries de valores serve, fazemos o
leste na equação que ainda não foi usada (III) e vemos que apenas a
primeira série serve, Assim, o conjunto-solução ê:

S ={-3;-6:l)

14.4) Resolva a equaçao x3 - 7x2 +1 5x - 9 = 0, sabendo que admite uma raiz


dupla.

Solução

Jã resolvemos este exercício com o auxilio de derivada (veja exercido 12.2),


Agora vamos resolvê-lo usando as relações de Girard. Sendo a. b e c as
raizes da equação, temos:

a + b + c =------- = 7
1
ab + ac + bc = — =15
1
-9
abc =------- = 9
1
Já que a equaçao tem uma raiz dupla, façamos b = a. Substituindo nas três
equações anteriores, temos:

I 2a + c = 7 d)

ka' ■+ 2ac = 15
| a 2c -a9
,-----
(II)

d")
Da equação (1), tiramos: c = 7-2a(IV). Substituindo em (II), obtemos a
equação:
3a2 —14a+1 5 = 0

cujas raizes sao a1 = 3 e a" = — Substituindo em (IV), temos:


3

a ■ 3 c =1
5 11
a = — => c = —
3 3

Para sabermos qual destes pares de valores váo servir, fazemos a


verificação na equação que ainda não foi usada (III) e observamos que
serve apenas o par a = 3 e c=1.
Assim, o conjunto-solução é:

s = <3:1}
onde 3 éa raiz dupla.

219
14.5) Resolva a equação x3 7x2 +1 7x - 15 = 0, sabendo que uma de suas
raízes é n número 2 + i.

Solução
Este exercício já foi resolvido no capítulo 13 (exercício 1.5)- Vamos agora
resolvê-lo de outro modo (bem mais rápido).
Observemos cue os coeficientes da equação são todos reais; partanlo, sen
número 2 + j é raiz, □ número 2 - í também o é. Sendo a, becas caizes da
equação, pela primeira relação de Girard, temos:
-7
a ■+ b 4- c ~------- = 7
1
Fazendo a = 2 + i e b - 2 — i, temos:

(2 + r) + (2-i) + c = 7
donde:
c= 3
Portanto, o conjunto-soluça o é:

S = {2 + t 2-i; 3}

14.6) Determine as raízes do polinômío P(x) = x


s -x" +1Bx3-1Bx2 +81X-Ô1.
sabendo que o número 3i é raiz dupla.

Solução

Este exercício já foi resolvido no capitulo 73 (exercício 13.6).


Sejam a, h, c, d e e as raízes de P(x). Observando que os coeficientes de
P(x) são todos reais concluímos que se 3i é raiz dupla, — 3i lambém c é.
Assim, podemos fazer:

a=b = 3i e c=d = —3i

Mas, pela primeira relaçao de Girard, temos:

a + b + c-f-d + e= —- = 1
1

Isto é:

3r + 3i - 3I-* 3Í+e = 1

donde:
e = 1

Portanto, as raízes sac 3t, - 3i e 1.

220
14.7} Dê um polinòmío de grau 3, cujas raizes sejam os rnjmeros 2 (simples) e 4
(dupla).

Solução
Observe que este exercício já foi resolvido de outro modo no capítulo 11
(exercício 11.12),
Já que não houve exigência especial, vamos construir um polinòmío em que
o coeficiente de x3 é igual a 1:
P(x) = x3 + czx2 + px + Y
Sendo x-i. x2 e x3 as raizes de P(x). as relações de Girard ficam:

x, + x2 + x3 =-“ =-cx (I)

X-|Xa + *1*3 + *2*3 = £ = P (II)

Xl*2*3 = (III)

Como 2 ê raiz simples e 4 é raiz dupla, podemos fazer x, = 2, x2 = 4 e


x3 = 4. Substituindo em (I), (II) e (lll), temos;
2 + 4 + 4 = -a (I)
2{4) + 2(4) + 4(4) = p (II)
(2)(4)(4) = -y (III)
donde;
rx = -10. |3 = 32 e y = -32

Assim, P(x) = x3 - 10xz + 32x — 32.

14.0) Sendo a, b, c e d as raizes da equação. 5*^ -7x3 f6x2 -k9x + 13 = 0,


1111
calcule o valor da expressão — + — .
abcd
Solução
Vamos tentar calcular o valor da expressão dada, sem sabermos quais são
as raízes, usando apenas as relações de Girard:
7 7
a+b+c+d=- (!)
5 5
g
ab + ac + ad+ bc + bd + cd = — (lü
5
g
abc + abd + acd + bcd = (lll)

13
abcd - — ÜV>
5

221
Temos, então;
-S
1 1 1 1 bcd + acd + abd + abc - _5^ =
a b c rl abcd 13 13
5

14.9) Sendo a. b e c as raízes da equaçao x 3-4x2 +7X + 3 = 0, calcule os


valores de:
1 1 1
a) — + —+
ab ac bc

b) a2 + b2 + c2

c) a2b2 + a 2^2
c + b2c2

d) a3 + c3

Solução

As relações de Girard para essa equaçao sac;


a+b+c=4
■ ab + ac -t- bc = 7
abc = —3

a) -1 L J_. c+b+a 4 4
ab ac bc abc -3 3

b) Vamos nos lembrar da identidade:

(a + b + c)T = a + b'± + c2 + 2(ab + ac + bc)

Portanto:
a2 + b2 + c2 = (a + b + c)2 - 2(ab + ac + bc) = (4)5 -2(7) = 16-14=2

c) Temos:

(ab + ac + bc)2 = (ab)2+(ac)2 + (bc)2+2[(ab)(ac) + (ab)(bc) + (ac) (bc)] -

= a2b2 + a2ca + b2G2+ 2abc (a + b + c)

Portanto:

aV + a 2 c 2 b c = (ab + ac + bc)2 -2(abc)(a + b + c) =


— (7)2 - 2(-3)(4) = 49 +24 = 73

222
d) Já que a, b e c sao raizes da equação, xa-4x2 + 7x+3»C, devemos
ter:
a3J -4a= + 7a + 3 = 0
b3 - 4b2 + 7b + 3 = 0
C3 - 4c2 + 7c + 3 - 0

Somando membro a membro estas três equações, obtemos:

(a3 + b3+ ?) - 4(a2+ b2 + c2) + 7(a + b + c) + (3 + 3 + 3) = 0


2 4
Dai tiramos: a3 + b3 + c3 =-29.

14 10)Sejam x^ x2, x^,.., xn as raizes da equaçao algébrica de grau n (com n > 1):

n-1+
Vn+W''' +aW"“‘ + .,,atx + aa = 0

Calcule o valor de x2 + x2 + x2+... + x2.

Solução

Vamos partir da 'dentidade:


(x1 + x2 + x3+..>+xr1)2 =(xf+x| + x^+.,. + x2) + 2(x1x2 fx,x3 + ... + xnw1xn) (I)
Escrevamos a primeira e a segunda relações de Girard
a^
Xi+X2 + X3h... + Xn =- (II)

3n-?
X^j+X^j + ... + X 1xn ’---------- (III)
an
Introduzindo (II) e (111) em (I), obtemos:
/ '2
I a-n^L - (x2 + x|+.,.+ x2) + 2Í
J I an
donde:

_
xf + Xj+.+X n2 ~
2 2 | an-1

k an J

14.1 i)Sejam a, b. c e d as raizes da equação:

3x4 x/2X3+(1+Í)X2-r-X-(e + 3Í)= 0.

Calcule □ valor de a2+b2 + c2 + d2.

223
solução

e acordo com o que vimos nú exercício anterior, vamos partir da


identidade:

(a + b + c + d)2 = a2 + b2 + oz + dz + 2(ab + ac + bc -i-bd + cd) <0


A primeira e a segunda relações de Girard são*

J2
a+b+c+d=- PD
3
ab + ac + ad + bc + cd =----- (!!!)
3

introduzindo (II!) e (111) em (I), obtemos:


2
72 = a2 + b2 + c2 + d2 + 2
1+i
3 3

donde:
a2 + b2 + c2 +d3 = -4-6f
9

1 4.1 2) Consideremos a equação 2x3 + 7x 2 + Sx + 3 = 0, cujas raizes Sâo x1,x2 e

X3. Obtenha um polinõmio P(x) de raízes a,


a, b e c, lais que:

a = x^j, b^x1x3. c = x2X3.

Solução

As relações de Girard para a equação dada são;

-7
*l+*2 + *3 = —

5
*1*2 + *1*3 +*2*a

-3
*1*2*3 --re­

Seja P(x) = x3 + ax2 + Jlx + y o polinõmio de raizes a, b e c, As relações de


Girard para esse polinõmio são:

a + b + c = -a (D
ab + ao + bc = p (tf)
abc = -y (fH)

IVas, de acordo com □ enunciado, temos a - x1x2, b- xnx3 e c = x3*3

224
Assim:
5
a+b+c = x1x2 + x1x34-x2x3 = -

ab + ac + bc = x.x2x1x3 + x1x2x2x3 + x,x3x2x3 = (x^x^x, ■+• x Z + X3^ “

■(?)(?)■¥
2
-3 9
abc - x1x2x ,x3x 2x3 — —
2 4
-5 21 -9
Substituindo em (I), (I!) e (III), obtemos a = —, 0 = — e y = — Portanto:

5 X^2
P(X)= X3 ------- 21 9
+--- X-----
2 4 4

14.13)Determine as raizes do polinõmio P(x)=x 3 + Xa-Sx-125, sabendo qüe


elas têm o mesmo módulo.

Solução

Sendo a, b e c as raizes de P(x), vamos escrever a terceira relaçao de


Girard:
abc = -
1
A condição dada no enunciado é: ja| = |b|=|c|.
Temos, então:

abc = 125 = ] abc |=125 |a|>jb|^c| = 125 |ap = 125 =^>


1a|=Vl25 = 5

Observando que P(x) é de grau impar e possui todos os coeficientes reais,


concluimos que pela menos uma das raízes é real, supondo que a seja raiz
real, temos:
iaj — 5 » a = ±5

Podemos verificar que P(-5) ?* 0 e P(5) = 0: portanto, temos a 3 5.


Dividindo P(x) por x - 5, obtemos □ polinõmio A(x) = Xa + 6x + 25, cujas
raizes são -3 + 4i e -3-41 Assim, as raizes de P(x) são: 5,-3 + 4j e
-3-41

14.14) Determine as raizes do polinõmio P(x) = 2x3 - 15x2 + 37x - 30, sabendo que
elas formam uma progressão aritmética.

225
Solução

Sendo a, b e c as raizes de P(x), vamos escrever apenas as duas primeiras


relações de Girard:
-15 15
a -r b + c = -
2 2
(D
L 37
ab + ac+bc = — (10
2

A partir daqui, podemos encaminhar a resolução de vários modos.

1* modo
Supondo que a, b e c formem, nesta ordem, uma PA, podemos usar a
representação (veja capitulo 4 do volume 2 desta coleção):

a = b - r e c = b +r

onde rêa razao de PA. Substituindo em (I) em (II), obtemos:

15
(b - r) i b + (b + r) = — (III)

(b-r)(b) + (b-r)(b + r) + b(b + r) = (IV)

5
□ a equaçao (III), tiramos b = —. Substituindo em (iV):

5
---- r
)[|k[5_Y5+/|1V''
+ 5+r',i=az I -I-------- 1--------- r I i
2 A2j (2 A2 ; 2^2 ) 2
_ - _ _ T
Efetuando as multiplicações, chegamos à equação 4r = 1,

donoe:

r-±l
2
. 5
a - b-r =-------- =2
1
1
Para r = temos: 2 2
2
c-b + r = -H— = 3
2 2

5 f
a - b — r — -—| =3
2
Para r = - —. temos: 2 k 2
2
. = bl+..-
c 5-2
r =— =2
2 2
Em resumo, tanto para r = —, como para r =----- „ as raízes são- - 2 e 3.
2 2 2

226
2* modo
Podemos começar como no 1° modo e ir até o ponto em que obtemos
b= A partir dai, abandonamos as relações de Gtrard e fazemos a divisão

de P(x) por x--|, obtendo A(x) = 2x2 - 1Ox +1 2. cujas raízes são 2 e 3.

3* modo
Um outro modo de indicar que a, b e c formam uma PA (nessa ordem) è
colocar a + c = 2b.
Substituindo na relação (I) temos:

2b + .b — —
15
2
donde:
b-s
2
A partir dai procedemos com no 2D modo.

14.15) Obtenha as raizes do pclinômio P(x) = x3 - 7x2 + 14x - 8, sabendo que elas
são reais e formam uma progressão geométrica.

Solução
Sejam a, b e c as raízes; as relações de Girard ficam:
, -7
a + b + c - ------- =■ 7
1
0)
ab 4-ac + bc - — = 14 (II)
1
abc =---- - — S (lll)
1
1o modo
Supondo que a, b e c formem, nessa ordem, uma PG, temos [veja capítulo
6 do capítulo 2 desta coleção):
ac " b 2 (IV)
Introduzindo (IV) em (III), obtemos b3 = 8.
Em princípio, deveriamos agora extrair todas as raizes cúbicas do número 8;
mas, como o enunciado do exercício diz que as raízes do polinómio são
reais, ficaremos apenas com a raiz cúbica real de 8:
b3=8»b=^8-2
2 m j
Dividindo P(x) por x 2. obtemos o pclinômio A(x) = x -5X + 4, cujas
raizes são 1 e 4. Assim, as raizes de P(x) são: 1,4 e 2.

227
2° modo
Suponhamos que não soubéssemos que as raizes do polinòmio são Iodas
reais. Então, para evitar o problema surgido com as raizes cúbicas do
número 8. introduzimos (IV) em (II), obtendo:
ab + bJ + bc = 14
isto é,
b(a + b + c) = 14
7
donce.
b=2
Daí em diante, procederiamos como no 1o modo.

3° modo
Uma outra maneira de indicar que a, b e c formam uma PG (nessa ordem) ê
usar a representação:
b .
a = — e c = bq
q
Onde q é a razão da PG. Substituindo em (III), obtemos:

- ](b)Cbq) = e
donde:
b=2
Daí em diante, fazemos como no 1* modo.

1 4.16) Obtenha as raizes do polinòmio P(x) = 2x3 - 11x2+ 18x-9, sabendo que
elas formam uma progressão harmônica.

Solução
Sendo a, b e c as raizes de P(x), as relações de Girard são:
. -11 11
a +b + c =--------*= —
2 2
(D
18 = „S
ab + ac + bc - —
9
(H)
. -99
abc - —— = — (III)
2 2
Supondo que a, b e c formem, nessa ordem, uma PH, então seus inversos
formam uma P.A., ou seja:
1 1 2 1 . 2ac
:=o b = ——
b a c b a +c
donde:
ab + bc = 2ac (IV)

228
Introduzindo (IV) em (II), obtemos 3ac = 9,
isto é:
ac = 3 (V)

9
Introduzindo (V) em (III), obtemos 3b = —
2
donde:
b=5
2
3
Dividindo P(x) por x--, obtemos o polinômio A(x) = 2x -Bx + 6, cujas
2
3
raizes sao 1 e 3. Assim, as raizes de P(x) são: —, 1 e 3,

14.17)Determine as raizes do polinômio P(x) = x4 -8x3 + 14x2 + 8x-1 5, sabendo


que elas formam uma progressão aritmética.

Solução
Sendo a, b, c e d as raizes de P(x). vamos escrever apenas as duas
primeiras relações e Girard:

—8
a + b + c + d =------ = 8
1
(D
14
ab + ac + ad + bc + bd + cd = — = 14 (II)
1
Uma maneira de indicar que a. b, c e d formam uma PA, nessa ordem, é
fazer as substituições;
a b c d
4 1 i
Cy-3k) (y-k) (y + k) (y + 3k)
Introduzindo em (!) e (II) obtemos, apos as simplificações:

4y=8 (III)
6y=-l0k2 =14 (IV)

De (III), tiramos y = 2; substituindo em (IV), obtemos k = ±1 Assim;

íy = 2 e k=1=>a = -1,b=1,C = 3,d = 5


[y = 2 e k = -1 => a = 5, b = 3, c = 1, d = -1

Tanto num caso como no outro as raízes de P(x) são —1.1.3 e 5.

229
14.1 3) Determine as raízes dopolinômio P(x) = x 4-3xs-7x2 +27X-1Ê. sabendo
que duas delas são simétricas.
Solução
Sendo a, b. c e d as raízes de P(x), vamos escrever apenas a primeira e a
terceira relações de Girard:

a+b + G+ d= -3 (I)
abc + abd + acd + bcd = -27 (II)

Supondo que as raízes simétricas sejam a e b, temos:


a + b-0 (!!!)
Substituindo (!!!) em (I), obtemos:
c+ d =3 (IV)
A relação (II) pode ser transformada em:
ab (c + d) +■ cd (a +■ b) = -27
3 " '5
donce:
ab = -9 (V)
Consideremos agora o sistema formado peJas equações (V) e (III):
Ja+ b = 0
|ab = -9
Resolvendo este sistema, a=3eb=-3 (ou então a = - 3 e b = 3, o que
dá no mesmo). Agora que já obtivemos as raizes 3 e — 3, podemos fazer a
divisão de P(x) por (x-3) (x + 3), obtendo o polínõmío A(x) = x2-3X *2.
cujas raizes são 1 e2. Portanto, as raizes de P(x) são: 3, - 3, 1 e 2.

1 4.19) Determine as raízes do polínõmío P(x) = 2X*1 -7x3-5x2 +28x-12, saben­


do que duas delas são reciprocas (isto é, uma delas é o inverso da oulra).
Solução
Sendo a, b, c e d as raízes de P(x), vamos escrever apenas a primeira, a
terceira e a quarta relações de Girard:

7
a +b + c +d =— (D
2
. . 28
abc + abd + acd + bcd =------ = -14 CD
2
abcd = —— - -6 (Jll)
2

230
Sendo a e b as raízes recíprocas, temos:
ab - 1 (IV)
Substituindo (IV) em (lll), obtemos:
cd = -6 (V>
A relação (II) pode ser transformada em:
ab (c + d) +■ cd (a + b) = -14
1 ^6
isto è
(c + d)-6(a + b) = -14 (VI)
Da relação (I), tiramos:

c + d = ~-(a + b) (Vil)

Substituindo (VII) em (VI), temos


— - (a+ b) - 6(a + b) =-14

donde:

a +b = — (VIII)
2
Consideremos agora o sistema formado petas equações (VIII) e (IV)
|a + b = |
l 2
[ab =1
1 1
Resolvendo este sistema, obtemos a = 2 e b = — (ou a — e b = 2. o que
2 2
dá no mesmo). Daqui por diante, podemos completar o probiema de dois
modos.
1° modo
1 podemos dividir P(x) por (x-2)í x-^ L
Como jã temos as raizes 2 e -,

obtendo o polinõmio A(x)=x2 -x-R, cujas raizes são - 2 e 3, Assim, as


1
raizes de P(x) são: 2, —, - 2 e 3.

2’ modo
Podemos substituir (VIII) em (VII). obtendo:
c+d =1 (IX)
Consideramos em seguida □ sistema formado pelas equações (IX) e (V):
íc + d = 1
|cd = -6

cuja solução é c - - 2 e d = 3 (ou c = 3 e d = — 2, o que dá no mesmo)

231
14,20) Consideremos um número complexo z 0 e um número natural n > 1.
Sejam xt xn as raízes n-ésimas do número z. Mostre que:

x1 + x2+ +xn = 0
Solução

As raizes n-ésimas do número z são as raizes da equaçao xn - z, a qual é


equivalente a;
xn + Oxx1 + ... -2=0 (I)

Mas, de acordo com a primeira relação de Girard, temos:

+ x2 +. °=0
1

14,21) Seja n um número natural qualquer tal que n > 1. Mostre que:

2jt 4jt
COS----- + COS—+ ... + COS
n n n
2rt 4rr 2(n-1)rc
sen — + sen — + ... + sen - 0
n n n

Solução
Sejam íüq, íü2, .... üj^ as rarzes n-ésimas do número 1. De acordo
com o exercício anterior, temos:

<d0 +-üj1 + ío2 +... + = 0 (!)


Porém:
íad =1
2rr 2n
üj, = cos — + j sen —
nn
4ji . 4n
í£2 = cos------ F-1 sen-----
n n

ÚJ. = CDS
2(n-1)x
+ j sen
2jn -1)tc
n n
Substituir em (I) e separando as partes real e imaginária, temos:

. 2jt 2(n-1)n]
4 ji
1 + COS—- + CCS“— +•...+ COS
n n n JJ
,r 2rt 4n
+ H Sen — + sen — ■r. , + sen
2(n - 1)jt
= 0
L n n n

232
Para que esta igualdade seja verdadeira, devemos ter:

. 2n Alt 2(n-1)n
=O
1 + COS------- k COS------- k... + COS
n n n
2n An 2(n-1)n
sen — + sen — + ... + sen =
n n n
o que nos leva à tese:

14.22)As raízes do polinômio P(x) = x3 -(5 + 7õ)x2 + (6 + õ75)x-675 são me-


didas dos lados de um triângulo retângulo. Determine essas raizes:

Solução
As raízes a, b e c do polinômio P(x) devem ser números reais positivos.
Supondo a > b > c, pelo teorema de Pitágoras, devemos ter:

a2 = b2 + c2

Vamos escrever apenas a primeira e a segunda relações de Girard:

a + b + c = 5 + 75
ab + ac + bc = 6 + 575
Temos, então:
a + b + c = 5 + 75 => (a + b + c)2 = (5 + 75)2 => a2 +b2 + c2 +
a2
+2(ab + ac + bc) = 25 + 1075+5=> 2a2 = 18 => a = 3
6+575

Obtida a raiz 3, dividimos P(x) por x — 3, obtendo


o polinômio A(x) = x2 - (2 + 75 )x + 2^5, cujas raí­ 2
zes são 75 e 2. Assim, as raizes de P(x) são 3,
75 e2. <5

14.23)As raizes do polinômio P(x) = x3 -14x2 + 63x-90 são medidas dos lados
de um triângulo. Calcule a área desse triângulo:

Solução
Sendo a, b e c as raízes de P(x), vamos escrever apenas a primeira relação
de Girard:
a + b + c = 14
Seja p o semiperímetro do triângulo:
a+b+c
p=—— 11
2
=7

233
De acordo com a fórmula de Herao (veja volume 5 desta coleção), a área S
do triângulo é dada por:
S = 7PÍP ”a)(p-b)(p-c) 0)
Mas, observando que P(x) - (x-a)(x - b)(x-c). concluímos que

(p-a)(p-b)(p-c) -P(p) = P(7)= 73-14(7)2+83(7)-S0 = fl


Substituindo em (I), obtemos:
S = 7(7) (8) = 756 = 2d14

1 4.24) Escreva uma igualdade ligando os coeficientes da equação x.3J + kx + m - 0


(onde m / 0), de modo que uma das raízes da equação seja igual ã soma
des inversos das outras duas.
Solução
Sendo a, becas raizes da equação, a condição do enunciado é:
1 1
a - —+- (I)
b c
Vamos escrever a primeira e a terceira relações de Girsrd:
ia + b + c = 0 (II)
[abc = -m (lll)
De (I), tiramos: b + c = abc (IV)
De (II). tiramos: b + c = -a (V)
De (FV) e (V), vem: abc-a (VI)

Substituindo (VI) em (III), temos a = m, isto é. uma das raízes da equação é


© número m. Portanto:
ni3 + km + m - 0

Mas. como m # 0. temos: m2+k + 1 = 0

14.25)Escreva uma igualdade ligando os coeficientes da equação x33 + kx2 + mx +


+ t = 0, sabendo que as raizes dessa equação estão em progressão
geométrica.
Solução
Sendo a, b e c as raizes da equação, suponhamos que efas formem uma
progressão geométrica, nessa ordem:
b2 = ac (J)
As relações de Girard são:
ía +b + c = -k
I (II)
• ab + ac +bc = m (Hl)
abc = -t (IV)
234
Substituindo (I) em (IV), vem: b3 = —t (V)
Substituindo ÇL) em (III), temos:
ab + b2 + bc = m
Isto é, b(a+b + c) = m, donde bk = - m, ou ainda, b3k3 = - m 3 (VI)
-k
Substituindo (V) em (VI), obtemos tk3 = m3

14.26) Escreva uma igualdade envolvendo os coeficientes da equaçao


x -rkx"+mx+ t = ü, sabendo que as raizes da equação formam uma
progressão aritmética.

Solução
Sendo a, b e c as raizes da equação, suponhamos que elas formem uma
progressão aritmética, nessa ordem;
2b = a + c (í)
A primeira relação de Girard é:
a +b + c — — k CO
k
Substituindo (I) em (II), obtemos b =—. Assim, concluímos que uma das
3

raizes da equação é o número — e, portanto:

3 2

donde:
2k3 -9mk + 27t = 0

Fxerciclos Propostas

14.27)Determine o conjunto-solução de cada equação a secuir, utilizando a


condição dada:
a) X3-6x2 + (12 + í)x-(9 + 3i) = 0
Uma das raizes é igual à soma das outras duas.
b) 3x3 - 2x2 - 12x + B = 0
Uma das raizes é o triplo da outra raiz.
C) X3-4x2-3X + 18 = 0
Hã uma raiz dupla.
d) x3-14x?+ 59X-116 = 0
Uma das raizes é 5 + 2i.
e) 2x5 + 3x*+1 5x3 + 24x2 + 32x + 48 = 0
O número 2i é raiz dupla.

235
f) x3 +3x2-16x-43 = □
Hã duas raizes simétricas.
g) 6x3-23x2 + I6x-3 = 0
Há duas raízes recíprocas (uma é o inverso da outra),
h) 2x3-5x2-2x+S = 0
7
A soma de duas raízes é igual a
2
i) x3 +x2 -14x-24 = 0
A diferença de duas de suas raizes é igual a 6.
j) 2x4 +25x3 + 103x2 + 176x + 64 = 0
Há uma raiz tripla,
k) 2x3 — 19x2+ 45X-18 = 0
Uma das raizes é igual ao produto das outras duas.
l) x3 -12x2+ 2ÜX + 96 = 0
4
A razao entre duas de suas raizes ê iguai a —.
3

14.23) Uma das raizes da equaçao 2x 3 - 9x2-t-x+k - 0 é igual à soma das outras
duas
a] Determine n vaJor de k,
b) Dê o conjunto-solução da equação

14.29) Dê uma equação de grau 3. cujas raízes sejam —,2 e 1 + L


3
1 4 30) Dê uma equação de grau 3, cujas rarzes sejam 5 (dupla) e 2 - i (simples).

14.31) Dê uma equação de grau 4, cujas raízes sejam 3 (dupla), 2 (simples) e


— 1 (simples)

1 4.32) Determine o valor de k na equação 3x2 — 7x + (k + 2) = 0, de mcdc que unna


de suas raizes seja o sêxtuplo da outra.

1 4.33) Determine o valor de k na equaçao 5x2 — 1 2x + 6k + 2 = 0, de modo que


suas raizes sejam recíprocas.

14.34) Sejam a e b as raízes da equaçao 3x2 +4x + 5 = 0 Calcule os valores de.

1
a) ~ i—
a b
b) a2-rb 2
c) ~
a2
+4-
b2
d) a4 + b< e) a3 + b3

(Sugesíâo: para os exercícios b e e, use as identidades (a + b)2: = a 2 + 2ab + b?.


(a + b)3 = a3 + 3a2b + 3ab2 + b3.)

236
14 35)Calcule a soma dos quadrados das raizes da equaçao:
3x? + (5-i)x + (2-3Í) - 0
14.36)0etermine o vaiar de k, de modo que a soma dos quadrados das raízes da
equação 2x2-5x + k = 0 seja a —.
4

14.37)Sejam, a, b e c as raízes da equação 4x 3 - 3x2 + 2x + 6 = □. Calcule os va-


lores de:
.111 r’ 1 1 1
a) -+-
-+—+- e) T2+Z2+7T
abc a' b ‘ c"
b) a2+b2 + c 2 f) a4 + b4 + c4
c) a2b2 + a2c2+ b2^2
2c g) a 3 +b 3 + c3
1 1 1 . , a b c
d) —- + —- + — h)
c — k + — + -—
ab ac bc bc ac ab

14.33) Calcule a soma dos quadrados das raízes da equação 4x9 fl + (3 + Í)X7 4-

+ (6-i)x3+4x — 1 = 0.

14.39)Sejam a, b e c as raizes da equação 2x3 + 3x2 + 4x + 6 = 0. Em cada caso


a seguir, dê um polinõmio P(x) de raizes x^ x2 e x3, satisfazendo a con­
dição dada:

a) x, - ab; x2 = ac; x3 = bc
1 u 1 1
b) x. = ------ ; x, =b =------ ; x3=c-------
bc ac 3 ab
1 1 1
c) x, ~ ; X-, - — • x, = —
a b c

14.4Q)Sejam a e b as raizes da equação x2+2x + 3 = 0. Dê um polinõmio de


raizes Xí e x9
2'
, tais que Xi=a 2 +b2 e x 2 =
x1 = a 2 -= + b3
a33-*-^3.

14.41) Sejam a. b e c os números complexos distintos, tais que:


a3 4*73 + 13 = 0
b3+7b + l3 = 0
c3 + 7c +13 = 0

Calcule □ valor de a + b

237
14.42) Consideremos a equação x3+kx + m-0, onde k e m são reais e m x ü.
Mostre que, se todas as raízes da equação são reais, então k < 0.

14.43) Determine as raízes da equação 5x3 - 2x2 - 4x + 40 = 0, sabendo que elas


tèm o mesmo módulo.

14.44) Resolva cada uma das equações a seguir, utilizando a condição dada:
a) x3— 3x2-13x + 15 =0
As raízes estão em progressão aritmética.
b) 2x3 -21x2 +42X-16 = 0
As raízes estão em progressão geomêtrica.
c) x3-9x2 +11x + 21 = 0
As raizes estão em progressão aritmética.
d) x3+7x2 + 14x + 8 = 0
As raizes estão em progressão geométrica.
e) 3x3-4x2 -48x + 64 = 0
As raízes estão em progressão harmônica (ou seja, seus inversos eslao
em P.A.)
0 X4 - 1 2x3+1 4x2 + 1 32x - 1 35 - 0
As raízes estão em progressão aritmética.

valer de k de modo que as raizes


14,45) Determine o valor da equação
3 2
x3 +■ 6x2 +kx-24 = ü formem erma progressão aritmética.

14.46) Resolva a equação x4 - 6x3 + 6x2 + 24x-40 = 0. sabendo que há duas


raizes simétricas.

14,47) Resolva a equaçao 3x4 + 2x3 - 28x2-13x + 9 = 0, sabendo que há duas


raizes reciprocas.

14 4S)Resolva a equação 2x4-13x3 + 4x2 +13x-6 = 0, sabendo que □ produto


de duas raizes é igual a 3.

14 49) Consideremos a equaçao 2x4 + kx3+mx2+14x-8 = 0. Duas de suas


raizes têm soma igual a 1 e as outras duas têm produto igual a 2.
a) Dê o conjunto-solução da equação
b) Determine os valores de ke m.

14.50) As raizes do polinõmic P(x) = x3 - (7 + %/?)x2 + (12 + 7V7)x +12-77 são me­
didas nos dois lados de um triângulo retângulo. Determine essas raizes.

238
14 51)As raízes do polínõmio P(x) = x3 -6x2 + kx + m sao medidas das lados de
um triângulo Obtenha a área desse triângulo em função de k e m

14 S2)Sendo a, b e c as raízes da equação x3 -Sx2 + 24x-16 - 0, calcule os va­


lores de:

a) log2(a2 +b2 + c2)


(ir tt
b] sen |
U*-- b c.
a o 0
2 c 0
—3 7 b

14.53) Sejam a, b e c números complexos não nulos, tais que a+b+c=0 e


ab + ac + bc = 0. Mostre que |a] = |b| = |c|.

14.54)Em cada caso a seguir, escreva uma igualdade ligando os coeficientes da


equação, utilizando a condição dada:

a) x3 + kx2 + mx + t = 0
Uma das raízes é igual à soma das outras duas.
b) x3 + kxz + mx + t = 0
Há duas raizes simétricas.
c) x3 + kx + m = 0 (com k * 0 e m * 0)
Há uma raiz dupla.
d) x3 + kx2 + mx + t = 0
As raizes formam uma progressão harmônica (ou seja, seus inversos
estão em P.A.)
e) x4 + kx3 T-mx2 + tx + n = 0
O produto de duas raízes é iguaí ao produto das outras duas.
n x4 +kx3 + mxs + tx + n = 0
A soma de duas raízes ê igual à soma das outras duas.

14.55)Determine os números a, b e c (nao nulos), sabendo que eles são raízes


da equação x3 - ax2 + bx - c = 0.

14.56)Sendo <í> = cos—+1 sen^, calcule o valor de w + w2 + ÍÜ3 + + oj5 + w6

(SupesJáo: observe que uj ê uma cas raizes sétimas da unidade: veja exercício 14.20)

239
27r 2?r
4.57) Consideremos a número natural n > 1, Sendo tu —COS— + iser> , calcuíe
n n
os vaiares de:

a) íil + tu2 + ÍI)3 + cd""1

b) ojn

4, 58] Sendo to= cos—+isen —, calcule as vaiares de:


3 3
T 4 S
a) ÍD+ + CO ++ ü)
ÍO4 + Co*
b) O)s

2n 2jí
4.59] Consideremos o número natural n > 1. Sendo co=cos—+isen —, calcule
n n
o valor de S;
S = 1* 2üj+ 3oj2 + 4üj3 + ... + noj'

(Suges fao: calcule S - cuSj

240
Capitulo

Raízes Racionais

15.1 —Teorema das Raízes Racionais

Consideremos a polinomio:
P(x) = anx'> + ari_1xft-1 +...+- a.jX + a0
de grau n > 0, cujos coeficientes são todos inteiros, com □. Consideremos
também um número racional — (pe Z* s q <= Z*J. com p e q primos entre si. No
q
item seguinte demcnstraremos que:

Se - é raiz de P(x), então p é divisor de aQ e q é divisor de an. (15-1)


q

Observemos que:
1a) esse teorema não garante que exista a raiz garante apenas que se existir,
9
p é divisor de ane q é divisor ce an;
2°) o teorema só vale quando os coeficientes são todos inteiros:

3a) ao colocarmos a 0, estamos excluindo o caso em que - = 0, pois este ê de


q
análise imediata.

Exemplos

a)Consideremcs o polinomio:

P(x) = Gx5 -7XJ + 9x3-X-15


I I
an aa

'divisores de a0 ; ±1; ± 3; ± 5;+15


divisores de an : ± 1; ± 2; ± 3; ± 6

241
Se o polinômio admitir uma raiz racionai — (com p e q inteiros não nulas, p
q
e q primos entre si), p ê divisor de afl e q é divisor de a0 Dividindo de
todos os modos possíveis, um divisor de a0 por um divisor de aa, obtemos

o conjunto dos possíveis valores da raiz — :


q
M = J±1;±3.±5; ±15;±-;±-;±-;±-;±-,±-;±-;+ —!■
l 23622362)

Isto não significa çue, obrigatoriamente, um dos elementos de M seja raiz de


P(x). Significa apenas que se P(x) tem uma raiz racional (não nula), ela é
obrigatoriamente um elemento de M.

b) Seja o polinômio
P(x) = 2x^ — 7x2 - 17x2 + 58x - 24

l I
a., aD
Tdivrsores deao : ±1; ±2; ±3; ±4; ±6; ±0; ±12; ±24
n : ±1; + 2
[ divisores de a„

Seja M o conjunto dos possíveis valores da raiz - :


q
3)
±T. ±2; ±3; ±4; ±6; ±8; ±12; ±24; ±^; ±->
M= ,- >
2 2]
Para sabermos se algum elemento de M é raiz de P(x), vamos
calculando os valores numéricos de P(x) para os diversos elementos de
M (obviamente começando pelos inteiros), Esse cálculo pode ser feito
pelo dispositivo de Briot-Rufíini:

P(1) = 12
2 -7 -17 50 -24
P(-1) = _go
P(2) = 0 2 é raiz 1 2 -5 -22 36 12

-1 2 -9 -8 66 -90

2 2 -3 -23 12 0

Q(x) - 2x3-3x2-23x + 12

As verificações seguintes podem ser feitas no polinômio Q(x). Devemos


recomeçar essas verificações experimentando novamente o número 2.
pois ele pode ser uma raiz de multiplicidade m > 1,

242
2 -3 -23 12

Q(2) = -30 2 2 1 -21 -30


Q(-2) = 30 -2 2 -7 -9 30
0(3) = -30 3 2 3 -14 -30
Q(-3) = 0 -3 é raiz 4
-3 2 -9 0

2x2 - 9x + 4
Resolvendo a equaçao 2x2 -9xi-4 = 0. obtemos as raízes que faltam: 4 e

Observemos que estas duas raizes são também elementos de M.

15.2- Demonstração do Teorema

Suponhamos que o número P.


(satisfazendo as condições do enunciado do
q
teorema) seja raiz do polincmic:
n
P(x) = anx + a. + ... + a1X + aQ
Devemos ter:
n
P □ -0
+a
q
Isto é,
anPn +3n-iPn” , , aiP.A
4-„ a ~0
qn q"“1 q

Multiplicando todos os termos por qrt, obtemos:

anpn +an_1pn 1q + .,, + a1 pqn"1+aoqn =


=00 (I)
Dividindo por p todos os termos de (|) e passando o último termo para o lado
direito, lemos:
_ — r»
q+.. + a,q _ apq
anP n-1 + a P (H)
p
0 lado esquerdo da igualdade (II) nos dã um número inteiro e. portanto, o lado
direita também deve ser inteiro. Mas, como p e q são primos entre si. para que o
número:
aqqn
P
seja inteiro, ao deve ser divisível por p, isto é, p é o divisor de ao-
Dividindo por q todos cs termos da igualdade (I), e passando o primeiro termo para
o lado direito, temos:

a iP + „. + a1 pqn-2
n + aoq = _^£2 (III)
q

243
O lado esquerdo as igualdade (lll) nos dá um número inteiro e, portanto, o laõo
direito também deve ser inteiro. Mas, como p e q são primos entre si, para que o
número:

q
Seja inteiro, an deve ser divisível por q, isto é.qé divisor de an

15.3 — Consequências

Do teorema anterior, tiramos imediatamente duas consequências:

1°) Se P(x) admite uma raiz inteira k x- 0, então k deve ser divisor de aQ,.
2”) Suponhamos que a„ = 1. Então, se P(x) admitir raízes racionais, estas serão
necessariamente inteiras.

Exercícios Resolvidos

1 5.1] Determine o conjunto-sofuçao da equação 3x s-4x4 + x3 + 6x2-2x = 0-

S I u çâ o
Neste caso podemos colocarx em evidência:

- 4x
3X5 - 4 -X
4x" -r- X 3 + 5X 2 — 2x = 0 e=> x(3x4 -4x3 + x2 + 6x -2) = 0
+ 5x
x = 0 ou Sx"1 - 4x3 + x3-f-6x - 2 = 0
■4 3
Vamos agora tentar determinar as raízes do polinõmio P(x]=3x -4x +
+ x2 + 6x — 2 usando □ teorema 15,1,

divisores de a0 : ±1: ± 2
divisores de an : ±1; ±3

Assim, o conjunto das possíveis raízes racionais de P(x) é:

1 21
M = ±1:±2;±3;±-;±-l
3 3J
Começaremos as tentativas pelos números inteiros:

3 1 6 -2
P(1) = 4
1 3 -1 0 6 4
P(-1)=0 =>-ié raiz
-1 3 -7 8 -2 0

Q(x) = 3x3-7x2 + 8x-2

244
Continuaremos as tentativas, agora usando o polinómio Q(x). experimen­
tando novamente a raiz —1, pois ela pode ser raiz de multiplicidade m < 1.

3 -7 8 -2
-1 3 -10 18 -20
2 3 -1 6 10
-2 3 -13 34 -70
3 3 2 14 40
-3 3 -16 56 -170
3 -6 i6 0
3

Q(i)-
« 1 - ■
□ => — e raiz 3x2-6x + 6
3
Resolvendo a equação 3x2 — 6x + 6 = 0, obtemos as raízes que faltam: 1 + i
e 1 - i.
Portanto, o conjunto-solução é:

0; „1; 1; 1 + i; 1-i
S =

15.2) Decomponha o polinómio A(x) = 3xs - 4/1 3 + 6xa-2x em fatores do


primeiro grau.

Solução
O polinómio A(x) coincido com o lado esquerdo da equação dada no
exercício anterior. Aproveitamos então as raizes que já foram obtidas e
temos:
A(x) = 3x(x + 1][ x--l(x-1-i)(x-1 + i)

7 2
15.3) Determine o conjunto-soluçao da equação x3 4- - X -5x-12 = 0.
2
Solução
Vamos tentar obter as raizes usando o teorema 15.1. Mas acontece que o
coeficiente de x2 na equação dada não é inteiro. Assim, vamos multiplicar
todos os termos por 2, para obtermos uma equação equivalente á equação
dada, cujos coeficientes são todos inteiros:

3 7 2 -5x-12 = 0^2x3 + 7xa-Wx-24 = 0


X + —X
2
Determinaremos, agora, as raizes do polinómio:
P(x) = 2x3 + 7xa — 10x —24

divisores de a 0 : ± 1, ± 2; ± 3; ± 4; ± 6; ± 8, ± 12; 1 24
divisores de a n:±t±2

245
Conjunto das possíveis raizes racionais de P(x):
I
M 7= S ± 1; ±2; ±3; ±4; ±6; +8; ±12; ± 24; +-; ±-l
131

2 7 -10 -24

1 2 9 -1 -25
-1 2 5 -15 -9
2 2 11 12 0
_ _________ f

P(2J = 0 => 2 é raiz 2x2 + 11x + 12


Resolvendo a equação 2x 2
+11x +12 = 0, obtemos as outras raízes: - 4 e
_3 "
—. Portanto, o conjunto-solução da equaçao dada é:
2

S = * -4:

15.4) Mostre que o número 1 + 72 é irracional,

Solução
Sabemos que os números 1 e 72 são reais. Sabemos lambam que a soma
de dois números reais é ainda um número real. Portanto, c número 1 + 72 é
real, resta saber se é racional ou irracional.
Seja x=1 + 72. Temos:

x 1+ 75 =s x2 = (1+ 75)2 z=> x2 = 3 + 275 =* x2 -3 = 2 /2 =>


=> (x2- 2)2 = [2 72)2 => x4 - 6x2 + 9 = 8 => x4 -6x2 + 1 = 0
Esta última equação tem todos os coeficientes inteiros. Aplicando o teorema
15.1, concluímos que os únicos números racionais que poderíam ser raizes
dessa equação são 1 e-1. Assim, concluímos que 1+72 é irracional.

1 5.5) Mestre que o número ^7 + 572 + $7 - 572 é inteiro.

Solução
Seja x = 7? + 572 + y/^ - S\/2 É fácil verificar que o número x è real;
resta mostrar que ele é inteiro. Para facilitar a notação, faremos:
a = ^7 + 5 v'2 e
e b = 7?- 5 72
Lembrando que:
(a+bj3 = a 3 3a 2b + 3ab2 +b3 = a3 + b3 + 3ab(a + b)
temos:
x = a+ b=>x3 = (a + b)3 = a3 + b3 +3ab(a + b) = a3 +ba + 3abx
x
246
Porém:

^7+575
a = ^7 :=> aa3J = 7 + 5^2
+ 5V2 =

b = 7? -572 => b33 = 7 - 572


ab ^7 + 572 x^7-572 - ^(7 + 5T2)(7-5T2) = $7Z-(5T2)Z = -1

Assim:
x3 = a3+b3 + 3abx = 7 572+7 - 572 + 3(-1)x = 1 4 - 3x
isto é:
X3 + 3X-14 = 0
O conjunto das possíveis raizes inteiras desta última equação é:
M - {+1; ± 2; ± 7; ± 14}
Fazendo-se as tentativas:

1 O 3 —14
1 1 1 4 -W
-1 1 -1 4 -18
2 ,1 2 7, 0
2 é raiz A(x) = x* + 2x + 7

C discriminante do polinõmio A(x) é:


a — —>
A = 2 — 4(1X7) =4 -28 = -24 < O
e portanto, as raizes de A(x) sãc imaginárias. Concluímos, então, que .
única raiz real da equação:
x3 + 3x-14 = 0
é o número 2. Como o número x é real, concluímos que x = 2.

15.6) Um número complexo é chamado de número algébrico se ele pode ser raiz
de um polinõmio (não identicamente nulo) de coeficientes inteiros; um
número complexo que não seja algébrico ê chamado de número
transcendente. Na realidade, praticamente todos os números com que
tratamos em Matemática Elementar são algébricos. Todos os números
racionais são algébricos; alguns números irracionais, como, por exemplo:
72, ^7, 1 - 72
são também algébricos, Como exemplo de número transcendente podemos
dar o número k, Outro exemplo de número transcendente é o número e.
base do sistema de logaritmos neperianos (veja capitulo 10 do volume 2
desta coleção). Com base no que acabamos de dizer, mostre que cs
seguintes números são algébricos:
a) 7s + 72 b)3 + 4i

247
Solução
a) Fazendo x = 75 + 72, temos:

X = 75 + 72 => xX'2 = (7S + 72)2 = 5 + 275 72 + 2 =>


x2 = 7 + 27w => x2 - 7 = 27lÕ => (X2-7)2 =(27W).2:
x4 - 14x2 + 49 - 40 = x4 - 14x2 + S =0

Portanto, 75 + é uma das raizes da equaçao x4-14x2 + 9 = 0, a qual


tem todos os coeficientes inteiros; assim, podemos dizer que o número
75 + 72 é um número algébrico.
b) x = 3+ 2Í x-3 = 2i=>(x-3)2 =(2i)2
=^x2-6x + 9 = -4=>X2-6x+13 = 0

O número 3 + 2i ê uma das raizes da equação x' -6x +13-0, a qual tem
todos os coeficientes inteiros, portanto, ele ê um número algébrico.

15.7) Determine todas as raízes do polírõmio P(x) = 2x5 - Sx4 + 16x3 - 23x! +
+ 32x -12, sabendo que ele admite uma raiz imaginária pura.

Solução
Note que jã resolvemos exercício semelhante a este no capítulo 11
(exercício 11.41). Seja ai a raiz imaginária pura procurada (com a* e R),
Devemos ter P(ai) = 0

2(ai)5 - 5(ai)4 + is(ai)3 - 23(ai)2 + 32(ai)-12 = 0

Efetuando as operações e separando as partes real e imaginária, obtemos:

(~Sa4 + 23a2 -12)+(2a5 - 16a3 + 32a)i = 0


Devemos ter, então:

-5a4 +23a2-12 = D (J)


2a5-16a3 + 32a = 0 (10
O conjunto-soluçáo da equaçao (I) é:

SH2: ’2: &


e o conjunto-solução da equação (II) é:

S2 = {0; 2; -2)

248
Os números que satisfazem simultaneamente as equações (I) e (I!) sao
2 e - 2, portanto, temos a = ± 2. Concluímos, então, que c pnlíncmio da­
do admite duas raizes imaginárias puras: 2i e — 2i
Assim, temos
p(x) = (X - 2t)(X ^2í)xA(x) = (X2 + 4)xA(x)
xa+4
Dividindo P(x) por x 2 + 4, obtemos A(x):

A(x) = 2x3-5x2 + 8x- 3


Vamos agora verificar se A(x) admite alguma raiz racional.
divisores de - 3: ±1; ±3
divisores de 2 : ±1; ±2
Assim, o conjunto das possíveis raízes racionais de A(x) é:

M= ±1; ±3; +-, ±- r


+-; ±J'
I 2 2J
2 -5 8 -3
1 2 -3 5 2
-1 2 -7 15 -18
3 2 1 11 30
-3 2 -11 41 -126
1 ^2 -4 0
6,
T
0 — é raiz 2x? -4x + 6
2

Resolvendo a equação 2x2 - 4x 4- 6 = 0, obtemos as raizes que faltam:


1 + Vii e 1 - Vii. Então, o conjunto de todas as raizes de P(x) ê:

2i; -2i; 1 + V2i; 1-Vãi


2

Exercícios propostos

15 0) Consideremos a equação 6x7 -14x5 + kx2 - 3x + 4 ~ 0, andeke?/


Determine:
a) as possíveis raizes inteiras dessa equação;
b) as possíveis raizes racionais não inteiras dessa equação.

15.9) Consideremos □ polinõmio P(x) - 7x3 + kx2 -3x4-6, onde k e O número


| poderia ser raiz de P(x)?

249
2
15.10) Seja A(x) = 4x37 -5x22 + kx 10 - 7. Sabe-se que □ número - ê raiz de A(x).
w

O número k É inteiro?

15,11) Resolva as equações7

a) -9x3 25x2-17x-12 = 0
b) 55xx2-52x + 15 = 0
4x^ — 24x3 + 55
c) x4 -3x3 -6x2 + 3x =
7 2 14 „ „
d) x3 -—■ x ----- x + a = o
3 3

15,12) Decomponha o polinômio P(x)= x 4-8x3 + 13x2 +12X-10 em fatores dü


primeiro grau.

15.13)Mostre que o número 72 + 1 é irracional.

15.14) Mostre que o número ^26+ 1573 +#26-1573 é inteiro.

15.15)Mostre que o número ?/2+i é algébrico.

1 5.1 6) Resolva a equação x5s — 2x*


2x4 - 5x3+14x2 + 6x - 20 - 0, sabendo que uma
de suas raizes è o número 2 + i.

"5.17) Resolva a equação xí,+4x 3 + 1 3x 2 + 36x + 36 = 0, sabendo que ela admite


uma raiz dupla.

15.1 Ô) Resolva a equação 2x5-9xJ + 14x 3-13x2 + 12x -4 = 0. sabendo que ela
admite urra raiz imaginária pura.

l5.19)Resolva a equação xB-7x'l+16x2-12 = 0.


(Sugesfão: taça a mudança de variável x2 - y. )

15,20) Resolva a equação x6 + 2xs +2x4 + 2x3 + 2x2 + 2x +1 = 0.

1 5.21) Consideremos a equação x" + 2kx - 2 = 0, onde ktZ, ení2. Essa equa­
ção pode ter raizes racionais?

250
15 4 - Propriedades

Consideremos um polinòmio P(x) nao identicamente nuloi e de coeficientes


racionais. Consideremos ainda os números racionais a, b.
I m e n tais que
7m e 7n sejam irracionais. Pode-se demonstrar que:
1fl)sea + 7m éraizdeP(x), a-7m também o é;
2*) se 7m + 7n é raiz de P(x), os números 7m - 7n, - 7m + Jn e m - 7n
também são raízes de P(x).

Assim, por exemplo, se 2 + 73 é raiz de P(x), 2 - 'ã também é raiz. Se 7b + 75


é raiz de P(x), os números 75-75, -75+7? e /& - 75 também serão raizes.

Exercícios Resolvidos

15.22)Sabe-se que o número 1+72 è raiz do polinómio:


P{x)=x4-2x3+3x2-8x-a
Determine as outras raizes.

Solução
Os coeficientes de P(x) são racionais; portanto, se 1 + 72 é raiz, 1-i 2
também c é. Assim:
P(x) = (x - (1 + 75)][x -(1 - 75)]>A{x) = (x2 - 2x - 1)i«A(x)
x2-2x-1
Dividindo P£x) por x1 - 2x -1, obtemos A(x):

A(x) = x3 +4
r2
Resolvendo a equaçao x + 4 = 0 obtemos as raizes que faltam: 2i e
-2i. Portanto, as raízes de P(x) são:
1 + 75,1-75, 2i e -2i
15 23) Determine todas as raizes do polinómio P(x)=x6 -x4+7x2 + 9, sabendo
que uma de suas raizes è 75 + 75.

Solução
Os coeficientes de P(x) são todos racionais; portanto, se 72 + 73 é raiz, os
números 72 - 73, ** J2 + 73 e - 75 - 73 também serão raizes.

P(x) = [X-(72 4 7ã)][x ^(72-^73)][x-(-72+73 )][x-{-72-73)]xA(x) =


x2-272x-1 x* +272-1

= (x2-272x-1)(x2 + 272x-1)xA(x) - (x4-Wx2i1)>A(x)

251
Dividindo P(x) por x 4 - 10x 2 +1. obtemos A(x):

A(k)= x3 + 1
As raizes de A(x) são ± r Portanto, as raízes de P(x) são

72 - 7ã. Tã - Tãt - 72 + Tã. -7? - 7M -1

15 24) Resolva a equação x 4 - 4x33 + 2x2 - 4x+1 = 0. sabendo que uma de suas
raízes ê o número 2 + 73,

15.25)Resolva a equação x55 + x4-14x3-14x2 + 9x + 9 = 0, sabendo que uma de


suas raizes é o número 72-75.

15.26) Resolva a equação x4 - 5x3 + 4x2 + 10x-12 = 0, sabendo que uma de suas
raizes é o número 72.

1 5.27) Determine as raizes do polinõmio P(x) = x 4 - 4x3 - 4x -1, sabendo que uma
de suas raizes é □ número 79 + 780.

(SugesJáo. escreva o número <9 + TâÕ na forma 7r + 7s onde r e s são racionais


— veja volume 1 desta coleção, p. 261.)

252
z
Capitulo

M. Equações recíprocas

16,1 - Definição

Uma equaçao algébrica de grau n > 0 è chamada de equação reciproca se, e


somente se, fúr satisfeita a condição:

1
ré raiz da equação =£>- é raiz da equação, com a mesma multiplicidade,

Em oulras palavras, dada uma equação recíproca, se um número é raiz, □ seu


reciproco também é raiz, com a mesma multiplicidade. É obvio, então, que o
número zero nunca é raiz de uma equação recíproca e, portanto, o termo
independente da equação reciproca é sempre diferente de zero

Exempla

a) A equação 3x2-8>ix+3 = 0 tem conjunto-solução S = p;- : portanto ê ll


2j
uma equação reciproca.
-i lj-
b) A equação 2x2-5x+2 = 0 tem conjunto-solução S = ^3i: —
Observando

qu4= 3 '
concluímos que a equação è recíproca.

c) A equação 6x3 - 19x2 +14x-1 = 0 tem conjunto-soluçao S=


i J 2]
Observando que:
2 3
o reciproco (ou inversa^ de - é y

o reciproco de 1 é 1

concluímos que a equação é reciproca.


/ 1
d] Consideremos a equaçao (x-4)3ix— = 0, cujas raizes sao 4 e —
4J 4
1
Embora — seja □ reciproca de 4, a equaçao não ê reciproca, pois as raizes

não têm a mesma multiplicidade.

253
3 2
1
e) A equação (X + 5)2 X+ (x + 1) = 0 é recíproca.
5
f) A equação (x -4)(x -7) não é reciproca

Devemos ressaltar que:


1
r = - <=> r 2
r
isto é, os únicos números que são iguais aos seus recíprocos são os números 1
e — 1. Portanto, para r * ± 1, devemos ter r Dai, concluímos que:
r
1a) se uma equação reciproca não admite a raiz 1 nem a raiz - 1. ela deve ser
de grau par;
2°) se uma equação recíproca é de grau impar, ela deve admitir a raiz 1 ou s
raiz - 1

16.2 - Reconhecimento de uma Equação Reciproca

Consideremos a equaçao reciproca:


anxn + 3,_,x' a fl_2xri'z + . . + a2x3 4-a^ + a0 = 0
a 1* r l + a (I)
1 ■
onde, obviamente, an # 0 e a0 r 0. Se x ê uma das raizes de (I), — também e
x
raiz.
Substituindo em (!}, temos:

aan-2 flY
+ a,
+ a,bd + a0 ^0

Multiplicando todos os termos por xn, obtemos:


ar + an-i x-rãn_2x2+ .. + a2
2-xo-2+atx + aax" = 0
isto ê:
2
nx + a1xf*-1 + a2xn‘2 + ... + an_2x2 +
ao*" +aaJl_1x + an = 0
As equações (J) e (II) devem ter as resmas raizes, com as mesmas
multiplicidades; portanto (veja 11.8) os coeficientes dos termos de mesmo grau
devem ser proporcionais, Sendo k a constante de proporcionalidade, vem:
ao =k^n

a2 = k >an_2
........... (HO
an-2 = k ;<a2
a-. —■ k ?*a.

= k >a0

254
onde:

e a n são coeficientes doa termos extremos


a, b são coeficientes de termos equidistantes dos extremos
a; e a n-2, sáo coeficientes de termos equidistantes dos extremos

De a0 - e an = k>ao vem k = ±1,


Substituindo nas igualdades (IIt), concluímos que:

1°) para k =1, os coeficientes dos termos equídístantes dos


extremos (e os dos extremos) são iguais:
(IV)
2,°) para k = -1, os coeficientes dos termos equídístantes des
extremos (e os dos extremos) são simétricos.

Todos os passos que demos neste item podem ser dados “para trás", isto é,
supondo que as condições (IV) são satisfeitas, podemos concluir que a equação é
reciproca. Assim, podemos dizer que:
Uma condição necessária e suficiente para que uma equação algébrica de
grau n > 0 seja reciproca é que os coeficientes dos termos equídístantes
dos extremos (e dos extremos) sejam iguais ou simétricos
Quando ocorrer k = 1, diremos que a equação reciproca é de primeira classe;
quando ocorrer k = ~ 1, diremos que a equação reciproca é de segunda classe.

Exemplas

a) Seja a equaçao 7X4-9x" + Sx1 + 6x}-9x + 7 = Ü observe que os coeficientes dos


l ' « ------- ! i
termos equidistantes dos extremos (e dos extremos) são iguais Portanto, é
uma equação recíproca de primeira classe.

b) 6x5-7x< + 9x’-9x1+7x-6 = D
1 1 5----- J -! I
Os coeficientes dos termos equidistantes des extremos (e os dos extremos) sãc
simétricos; portanto, ê uma equação recíproca de segunda classe.

C) 5x4i-6x3 -gx2 + 6x4-5 = 0


l i i |
Os coeficientes des termos equídístantes dos extremos (e os das extremos) são
iguais (Observe que neste caso o termo central é igual a ele mesmo.) Trata-se
de uma equação reciproca de primeira classe.

d) A equação 5x4 + 6x3-6x-5 = 0 pode ser escrita:


5x4 + Sxa +• Ox2 - 6x - 5 - 0
I i i I

2S5
Neste caso, os coeficientes dos termos equidísiantes das extremos (e os dos
extremos) são simétricos e, portanto, a equação, para ser reciproca, não pode
ter o termo central.

e) Consideremos a equação 7x4 -9x3 + 2xz + 9x-7 = 0. De acorde com o


que falamos no exemplo anterior, podemos dizer que esta equação não e
reciproca.

f) 6x* - 9x + 6 = 0
Esta equação é reciproca de primeira classe.

g) A equação 4x6+(2 + 5i)x4 4-9x3 + £2 + 5i)X2+4 = 0 pode ser escrita:

4xb + Ox6, + (2 + 5i)x4 + 9x3 + (2 + 5i)x2 + üx 4-4 = □


Trata~se de uma equação reciproca de primeira classe.

h) A equaçac 2x5-7xd + 7x-2 = 0 pode ser escrita.

2x5—7x4+0xa+0xz+7x-2=0
Trata-se de uma equação recíproca de segunda classe.

16.3 — Resolução de uma Equação Recíproca

Consideremos o poíinõmio
P(x) = a^x" + art_4xn“1 + ... + a1x + a0,
Tal que a equação:
P(x) = 0
seja recíproca. Suponhamos que os números 1 e — 1 sejam raizes dessa equação,
com multiplicidades p e q, respectiva mente (eventualmente podemos ler p = 0 ou
q “ 0). Admitindo que haja outras raízes, temos:
P(x) = (x-1)D>(x + 1)<í>fi(x) (!)
de modo que (de acordo com o que vimos no item 16.1) a equação:
B(x) = 0
é recíproca de grau par, que não admite a raiz 1 nem a raiz -1. Resta saber se a
equação ê de 1a ou de 2' classe. Assim, sendo:
B(x) = brixí’-i-bn_1xr,'’ + b^2xn"2+,,,+b2x2 + b1x + b0
teriamos:
B(1) = bn + bn_, + bn_a +... + b2 +bq +b0 - 0
pois, a equação sendo de segunda classe, os coeficientes dos termos
equidistantes dos extremos são simétricos Mas aí, concluiriamos que o número 1
é raiz de B(x) o que não pode ser. Então concluímos que a equação B(x) = 0
deve ser reciproca de grau par e 1“ classe.
256
Portanto, só precisaremos de um processo especial para resolvermos uma
equação recíproca de 1* classe e grau par que não admita a raiz 1 nem a raiz —1.
Tal equação será chamada, daqui em diante, de equação recíproca normal.

Exemplo
Consideremos a equação reciproca;
2x7 -6xe + 3x5 + x4 + x3 + 3x2 -6x + 2 = 0
Experimentando os números 1 e -1, verificamos que 1 é raiz dupla e -1 ê raiz
simples.

2 *-6 3 1 1 3 -6 2

1 2 -4 -1 0 1 4 -2 0
1 2 -2 -3 -3 -2 2 0
2 -4 1 -4 2 0
B(x) = 2x4-4x3 + x2 —4x + 2
A equação B(x) = 0 ê recíproca, de grau par, de 1a classe e nao admite a raiz 1
nem a raiz — 1. Portanto, é uma equação recíproca normal.

16.4- Resolução da Equação Reciproca Normal

Consideremos uma equaçao reciproca normal de grau n. O artificio usado para


n
resolvê-la consiste em dividir todos os termos por x2, e, em seguida, fazer a
seguinte mudança de variável:
1
x + -= y
x
Para facilitar a nossa trabalho nas exercícios, varras deixar prontos alguns
cálculos:
2
1 1
= y2‘ => x 2 + 2+X = y2
x + — = y => x+~
x X x2

Assim: x2 + -^- = y2 -2
x2
3
1 3 .2 1 1
y = x +— yJ =>y3 = x3 + 3x2 x— + 3xxJ^ + =■
x x2 x3

=>y3 = x3+~ 3 1 + 3y
x3 X +~
XJ

Assim: Xd + —= yJ -3y
x3

257
De medo análogo, podemos concluir:
x4+—L = y4 -4ys + 2
x
e assino por diante.

Exercícios Resolvidos

16.1) Resolva a equação 6x€ + 47x4 +13BxJ +154 xJ +138x? + +47x + 6 = D

Solução
1° modo

A equação dada é recíproca, de grau par, porem não sabemos ainda se é


normal. Devemos começar experimentando os números 1 e - 1. Fazendo
isso, podemos verificar que o número 1 não é raiz e que o número - 1 ê raiz
dupla.
0 47 138 194 138 47 6
-1 6 41
41 97 97
97 41 6 0
-1 6 35
35 62 35
35 6 0
Ficamos agora com a equação normal Gx^ + 35x3 + G2x2 + 35xt6 = ü. a
n
qual tem grau n = 4. Vamos dividir todos os termos por xz, isto é, por x2.
obtenrio:
6x4
x2
35x3
X2
62x
X2
i+^
35x
x
+4-o
x

7 _ 35 6
isto é; 6x2 + 35x + 62 + — + — - 0
x x

ou, ainda: 6 f x2 + —^-'|+35 f x. +. — . + 62 = 0 d)


k x
1 . 2^2
Fazendo a mudança de variável — = y, temos x+ —~-y -2. Subs­
x X2
tituindo em (I):
6(y2~2) + 35y + 52 = 0
donde: 6y=+35y + 50 = 0
5 10
cujas raízes sao y' =---- e y" = —
3

25B
Como x + ~ = y* temos as equações:

1 5 1
x+- - cujas raízes são x’ = -2 e x " = - — e
X 2* 2
1 10
x + - = - — , cujas raízes são x1 = -3 e x" = -
2
x 3 ' 3

Assim, o conjunto-solução da equação dada é'

S = '-1;-2;-3;-3;-3
I 2 3
2o modo

Já que os coeficientes da equação são todos inteiros, poderiamos ter


tentado o processo desenvolvido no capitulo 15, isto é, a pesquisa de raizes
racionais.

16.2) Resolva a equação 12x6-40x5-39x'’+170x3 -39x2 -40x + 12 = 0.

Solução

1° modo

Temos uma equação reciproca, de grau par e 1a classe. Fazendo a


verificação, concluímos que 1 e - 1 não são raizes. Vamos então usar o
artificio. Como o grau da equação é n = 6, vamos dividir todos os termos por
n
x2, isto é, por x3, obtendo:
12x3 - 40x2 -39X + 170---------^? + -^4 = 0
x x2 xJ

1 1 | +170 = 0
-40| x2 + —X-| — 39| x + —
ou, 12 x3 + x3
\ x2 x
y3-3y y2-2
donde:
12y3 -40y2 ~75y+ 250 = 0 (D

Fazendo a pesquisa das raizes racionais, descobrimos que uma das raizes
5 5
da equação (I) é y=2’ Dividindo o lado esquerdo de (I) por y~ —,

„ W
obtemos o polinõmio 12y2-10y-100, cujas raizes são y e

H. 5
y *“

259
Lembrando que x + — = y. temos as equações:
x
5 1
x + —= —, cujas raizes são x’= 2 e x" = —
x° 2 2
2=—
10
, cujas raízes são x'= 3 e x" = —
1
X 3 ’ 3
^2 = —5 , cujas raízes são x'=-2 e x" = -
2
X 2* 2
Assim, o conjunto-solução é

„ 1 o 1 „ 1
S= 2;-; 3; -2; —
2 3 2

2° modo

Poderiamos ter feito a pesquisa de raízes racionais já na equação dada.

16.3) Resolva a equaçao x4 3 + x2 + x + 1 = 0.

Solução
Este exercício já foi resolvido no capítulo 11 (exercício 11.38). Vamos agora
resolvê-lo de outro modo.
A equação dada é reciproca normal. Vamos então dividir todos os termos
por x , obtendo:
2 4 1 1
x+x + 1 + — + — = 0
X x‘

ou, X2

y2 + y-1 = O

Esta última equação tem raízes y' =


-1 + 75 -1-75 . Obtemos en-
e y =
2 2
tão as equações:

2 -1+75
, cujas raízes são
-1 + j5+ijl 0 + 2^5
e
X 2 4
1 -1-75 . . -1-75 ±i7i0-275
, cujas raízes
x + — - ----------- .cuias sao --------------- --------------
raízes sao
x 2 4
Comparando a solução dada no capítulo 11 com a solução dada agora,
teriamos um novo processo (além daquele visto no volume 3 desta coleção)
de calcular o seno e o cosseno de —
5
260
16.4) Resolva a equaçao 6x4 - 2Sx3 + 12x2 + 25x + 6 - 0.

Solução
Esta equação n§o é recíproca, mas podemos resolvê-la usando um artificio
semelhante ao que usamos no caso da recíproca normal.
Vamos dividir todos os termos porx":
6x2-25x + 12 + — +-^ = 0
x x2
1 - 25| x — — ) +12 - 0
6| x2 (D
l xj
1
Fazendo x—= y, temos:
x
2
2 \ 1 1
y.2 = x — = X2 +
X2 —2

X2
1
isto é: + —= y 2 +2
x2
Substituindo em (l), obtemos:
6(y2+ 2)-25(y) + 12 = 0

ou, 6yz-2Sy + 24 = 0

- y, = —
8 e
cujas raizes sao
3 ’"4
Temos então as equações:
1 8
x — = —„ cujas raizes são x1 = 3 e x"=-
2
3
1 3
cujas raízes sao x'= 2 e x"=-
2
I. x 2 2
Assim, o conjunto-solução é:
„ í_ 1 o 1
S = {3; 2: --
I 3 2
Exercicios Propostos

16,5) Determine os valores de a e b de modo que a equação Sx4 + (a-b)x3 +


+ (3a - 2b)x 2 4- x - 5 - 0 seja recíproca.

16 6) Determine os valores de a e b de modo que a equaçao 4x3+ax2 + 7x+


+b= seja reciproca.

261
16 7) Resolva as equações:
a) 3X4 -10x3 + 6k2-10x + 3 = 0
b) 2x8 - 6x7 4-5x6 + 6x5~14x4 + 6x3 -b5x2 - 6x + 2 = 0
c) 12x6 — 4x5 - 53x4 + 53x2 + 4x-12 = 0
5x3 + 5x 3x^+31x2 + 3
d) 2 = 7

16 8) Dada a equação ax A + 7x3 + 5x3+bx + 9 = 0, determine cs valores de a e b

de modo que, se um número a é raiz da equação, a numera —- também


a
seja raiz da equação, com a mesma multiplicidade.

252
Capítulo

17 Raízes comuns

17.1 - Introdução

Consideremos dois polinômios: A(x) e B(x). Neste capitulo, vamos esta­


belecer um processo que permita obter os números que são ao mesmo tempo
raizes de A(x) e B(x).

17.2 - Raízes Comuns e MDC

Sejam então dois polinômios A(x) e B(x), de graus maiores que zero. Conforme
vimos no capítulo 10, o máximo divisor comum de A(x) e B(x) pode ser obtido
multiplicando-se os fatores comuns, com os menores expoentes. Mas,
obviamente, ao pegarmos os fatores comuns, estamos pegando os fatores que
trazem as raizes comuns. Dai temos a propriedade:
As raizes comuns de A(x) e B(x) são as raizes do mdc de A(x) e B(x). j

Exercícios Resolvidos

17.1) Obtenha as raízes comuns aos polinômios:


ÍA(x) = x5 -5x4 + 5x3 + 6x2 -11x + 6
|B(x) = x 4 -4x3 + 2x2 + X + 6
Solução
Seja D(x) o mdc normalizado (veja capítulo 10) de A(x) e B(x):
D(x) = x2-5x + 6

Como as raizes de D(x) são 2 e 3, concluímos que as raízes comuns de A(x) e B(x)
são 2 e 3.

17.2) Determine o valor de k de modo que os polinômios:


ÍA(x) = x3 -3x2 + kx+12
[B(x) = x 2-x-2
Admitam uma raiz comum.

263
Solução
Neste caso não é necessário recorrer ao mdc. As raizes de B(x) são 2 e ~ 1.
Temos, então:
[A(2) = 23 -3(2)2+k(2) + 12 = 2k + 8
| A(-1) = (-1)3 - 3(-1)2 + k(-1) +12 = -k + 8
[A(2)^0=>k =-4
^A(-i) = o=5k = a
Assim, podemos ter k = —4 ou k = &

1 7.3) Sendo k e R*. Determine a raiz comum aos polinõmios:


A(x) = kx3 + (3k-2)x2+(3k-4)x + (k-2)e
B(x) = kx2 +(3k- 2)x+(2k-4)

Solução
Vamos determinar um mdc de A(x) e B(x):
kx3 + [3k -2)x2 + (3k- 4)x + (k - 2) kx2 + (3k~2)x + (2k-4)
-kx3-r(-3k + 2)x2+(-2k + 4)x x

kx + (k-2)

kx2 + (3k -2)x + (2k -4) kx + (k-2)


Lv^ ■ f L . Tlv
-kx2 + (-k + 2)x x+ 2
- 2kx+(-2k + 4)
0

O mdc de A(x) e B(x) ê o polinõmio D(x) = kx + (k-2), cuja raiz é


2-k 2-k
——. Portanto, a raiz comum aos dois polinõmios é
k

Exercícios Propostos

17.4) Obtenha as raízes comuns aos polinõmios:


A(x)= x4 +2x3-7x2-8x + 12 e B(x) = x3-4x2 + x + 6

17.5) Obtenha as raizes comuns aos polinõmios:


2x4
A(x)- xs-3x5’ + 2X —4x2 + 12x^8 e B(X) = X4 - 3x3 + 5x2 - 9x + 6

17.6) Determine o valor de k de modo que os polinõmios A(x) = x 3 - x2 + kx -1 e


B(x)=x2-4 tenham uma raiz comum.

264
17.7) Determine os valores de k e m, de modo que os poíínômios A(x) = x 3 +
+ xí-F-kx + m e B(x) = x2-4 tenham duas raizes comuns

17.8) Sendo k e IR*, determine a raiz comum acs poíínômios:

A(x) = kx3 + (2 - k)x2 - (k + 4)x + (k + 2) e B(x) = kx2 + (2-k)x-(2k +4)

17.3 - Raízes Múltiplas e MDC

Consideremos um polinômio P(x) de grau n > 1 e suponhamos que um número c


seja raiz de P(x), com multiplicidade m > 1 De acordo com o que vimas no capitulo
12. podemos dizer que o número c é raiz de multiplicidade m-1 do polinômio P'(x)
(polinômio derivado de P(x)). Portanto, o número c é raiz comum dos polinómlos
P(x) e P'(x); a partir disto, concluímos:

O número c é raiz de multiplicidade m - 1 do mdc dos polinòmios P(x) e P'(x)

Exercícios Resolvidos

17.9) Obtenha as raizes do polinômio P(x)~ x4 - X3-3x2 + 5x - 2, sabendo que


há uma raiz de multiplicidade maior 1.
Solução
P(x) = x4-x3-3x2+ 5x-2
Temos:
P,(x) = 4x3”3x2-6x + 5
Podemos determinar o mdc dos poíínômios P(x) e P'(x), que é
M(x) - x2-2x + 1. É fácil concluir que o número 1 é raiz dupla de M(x);
portanto, o número 1 é raiz tripla de P(x):
P(x) = (x-1)3 xA(x)
Fazendo três divisões sucessivas por X — 1, podemos obter A(x)=x + 2
cuja raiz è- 2. Portanto, as raizes de P(x) são 1 (tripla) e- 2(simples)

17.10) Obtenha as raizes do polinômio P(x) - x4 -10x3 + 37x2 -SOx + 35, sabendo
que há pelo menos uma raiz de multiplicidade maior que 1.

Solução
ÍP(x) = x4-10x3 + 37x2 -60X + 36
(P'(x) = 4x3 -30x2 +74X-60
O mdc de P(x) e P’(x) é o polinômio M(x) - x2 - 5x + 6, cujas raizes são 2
(simples) e 3 (simples). Dai concluímos que o número 2 é raiz dupla de P(x)
e o número 3 é raiz dupla de P(x). Como P(x) ê de grau 4, estas são as
Únicas raizes. Devemos ter, então:
P(x) - (X-2)2(X-3)2

265
Exercícios Propostos
17.11) Resolva a equação x* + x 3 - 3x2 - 5x - 2 = 0, sabendo que há uma raiz de
multiplicidade maior que 1.

17.12) Resolva a equação x5 - õx^ + 7x3 - 2x2 + 4x - 8 = 0, sabendo que hã urna


rais de multiplicidade maior que 1

266
Capitulo

18 Raízes Reais

18.1 ~ Introdução

Neste capítulo, apresentaremos algumas propriedades referentes às raizes reais


de um polinômio de coeficientes reais. Para melhor entendermos, é útil
encararmos os polinômios como funções. (Observe que o item 10.4 deste volume
se refere a isto.) Para tal, usaremos vários conceitos sobre funções, como os de
função crescente, decrescente, gráfico de uma função, etc., já estudados no
volume 1 desta coleção.
Observamos que teremos de lançar mão de várias propriedades que só serão
justificadas no volume 8 (onde serão estudados os conceitos de limite e de
derivada); assim, estas propriedades serão aqui apresentadas sem demonstração.

18.2 - Gráfico da Função Polinomial


Consideremos o polinômio P(x) de grau n > 0 e coeficientes reais:
P(x) = anxn + a^x"-1 +... + a,x + a0
Obviamente, para cada valor real atribuído a x teremos um único vator de P(x) e.
portanto, podemos considerar a função de R em R que a cada x associa o valor
P(x). Pode-se demonstrar que o gráfico desta função é sempre uma linha
continua (como, por exemplo, na figura 18.1), não podendo apresentar "buracos"
ou "interrupções" (como, por exemplo, na figura 18.2).

|P(x) |P(x)

■>
>
x X
Fig. 18.1 Fig. 18.2

Demonstra-se ainda que o gráfico não pode apresentar "bicos" do tipo que
aparece na figura 18.3. É fácil ainda verificar que a insterseção do gráfico com o
eixo vertical nos dá o termo independente a0, e uma interseção qualquer com o
eixo horizontal nos dá uma raiz real r (veja figura 18.4)

267
àP(x) àP(x)

'5*0

X X
Fig. 18.3 Fig. 18.4

Quando ocorrer o que vemos na figura 18.5, a raiz r tem multiplicidade par (a
gráfico tangencia o eixo horizontal sem “cortá-lo). Quando ocorrer o que vemos na
figura 18 6, a raiz r tem multiplicidade ímpar maior que 1 (o gráfico corta o eixo
horizontal, “tangenciando-o"). Se a raiz r for simples, ocorre o que vemos na figura
18.4.

+ P(x) AP(x)

r
>
r x r X

Fig. 18.5 Fig. 18.6

Para aqueles valores de x para os quais a função é crescente (fig. 18.7), o


polinômio derivado P*1'(x) assume valores positivos. Para os valores de x para

os quais a função é decrescente (fig. 18.8), o polinômio derivado P<1)(x) assume


valores negativos.

ÀP(x) àP(x)

P'(x)> 0 p’(x)< 0

a-
x X
Fig. 18.7 Fig. 18.8

Seja k um número real tal que P<1,(k) = 0 Então, a reta tangente ao gráfico o ponto
(k; P(k)) é paralela ao eixo horizontal (veja figuras 18.9, 18.10 e 1 8.11).

268
P(x) aP(x) P(x)

k *x k X k X
Fig. 18.9 Fig. 18.10 Fig. 18.11

Suponhamos que o polinômio seja de grau par. Sendo an o coeficiente do termo


de mais alto grau, demonstra-se que:
1°) se an > 0, tanto para x —>+“ como para x—»— <« teremos P(x) • ~ (como.
por exemplo, na figura 18.12);
2°) se an< 0, tanto para x—»+« como para x —>-<» teremos P(x)-» (como.
por exemplo, na figura 18.13).
aP(x) ÀP(x)

X X

Fig. 18.12 Fig 18.13


Suponhamos agora que o polinômio seja de grau ímpar Demonstra-se que:
1a) an>0
parax +« vem P(x) -» +«>
(veja figura 18.14)
para x —»vem P(x) —> - oo
2°) an<0
parax —> +« vem P(x) —♦
(veja figura 18.15)
parax -»vem P(x) —» +«>

4P(x) / P(x)

■>

x x

Fig. 18.14 Fig. 18.15

269
Exercícios Resolvidos

10.1) Consideremos o poiinõmio P(x) - x3 + 3x2 + 5x + 2.

a) Mostre que P(x) tem apenas uma raiz real.


b) Mostre que essa raiz real ê negativa.
c) Mostre que essa raiz real é irracional.

Solução
a) Como P(x) é de grau ímpar e tem coeficientes reais, podemos garantir
que ele possui pelo menos uma raiz real r; resta mostrar que rea única
raiz real. Para isso, determinemos o polinomio derivado de P(x):
P<1,(x) = 3x2+6x + 5

O discriminanie A desse poiinõmio é:


A =- 62 - 4(3) (5) = 36 - 60 - -24 < 0
Concluímos então (veja capítulo 4 do volume 1 desta coleção) que, para
qualquer x e Et, teremos Pl'1)(x)>0 e, portanto, a função polinomial P(x) é
sempre crescente. Isto significa que o gráfico corta o eixo horizontal apenas
uma vez (sem tangencjá-lo, pois * 0) e deve ter o aspecto da figura
a ou da figura b.

Fig. a Fig. b
àP(x)
b)O termo independente é a a0 = 2 e o
coeficiente do termo de maior grau é
2
= 1 (isto é, an>0). Como □
poiinõmio é de grau impar, o seu gráfico
deve ter um aspecto parecida com o da ■>

figura c e. portanto, a raiz real r é r X


negativa.
Fig. c

270
c) De acordo com o teorema visto no capitulo 15. as únicas raizes racionais
seriam 1, — 1, 2 e - 2. Mas, já sabemos que a raiz real è negativa,
assim, restam — 1 e - 2 No entanto, fazendo a verificação, vemos que
P£-1) * 0 e P(-2) st 0. Portanto. P(x) não admite raizes racionais e a
raiz real r deve ser irracional.

13.2) Consideremos o polinomio P(x) = anxn +... + anx - a0, de coeficientes reais e
de grau impar Mostre que, se an > Q, o polinõmio P(x) admite pelo menos

uma raiz real de sinal contrário ao do termo independente 3a


a0 (supondo
aQ # 0).

Solução
O polinõmio é de grau impar e tem coeficientes reais; podemos então
concluir que ele tem pelo menos uma raiz real. Consideremos então dois
casos:

1a caso aD > 0 àP(x)

Como ar > 0 e o polinõmio é de


grau ímpar, temos

[para x -+ » vem P(x) oo

[para x —»- vem P(x) —+ —»


r X

Portanto, há pelo menos uma raiz real r negativa, isto ê, de sinal contrário ao
de a^ (obviamente, poderá ter também raizes positivas].

2o caso: a0 < G |P(x)


Como an> 0 e o polinõmio é de grau
ímpar, temos:

para x —*» vem P(x) -4 «5 X

para x —> - « vem P(x) -» -«

ao

Assim, há pelo menos uma raiz positiva r (de sinal contrário ao de a0.).

10.3) Consideremos o polinõmio P(x) = a^x" +... + a,x + a0, de grau par (não
nulo), tal que an > 0 e a0 < 0. Mostre que esse polinõmio admite pelo me­
nos duas raizes reais, sendo uma positiva e outra negativa.

271
Solução

O polinômio é de grau par e


an > 0. Portanto temos:

para x —> + <*> vem P(x) —> +«>


para x —» vem P(x)
r2
'a0
Como a0 < 0, o gráfico deve ser
"algo parecido" com o gráfico da
figura ao lado e, portanto, o
polinômio admite pelo menos uma
raiz positiva e pelo menos uma
raiz negativa rj.

18.4) Determine o polinômio P(x), cujo P(X)


gráfico é dado ao lado, sabendo
que seu grau é 3.
18
Solução
Vemos que - 2 é raiz simples e 3 é
raiz de multiplicidade par. Mas,
como P(x) é de grau 3 (e portanto -2
tem três raízes), concluímos que o 3 x
número 3 é raiz dupla. Portanto
temos:

P(x) = a(x + 2)(x - 3)2

Mas o gráfico nos informa ainda que P(0) = 18. Assim:

P(0) = a(0 + 2)(0-3)2 = 18

donde tiramos a = 1.
Portanto, P(x) = (x + 2)(x - 3)2

ou, desenvolvendo, P(x) = x3-4x2 -3x + 18.

18.5) Temos ao lado o gráfico de um AA(x)


polinômio A(x). Esboce o gráfico do
polinômio B(x) tal que B(x) = A(x) - 1.

1
2 X
-1

272
Solução
Ao subtrairmos 1 unidade do . A(x)
polinòmio A(x). o seu gráfico B(x)
deve “descer" 1 unidade {veja 1
capitulo S do volume 1 desta > 2 ,3 4
coleção).

16.6) Consideremos o polinòmio P(x) = x3-2x2 +■ 4x + (k2 - 6k +1), onde k é real.


a) Mostre que, qualquer que seja k e R, P(x) admite uma única raiz real.
b) Determine o valor de k para o qual essa raiz real tenha o maior valor
possível.
c) Determine essa maior raiz real

Solução

a) P(x)= x3-2x2 ~4x + {k2-6k+ 1)


ac

onde a0 é uma constante (para cada valor de k), isto é, ê o termo


independente. Derivando P(x), temos:
Pí1J(x) = 3x2 -4x + 4
cujo discriminante é:
A = (-4)22-4(3) (4) = -32 <0

e, portanto, P^(x) é sempre |P(x)


positivo, o que acarreta que P(x)
é sempre crescente. Como o
grau de P(x) é Impar e o /
coeficiente do termo de mais
alto grau ê positivo, o gráfico é
r X
“algo parecido" com o grãfico ao
lado, e o polinòmio admite uma
3|i
única raiz r.

b) Observando o gráfico, percebemos que. para que r aumente ao deve


diminuir. Portanto, a raiz r atinge o seu vaiar máximo quando ao atingir
seu valor minimo. Mas:
a0 =k?-6k + l
e o valor mínimo de aQ (veja capítulo 4 do volume 1 desta coleção) è
obtido quando k = 3.

273
c) Para k = 3 temos aQ = 32 - 6(3) +1 = -8 e o polinõmio fica:

P(x) - x3 - 2x22 + 4x-8

Fazendo a pesquisa das raizes racionais, concluímos que 2 é raiz e,


portanto, a maior raiz reaí possível para P(x) é o número 2.

Exercícios Propostos

1 fl.7) Consideremos o polinomio P(x) = 5x3 -4x2+2x + 7. Mostre que ele éserri-
pre crescente.

18.8) Mostre que o polinomio P(x) = -x3 4-x2 + 3x-i-4 é sempre decrescente.

18,9) Considere novamente o polinõmio do exercício anterior. Mostre que P(x)tem


apenas uma raiz real, que é positiva e irracional.
18.10) Damos a seguir apenas o termo de mais alto grau e o termo independenle
de uma série de polinômios de coeficientes reais.
A(x) = 6x33 + ... + 7 E(x) = Bx42 +... + 7

B(x) = 6xSa +.,.-7 F(x) = 8x42 + ...-7

C(x) = ~6x53 + ...+ 9 G(x) = -8x42 +... + 7

D(x) = -6x53 + ,..-9 H(x) = -8x43 +„,-7

Analise cada sentença a seguir e diga se é verdadeira ou falsa.

a) A(X) tem pelo menos uma raiz real positiva.


b) A(x) tem pelo menos uma raiz real negativa.
c) B(x) tem pelo menos uma raiz real positiva.
d) B(x) tem pelo menos uma raiz real negativa.

e) C(x) tem pelo menos uma raiz real positiva.


0 C(x) tem pelo menos uma raiz real negativa.

9) D(x) tem pelo menos uma raiz real positiva.

h) D(x) tem pelo menos uma raiz real negativa.


i) E(x) tem pelo menos duas raízes reais.
j) F(x) tem pelo menos duas raízes reais, sendo uma positiva e outra
negativa.
k) G(x) tem pelo menos duas raizes reais, sendo uma positiva e outra
negativa,
I) H(X) tem pelo menos duas raizes reais, Sendo uma positiva e outra
negativa.

274
l8.1T)Temos, ao lado, o gráfica de àP(x)
um polinõmio de coeficientes
reais.
a) Determine esse polinõmio,
supondo que ele seja do
terceiro grau.
b) Determine esse polinõmio,
. 2
supondo que ele seja de -6
grau 5.

18.12)Temos, ao lado, a gráfico de um


polinõmio de grau 4 e coefi­
cientes reais. Determine o poli- /
/
nômio, /

-2 7í

-10

18.13]Consideremos o polinõmio P(x) = 2x3 + 3xz -12x + k,, onde k é real.

a) Determine os valores de k para os quais o polinõmio admite uma raiz


dupla.
bj Determine os valores de k para os quais o polinõmio tem 3 raizes reais
distintas.
c) Determine os valores de k para os quais o polinõmio admite uma raiz
real e duas raizes imaginárias.

18.3 - Teorema de Bolzano


Consideremos um polinõmio P(xj, de grau n > D e de coeficientes reais.
Consideremos também dois números reais quaisquer a e b, que não sejam raízes
de P(X), com a < b. O teorema de Bolzano (matemático tcheco, de origem italiana,
1781-1848) afirma que;

1 0) Se P(a) e P(b) têm sin ais contrários, há um número Impar de raízes


reais entre a e b.
2 “) Se P(a) e P(b) tém o mesmo sin al, há um número par de raízes reais
entre a e b.

Ao enunciarmos esse teorema, estamos considerando que o número 0 também é


par.
A demonstração deste teorema será feita no exercício 13.17; agora daremos
apenas algumas ilustrações gráficas.

275
Exemplos
a) Ma figura 81 >16 temos P(a) e P(b) de sinais contrários e uma raiz real
entre a e b (isto é, um número impar de raizes reafs entre a e b).
Na figura 18.17 temos P(a) e P(b) de sinais contrários e três (número
ímpar) raízes reais entre a e b.

P(a)
P(a) -s

b .
a

P(b)
P(b) ■

Fig, 18.16 Fig 16.17

b) Na figura 18.18 temos P(a) e P(b) de sinais contrários. O número p é


raiz de multiplicidade par. O número r2 ê raiz de multiplicidade 1 (raiz
simples). Portanto, no total, temos um número ímpar de raizes reais
entre a e b.
Na figura 13.19 temos P(a) e P(b) de sinais contrários. O número q è
raiz de multiplicidade impar e o número rj é raiz de multiplicidade par.
Portanto no total, temos um número rmpar de raizes entre a e b.

P(b)
P(b)

a a
"T
i
P(a) ---> P(a) .5

Fig. 18.18 Fig. 18.19

c) Na figura 18.20. P(a) e P(b) têm o mesmo sinal. Há duas (número par)
raizes entre a e b.
Na figura 18.21. P(a) e P(b) têm o mesmo sinal. Não hã raízes reais
entre a e b, isto é. o número de raízes reais entre a e b é zero (número
par).

276
J

P(b) a b

P(a) PO)
/
P(b)
a b

Fig. 18.20 Fig. 18.21

d) Na figura 18.22, P(a) e P(b)


têm o mesma sinal. O número
P(b) -
Fi ê raiz de multiplicidade par e
os números rj e sãa raizes
simples. Portanto, no total, P(a)b’”\
temos um número par de
raizes reais entre a e b b*
a ri r2' r3

Fig. 18.22

Exercícios Resolvidos

18.14) Consideremos o polinõmio P(x) = x3 - 2xz + 4x -1. Quantas raizes reais ele
podería ter no intervale ]-1; 1 [ ?

Solução
P(-1) = (-1)3-2(-1)2 + 4(-1)-1= -a
P(1) = 13 - 2(1)z + 4(1)-1 = 2

Como P(-1) e P(1) têm sinais contrários, há um número ímpar de raizes


reais entre -1 e 1: pode ser uma ou três raízes reais.

1S. 15) Mostre que o polinàmio P(x) = x3 + 7xz + 6x-20 admite pelo menos uma
raiz positiva menor que 2.

Solução
P(0) --20
P(2) = 23 + 7(2)5 + 6(2)-20 = 28

277
Como P(Oj e P(2) têm sinais contrários, concluímos que há um níimeno
impar (pelo menos uma) de raízes reais entre D e 2, isto é, há pelo menos
uma raiz menor que 2 e positiva.

18.16) Determine os valores reais de k de modo que o polinômío P(x) - x3J -3x2 +

-r 4x + (k + 7) admita um número par de raízes reais entre os números 1 e 2


(mas de modo que 1 e 2 não sejam raizes)

Solução
í P(1) = 1- 3 + 4+(k + 7) = k + 9
(P(2} = 23 - 3(2)2 + 4(2) + k + 7 sk +11

Para que haja um número par de raízes reais entre 1 e 2, devemos ter P(1)
e P(2) com o mesmo sinal, isto ê:
P(1)>P(2) > 0
Assim, temos a inequação:
(k + 9)(k + 11) > 0
que, resolvida, nos dá:
k < -11 ou k >-9

1 8.17) Demonstre o teorema de Bolzano,

Solução
Sejam:
Z2 2p as raízes imaginárias de P(x)
*1. *2....... as raízes reais entre a e b

r1, r2,.... rh as raizes reais fora do intervalo (a, bj


Temos:
P(x) = an<x- (x -ZpHx-XiXx- X2)...(x-xq)(x-r,)...(x-rtl)
Ã(xj ' ' B(xj ' C(x)

Como os coeficientes de P(x) são reais, as raízes imaginárias (se existirem)


virão aos pares e ao decompormos A(x), obteremos pares do tipo
(x - z) (X - z) Sendo z = a + pi e Z = o: - pi {com a e X e p e R‘) temas:

(x-z)(x-z) = [x - (cr+ pi)] [x —(a-pi)] = (x^a)2 +j32

Mas. para qualquer valor de x € R, temos (x-o)2 + p2 > 0 e, portanto,

A(x) > 0 para todo xe B.

jP(a) = an A(a)B(a)C(a)
{P(b)=anA(b)B(b)C(b)

P(a) >P(b) = a2 xA(a)xA(b):43(a) >B(b)>C(a)>C(b)

27B
Como as raízes r1, r2 rh estão fora do intervalo [a, b], o produto
C(a)>C(b) será sempre positivo. Assim, temos:
P(a)>P(b) = [a2 xA(a)xA(b)>C{a) >C(b)]>8(a)>B(b) = D>8(a)>6(b)
1 --------------------- .____________—___ _J
D
onde D > 0. Portanto, o sinal de P(a) P(b) depende do sinal de Bfa) B(b).
B(a)>8(b) = (a -x1)(b-x1)(a - x2)(b- x3)...(a - xq)(b- xq)
<0 <õ 1 <0

Cada um dos produtos (a-x^íb-x^ é negativo. Dai concluímos que:


1“) se q é impar, B(a)>8(b) < 0 e P(a)>P(b) < 0 Isto significa que P(a) e P(b)
têm sinais contrários.
2’) se q é par. B(a)>B(b) > 0 e P(a)>P(b) 0, Isto quer dizer que P(a) e P(b)
têm o mesmo sinal.

Exercícios Propostos

18 18)Consideremos o polinõmio P(x) = x 3 - 3x2 +5x -1. Quantas raízes reais ele
podería ter no intervalo ]1; 4[ ?

ia i9)Mostre que o polinõmio P(x) = x3 + 2x2 -7x + 3 admite pelo menos uma
raiz irracional no intervalo ]1; 3[.

18.20) Determine k e í? de modo que o polinõmio P(x) - x3 2 - 8x + k admita um


número ímpar de raizes reais no intervalo ]1; 3[.

Exercícios Suplementares

III.1) Resolva a equaçao 2x 4 + x3 -4x2 +6x + 4 =0. sabendo que duas de suas

raizes sao os números -2 e —1.


2

111.2) Resolva a equaçao 2xB+ xs+ 13x4+8x3+ Sx2+16x - 48 = 0, sabendo que


2i ê uma raiz dupla.

BI.3) Resolva a equaçao x3 + 3x2 - 6x- 8 = 0, sabendo que suas raizes formam
uma progressão aritmética.

III 4) Resolva as equações:


a) x3+ 12x2+17X + 15 = 0
b) 2x3 - 17x2 + 48x- 20 = 0

279
111.5) Resolva a equação x3 + Êx2 + 5x-50 = 0, sabendo que possui uma raiz
dupla.

II 6) Verifique se o número x = ^45 + 2972 + ^45 - 29^2 é inteiro.

III .7) Verifique se o número 73 +• Vfi + ^3 -78 é racional ou irracional.

III 8) Seja rt um número natural tal que n > 1, Consideremos também um número
real k * 0. Dé o valor verdadeiro ou fetóo a cada sentença a seguir, as quais
se referem às raízes n-ésimas do número k.
3] Se n é ímpar, apenas uma das raizes é real.
b) Se n é par e k < 0, nenhuma das raizes é reai.
c) Se n é par e k > 0, há duas raizes reais.
d) Para lodo n, desde que haja raízes imaginárias, eles formam pares de
números conjugados.

280
TESTES OE VERSTIBULARES

Números Complexos

(MACKENZIE-SP) O valor da expressão y - i + ia + i* + t5 +. + r1001 ê:


1}
a) 1 b) i c) — i d) —1 e) 1 + i

2) (PUC-RJ) Seja i o número complexo (0; 1). Então, a soma:


1 + i + i2+P+j.j+í^-1

a) Ê um imaginário puro
b) é um real positive
c) é real se e só se n é impar
d) é uma potência inteira de i
e) é nula

3) (MACKENZIE-SP) A igualdade (1 + i)n= (1-i)rt verifica-se para todos os


números naturais divisíveis par:

a) 1 b) 2 c) 3 d) 4 e) n.r.a.

4) (F.M ABC-SP) Chama-se "rival" de um número complexo z = x + iy o


número complexo z tal que z+z = 2yi.

a) z= —
z
b) z = x - ly

c) z - -z
d) z>z=x2 + y

e) 2x
5
53 (PUC-RJ) Considere os números complexos z = 2 - i e C° “ 2 - i

Então, Se uj indica o complexo conjugado de tu:


1
a) z - -tu b) z - w c) z - -<u d)z =-
to
e) z = íd

6) (U.F.BA) O número complexo z que satisfaz a igualdade (2 -i)xz + 7 +


5i = 8 - 3i é

14 17 6 17. 32
a) b) --5 c) -líl d) 2-—1 e) 2-yi
5 ~5 5 ' T 5 3

261
7) (U.C.MG) O produto (x + yi) (2 + 3i) é um número real, quando x e y sao
reais e:
a) x — 3y - 0
b) 2y - 3x = D
c) 2x + 2y = 0
d) 2x + 3y = 0
e) 3x + 2y = 0
8) (EPUSP) O quocíente do número complexo a + bx pelo número complexo
não nulo c + rd será um número reaí se:
a c
a) — = — b) a 4- b = c + d c) ac — bd d) a4-c + b + d = 0
b d
e) n.r.a

9) (CESGRANRIO) Seja z = x + ry um número complexo nao nulo, onde x e y


x- iy
sâo reais. Se a e b são números reais tais que: ------ ~ - a + Jh
x + iy
podemos afirmar que:

a) fa| + |b| < 1


b) a = -b
c) a = t> = 1
d) a2 + b2 = 1
e) a >0 e b >0

10) (OSEC-SP) Determinando-se os valores reais de m e n de modo que se


tenha: 2{m — rti) + i(m 4- ni) — í = 0, pode-se afirmar que a soma de m e n é
igual a:
a) - 1 b) 0 c) 1 d) 2 e) 3

11) (CESESP—SP) Assinale a alternativa que completa corretamente a sen­


tença: A equação za = z, onde z = a + bi coma e b reais, definida em C.
não admite soluções:
a) reais
b) da forma bi. com b -# 0
c) da forma a + bi, com a * D e b * 0
d) em 'C
e) inteiras

282
12) (U-F. UBERLÂNDIA) Dados os números complexos:
= p(cos6 + i sen 6)
z2 = p(sen 8 + icos 6)

então z, - lz2 é igual a:

a) IzJ - 3|z2|
b) 2pcos8
c) pcosB

d) l*il + 2 l z2 I

e) I Zi I + IZ21

1
13) (FATEC-SP) Sejam jz = -1 e .
2 -------------------- Se A ={Re £|[z| = 1}.
1 + i sen 0
então. A è igual é igual a;

a) {krr, ke £} b)
í^.keZ
2

c) íí.keZ d) {(2k + 1)te, k e Z}


4
íkit . krr , „
e) t—.ks Z ~~, k e Z
l3 S

14) (MACKEN2IE-SP) O número de soluções distintas do sistema;

í|2| - 2
[|z-l| = 1
ê;
a) zero b)1 c)2 d) 3 e) maior que 3

15) (CESGRANRIO) Seja z = x + iy um número complexo não nulo, onde x e y


x-iy 'u
são reais. Se a e b sào números reais tais que: ----- - - a + ic podemos
x + iy
afirmar que:
a) | a | +• | b | < 1 b) a = -b

c) a = b = 1 d) a2+b2 = 1
e) a > 0 e b > Q

2B3
1
16) (MACKENZIE-SP) Se z+- = -1. então o valor de |zfé:
Z

b) 0 c) 1 d) 2 e) d
a,i
17) (MACKENZIE-SP) O número complexo z = a + bi é tal que M=i
Z-1|

Então:
a) a = - b
b) a - b
c) a — 2b
d) a = 3b2
e) a = - 7b

1B) (F.F C.L -USP)Sez e w são dois números complexos quaisquer tars que
|zj - [w| = 1 e 1 + zw *■ 0
z+w .
então o número complexo —■—— e:
1+ zw
a) de valor absoluto 1
b) imaginário puro
c) não real
d) real
e) n.r.a

15) (ITA-SP) Suponhamos que z1 = a+xí e z2=a + yi. a * 0, x * 0. são dois


números complexos, tais que Zl>z2 = 2. Então lemos:
a) zi
z, = Zi e | Zl | = | z3 | = 2
b) = z2 e |z, | = | z2 | = ^2
C) 2. =Z3 e | z, | - | z2 | - 75
d) + Z2 = 2a e- —2 -4
a' +y
e)

20) (MACKENZIE-SP) O número compiexo z = x + ry é Cal que |z-3| = 2.


Então, necessariamente:
a) a<x<2eü<y<2
b) 1 £x<5e-2íy<2
c) -1 £x<2e-3iy£3
d) üíx<3eúsy$3
e) üíx£3ey não tem restrição

264
21) iMACKENZIE-SP) Representando-se graficamente no plano de Argand-
Gauss. os números complexos z tais que z2 - zí, o número de pontos
obtidos é;
a)1 b)2 c) 3 d) 4 e) não sei

22) (CESGRANRIO) O conjunto dos pontos z = x + íy do plano complexo que


satisfazem | z-1 [2= 2x e y > 2 é:
a) o conjunto vazio
b) uma região não limitada do plano
c) todos os pontos x + yi e y>2é:
d) uma reta
e) diferente dos quatro anteriores

23) (MACKEbJZIE-SP) Os números complexos z tais que (z-a)(z - a) = r2.


cem a complexo e r real, representados no plano de Argand-Gauss,
formam;
a) uma reta
b) uma parábola
c) lima elipse com focos em a e ã
d) uma circunferência com centro em a e raio r
e) uma hipérbole

24) (MACKENZIE“SP) Seja t = 2 + 3r um número complexo. Se


A = {ze C] | z-t | < 1} e
B = {ze C|| z = a + bi e b < 3}

então no ptano de Argand-Gauss, Ar. E c:


a) um conjunto vazio
b) uma semicircunferência
c) um semicírculo
d) uma circunferência
e) um circulo
1
25) (ITA-SP) Seja z um número complexo. Se z+— ê um número real, então
z
podemos afirmar:
a) z * 0 e Re(z) > 0
b) I^fzj-O ou |z|=1
c) é necessariamente um número real
d) z2--1
e) n.r.a.
265
26) (ITA-SP) O lugar geométrico no plano complexo, representado pela equa­
ção;
zxz — zQ xz — zoxz + k - 0

onde k é um número real positivo e | Zr ] > k, é:

a) uma hipérbofe com centro Zq


b) uma elipse com um dos focos em Zo
c) uma circunferência com centro em Zo
d) uma parábola com vértice em zD
e) n.r.a

27) (U,F. UBERLÂNDIA) Sejam O. e Z2 as representações gráficas rics


complexos (0 + Oi). (2 + 3i) e (—5 - i), respectiva mente. A menor deter­
minação positiva do ângulo Sq Õ Z? é:

a) 135“ b) 150“ c) 165° d) 120’ e) 175*


4
2fl) (CESGRANRIO) O módulo do número complexo (1 + 3r) é:
a) 256 b)100 C) 31 d) 64 e) 16

29) (FATEC-SP) A representação gráfica do conjunto do z, z e C, tais que

1 —
z !i = 2. no plano de Argand“Gauss é uma circunferência de rato - e
2
|z-1|
W

centro em:
f 4 f 4
a) o b) l;0 C) d} e) (0:0)
3 3 l 3 X ***

SO) (CESESP-PE) Sendo z um número complexo, assinale a alternativa que


não está correta.
a) z+z é um número real
b) zxz é um número real

c) se ?z[ = 1 então z = —
1 z
d) z - z é sempre um número complexo não real
e) z+z é sempre um número real

1) (CESGRANRIO) No plano complexo, o conjunto dos pontos z = x + íy tais


que | Z | 5 í e y 2 0 é;
a) uma circunferência
b) um círculo
C) um quadrado centrado na Origem
d) um semicírculo
e) um segmento de reta
266
32) (MACKENZIE-SP) A função f associa a cada complexo seu argumento. O
vafor de cotg|f (-1 - i)] é:
. '/3
a)-1 b) 0 c)1 d) V2
e)T
33) (CESGRANRIO) Determinando seu argumento, pode-se concluir que o
número complexo (1 + i)12:
a) tem parte real igual à parte imaginária
b) tem parte real positiva e parte imaginária negativa
c) ê real positivo
d) é real negativo
e) é imaginário puro
34) (F.M. ABC-SP) Os números complexos e z2 ambos diferentes de zero,
têm os ângulos polares opostos e módulos inversos.

a) |z1 + z2| = r1-r2


b) z1xz2=1
ri
C) Z1:Z2 = 1

,. — 1
e
d) z2= —
Z1 '/-O
r2\
e) z2=- J_ Z2

35) (F. FRANCISCANA-SP) O número complexo z = - 2 - 2i é escrito na forma


trigonométrica como:

a) z = x/ô rc . n
7t )
cos - + isen —
2 2J
3n 3n
b) z = V§ cos—+isen—
4 4
, 5ti . 5n
c) z = 2V2,cos— + isen —
<44 4 4

7t . 71
d) z = 2 cos—+ isen —
4 4

e) z = 2V21 cos—+ isen —


3 3

287
36) (PUC-SP) Se b ~ 2 (cos 30° + t-sen 30°) e z = p (cos 8 + i xsen 0), os
a fixos correspondentes a b. b + z, b + z + iz, b * iz são os vértices de um:

a) trapézio b) losango
c) quadrado d) quadrilátero qualquer
e) n.r.a.

37) (CESCEM-SP) As funções hiperbólicas cosh z e sinh z sao definidas como:

e^e'* . , e’-e"z
sinh zz == —----------
cosh z = -------------- , . sinh
2 2

ondezs=x + iy e e2 - eli+,v = eK(cos y + i sin y).

Nestas condições, podemos dizer que cosh z é igual a:

a) sinh x xsin y + i cosh x xcos y

b) sinh x xcos y + i cosh x xsin y

c) cosh x xcos y + i sinh x xsin y

d) ccsh x xsin y + 1 sinh x xcos y

e) cosh x xsinh x + i sinh x xcos y

38) (F.C. CHAGAS-SP) Dado c número complexo z = cos^+ isen—. o valor


16 lo
de z’2 é:

. -72 .72
72 ’-^/2 72
a)------- + 1----- b) —
' 2 2 2
c) —72 +i d) -1 + i7s
e) -J2 + í72

39) (F.M. SANTA CASA-SP) Se. x3 - 473 +4Í, x é dado por:

a) 2 cos (3O’ + kxl 50°) +■ 2i sen (3Q° + kXl50°)

b) Ví [cos (60°+kxi20"») + i sen (60°+kx120°)]

c) V3 [cos (60° +k >420°) + j sen (30°+ k xl 20°)]

. r f n 2krt'j (' n 2krr


d) 2 cos — d------- + i sen — +----
L UB 3 J l 18 3

rt 2kn í tc 2kn
e) 2 cos —+■------ + I COS -----4---------
18 3 t18 3

288
40) (F, FRANCISCANA-SP) Dados dois números complexos zi e z21 repre­
K
sentados geometricamente abaixo, e sabendo que 0 < 0f < — e 0 < 9Z < —.
4>• 4
então podemos afirmar que:

a) RÉ(z,>Zj)<0 e lm£z1xzz) = O Z2

b) Re(z1xz2)>0 e lrrí(z1>z2)>0
zi
c) Re(z1xz2} = 0 e li11(z1>z2)<0

d) Re(z,>z2)<0 e lm(z1>c2)<0 «1

e) n.r.a.

41) (U.F.PR) Escrevendo-se Ex = cosx + isenx, onde i ê a unidade imagi-


2
nãria (i = - 1), então Eu + Ew é igual a:

a) Ey + Ev b) Eu + i Ev c) d) E„2, e) Eu + v

42) (F.M.SANTOS-SP) As cinco raízes quintas de z = 15 -1 B^ãi têm o mesmo


módulo e seus argumentos formam uma PA cuja razão ê:
a) 60° b)120° c) 204° d) 216“ e) n.r.a.

43 (F.C. CHAGAS-SP) O número complexo z tem modulo e argumento


■ - - - - 1 .... ,3
respectivamente iguais a 2 e Se cos9 =—, então "z‘
2’

a) tem sua imagem no eixo real


b) tem módulo igual a 5
c) tem módulo igual a 3 + 373 i

d) imaginário puro
e) tem argumento igual a 120°

44) (F.M. SANTA CASA-SP) Seja o número complexo z = (2-2í)n. onde


n e N *. Se |z| = 512 o número n é:

a) primo
b) quadrado perfeito
c) divisível por 5
d) múltiplo de 4
e) divisível por 3

289
45) (ITA-SP) Seja z um número complexo de módulo 1 e de argumento 6. Se n
é um número inteiro positivo, zn +-^- é igual a:
z'
a) cos (n 6)
b) 2 cos (n 9)
c) sen (n fl)
d) 2 sen (n 6)
e) sen (n fi) + cos (nR)

46) (F,M. SANTA CASA-SP) O menor valor de n inteiro e pasilivo para o qual
y° = (2 + 2^3 i)n seja real positivo ê (obs.: i = ):

a) 3 b) 12 c) 6 d) 9 e) n.r a.

cos— + i sen 75-'j


47) (F.M SANTA CASA-SP) O número complexa z = ^2 ê
16 16 J
uma das raízes quartas do número complexo:
1 1. 1 .
a) 1 - i b) 1 + t c) —+ -i
2 2
t 72 J2
e)---- +------
2 2

48) [F.M. SANTA CASA-SP) Considere o produto (x-z)x(x-z}. onde


z = tx + fii, com ae jj pertencentes ao conjunto dos números reais ct>p * 0-
Então podemos afirmar que:
[(X-2)Xx-z)]m. VXE R
a) é negativo caso m seja ímpar
b) é sempre um número par
c) é positivo para qualquer m real
d) vale 3
e) não satisfaz a nenhuma das alternativas anteriores

49) (ITA-SP) Sejam a e k constantes reais, a>0e0<k<1. De todas os


números complexos z que satisfazem a relação |z-ai| £ ak, qual é o de
menor argumento?

a) z = ak7l-k?+ia(1-k2) b) z= -k2 + ia (1 -k2)


c) z=k7i-k2 -i Jd-k2) d) z = - k7l-k2-ia (1-k2)
e] z — a + ik

290
50) (MACKENZÍE-SP) O número de soluções da equação z2+|z| = 0, onde
z = r (cos 0 + j sen 0). ê:
a)0 b) 1 0)2 d) 3
e) 4

Polinómíos

51) (ESAN-SP) Sendo P(x) = Q(x) + x2 + x + 1 e sabendo que 2 é raiz de P(x) e


que 1 é raiz de Q(x). então P(1) - Q(2) vaie:
a) 0 b) 2 c) 3 d) 6 e) 10

52) (MACKENZIE-SP) P(x) = anxn + an_fxn*1 +a,x + aD é um polinõmio:


an + an_1 +... + a1 + aD é a soma dos coeficientes do polinõmio P(x). A soma
dos coeficientes do polinôrnío (4x‘ -2x2-2x-1)36 é:
a) 0 b) -36
c) 1 d) -1
e) impossível de calcular no tempo disponível

53) [ITA-SP) O coeficiente da maior potência de um poilnõmio P£x) do 3o grau


é 1. Sabendo-se que P(1) = P(2) = e P(3) = 30. então P(- 1) vale:
a) 48 b) 06
c) 18 d) -2
e) 68

54) (CESPESP-PE Considere o polinõmio:


n
P(x) = j>ix’
i=o
Assinale a alternativa falsa'.
a) P(—1) = 0, se e somente se a soma dos coeficientes de ordem par for
igual à soma dos coeficientes de ordem impar
b) P(1) s 0, se e somente se a soma dos coeficientes de ordem par for igual
ao simétrico da soma dos coeficientes de ordem ímpar
c) P(-2) = 0, se e somente se a soma dos coeficientes de ordem par for o
dobre da soma dos coeficientes de ordem impar.
d) R(0) = 0, se e somente se a0 = 0.
e) P(1) = P(-1) = 0, see somente se a soma dos coeficientes de ordem par
for igual á sorra dos coeficientes de ordem ímpar e igual a zero.

291
55) (GESGRANRIO) O polínõmío P(x) - aox3 + anx2 +a3x + a:1 se anula para 4

valores distintos de x. Podemos concluir que:


a) aQ-j-g-i + aj + aj — 1

b)

C) aO <«1 3 j- < a3

d) a D > af > a 2 >


e) a a ~ ai ~ a? — a3 — 0

56) (MACKENZIE-SP) Os valores de m, n e f para cs quais


p(x)= (2m-1)x3 -(5n-2)x2+(3-2í)

é um polinôrrijo nulo são, respectiva mente:

1_ _ 2 3
a)
2' 5 2

2 2 3
b)
2'5'2

c) -t - 3. 1
d) -2, -5.2

e) 0, 0.0

57) (PUC-SP) Os valores de m, ri e p de modo que sejam idênticos os


pohnõmios
P.(x) = (m + n + p)xJ- (p + 1)x3 + mx2 + (n-p)>x + n

e
P2(x) = 2mx3 + (2p+7)x2 + 5mx + 2m

são, respeútivamente:
a) 1, 2, -3 b) 2. 3, 1
c)-1. 2, 2 d) 2, 1,-3
e) 1. -3. 2

55) (UFRS)Se r(X) = a >p(x) + b>q(x), com r(x) = 4k2+kx-fi, p(x) = 2x2-3k-2h

q(x) = x - 5x +1, ae R., b e R e k e R, então a + b + k é;

a) 0 b) 1 C) 2 d) 3 e) 4

292
x‘2 b3x
59) —+ -—
(OSEC-SP) Escolha o termo que se deve acrescentar ao binômio —
(OSEC~SP)

de maneira a obter trinômio que seja quadrado perfeito, entre as alternativas


abaixo:
t b6
b) ÈÍ
a> T 9
bB
C)
3 d) T
e)
6

60) (ITA-SP) Considere o conjunto C dos polinõmios P(x) de grau 3, tais que
P(x) = P(- x} para todo x real Temos, então, que:
a) C tem apenas dois elementos
b) C é o conjunto de todos os polinõmios da forma P(x) = anx3 + bx.
c) C tem apenas um elemento.
d) C tem uma infinidade de elementos.
e) n.r.a.

61) (EPUSP) Seja ar o coeficiente de xn num polinõmio de coeficientes


complexos de grau 30 Sendo a0 = -1 e an+1 = 1 + ían(n 2 0), então a30 ê
igual a:
a) -i b) 1-i
c) i d) 2i
e) n.r.a.

62) (ITA-SP) Os coeficientes A, B, C e □ do polinõmio P(x) = Ax3+ Bx2 +


+ Cx + D devem satisfazer certas relações para que P(x) seja um cubo
perfeito Assinale a opção correta para que isto se verifique:
. n c2a
a D =-----
3B
B B2
b) C = —- e D =
3A3 27A3

c) BC = 3A e CD2 = BzA 2

d) C = |^ e D=X
d) C = — e D =
3A 27A2
e) n.r.a.

293
5-3x A B C então os valores rfe A, Be
63) (PUC-SP) Se - x-2 + x-3’
x 3-5x2 + 6x X
C são. respectivamente:
1 1 1
a)
235
5 1 4
b)
S 2' 3
] ] 2
c)
2'35
5 1 2
d)
6'2'3
2 4 J
e)
3 5 2

2X - 3 A □
64) (U.F.SE) Se 4-------- , então:
X2 — 6x + 5 x-5 x-1
a) B - A = 6 b) B = 2A c) A=1q d) A + B = 2

’>X=7

65) (CEUB-DF) A condição para que o polinòmio f = (ax + b)2 + (cx + d)2, onde
a, b e c são reais e não nulos seja um quadrado perfeito é:
a) ad = bc
b) cd - ah
c) abc = d
2
d) ad = b c + a

56) (ITA-SP) Dizemos que os polinõmios pi(x), p2(x) e p3(x) são íínearmente
independentes (L.L) se a relação ai>p»(x) + a2p2(x) + a3p3(x) = 0 implica
a-t = a2 = a3 = 0, onde ab a?. a3 são números reais. Caso contrário, dizemos
que p-ifx), p2(x), e p3(x) são linearmente dependentes (L.D.). Os
polinõmios p-^x) = x2 + 2x + 1, p2(x) = x2+1 e p3(x) = x2 + 2x+ 2 são:
a) LI.
b) nem L.l. nem L.D.
c) L.l. se pi(x), pz(x) e p3(x) tiverem as raízes reais
d) L D.
e) n.r.a.

294
67) (MACKENZIE-SP) O valor de
1 1 1 1 1 f é.
1>ô + 3>6 + 5x7 + +" + (2n-1)(2n + 1)

1 A e
Sttgestec .determine duas constantes AeB tais que
(2n-1)(2n + 1) 2n-1 2n + 1

a)1 b)-1
4 d) 2 e) não sei

60) (CESCEA-SP) Determine a e b de modo que a expressão:


3x2 + 5x-6
ax2 -10x+b

não dependa de x. Então a + b é igual a:

a)-10 b) 10 c) 8 d) naa sei

69) (CESCEM-SP) Seja r(x) uma função racional. Ela pode ser representada de
P(x)
forma única como □ quociente de dois polinâmios primos entre si, —,

onde q(x) tem o coeficiente do termo de maior grau, unitário. Nestas


Condições, definimos ordem k de r{x) como:
k= Í2 x grau(q(x)) se grau (p(x)) » grau (q(x))
|z x grau de (p(x))-l se grau (p(x)) > grau(q(x))

3xa + 2x-1
Assim, dado opolinâmio r(x) = a ordem de r(x) é;
x-4
a)1 b) 2 c) 3 d) 4 e) 5

70) (CESCEM-SP) Pq(x) é um polinômio de grau n tal que:

Pi(0) * 0, isto é, P1(x) = aoxr + a1xn'1+ _(a0 *0)

x^.í-j; x*0
e P2(x) = (,x;
a0; x = 0
Então P2(x) é:
a) um polinomio de grau n
b) uma função racional não inteira
c) uma função irracional
d) uma função transcendente
e) n.r.a.

295
71) (MACKENZIE-SP) O polinõmio P(x) = (m - 4)x3 + (m3 - 16)x2 +(m + 4)x + 4
é de grau 2:
a) se e somente se m = 4 ou m = - 4
b) se e somente se m 4
c) se e somente se m * — 4
d) se e somente sem^4em* — 4
e) para nenhuma valor de m.

72) (U.F.AL) O grau dos polinômios f e g sao, respectiva mente, 2 e 3, a grau da


polinõmio h = f2 - 4g é:
a) 4 b) 3 c) 2 d) 1
e) 0

73) (FUVEST-SP) O grau dos polinômios f, g e h é 3. O número natural n pode


ser □ grau do polinõmio não nulo fXg + h)se e somente se:
a) n = 6 b) n = 9 c) < n<6 d)3ání9
e) 3í n í 6

74) (MACKENZIE-SP) Dados três polinômios de graus 3, 5 e 5. respecli-


vamente:
a) pode-se somar esses polinômios e o grau da soma é necessariamente 5
b) pode-se somar esses polinômios e o grau da soma pode ser menor que
5
c) nãc se pode somar esses polinômios, porque seus graus são diferentes
d) não se pode multiplicar esses polinômios, porque seus graus são
diferentes
e) a produto desses polinômios tem grau 75.

75) (CESESP-PE) Sejam f e g dois polinômios não nulas de coeficientes reais.


Assinale a alternativa carreta.
a) grau (bg) = grau (f)>grau (g) b) grau (f) > grau (f>g)
c) grau (bg) = grau (f) + grau (g) d) grau (f +g) = grau (f) + grau (g)
e) grau (f+g) = max.{grau (f). grau (g)}

76) (FM. SANTA CASA-SP) Indica-se o grau do polinõmio f por t)f. Sejam
f e g dois polinômios tais que 3f = 2n e dg = = n - 1, andem e hf e n > 1.
Se na divisão de f par g obtém-se quociente q e resto r, r ± 0, podemos
afirmar que:
a) dq = í)r = n - 1 b) í)q - 3r ~ n + 1
c) dq = n - 1 e 3r = n - 2 d) dq “ n - 1 e 3r < n - 1
e) rlq = n + 1 e 3r < n - 1

296
77) (CESCEM-SP) Dividindo (x3 - 4x2 + 7x - 3) por um cedo polinõmio p(x)
obtemos quociente (x- 1) e resto (2x- 1). O polinõmio p(x) é igual a
a) 2x2 -3x + 2

b) xz-3x + 2

c) x2-x + 1
d) 2x2~3x + 1
e) n.r.a.

70) (ENG S. CARLOS-USP) Seja o quociente e Ro resto da divisão de um


polinõmio A por um polinõmio B. Então, quando A é dividido por 2B
a) a quociente é 2Q e o resto 2R

Q R
b) o quociente ê — e o resto —
2 2

Q
c) o quociente é - è o resto é R
2
d) o quociente ê 20 e o resto R
R
e) o quociente é 2Q e o resto —
2

79) (U.F.PR) Determine m e n de modo que o resto da divisão do polinõmio


y5-my3+n por ya + 3y2 seja 5.
a) m = 9 e n = - 5
b) m = 9 e n = 5
c) m = “ 4 e n = - 5
d) m - 4 e n = 5
e) m = - 9 e n = - 5

80) (MACKENZIE-SP) Se A(x) = 3(x- 2)(x2 -1)- (2x-4)(x2 + 3)


B(x) =-2x-* 6 +(3 - x)(x - 4)
F(x)=^>
B(x)
então, para todo x do domínio de F tem-se:

a) F(x) = x + 3 b) F(x) = -x - 3
c) F(x) = - x + 3 d) Ffx) = x - 3
e) n.r,a.

297
81) (PUC-SP) Para que valores de m o resto da divisão de P1(x) = 4x3-3xz +

+ mx + 1 por P2(x) = 2x2 -x + 1 ê independente de x?

2 b)m =-!• 3 d)m^|


a) m = — G)m = -
5 •J D

e) n.r a

02) (U.F.GO) Se o polinõmio x5 + kx2-2x + 3 é divisível pelo poliriómio


x2 -x + 1, então o quociente é:

a) x -3 b) x + 3 c)x-1 d)X+1
e) X + 2

83) (CESGRANRIO) O polrnõmio x3+px + q é divisível por x3+2x + 5 Os


valores de p e q são. respectivamente:
a) 2 e 5 b) 5 e 2 c) 1 e 5 d) 1 e-10
e) 3 e 6

34) (F.M. SANTA CASA-SP) O polinõmio P^x) - x3, + px + q é divisível por


P2(x) = x2 + mx -1 se (Obs : m. p e q são reais.)

a) p = - q -1 e m - q
2
b) m —1-pem-q=0

c) q + p = 1 e m = 0
3
dJ m = — q e p = 1 + q

e) n.r.a.

S5) (U.F.SC) A divisão de (xs-6x-1) por (mx2 + nx + p) apresenta como quo-


cíente (x-3) e como resto ( x + 5 ). Os vafores de m, n e p são.
respectiva mente:
a) (3; 2:1) b) (2; 1;3) c) (1:3; 2) d) (2; 3; 1) e)(1;2; 3)

86) (F.M. SANTA CASA-SP) Um polinõmio P(x) dividido por X - 2 dá resto 3 e


dividido por xí-2 dá resto 3x — 1. O resto da divisão de P(x) por (x - 2)
(*2 - 2) é

a) —x7 + 3x +1 b)9x —3 c)xJ -3x-1 d)4x2 + 2x^3 e) n.r.a.

293
67) (PUC-SP) O resto da divisão do polinômio P(x)- 2x4 - 3x +1 por
g(x) = 2x-l, é:

4 b)-í
4 <4 4
08) (F.G V.-SP) O resto da divisão do polinômio:
x 1Q +xv9 +x
,
+ X +X - X „4
-X - X - X2 -x + 1
pelo binômio x ♦ 1 é:
a) 0 b) 1 C)- 1 d) 3 e)2
too
B9) (CESGRANRIO) O resto da divisão do polinômio x por x + 1 é
a) x - 1 b) x c) — 1 d)0 e) 1

90) (CESCEA-SP) Se n > 1 é um número natural, então o polrnômio


P(x) = 5xr -4/1-1 -1 ê tal que:
a) P(x) ê divisível por x- n
b) P(x) ê divisível por x + 1
c) P(x) é divisível por x - 1
d) P(x) é divisível por xJ - 1
e) P(x) é divisível por x + 2

91) (CESESp-PE) Seja p(x) o polinômio definido por:


100
p(x) = £ix'
i=1

Assinale a alternativa que corresponde ao resto da divisão de p(x) por x - 1.


a) i2 b) i! c) 0 d) 1 e) 5.050

92) 2V2 -5a3x + a4 é divisível por x - a,


(UNIFOR-CE) Sabendo que x4 +ma¥
a*0, então m vale;
a) 1 b) 3 c) 5 d) 7

93) (CESCEA-SP) Os coeficientes aD, 31,... , an do polinômio:

P(x} = a0 +a.|X + ..,-í-a n*x"

formam, nesta ordem, uma PG de razao — . Então, o resto da divisão de

P(x) por x + 2 é;
a) 0 se n é ímpar b) O se n è par
c) 0 se □□ = 1 d) não sei
299
94) (MAGKENZiE-SP) O resto da divisão de p(x) = x 2í1 +1 (n, número natural
não nulo) por q(x) = x + 1 é:
a) sempre 0
b) sempre 2
c) 0 se e somente se n for ímpar
d) 2 se e somente se n for par
e) n.r.a.

95) (MACKENZlE-SP) O resto da divisão de p(x) = 4x3 + 2x3-mx+5 por x + 2


é 1. Então, m e igual aí
a) 2?
b) - 20
c) 20
d) 10
e) - 10

96) (FACESP) Dados os polinómios f = x3 + (p - q)x +2p e g = x2-(p + q) com


p e q reais, para que ambos os polinómios sejam divisíveis por 3 - X,
devemos ter:
a) p = q = 0
b) p = q = 3
c) p = 3. q - 0
d) p = 0, q = 3
e) n.r.a

97) (MACKENZIE-SP) O resto da divisão por x - b do polinômio:

1 x Xa x3 x4
1 a a2 a3 a4
P(x) = 1 b b2 bJ b4 é
1 c c2 c3 o4
1 d d2 d3 d4

a) (x - a) (x - b) (x - c) (x - d) se abcd r
b) em geral, um polinômio não nulo de grau 3
c) o polinômio nulo, se e somente sea = b = c = d
d) sempre □ polinômio nulo
e) não sei

300
99) fFATEC-SP) Seja R[x] o conjunto dos polínômios com coeficientes reais,
p e R[x], p = ax2 - 3x + b. Se os coeficientes a - 3, b, nessa ordem, formam
uma progressão aritmética, então o resto da divisão de p por x + 1, em fé[x],
é:
a)-3 b)-B c) D d) - 1 e) 1

99) (CESCEM-SP) Se P(x) é um polinõmio divisível por x - a e por X - b,


podemos concluir que (x - a) (x - b) divide P(x);
a) sempre
b) desde que P'(a) = P’(b) = D
c) desde que P(a + b) - P(ab) - 0
d) desde que a * b
e) n.r.a

100) (U.F.CE) Sejam a, b e c números reais distintos, nao nulos, tais que o
polinõmio p(x) = x2 + x + c é dividido exatamente por x - a e por x - b,
Então a + b è igual a:
a) -1
b) 2
c) -2
d) 1
101) (UFBA) Na divisão de um polinõmio pelo binômio ax + b. usou-se o
dispositivo prático de Bnot-Ruffini e encontrou-se
1 p -3 4 -5
-2 q —4 5 r 7

Os valores de a. b. p, q e r são, respectivamente:


a) 1,-2, 1.-6, 6
b) 1.-2, 1, 1.4
c) 1,2,-2,-2.-6
d) 1, 2, 1, -4.4
e) 1.2,-2, 1,-6

1Ü2) (CESCEA-SP) O quadro

1 0 -0,52 -1,626
1.32
1,32 1,7424 1,613568

1 1.32 1,2224 -0.012^32

301
É o dispositivo prático de Briot-Ruffini, da divisão de determinado polinômio
P(x) por determinado binômia linear D(x). Então, □ valor de P(x) + D(X) na
ponto x - 1 é:
a) -1 466 b) -0.326 c)l d) -0,332432 e) 0.854

103) (MACKENZIE-SP) Na divisão do polinômio 5xs+ax3+ bx2+3x + 1 por


x — 2 encontrou-se o quociente 5x* +■ cx3 + dx2 -s- ex -r 115, O resto ê’
a) -229
b) 229
c) — 231
d) 231
e) impossível de determinar sem conhecera, b, c, d, e

104) (CONSART-RJ) O poltnómio P(x) que satisfaz a igualdade


(3x + 2)M=(x) = 3x3 + x2-5x-2 + P(x)
é:

a) x3-2x-2

t>) x2 -2
c) x3 +3
d) x3-6x-2

e) (x-1)(x2 x +1)

105) (ITA“SP) Os valores reais a e b tais que os polinômios


x3 -2ax?+(3a + b)x-3b ei x3 - (a + 2b)x + 2a sejam divisíveis por x + 1,
são;
a) dois números inteiros positivos
b) dois números inteiros negativos
c) números inteiros, sendo que um é positivo e o outro negativo
d) dois números reais, sendo um racional e o outro irracional
e) n.r.a.

106) (F.M. SANTA CASA-SP) O polinômio f = x33 + ax2 + (a-18)x + 1 é divisível


por x — 1. O polinômio g = ax3 + bx2 + bx + a é um cubo perfeito se b for
igual:
a) 24 b) 18 c) 12 d) 6 e) 6

302
107) (PUC-SP) A divisão do polinõmio P(x) por x-a fornece o quociente
q(x) = x3 + x2 + x +1 e o resto P(a) = 1. Sabendo que P(0) - -15r o valor de
a é:
a) 13
b) -13
C) 14
d) -16
e) 16

108) (CESCEA-SP) As raizes da polinõmio P(x) = xa + mx2 -é-nx + p são 1r2e 3.


O quociente de P(x) por x -* 3 é:
a) x2 + 2
b) x2-3x + 2

c) x3 + 3x-2

d) x2-2x + 1

í e) x2-2

1Ü9) (U.F.GO) Observação: um polinõmio é chamado mõnicc quando o coefi­


ciente do termo de maiargrau é 1.
Seja P(x) um polinõmio mõnico de grau 2, divisível por x - 1 e assumindo
valor 2 em x= 3. Seja Q(x) um polinõmio mônico de grau 3, divisível por x -
1, divisível por x - 2 e assumindo valor 6 em x = 3, O quociente de P(x) po-
Q(x) é:
2
a)Ax-1
b) —
x+1 c)7 d) —
x-2
e)
6(X-2)

110) (FACESP) Para que o polinõmio x3 + 2x: +(a + 5b)x + (a - 2b) seja divisível
por x: -x, os valores de a e b devem ser, respectiva mente:

a)-2e1 b) 1 e 2 c)2e-1 d)-1e-2


e) n,r.a.

111) (FUMEC-MG) Para que o polinõmio 2xd-x a + mx2-nx + 2 seja divisível


por x2-x-2, devemos ter,

a) m - 1 e n - - 6 b)m=-6en=-1
c) m = 6 e n = 1 d)m--5e n = 1
e)m = 6en=-1

303
112) (F.G.V.-SP) Determinando-se m e n de forma que xJ -x33-22x2 mx + n

seja divisível por x2 -5x-6, o quociente dessa divisão será:

a)
x2 ’4
b)
c) x2-4x-10

d) x3 - 3x +1

e) (X - 3)(x + 4)

113) (MACKENZIE-SP)O polmõmia P(x) = (cos 6 + x sen 9)" -cos n9 - x sen n6.
n e N*. ê divisível por P^x) - x2 +1;

7t
a) somente se 0<íJS -
2
71
b) somente se — < 0 < ?t
2
n 71
c) somente se 0 —+k —
2 3
d) somente se n& *

e) sempre

114) (MACKENZIE-SP) Um polinõmío desconhecido, ao ser dividido por x - 1,


deixa resto 2 e, ao ser dividido per x - 2, deixa resto 1. Então, □ resto da
divisão desse poünômio por (x - 1) (x - 2) ê:
a) x - 3 b) -x + 3 c) x + 3 d) x-5
e) -x + 5

115) (CESCEA-SP) O quociente de x3 na poünômio P(x) do terceiro grau que se


anula para x = -1 e tal que dividido separadamente par x - 1, x + 2 e x + 3
deixa resto 10, é:

a) 5 b) 10 c) 1
4 e) naa sei

116) (PUC-SP) Os restos das divisões de um poünômio P(x) pelos binômios


(x + 1). (x - 1) e (x - 2) são, respectiva mente, 5, - 11. Então, □ resto da
divisão de P(x) por (x + 1) (x- 1)(x- 2) é:

a) x2 + 3x -1 b) x2 + 3x + 2 c) x2-3x + 2
d) x2-3x + 1 e) x2 + 3x + 3

304
117) (ITA-SP) Suponhamos que os polmõmios P(x). Qfx). p(x) e q(x) satisfazem
as seguintes condições:
P(x)>p(x) + Q(x)>q(x)=1 para todo x complexo
P(q(1))= 0. Q(0) = 0
Assinale a afirmação correta:
a) P(x) é divisível por S(x) = x
b) P(x) e Q(x) não são primos entre si
c) Q(p(1)) = 0
d) p(x) não é divisível por R(x) = x-1
e) p{0) = 0

118) (ITA~SP) Seja P(x) = a0 + ap< + a2x2 +a-jx3 + ... + alÇjOx’aa □nde a 1QO — 1'

um polinõmio divisível por (x + 9) 100 Nestas condições, temos:

a) a2-50 >99 >9 33

100!
b) a2»
2'98!
99!
c) a2 =
2! 98!

100!92
d) a? =
2'98!
e) n.r.a.

119) (UnB-DF) P,(x) e P2(x) são polinômíos do 2° grau que se anulam quando
x = 0 O resto divisão de P^x) por (x - 1) (x <■ 2) é 3x + 1. resto da divi­
são de P2(x) por (x + 1) (x + 2) é 2x - 1. Então, o quociente da divisão de
Pi(x) por Pz(x) é:
a) 1
b) 0
c) x + 1
d) n.r.a.

120) (PUC-SP) Os valores de a e b de modo que o polinõmio P,{x) = x3 + ax + b

seja divisível pelo polinõmio P2(x) = (x-1)2 são:

a) a = 2 e b = -3 b)a = 3eb--2 c) a 1 eb= 3


d) a - 3 e b - - 1 e) a = -3 e b = 2

305
3 2
121) (U.F RS) O máximo divisor comum entre □s polinómíos o(x)=x t2x -

x-2 e q(x) ss x3 + 3xz -4 é:

a) (x + 2) (x-2)
b) (X + 1) (X- 1)
c) (x - 1 )(x - 2)
d) {x+ 1)(x —2)
e) (x- 1) (x* 2)

1 22) (F.G.V.-SP) O máximo divisor comum entre os polinómíos


A(x) - 2x2 + 5x“3

B(x) — x3 + 9x2 + 27x + 27


C(x) = 2x3 -x2 -1ÔX + 9
é:
a) 1
b) X- 3
c) x + 3
d) (x + 3) (2k~ 1) (x,22 - 3x + 9) (x-3)

e) (x + 3) (2x - 1)

P(X)
123) (CESCEM-SP) Se p(x) =s 2x3 + X2 - flx e q(X) = X2-4, então - - ét
q(x)
a) 2x + 1
b) 2x + 5

4
c) 2x + 1 +
x2-4

4
d) 2x + 1-—y—■
x 3-4
1
e) 2x + 1+ —
x

124) (CESCEM-SP) Um polinômio de coeficientes inteiros na variável x tem grau


par, seu termo independente ê impar e o coeficiente do termo de maior grau
é igual a 1. Assinale a resposta falsa:
a) o valor do polinômio para x - 0 é um número ímpar
b) o zero não é raiz desse polinômio
c) o polinômio derivado tem grau impar
d) o coeficiente do termo de maior grau do polinômio derivado é um número
Empar
e) nenhum número par pode ser raiz desse pclincmio

306
125) (CESGRANRIC) O polinômio x3 + 2xz + mx + n é divisível por x2 + x + 1.
O valor dem + né:
a) ~3 b)-1 c) 1 d) 2 e) 3

126) (EPUSP) Sejam u e v dois números complexos tais que u2 -vz = 6 e


u + v = 1-i (u e v conjugados de u e v). Então u - v é igual a
a)1-i b) 1 + i c)3-3í d) 3 + 3i e) n.r a

127) (II.F, UBERLÂMDIA-MG) O polinômio p(x) = x5-3xJ + 0x2 -9x + 3 é dlvi-


SÍVel por?

a) x^2 b) x + 1 C)x d) x + 2 e) X — 1

120) (FACESP) Se A(x - 1) t (Sx + C) (x + 1) = 3x(x + 3) è uma identidade, então


o valor de A + B + C é:

a) 9 b) 12 c) 3 d) 6 e) n.r.a.

2 ? 2 1*
129) (CICE-RJ) Seja z = x + iy. X + y * 0 (í - - 1, x e y reais), z - - é rea I se
z
e somente se:
a) x2 + y2-1-0

b) x2 + y2 + 1 = 0

c) y = 0 e x2 -i-y2 - 1

d) x = 0e|z| = 1

e) y = 0 e [z| = 1

a + bi , ,
130) (F.C, CHAGAS-SP) O módulo do número complexo z ------- , onde a e b
a-bi
são números reais é:

a) 0
b) 1

c) 72
d) Va3 +b2
a +b
e)
a2 + b2

307
Equações Algébricas

131) (PUC-RJ) Sobre as raízes xa — xí+3x-3 = 0 podemos afirmar:


a) nenhuma raiz é real
b) hã uma raiz real e duas imaginárias conjugadas
c) há três raizes reais cuja soma é 3
d) hã três raizes reais cuja soma é 1
e) há três raizes reais cuja soma é“3

132) (F.G.V.-SP) Sabendo que P(x) = -x4 +11x3 - 36x2 + 52x-24 tem uma raiz
dupla x = 2, o domínio de definição da função f£x) = log[P(x)] é:
a) {xe l-L|x x 2 e x * 3}

b) {xe iíc | 2 < x < 3}


c) {x e |1 < x < 6}

d) {xe ít|x < -1 ou x > 7}

€ (xe |1 < x < 2 ou 2 < x <6}

133) (MACKENZlE-SP) Sabe-se que o número complexo I ê a solução da


equação x4^3x2-4=0, Então:
a) essa equação tem uma solução de multiplicidade 2
b) as soluções dessa equação foram uma progressão
c) a equação tem 2 soluções reais irracionais
d) a equação tem 2 soluções reais racionais
e) a equação não tem soluções reais

134) (LLF.RS) O maior número real, cuja soma com o próprio quadrado é igual ao
seu cubo, é:
a) 0
-1- 73
b)
2
I^VÊ
c)
2

d)
2
3_^/f
e)
2

308
135) (U.F.RS) Se P(x)= x3-3x2+ 2x, o conjunto {x e F?|P(x) > 0} é:

a) (0;1)
b) (1;2)
c) (-~;1)uj(2;+«)
d) (0;1)u(2; + «)
e) (-«;0)u(1;2)

136) (CESGRANRIO) Uma das raizes do polinõmio x3 + 4x2 + x-6 é 1, Com


relação ás autras raizes do polinõmio, podemos afirmar que:
a) ambas sáo negativas
b) uma é negativa e a outra positiva
c) ambas são positivas
d) uma delas é nula
e) são complexas com a mesma parte real

137) (PUC-SP) O produto das raízes da equação x(x + 1)(x + 2).,.(x + 9) = 0 é:


,9
a) 9
b) 9)
g) -9
d) 45
e) 0

138) (FATEC-SP) O número 2 é uma das raizes do polinõmio P(x) = xJ - 6x‘ +


+ 3x + m, onde me R. Quanto ãs raizes reais de P(x) podemos afirmar que:
a) sua soma é - 4
b) o quadrado da sua soma é 30
c) a soma de seus quadrados é 36
d) formam uma progreessão geométrica
e) formam uma progressão aritmética

139) (CESGRANRIO) Dada a equação xE -13x“ + 36 = 0. tem-se que:


a) admite 4 raízes reais irracionais
b) asmite 3 raizes reais
c) não admite raizes reais
d) admite 4 raizes reais inteiras
e) as 4 alternativas anteriores são falsas

309
6 5x A 8 c
140) (ITA-SP) Se --------, onde A, B e C são reais
X3 - 5x2 + 6x x -a x-b x -c

e a. b e c sao raizes da equação X 3-Sxz+6x = 0, então:

a) A =- 2, B - - 1 e C = 0
b) A = 2. B = 4 e C = 1
c) A = 1„ B = - 3 e C = 2
d) A= 5. B = 2 e C=1
e) n.r.a.

141) (EPUSP) O polinômio


x100 + (sen 1a + cos 1° )x179 +(sen f + cos 1° )(sen 2a + cos 2°)x1,e +
+ (sen r + cos lfl)(sen 2fl + cos 2°)(sen 3° + cos3°)x177+ ...

a) tem o produto das raizes igual a 1


b) admite zero como raiz simples
c) admite zero como raiz de multiplicidade 46
d) não admite raizes reais
e) n.r.a.

142) (CESCEM-SP) Os valores de m e n a fim de que 3 equação

í3
x7 -5xe + 4x5 -3x4 + 2x3 + (m-5n)x* + — m -n + 2 x + (5-mx)) = 0 admí-
\5
ta duas, e apenas duas raizes nulas, sáo, respectiva mente

a) — e - 5 b) -5 e 3
c)| e 3
d)-5 e -1 e)| e -1
3 ■J w

143) (CESCEA-SP Sabendo~se que um polinômio P(x) do 4° grau é divisível por


(x - 2)3 e que P(0) — 8 e P(1) = - 3, o vaior de P(3) é:

a) 10 b) — 8 e) 0 d) 1 e) 3

144) (EPUSP) Qual é o coeficiente de xn+1


n+1 no polinômio xn*3 + axn+7+... cujos

zeros são 0 com multiplicidade 3, 2 ccm multiplicidade n?

a) 0
b) 2n
c) 2n(n + 1)
d) 2n(n — 1)
e) n.r.a

310
145) (ITA-SP) A equação xr -1 = 0, onde n é um número natural maior que 5,
tem:
a) 1 raiz positiva, 1 raiz negativa e (n - 2) raízes imaginárias quando n é
par
b) 1 raiz positiva, (n - 1) raizes não reais quando n é par
c) 1 raiz negativa, (n - 1) raizes imaginárias quando n é ímpar
d} 1 raiz positiva, 1 raiz negativa e (n ~ 2) raizes imaginárias quando né um
número natural qualquer
e) n.r.a.

146) (MACKENZIE-SP) Dado o polinõmio P(x) = xn-1, ne N* cujas raízes são


1, a, b, c t, então (1 - a) (1 - b)... (1 — t) vale:
a) n, somente se o grau dc polinõmio for par
b) n, somente se o grau dc polinõmio for ímpar
c) 2n, somente se o grau do polinõmio for impar
d) 3n, somente se o grau do polinõmio for ímpar
e) n, qualquer que seja o grau do polinõmio

147} (PUC-SP) Sobre um poíínômio na indeterminada x de coeficientes inteiros e


de grau no máximo n que se anula para n + 1 valores distintos de x.
pode-se afirmar:
a) ê constante
b) é de grau impar
c) é de grau par
d) é identicamente nulo
e) n.r.a.

148} (ITA-SP) Seja zk um número complexo, solução da equaçao


(z + 1)s +z 5 - 0. k - 0. 1, 2, 3. 4. Podemos afirmar que:
a) todos os zr (para k - 0. 1,2, 3, 4) estão sobre uma circunferência
b) todos os Zr estão sobre uma reta paralela ao eixo real
c) todos os Zk estão sobre uma reta paralela ao eixo Imaginário
d) a equação não admire solução
e) n.r.a.

149) (FATEC-SP) O polinõmio x3 + 6x2 + 12x + 8 :


a) tem uma raiz real com multiplicidade 3
b) tem três raizes reais distintas entre si
c) tem duas raizes complexas e não reais
d) tem exatamente uma raiz complexa e não real
e) não tem raizes reais
311
150) (MOJI-SP) A equação H-px3 + 3x2 + qx-4 = admite a raiz 2 com
multiplicidade superior a 1. Então;
a) p - 4 e q = 4
b) p = -4 e q = — 4
c) p - — 4 e q — 4
d) p = — 2 e q - — 2
e) p = 2 e q = - 2

151) (F.F.C.L.-USP) Para que a equaçao x 3 _(4-i-m)x2 + (4 + 4m)x


onde m e R , tenha uma raiz dupla igual a 2. devemos ter;
a) m 2
b) m = 2
c) m > 0
d) m < 3
e) n.r.a.

152) (F.M.SANTA CASA-SP) Os valores de p e q para os quais a equação

— -2x-i-px + q = 0 admite uma raiz de multiplicidade 3 são, respecti-

vamente:
b) |e-8
a) 3 e 4 c)4 e -1 e4 e) n.r.a.

153) (MACKENZIE-SP) Os números complexos 1 + í, 1 + i2 e 2 — i sao raizes do


polinômio P de coeficientes reais. Podemos afirmar que o grau de P ê,
necessariamente:
a) par
b) impar
c) maior ou igual a seis
d) maior ou igual a quatro
e) igual a três

154) (F.C.CHAGAS-SP) Se o número complexo 2 + í é uma das raizes da


equação x2 + kx + t = 0, onde k e t são reais, o valor de k + t é:
a) - 2
b) - 1
C) 0

d) 2
e) 1

312
2
155) (MACKENZIE-SP) Uma das raizes da equaçao x +ax + 2b = 0, a e b
reais, ê 1-i/2i. Os valores de a e b são, respectivamente:

a)-2 e-
2
3
b)-2 e
2

c)2 e - —
2
2

e) 2 e —
2

156) (FUVEST-SP) A equação x3+ mx2+ 2x+ n = ü, onde m e n sao números


reais, admite 1 + i (sendo i a unidade imaginária) como raiz. Então m e n
valem, respectivamente:
a) 2 e 2
b) 2 e 0
c) 0 e 2
d) 2 e -2
e) -2 e 0

157) (CESCEM-SP) 0 número 2 + 3i é raiz da equaçao x2 + bx + c ~ 0, com b e


c reais. Podemos estão afirmar que:
a) c é um número ímpar
b) bé um número irracional
c) c é um número irracional
d) b e c não estão univocamente determinados
e) b é um número impar

158) (U.F.RS) Se os números -3, a e b são as raízes da equação


x3 + 5x2 -2x-24 = 0, o valor de a + b é:

a) -6 b)-2 c)-1 d) 2 e) 6

159) (MACKENZIE-SP) A equação x3-5x + 4 = D admite a raiz 1. Se a e b


correspondem às outras raízes, o valor de a + b é:
a) +1 b) ~1 c) -5 d) 5 e) - 4

313
3 2
160) (U.F.MG) A média aritmética das raízes da equaçao x +2x -3x = 0 é:
2 1
a)-1 b) -- c) 0 d) —
o o

161) (PUC^SP) Qual é a raiz real de 3x3 -7x2+flx-2 = 0, se uma de suas


raízes é 1 - í?

a)- b)—— d)-3 e) -1


3 3

162) (CESGRANRIO) O produto de duas das raízes da equaçao 2x3-19x2 +


+ 37x -14 = 0 é 1. A soma das duas maiores raízes da equação é:

3)7 b) 6 c) 9 d) 19^2 e) 19

163) (F.F. UBERLANDIA-MG) C número 2 + 3i é raiz da equação x2 + bx + c = 0.


com b e c reais. Podemos, então, afirmar que:
a) b é um número irracional
b) c é um número imaginário puro
c) c é um número irracional
d) c ê número ímpar
e) c ê um número par

164) (CESGRANRIO) As raizes da equaçao x2+bx + 47 = 0 sao inteiras, Pode­


mos afirmar que:
a) a diferença entre as duas raizes tem módulo 46
b) a soma das duas raízes tem módulo 2
c) b é positivo
d) o módulo da soma das duas raízes é igual a 94
e) b é negativo

165) (MACKENZIE-SP) Sejam a e b as raízes da equaçáo x 2 -3kx-f-ka = 0, tars


que a2+b2 = 1,75. Ovalordek2é:

a) (1.75)2 b) 17,5 c) 175 d) 0,5 e)0,25

314
166) (MACKENZJE-SP) Um valor de k para o qual uma das raizes da equação
x2-3kx + 5k = 0 é o dobro da outra é:

b)2 c) — 5 d) — 2
e)-*
=)
2 2

167) (F.M. SANTA CASA-SP) Seja a equação x3 + x3 + kx +1 = 0, onde k e t sao


coeficientes reais. Se o complexo 1 - 2í é uma das raizes dessa equação, o
produtos das três raízes ê:
a)-15 b)-12 c) - 9 d) 9 e) 15

168) (CESGRANRIQ) Se zn + a1zn-1+. . + z + an é um polinómio com coefi­


cientes complexos, cujas raizes são todas de módulo menor do que 1,
podemos afirmar que:

a) |aj = 1 b) |aj<1 C)!an l>1 d)|an| = Q e) faJrO

159) (UNESP) As três raizes da equação x3-12x2+ITIX-8 = 0 estão em pro­


gressão aritmética. Então:
a) m = 25 b) m = 28 c) m = 30 d) m = 32 e) m = 34

170) (ITA-SP) Considere a equação x3+px3 + qx+r = 0, de coeficientes reais,


cujas raízes estão em progressão geométrica. Qual das relações ê
verdedeira?

a)p2 = rq b) 2p + r = q c) 3p2 = r3q d) p3 = rq3 e) q3 = rp3

171) (ITA-SP) Sendo a e R e a > 1, o valor real de m para o qual a equação

x3 -gx^+ílogga"1 + 3)x -loge am = 0


tenha raízes em progressão aritmética, é dado por

a) m = logea - B ou m = -9a b) m-logea - 9

c) m = 15/logea d) m=--logea
D

e) n.r.a.

31S
172) (FEI-SP) Sendo a, b e c as raízes da equação 2x3-3x2 +5x4-1 = 0. o

vaíor da expressão a2b2+b2o2 4-c2a? é?

19
a) 19 b) 31 e) n.r.a.
C)T
173) (MACKENZIE-SP) O determinante da matriz

a a -c
0 b c
1 0 1

onde a b e c são raizes da equação x


3
-5x22 + 4 = 0 é:
a)-1 b) 0 c) 1 dj 2 e) 3

174) (ÍTA-SP) Se a, b e c são as raízes da equação x3-rx4-20 = 0, onde ré

um número real, o valor de a 3 + b3 + c'3 é:


5 d) 62 + r3
a) — 60 b) 62 + r c) 62 + r e) 62 - r

175) (ITA-SP) Seja M - —, onde a, b e c são as raízes da equaçao


az b‘ c
x3 - v3x2 + 54 = 0. Então:

a) log3W é um número irracional

b) log3Mê um número primo

XI U 5
c) —
log3 M = -

5
d) log3 M = -
2
e) n.r.a.

176) (FATEC-SP) Sabe-se que x,, x? e — 2 sao as raízes reais da equação

2x3 + (4-m)x? + x + 4m + 2 = 0
1 1
onde m e nt. Se — h—=-1, então:

d)m=-l
a) m ~ — 1
b’m4 c) m “ 1 e) n.r.a

316
177) (ITA“SP) Se as dimensões, em centímetro, de um paralelepípedo re-
to-retangular são dadas pelas raízes da equação 24x3 - 26x2 + 9x -1 - 0,
então o comprimento da diagonal é igual a:
a) 7/12 cm
b) 9/24 cm
c) 724 /12 cm
d) JÕ1/l2cm
e) 7/3/12 cm

178) (U.F.M G.) Se a, b e c sao as raízes da equaçao x 3 + x-1 = 0, então


fí 1 1V .
log1 — + — + — I e igual a;
A a b cj
a) -1
b) 0
c) 1
d) 2
e) n.r.a.

179) (ITA-SP) Sendo a. b. c e d as raízes da equação 2x'í-7x3 + 9x2


- 7x + 2 = 0, x, podemos afirmar que:
a) a. b, c e d são reais positivas
- - - 13
b) a2 + b2 + c2 + d2 ê iguala—*
5
c) a, b, c e d não sãc reais
1111
d) ----- +------ +------ +------ é a soma das raizes
bod acd abd abc
e) n.r.a.

180) (FM SANTA CASA-SP) Sejam x1n x2 e xa as raizes da equação


x3-2x2+ x + 1 = 0. Uma outra equação do terceiro grau, cujas raízes são
Vi = x2 >3(3, y2~x3x1 e y3 = x1x2, é.

a) y3-y2-2y + 1 = 0

b) y3 + y2 + 2y-1 = 0

c) -2y3 -y2 + y + 1 = 0

d) y 3 - y2 - 2y-1 = 0
e) n.r.a.

317
181) (CESCEM-SP) Dentre as frações 77.77,77, podem
' y 47881011121315 16
ser raizes da equação 16x 6 + axs + bx4 + cx3 + dx2 + ex + 45 = 0 (a, b, c. d
e e inteiros)
5 7 J3 5
3 5
a)-e- b>AeZ . 3 11
c) —e — d) — e — 6 16ei1
4 8 15 6 7 13 11 12

182) (PUC-SP) Qual dos números abaixo é a raiz da equação


15xa +7x?-7x + 1 = □?

4 4 4 4 4
183) (ITA-SP) Calculando as raízes simples e múltiplas da equação
x6-3x5 + 6x--3x2-3x + 2- 0
podemos afirmar que esta equação tem:
a) uma raiz simples, duas duplas e uma tripla
b) uma raiz simples, uma dupla e uma tripla
c) duas raizes simples, uma dupla e uma tripla
d) duas raizes simples e duas duplas
e) duas raizes simples e uma tripla

184) (F.M, ABC-SP) A equação algébrica do terceiro grau, a coeficientes inteiros,

da qual —1 e
1 + Vs são raízes é:
2
a) x3 -1 = 0 b) x3+1 = 0
C) x3 -2x-1 = 0 d) x3+2x-1 = 0
e) x3 - x2 + x - 0

105) (CESCEM-SP) Um número de coeficientes inteiros tem uma raiz dupla igual
a a + v'b, onde a e b são números primos. Podemos concluir que este
polinómio:
a) tem grau 2
b) tem grau pelo menos 4
c) tem uma única rarz complexa dupla
d) admite pelo menos uma raiz complexa
e) não existe

318
186) (MACKENZIE-SP) Considere as afirmações:
(I) Qualquer raiz racional da equação x3+ 3x2-3x-9 = é inteira.
(II) O menor grau da equação polinomíal de coeficientes reais, que admite
as raizes 3, 2 + j e — i, é 5.
(lll)Toda equaçao polinomial da forma ax^4 + bx3 + cx2 + dx + e = 0, de
coeficientes reais e a * □, necessariamente possui raiz real.

Então:
a) todas as afirmações são verdadeiras
b) somente as afirmações t e II são verdadeiras
c) somente I e lll são verdadeiras
d) somente II e lll são verdaderras
e) nenhuma afirmação é verdadeira

187) (PUC-SP) Os valores dea, b e c, de modo que a equação:


xB + 8ax5 + (b-2}xJ + (4a+b + c)x3 + 2ax2 + (b-2a)x-1 - 0
seja recíproca, são:
1
a) a = 1; b = 3; c = -
2
b) a = 1; b = 2; c = 1

c) a = — ; b = 1; c = -1
2

d) a =“ ; b = 1, c = 3

1
e) a : b = 3; c = -1

180) (ITA-SP) O valor absoluto da soma das duas menores raizes da equação:
2 1 1 >
x + —? + x+- = 4
x2 x
é;
4-^3
a) 2 b) 3 c) d) 4 e) n.r.a.
2

189) (ITA-SP) Os valores reais de a e b, para os quais as equações


x3 + ax2 +18 = 0 e x3 + bx + 12 = 0 têm duas raízes comuns, são:
a)a-1;b-2 h)a=-1;b = 4
c) a = 5; b = 3 d)a = -4;b=1
e) n.r.a.

319
(MACKENZIE-SP) As equações 3 -x2-x-(k + 1) = 0
kx3 e kx2 -x-
190)
- (k + 1) = 0, (k e R*), possuem uma raiz comum:

a) somente se k = -1
b) somente se k = 3
c) somente se k = 4
d) qualquer que seja k * 0
e) n.r.a.

191) (F.C. CHAGAS-SP) Se as equações x3 + ax + b = 0 e x2+x-2=0 têm


duas soluções comuns, os números a e b são tais que:

a) a + b = 1 b) a > 4 C) £b < -1 d) (a-b) =5 e) log ba > 0

192) (F.F.C.L.-USP) Se (x-ct) é mdc de P(x) = xn + a1x + ... + an_1x + an e de

Q(x)=nx + (n-1)a1xn"2- , então:

a) a é raiz simples de P(x)


b) a é raiz dupla de P(x)
c) a é raiz tripla de P(x)
d) a não é raiz de P(x)
e) n.r.a.

193) (CESCEM-SP) O gráfico abaixo é o de um polinômio cujas raizes reais


estão todas no trecho desenhado. Esse polinômio:
a) pode ser do 3o grau
y
b) pode ser do 5° grau
c) pode ser do 6° grau
d) pode ser quadrado X
perfeito
e) n.r.a.

194) (PUC-RS) O gráfico da figura é de uma função f : R -» R em, que f(x) é um


polinômio do terceiro grau. Para f(x), afirmamos:
(I) O termo independente é igual a 2
(II) Suas raízes são - 2, 2 e 1.
(III) Suas raízes são — 2, - 2, e 1
(IV) Suas raízes são — 2, 1 e 1
-2/

1 X

320
Está(ão) correta(s) a(s) afirmativa(s):
a) II b) tll c) I e II d) I e III e) I e IV

195) (CESCEM-SP) Sobre o polinômio P(x), de grau 3, cujo gráfico está esbo­
çado na figura, fazemos as afirmações:
(I) P(x) tem uma raiz igual a -2, uma raiz igual a 3 e uma raiz imaginária.
(II) P(x) tem um termo independente igual a -3
(III) P(x) tem uma raiz real e duas imaginárias
Podemos dizer que:
a) somente I é correta
b) somente II é correta
c) somente III é correta
d) apenas I e II são corretas
e) apenas II e III são corretas

196) (PUC-CAMPINAS) O polinômio P(x) = x4 - x — x + x— tem:


4 4
a) duas raízes reais no intervalo fechado 1; 3]
b) duas raizes reais no intervalo fechado -1;0]

c) pelo menos uma raiz real no intervalo

d) pelo menos uma raíz real no intervalo aberto ] 2; 3 [


e) n.r.a.

197) (ITA-SP) A equação 3x5 - x3 +2x2 + x-1 = 0 possui:


a) três raizes imaginárias e duas reais
b) pelo menos uma raiz real positiva
c) todas as raizes inteiras
d) uma raiz imaginária
e) n.r.a.

198) (CEUB-DF) Determine m de modo que a equação x5-2x4+3x3-5x2 +


+ x + (m - 3) = 0 tenha ao menos uma raiz compreendida entre 0 e 2:
a)-1<m<4 b)-5ím£5 c)-2<m<2 d)-3<m<3

199) (F.M. SANTA CASA-SP) Admitamos P(x) = anxn + a^x"’1+ ... +a0 um
polinômio de coeficientes reais que tem apenas raizes imaginárias. Então,
para um intervalo aberto qualquer ] a; b [, onde a e b são reais, temos:

a) P(a) + P(b) = 0 b) P(a)>P(b)<0 c) P(a)>P(b)>0


P(b)
e) n.r.a.

321
200) (ITA-SP) O conjunto dos valores de k para os quais f(x) = x 3-2x2 +
-t-3x-k tem uma ou três raizes reais entre 1 e 2 é:
a) k < 2 b) 1 < k < 2 c) 2 > k ou k > S d) k > 7 e) nj.a.

322
RESPOSTAS DOS EXERCÍCIOS PROPOSTOS

CAPÍTULO 1

1.7) a) S = {-3i; 3i) b) S = {-<75. i7S}


c) S = {-1; 1; -í; í} d) s = {-4; 4; -4i; 4i)

1.0) a) S = {1 + i; 1-i} b) S = p ++ l;
b) sUa -r; 2-l||
2--r

c) S = p + 2iV2: 3-2i72) d) S-[5+iJÍS; 5-iVÍ5}

1.9) S
I „ 1 + —
73 . -----
1
"l1:í2 2 2 2

1.10) Faça as substituições.

1.14) a) (- 1;2) b) (-1:5) c) (6;-10) d) (30; 10)


e) (12;-16) n <- 24.0) g) (“3;-4)

5
1 15) a) x = 0. y = - b) x = 5; y = - 2
2

1.16) a) z = (- 5; 5)
11
b)z= p:--j
C) z = í- 27 , í 1 3>
d)Z" 25'25;
5

1.17) S={(2; -2); (-2; 2)}

1.19) a) —
12 -5
13'13 b)ír° c,(o:4
(17 _7J b) (0; -1) c) (0,1)
12°) 7Z
L13’13J

323
-b
1 21) O inversa de z = (a: b) ê z (c;d)-
Va + b a2 +b 2
-b
Da definição de igualdade, vem que c = ——- e d ~ Então:
a +b a + b2
'/ _ \2 f
a -b
[a2 + b + d2 ] = [a^b= a2 4-b 2 a- b2

' a22+b2 1 [a2 + b21]z2


= [a2 + b2] J^ = 1
(a2+b2)2 (a2+b2)2
1.22) z = ( -6; 6)

CAPÍTULO 2

2.11) a) 4 + 4i
b) - 4 - 1 ai
o) - 27 + 5i
d) - 5
e) 16 + 2&i
f) 28 — 30i

2.12) a)-243í b)^ c) -Bi d) —32i

3
2.13) a) X = ±2 b) x * 2 e Xíé — 2 c)x = -
4
2
2.14) x2+y *0

2.15) a) a à 5 b) -3 < a < 3 c) aí-3 au 3 < a < 5

2.16) a) x = 11 e y = -1 b) [ x - - e y = 1 ou x = -- e y = -1
V 2 2

2.17) a)x = -3 b) Náa existe x

3 373
2.18) z = 2 + i ou z = —2 — í b) z = - 3 ou z - —-------
2 2

2.19) a) z=1 + iV2 ou b) z - — 7 + i ou z = — 7 — i

2.20) Seja x - yi, (y # 0), o imaginário puro raiz da equação. Entào:

(yi)2 + (a + bi)yi + c + di = 0
-y2 + ayi-by + c + di = O
(-y2 -by + c) + (ay+ d) i = O+Oi

324
Portanto:
|-y2-by + c = 0 íl)
i ay + d = 0 (H)

De (l[), y = substituindo em (I):


a

+ c = 0 => -d2 + abd T a 3c = 0

Logo: abd = d3 -a2c.

2,21) a) — + -r b) -J2+K/2
2 2

2,22) a -i-bi = sen ricos a-*! sen2a + i cos22 a - i2 sen nt cosa


a + bi = (2sena cos ra) + (cos2 a - sen2 a) i
a + bi - sen 2a + i cos 2a
Então: a = sen 2a e b = cos 2a:
logo, a2 +b2 = sen2 2o, + cos2 2a = 1. Vae R

2.23} 2 = (1+i tg o)(1 -i tg nt) = 1-i2 tg 2 a


z = (1 + tg2 a) + Oi = sec 2 a + Oi
Então: lm(z) = 0 e Re(z) = seo2a.
2
Cdrno, na Trígonometria, sec a < -1 ou sec a > 1. temos sec a > 1. isto é,
Re(z) ? 1,

2.26} a)-1 b)-i C) 1 d}i


e) 2 - 31 f) - 16384

2.27} a) A = ± 1
b) A = 1 + i; -1 + i; -1 - i; 1 - i
C) A = ± i

2.28) um só valor (A = 0)

2.29) a) m + n = 4k, k e Z b) m - n - 4k +■ 3. k e £

7 4
2.37) a) —" H---- i b)^i c„ —4 3.
+ -(
13 13 '10 10 5 5
d)i e) (V3 4-2)+(V3-2)Í f} -3 + 3i

325
1 1.
2.3B) a) — b)-------- 1 d) cosx-i sen x
3 2 2

2.39) a) x = ±2^2 b) x * -5, x # 2V2 e x í -2^2

2,40) ac = — bd e bc *■ ad

2.41) w = - i ou w = 4i

2.42)
£+ 1 (cos a + i sen a)2 +1
z cos a + i sen a
cos2 a
(cos2 a + 2i cos a sen a -sen2 a +1
cosa + i sen a
(2 cos2 a + 2icosa sen a)(cosa - í sen a)
(cosa + i sen a)(cosa-> sen a)
2cos3a-2icos2a sen a+ 2icos2a sen a + 2cosa sen2 a
_ _ -
cos cr. + sen a
= 2cos a (cos2 a + sen2 ct) + Oi = 2 cos ae R

1+z 1 + sen g + i cos ct 1 + sen a + i casa [(1 + sen a) + icosa]2


2.43)
1 +z 1 + sen (i-i cosa 1 + sen a + icasa (1 + sen a)z + cos3 a

1 + 2 sen a + sen2cx + 2 (1 + sen a) i cosa - ccs2 a


1+2 sen a + sen2a + cos2 a

1 + 2 sen a + sen2 a - (1 - sen2 a) + 2 (1 + sen a) i casa


2 + 2 sen a
2 sen g + 2 sen2 a 2(1 +sen g)i cos ft
= sen ct + í cosa = z
2(1 + sen a) 2(1 + sen a)

2.44) a)z = -4-i


b) z = 5 + 1 oi

1 73 1 /3
c) z - 0 OU Z = 1 OU Z =------ 1------ j ou z=—
2 2 2 2
d) z = a(1 + i), Va e R

5 x 4 3-
2.45) a) z = 4 ~ j b)z = - --Li c)z = — + -i
13 13 5 5

326
2.46) a) 4 - 2i
b) 4 - 2i
c) 5 - 5i
d) 5 - 5i
e) í
f) i
2.47) Sejam zi = a + bí e z2 = c + di, a, b, c e d reais
1 ?) 2, + z2 = (a + bi) + (c + di) = (a + c) + (b +■ d)i =
= {a + c)-(b+d) i - a + c -bi-di =
= (a -bi) + (c-d>) = zt+ z2

2.°) z1xz2 = (a + bi) (c + di) = ac + adi + bci-bd =


= (ac-bd) + (ad + bc) i = (aC-bd) - (ad + bc) i
z,^ = ía"+bi)>(c + di} = (a-bi)(c-di) =
= ac - adi-bci - bd = {ac - bd) - (ad + bc) i
Logo: z,^ - zdxz2

(a + bi)(c - di] ~ac-adi i-bci + bd


3.°)
(c + di)(c-di)J _ 2 4-d
c2 +d 2

ac + bd bc-ad . ac + bd ibc-ad
2 cz + d2 t
a hbi a - bi (a " bi)(c + di)
c + di c-di (c-di)(c + di)
ac + adi-bci + bd ac+bd bc-ad
c2 + d2 c2 + d2 c3 + d2

Zi
Logo: = =, z2 # 0.
Zz Z2
2 48) Utilizando o Principio da Indução Matemática, temos:
Teorema 1: a igualdade se verifica para n = 0, pois:
(z°j = 1 = 1 e (z)° = 1
Teorema 2: admitindo que a igualdade é verdadeira para n, vamos provar que é
verdadeira para n + 1, isto é:

(?).£)"=, (z^= (ir1


(zn+1) = zn xz1 = (zn) ><z1) = (z)n Xz)' = (z)"‘ ’
Está então provado que (z*1) = (z)n.

327
2.49) 2(Z)3 + 3(z)2 = 2(z3) + 3(z2) = 2z3 +3z2 =2z3 + 3z2 =a + bi = a-bi

2.50) Como z ê raiz, temos rez2+pz + y = 0. Substituindo no primeiro membro da


equação dada x por z, temos:
ct(z)2 + pz + y= a(z2) + pz +y = az2 + pz + y = az2 + pz + Y = Õ = 0
Logo; z é raiz de (E).

CAPÍTULO 3

3.2)
a) y- b) yx
3
0 3 >
I x

1 -2
x

yA
c) 3^

\
- 3' 'x

-3

3.3) O lugar é a reunião das bissetrizes


dos quadrantes pares e impares,
excetuando-se a origem O.

3-4)
y.
2 Elipse da equação
2 2
-x- + r- = i
16 4
\-2W 2
-2

328
3.12) a)25 c) 10 d) 16

3.13)

c)
2
yt2
\
x 1 X

I2 \
T" -2 2 •

-2 -2

3.14) a) o ponto de coordenadas (-3; 4)


b) reta de equação 4x + 6y - 5 = 0
c) circunferência de centro C(1; -2) e raio r = 2
d) circulo de centro C(1;-2) e r = 2

3.15)
a) b)
y*

V5

Fi ____ Fi
2 I3 x -3 -2 2 3 x
^=1
•T
-V5 -V5
3.16) |z| = 2V2

z
3.17) ReI — | = 4
W I

3.20) Escrevendo |z-w| como |z + (-w)| e utilizando a desigualdade triangular,


temos:
| z — w | = | z + (—w) | í [ z j +1 —w |
Assim,
I z —w I <| z| +1 (-1)>w I
| Z-W|£|z| + |-1|xJ W|

Logo: | z- w | < | z | + | w |

329
3.21) Pela desigualdade triangular, podemos escrever:
I Zl + (zz+Z3)l £ |ZJ-I-|Z2+Z3 I (I)
e
|z2fZ3| < |z2 | + |z3| (10

Somando membro a membro (I) e (II), temos:


[Zl+(Z2-rZj)| + | Z,+Z3 | < |zd+|22 + z3| + |z2 | + fz3|
Logo: | + z2 + z3 | í | zd | + | z2 | + | z3 |

CAPÍTULO 4

.. « ■ 11 'l
4.1) a = 3 (cos 0 + i sen 0) b) z = 2 cos- + l sen -
l 6 Ê7
I 71 71 i' 2ir . 2rt')
c) Z = 5 cos - +1 sen - d) z = 6 cos — 4-Jsen— I
\ 2 2 1 3 3J
r-( 5tt . 571^
e) z - cos 7T + i sen n f) 2 = 2 cos — + i sen— l
l 4 4 )
f 3n
3rc 33jtjt „ -f 7k . 7n'
q) z = 4; cos — sen — h) z = 3s/2 cos—- +1 sen —
.22 2 t 4 4 ,

4.2) a) z = 3 + 3./3Í

Jz
b) z =-------- r — i
J2
2 2
c) z = -s/ã -i

?
4.3) Escrevendo z = p(cos 0 + i sen 0), temos: X = p2fcos 9 + i sen 9)J =
2 2 2
p (cos 0 - sen 9 + i x2 sen Gxcos 6).
2 2
Como cos 9 - sen 9 = cos 26 e 2 sen 9 cos 6 = sen 29, vem:
2 2
z = p (cos 20 + i sen 29)
T1
Lego: □ argumento de z é 29.

4.4) Sejam zi = p(cos St + i sen 61) e zj = p(cos 92 + i sen 82)-


Como = 2n - Gr podemos escrever:
z2 = p[cos(2rc-9d) h-í sen (2te - 01)J
Mas. cos (2jt- 9i) = cos e sen (2n - &i) = - sen 9i.
Então: z.2- p[costi1 + r(-sen 6 = p cos 9-| - i p sen 01 = z1

330
CAPÍTULO 5
1 ';3
5.5) a) 2lS>i b)1 ' 21' + 2111
5.6) 72 - 72 i

5.7) a) Sendo z = cos6 + í sen 0, temos z3 = cos 30 +i sen 39 e, também:


z3 = (cosG + i sen O)3 =
= cos30 + 3icosJ0sen0 + 3 i2 cosS sen20 + i3sen30-
= (cos3 0 - 3cos0sen20) + i(3cos20senfl - sen3 9)
Então:
cos30 = cos3 S- 3cos9 sen29
sen30 = 3cosz Gsen 0- sen30
b) sen 40 = 4 senft cose (cos^ - sen20); cos 40 = cos4 0 + sen4 9 - 6 sen2e
2
cos 0

5.8) Sendo (1 + i) = V2 cos- + isEn— , temos:


l 4 4)
nn + i sen
(1 + i)rt = (V2)n fcos-^- |
4
nn
Para que (1 + 1)" seja imaginário puro, devemos ter cos—-Q logo.
4
D* 7t , n 1 .
— = - + kir, donde - = - + k e, assim, n = 2 + 4k, expressão que repre-
4 2 ■ 4 2
senta termos de uma PA de razão r = 4.

5.9) a) 6 b) 3

5.10) a) 3 b) 9

i7z)
-1'
1
5.16) a)2i; ±73 - i b) 1; -1; ±- ±
2

5.17) ajsía; -j ± -2; 1 ± Tãi b)S = J- + 2i: - -


2 2 12 2

5.18) -4 ± 473i

331
cini / 5* ■ 5tt'i 3^3 3
3^'3
5.19) 3I cos—+isen— l =——+ —i
V 6 6 ) 22 2
f 4jt 411) 3 3j3.
l 3 3 J 2 2
11rr . 11tt) 3^3 3
l 6 6 J 2 2
5.20)
'*2

CAPÍTULO 6

6.6) f(2) = 18

6.7} f(1 + 2i) = - 12- 2i

6.8) 2 é raiz

6.9) a = 2, b = - 1 e c = - 3

5.10) o. = p^--
2
6.11) zero

5.12) a = - 1 e b = D

6.13) Temos, sucessivamente:


2 3
b
— xí t —
j a
s 3
3 — 0
x? 4- —x, + — x 2-4
-X?
b ? a. bfx,+ x2 a X1 + X2
2 3 1 2 2 2l, 2
3 3 2
..
^(xf+ x|) + ^(xf+ xj) a (x, + xz)3
b (x^X;)- + ^-
o 2 2 3 4
a (X g + X 2 ) b 22 72 (X)+ X2)g
3 x? "2
4 + 2 + X2"
2
=0

a[*i(xf - x|) - x/x* - x|)] + b(Xl - x2)Z = 0


e sendo x1 * x2 :

a[x1(x1 + x2)-x2(x1+x2)] + b(xl-x2) = 0


a[(xt+ x2) (x1 -x2)] + b(x1 -x2) = Q
3(Xi-rX2) + b = 0,etc. x1 + x2 = -b/a

332
CAPÍTULO 7

7.8) a = 8, b -9 e c = 8

7,9) : -X3-2x + 1
a) 2x6-7x5 + Sx"’ -3x3

b) xs - x4 -4x3 + 3x + 1

7.10) a) P(0) = 3.P(1) = 2 e P(2) = 1


b) t}[P(X)J = r)(P(2 - X)]
3[P(x) -r x,>P(2-x)] = 21
j=>3[X1>f3(x)] = 2 = í’)[P(2-X)] = 1
3fx>P(2-x)]<a[P(x)]
e daí: <W)l = i
7.11) 3<n£6

7,12) b = 3, c = d = 2

7,13) Efetue as Operações.

----- 16 16
7.14) a = “— e b = -
3 9
7.15) 1°) Desenvolva os quadrados e apíique o resultado do exercício 7.4:
—a =—bJ
a‘ b
2°) Aplique □ item 1

7,16) Observe que a soma Sn dos n primeiros termos de uma PA é um polinómio


1 í 1
em n; identifique-o com — n'. Daí: a, - — e r = 1.
2 2
1 ,3 1 ? 1
7,17) g(x) = > + -x + — x; a veja exercício 7.3,
2" 2

1 1
7.18) g(y) = - x3 - — x; a veja exercício 7 3.
" ' 3 3
1 u 1
7.19) a = -, 11 ( 1 . 2 c = —11
b=- ou b=-
a = -L b«-~. □u
k k‘ C = kJ t, k k k
1
a = k' b = 0, c = 0 ou a= b= c-0
k k

7.20) (p = 0 e q = 0) ou (p = 1 e q = - 2)

7.21) Substitua x por 1.2. 3 e 4; dai A = 1L B = 6. C = 7 e D = 1.

333
e B-C-l
7.22) A= -
3
A=1 e
e B=-l
B=-
7 23)
2 2
7.24) Sejam: p(x) = a^x"'4-... e q(x) = bnxn+...
Como p(x) e q(x) não sáo nulos, tem-se am /O e b„ * 0.
O coeficiente de x1” + rt em p(x)xq(x) é antm e am>bn) * 0, pois an * 0 e
bn 0. Então, p(x)>q(x) não é nulo.

7.25) Sejam•p(x) = aQ + a,x + ... + alx' + q(x)= b0+b1x + ... + bix’+ ... e h(x) =
= Co -i-C^ 4-,..4-C|X' +...

O coeficiente de x1 em p[x)>q(x) é uf = aob( +a,b + ... + a-b0.


O coeficiente de xJ em [p(x)xq(x)] *h(x) é:

ZUiC, = uocn + u1cn_1 + ...+unco =


i-a
— a ob Qcr + (a Db1 + a -jbg )c n. + ,,. + (aobri+... + aI1bc)co

Obtém-se, assim, uma soma da forma Y a, >b: >0.


h JJij h
jt+j2+j:=n
Se calcularmos o coeficiente de x' no produto p(x) >[q(x) ^(x)], chegaremos
ac mesmo resultado.
7.26) Sejam p(x) = aQ-r-adx+..., q(x) = b0 + t^x+ „. e h(x) = c0 + c,x+...
O coeficiente de Xj em:

1o) p(x)>h(x) é Y aH-Cj.fc


k-Q

2°) q(x)>h(x) ê Yb kci-k

iguais, pois em
3fl) p(x) >h(x)q(x)>h(x) é YtakC,r-k +bkCj_K) C vale a
k=0
I distribui!vidade
4°) [p(x) + q(x))?ti(x) é X(ah+bk)Cj_h
k=0
3n + 1
7.27) Substitua x por 1 e -1:
2
CAPÍTULO S
8.9) a) Q(x) = x-4 e R(x) = 3x + 2
b) Q(x) = x3 -x2 + 5x-7 e R(x) = 11x-1C
c) Q(x) = 2x2 + 3X + 11 e R(x) = 25x-5
d) Q(x) = x2 + x- 2 e R(x)bD
334
6.10) a = 4 e b = 6

8.11) a =-2 e b = 4

8.12) m = 5 e q = 0

8.13) a) x2-ax + (a2-a-2) b) b = -a2 + 3a + 2 e c = -2a2 + 2a r 4

6 14) Efetue a divisão; R(x) = 0 => c = b 2 e d = b3, e dai a tese.

8.15) a(x) = b(x)>q(x) + r(x) e a(x) = x>b(x)q'(x) + r(x), e então


b(x)>q(x) + r(x) = x>b(x) q'(x) + r(x)
e dai a tese.
1
8.16) -<J(x); o resto é o mesmo.
2

8,17) m = -23, n = íJ e p = 21

8.18) 2x3-3x2 + 4x + 5

8.19) Efetue a divisão e R(x) = 0.

8.20) p = ±/z e q = 1

8,21) Efetue a divisão de A(x) por C(x) e R(x) s 0; em seguida, efetue a divisão
de Bfx) por C(x) e compare os restos.
3 _ Q
8.22) —x2 + 2x + —
2 2
8.23) Observe que x3 +1 = (x + 1)(x2 - X +1) Ache o resto r(X) da divisão de f(x)
por x3 + 1, usando o exercício 8 6; f(x)s (x3 + 1)Xx + 2)+r(x), e daí
f(x) = xi’ +2x3+3x2-x-1.

8.24) a = 2, b = 1 e c = 3

CAPÍTULO 9
9 18) a) q(x) = x3-x2 + 3x~3 e r = 5
b) q(x) = 2x3 -6x3 +13x2 - 39x +109 e r --327
C) q(x) = 4x?-(3 + 4í)x + £-1 + 7j) e r = 3-6i
d) q(x) = x 2 - 2ix - 5 - 2 e r = -9+8i
e) q(x) = 2x24-4x + 19 e r =162
f) q(X) = 2x - 5 e r = 28
335
9.19) f(x) é da forma ax2 + bx + c; f(1) = 1, f(2) = 8 e f<3) = 27; daí f(x) = 6x2-
-1lx + 6

9.20) f (— 2) = 0 e m = -73

9.21) 4x4 + 1 3x3 - 2x2-11x +12

n par. q(x) = x ~axn“3 + a2xn-3 .. - a e r=0


9.22)
n impar q(x) = X -axn"2 + a2xr-3 -... + a e rzz-ía11

9.23) q(x)-x + ax^3 + ... + a e r = 2art

9.24) a = 2 e b = -5

9.25) p é par e q impar

9.26) a) Verifique çue f(1 )= f(2) = 0.

b) Verifique que f(0) = f (—1) = f 0.

27) Verifique que f (- a) = f (- b) = 0.

9.26) q(x) = 2x2-1

9.29) Veja exercício 9.12, r(x)=x+2.

A(-Í)-Afi) A(i) + A(-i)


9.3D) Veja exercício 9.12; r(x) = ix? >x +
2 2

9.31) Veja exercício 9 15; r(x) = - 6x + 13.

2 7
9.32) ■— x
3 3

9.33) x3 - 3x +1

9.34) 7; 1
b b
9.35) (ax - b) xq,(x) = (bx - a)xqjx); substituindo x por-, vem -ü e.
a a
substituindo x por 1, vem q,(l) +■ qa (1) = o.

9.36) a) r b)-* X b2r


c) —
a a4
336
9.37) b-a

9.30} Veja exercício 9 16: ~(x - 1)(x - 2)(x - 3) + 6.

9.39) a - 3 e b = - 4

9.40) q(x) - x3+2ax2+3a2x + 4a 3

9.41) Aplique Briot-Ruffini: q(x) = xn+ xn-1 + ...+x + n.

9.42) Aplique Briot-Ruffini q(x) = X r’"2 + 2axn'3 + 3a2x,1-d +., 4(n-l)a

9.43) Aplique Briot-Ruffini; a = 2, b = 1 e m = 3.

9.44) O desenvolvimento do determinante nos dá -2{x -1)3.

9.45) p = 2 e a = 10

9.46) 2x3 + x2-Bx + 15

9.47) Note que x3 + x 2 + x + 1 = (x + 1)(x2+ 1) = (x +1)(x + i)(x-i); verifique que


p(-1)^p(i) = p(-i)-0.

9 48) x"(x2 + ax + b) s (x - 2)3 q(x) + 2n(x - 2); substituindo x por 2

2n(4 + 2a+b) = 0 e dai b = -2a-4; então:

xn(x2+ ax-2a-4) = (x-2)2q(x) + 2n(x-2)


x^fx2 - 4) + a(x - 2)) = (x - 2)2q(x) + 2n(x-2)
x"(x-2)(x + 2 + a)i (x-2)?q(x) + 2n(x-2)
xn(x + 2 + a)i(x-2)q(x) 4 2"
Substituindo x por 2. a + 4 = 1 e a = - 3 e b = 2

s.49) Note que x2 +1 = (x + i)(x-i); calcule f(i) e f(- r) usando a fórmula de De


Moivre,

CAPÍTULO 10

10.6) a) 15x2-6x + 1 b) x2-6x3 + 13x< -12x5 + 4k6


c) 3x2 + 1Qx + 5 d} 30x(5x2 + 7)2
e) 5(1 + 5x-8x?)S(5-16x) f) bm(a + bxf'1

337
10,7) -lx2 + 3x + 4
2
7
10.9) a = 21. b = 51 e c- -77 k = -
3

10.9) X2 + 2X + 3 ou -x 2-2x-3

10.10)a) 20x3, 60x2, 120x b) 72

10 11) Derive termo a termo, usando o exercício 1G.4.

10.12) Propriedade III


a) Demonstramos que a propriedade vale para p(x) = xh e q(x) = xk

Temos, p'(X) = e q'[x) = k>x


Também p(x)>q(x) = x hkk e (pfxj^íx)]^ (h+k}xh"FC-1

Temos, então: p'(x)xq(x) + p(x)xq'(x) = hx h'1 wk + kxk-‘ >xh


-(h+kjx^-^fpíxíxqíx)]-
r
k
b) Suponhamos agora os polinõmios: p(x) = "} ahx e
h-0 k*0
Então. p(x)x](x) = £ya ^kX^1* e dai:
h fc

[p(x)?q(x)]=X^ahbh(xribh)' = ^»hbk[(x'1)1xk + xíl(xk)-] =


h k h k

hXbk(*k)' = + PÍ*)>qXx)
h i< h k

Propriedade IV

Inicialmente. demosntramos pelo Método da Indução Matemática que se:

p(x) = q1(x)Xl2(X)xi3(x) ,qn(x)

tem-se: p’(x) = q lT(x)xq2(x),,,qr1(x) + q1(x)xq‘2(x)xq3(x)...qri(x) +


+q1(x)>q2{x)xq3(X)... qjx) + ... + q1(x)xÍ2(X)... q'rt(x)
Fazendo q^x) - q2(x) =... = qn(x) = q(x) vem a tese.

10.13)9) X 2 -2x +3
7 2
b) x 3 --X2 +2X + 2
2
cix + 3
d) 2x + 7
e) x2-2x + 5

338
10.14)(X + 1) (X — 1)

10,15) (x-1)2*X

10.16)1; 3

10.17)m = 21 e n * - 8

10.18) xe-14x*+ 49x*-36

10.19) x15-x 14 -x13 + x12; x-1

CAPITULOU

11.14) P(x) = (2 + i)(X - 1){x +i)

11.15)a) 13
b) 13
c) 4
3 2i
d) 5 (simples); - 6 (tripla); 72-4 (dupla) e (multiplicidade 7)
4

e) J3
11.16) S = {—4; 2; 3}

11.17) P(x) = 2fx-^(x-2-i)(x-2 + i)

11.18) P(x) = (x - 2) (x + 1) (x-1) (x-3)

11.19) A(x) = (x + ?f(x-3i)(x 3i)

11.20) B(x) = 3(x-2 + i)(x-i)[ x-^


X.

11.21) P(x) = (x-4)2(x + 3)

11.22) raiz tripla

11.23)p = 10 e q = -25

339
1 i.24)t = 9; s = |; r ?t6

3
11.25) k = - -

11.26) k <6-276 □u k > 6 + 276

11.27) P£x) = 5x4 +(10-1&t)x3 -(20j + 5)X2-10k

11.28) P(x) = 1X3 5


----- X 3 ‘--X + B
3 3

11.29)S = (4,^2, 3}

11.30)S = {0 2; 3; 4}

11.31) S = {2; 3}

11.32)S = {2. -1 + j; -1 -i}

11.44) P(x) = (x + 3)22(x-3)(x + 2i)(x-2i}

11.45) S ={xs R | x <3 e x * -3}

11.46) S = {73; - 2}

1 173
11.47) 8 = (1; w:w2;w4;ws}; onde w = — +-----
2 2

í 3 3 3 3 2ji 2ti
11.46) S = J—— onde w=cas—+isen —
|1-W 1-w2 ' 1-w3 ' 1 - w * j 5 5

11.49)8 = {2i; -2i; 2; 3}

3^-1
11.50)a) b) n
2

11.51) P(x)= (x-3)(x-2 + i)(x~1+i}

11.52) a, pey

1 1.53)20

340
11.54)Sendo P(x) = A(x) — B(x), P(x) é um polinõmio de grau menor ou igual a n
que se anula para os valores distintos rlB r21...rn. Portanto:
P(x) = k(x~r1)(x-r2)...(x-rn) (I)
onde k é uma constante. Temos, ainda, P(r0) - 0. Mas, como o valor x = ra
não anula nenhum dos fatores x-r. em (I), concluímos que k - Q e,
portanto, P(x) = 0, Assim:
A(x)-B(x) = 0 w A(x) = B(x)

CAPÍTULO 12

12 7) S = {-2 3}

12.fi) S = {1-i;3)

1
12.9) S = -1; -
2

12.10) k = 20 ou k = 16

i2.11)k = 20; S = {-2;-1)

7 5
12.12)(k = 5 e 1=8) ou k =- a t=-
4 ?6

40
12.13) k =-y

12.14) k *-2

CAPÍTULO 13

13.10) a) 4 b) 3

13.11)13

13.l2)a) V b) V c)F d) V e)F f)V

13.13)a) V b)F c)F d) v

13.14) P(x) = -2x5+Ux4-36x3 +44x2“34x + 30

13.15) A(x) = ^x ^_xfi + 7)(7_8x6 + 32x5_16x-' +48x3

341
13.16JS = {3 + i; 3-i; ”1; 4]

13,17)5 = {0; t; 2; 4i; - 4<)

13.18)8 = {- 1; 2; i; -j}

13.19)S = {1: -2; 5: 2i; - 2i)

13.20)8 = {- 1; 2; 1; 2i; - 2i}

13.21)não

6 8. 6 8.
13.22)a) m = 30 bl-- + -i: ~3
5 5 6 5

l3.23)m = 30 ou m = 24 ou m=-240

13.24) 8 = 1-J2Í; 1+V2Í; -3;

13.25) S={C; 2 + i; 2 - i: 2i; - 2i; 3; - 1}

CAPITULO 14

11)
14 27)a)S = {3; 1 + i; 2-t) 9) S =

2 h) S = ||
b) S 2; -2
' 1.3 ' '
c) 8 = {3; -2} i) S = {4;-2;-3)

d) S = {5 + 2i; 5-2i; 4} j) S=
21
2j
i 11
e) S =J2i;-2i;-^ k) S=|6;3;-J
f) S = [-3; 4; -4} D 5 = {8;6;-2}

Í3
14.28)a) k = 12 b) S=i^:-1;4

14.29) 3x3 -(1 0 + 3Í)X2 + (9 + 7Í)X - (2 + 2i) = 0

(4.30) X3 -(12-í)x2 +(45- 10Í)X - (50 - 25i) = 0

342
14.31) x4-7x3 + 13x2 + 3x-1B = 0

14.32) k = 0

14.33) k = l

14.34)a)-^

x 14 d)-«4
c)-- -
25 01
116
e)—
27

12 +ai
14.35)
9

g
14.36) k = |

14.37) a) 7
b) ~
16

d) --
2 a
e)7
8 256
x 339
9) "ZT h) —
□4 32

-23-8f
14.30)
16

2 64
14 39)a) x3-2x2+ x-9
9 b) x3J +2x32 ——x +—
2 3 9
1
2:r2 + -x
c) x3 + -x 1
+-
3 2 3

14.40) x2-8x-20

14.41) 0

343
1 4,42) Sendo a, b e c as raizes, temos:
a -?-b + c = 0 =s(a + b + c)2 = 09 aa22 +. b; + c2 + 2 (ab + ac + bc) = G => k = - -
t k
mas, como as raízes são reais, t > 0 e, portanto, k < 0.

14.43Í-2; ®,®t ® -
5 5 5 5

14,44) a) S = (-3; 1; 5} d) 3 = {-4; -2; -1}

b) S
b) S 2; 8sj-
=J-: 2; e) S=
e) S = 1-4;
<-4; 4;
4; -
I.2 J L j J
c) S = {-1; 3; 7} f) S = {-1, 3; 5; 9}

14.45)k = - 4

14.46] S ={2; -2; 3+i; 3 -f}

14.47) S = p: -1; -3

f 1
14.48) S = p; -1; 6; -

'4.49)a)S = -1: 2> 4 b) k = -11 e m = 9


2

4.50)4 3; 7

14,51) S - 2-74k + m - 64

14.52)a) 4 b)-1 c) 16

14.53) Imaginemos uma equação do terceiro grau cujas raízes são a, b e c. Como
a + b + c = 0 e ab + ac + bc = 0, essa equação é do tipo x3 -m = 0 (onde
m = abc), isto é, x3 = m. Portanto, a, b e c são as raízes cúbicas do núme­
ro me, assim, |a| = |bj=]c[.

14.54)a) k3 ■ 4m k + 8 t = 0 b) mk-t = 0
c) 4k3 + 27m2 = D d) 27t2 9mk t + 2m3 = 0
e)k2n - t = 0 f) k3 - 4 km + 8 t- 0

344
14.55)a - - 1; b = - 1 e c - 1

14.56) - 1

14.57) a) - 1 b) + 1

14.59) a) - 1 b) + 1

14 59) S=-!L_
w-1

CAPÍTULO 15

1 1 1 2. 4
15.8) a)±1; ±2; ±4 b) ±-: ±-; ±-; ±-:
2 3 6' 3‘ 3

15 9) não

15.10) nao

15.11)a) S = {3, 4; 1 + 72; 1-/2} b) S = < “, —, 2+i; 2-i


12 2

4
c) S = {0; 1, -2; 4} d) S = -2; 3;
3

15.12) P(x) = (x-5)(x + 1)(X-2-/2)(x-2+/2)

15.13) X=^2+1=>X~1 = ^2 =>(x-1)3 -(/2)3 =>


=> x3-3xz+ 3x-1 = 2 x3-3x2+ 3x-3 = 0
Os únicas números racionais que paderiam ser raizes desta última equação
são +1 e ± 3; no entanto, nenhum destes números satisfaz a equação.
Purtanta, o número dado é irracional.

15.14) Fazendo a /26 + 15/3 e b = /26-15/3 temos:

X = a + b =s x3 ss a3 + b3 + 3ab(a + b) = a3 + b3 + 3abx
x
^x3 = 26 + 15/3 +26-15/3 +3x = 52 + 3XZO
= x3-3x-52 = 0

A única raiz real desta equação ê o número 4.

345
15.15) x = 72 +i => x-i = 75 => (x-íp = (75)3
=> x3-3x2i + 3xi2-i3 - 2 x3-3x-2 = i(3x2 + 1M
(x3 -3x-2)2 =[ i(3x2 + 1)]2 =>
x6 -6x4 -4x3 +9xa + 12X + 4 = -(Sx” +6x2 + 1)=>
-■ x6 + 3x4 -4xa +15xz + 12x + 5 = 0
O número 75 + i ê uma das raízes desta última equação, a qual tem todos
os coeficientes inteiros.

15.16) S = {2+i; 2-i; -2; 72; -7z}

15.17) S = {-2; 3i; -3i}

15.16) S = J2; i; -i

15.19) S = {75; - 75; 75; - 75}

f -1 + í73 i-i7ã. 1 + íTã r i-i73


15.20) S = j-1;
2 2’2'2

15.21)não

5.24) S= [2 + 73; 2-75; í; -1}

1 5.25) S = {72 + 75; Jí - 75; - 75 + 75; 75 - 75; -1}

15.26) $ - {75; 75; 2; 3}

15.27) 2 + 75 ; 2-75; i; -í

CAPÍTULO 16

16.5) a = 2 e b = 3

16,6) (a = 7 e b - 4) ou (a= —7 e b = “4)


p —X . I rs- b) S = /l; H j . 1+ í 1-i
6.7)J Qa)J
Lz . I — 'i 3; - i; -i -1; 1 + i; 1-i, —;
3 2
í 1 2 31 1 1
d) S-{2; 3; -
2 3

6.0) a = g e b = -7

346
CAPÍTULO 17

17.4) 2

17.5) 1 e 2

3 13
17.6) k = — ou k = -
2 2

17.7) m = k = - 4

-k-2
17.8) k ’

17.11) S = {- 1; 2); - 1 é raiz tripla

f ■ 1 + i\/3 — 1 — iV3
17.12) S = í 2; ; 2 éraiztnpla.
2 ' 2

CAPÍTULO 18

18.7) P(1)(x) = 15x2-8x + 2 é positivo para todo x e ?.. Portanto. P(x) é sempre
crescente.

18.8) P(1’(x) = -3x2 + 2x + 3 é negativo para todo x e P. Portanto. P(x) é sempre


decrescente.

18.9) P(x) é de grau ímpar e tem coeficiente de termo de mais alto grau negativo.
Além disso, o termo independente é positivo. Assim, seu gráfico é 'algo
parecido" com:
ÀP(x)

18.10)a) F b) V c) V d) F e) V DF
g)F h) V i)F j) V k) V DF

18.11)a) P(x) = 3x3 - 9x - 6 b) P(x) = 3x5 + 6x4 - 6x3 - 24x 2 21 x-6

347
18.12) P(x) - 5x4 - 5x3 -15x3 + 25x -10

18.1 3) a) k = 7 ou k - -20

b) - 20 < k < 7

c)k < — 20 ou k > 7

1 8.1 B)duas ou nenhuma

18.19) P(1) e P{3) têm sinais contrários, portanto há pelo menos uma raiz real r no
intervalo dado. O único número racional que pertence ao intervalo dado e
poderia ser raiz de P(x) é o número 2, que no entanto não é raiz. Portanto, r
é irracional.

18.20) - 12 < k < 6

348
RESPOSTAS DOS EXERCÍCIOS SUPLEMENTARES

1-1) ±723
I.2) Re(z) = 0; lm(z) = tg a + cotg a

1-3) z = 0 ou z = -i ou z = ± — + —i
2 2
1.4) Se a e pi fossem raízes (a e p reais), então ot>pi = c + di, donde c = 0. con­
tra a hipóteses de que c não é nulo.

1.5) Teorema 1: para n = 1 temos:


i5" i5’=i5=i
Teorema 2:
Hipótese', i5
Tese: i5*' =
Temos:
,5' = i5h>£ = (j5k )5 =(i)5
I.6) 3 - 3i
I7) 2b-a + bi
1.8)
y

1 .« ■

1.9)
a) b)

2 2 /

1 1

d)

2
2

349
1.10) a} z = -1 + i

b) z = 2, arg(z) = 5í

c) 2 e
d)

1(1.1) A é divisor de P; A < P e Pé primo, daí A = 1.

n
II.2) a = 1 e b = -1;
n+7
11.3) Faça u- X e V = 0.

5 1
11.4) a—— e b =—
3 3
2 22,
11.5) (A + B + C) sen x cos x + B sen x + A cos x - 1 = 0
2
Dividindo por cos x:
(B -1) tg2x + (A + B + C)tg x + (A - 1) s 0

e dai: A = 1,B = 1eC=-2

..6) a = — ,b = — ,c = —. substitua n par 1,2 e 3.


3 2 6

11.7) 1

II.8) 4x + 7

11.9) (P = 0) e q = 0) ou (p = 0 e q=-1) ou (p = -1 e q = 0) OU ÍP- 1 e


q = 1) ou (p = -2 e q = 1)

11.10) 3x 3 + x? + 7x-i-1

11.11) Faça x-2=y e divida yn - 1 por y — 1 pelo Briot-Ruffini.

P(a)-P(b) e n = aP(b)-bP(a) = 22 200


2200
20a-1 +2
|.12) 3) m =r - a -b e n = a-b b)? m =
n - ------ -
3 e
3
c) Demonstre, pelo Método da Indução Matemática, que os números da
forma 22s -1 são múllrplos de 3; observe que r = m + 1.
350
4
b)lx2 + — 1
11.13) 3) a = -2, b = 4 e C = 1 x
3 3
11.14) 2x —7

II 15) a(x) = b(x)xj(x) + r(x) e a(a) = b{a)=O; então, r(a) = 0

!L16) a) x3 + x2-2-(x-1)(x2+2X + 2) e então, ct + p =-2 e nt£ = 2

g(nt)+ a = p + a = ~2
g(p) + p = a+ 0 =-2
Então, o polinòmio g(x) + x + 2 tem raizes nt e p
g(x) + x s \(x-a)(x-p)
g(x) + x a X(x2 +2x + 2)
g{1) = 1^g(x) = l(4xa + 3x-2)
5
b) g[g(X)] « l04*4 + 96x3 + 32x2 -3*- 64]

c) g[g(1)]-1 = 1-1 = 0
d) verifique que h(1) = h(a) = h(0) = 0

1117) As raizes de w do polinòmio x2+x+1 são tais que w 3 = 1


x=1
[x3 = 1(x-1)(x2 + x+1) = 0 w ou
xz + x + 1 = 0

Então, w 3in
W"+1 3pt2
+ wJ|J'í = w3m >1 + w3nxw’ + w3p:<w3
= 1mXÍ + Txw1 + 1pwz =
= 1 + w + w2 = 0

11.10) m nao é divisível por 3

11.13) a) 6 b) 6x

II.20) Teorema 1: a proposição ê verdadeira para n = 1.

Teorema 2:
Hipótese', todo polinòmio de grau n — 1 admite no máximo n-1 raízes.
Seja então p(x) um polinòmio de grau n; se p(x) não admite raízes, o
teorema é válido para p(x), Caso contrário, seja a uma raiz de p(x):
p(x) = (x-a)q(X)
onde [q(x)| = n - 1: então q(x) admite no máximo n - 1 raízes e, dai, p(x)
admite no máximo n raízes.

351
II 21) p = 1

II.22) 3 = - 3 e b = 2 b)p = 2

11.23) Lembre que:


a) [P^xJ + PjW-P^xJ + P /x) e que
b) [kP(x>]' » ksP (x)

IL24) (t=3 e u = - 2 ) oU (t = 2 e u = 0)

1
III. 1) S={-2; 1 + í. 1-i
2

3
llt.2) S = J 2i; -2i: 1; --
2

III.3) S={-4; t +2)

111 4) a) S ={-2 + i; -2-i; -3)

b) S = {4 + 2i: 4 -2i;

[||.5> S = {-5; 2}

III.6) É inteiro, e x = 6.

111. 7) irracional

III 8) a) V b) V c) V d) V

352
RESPOSTAS DOS TESTES DE VESTIBULARES

46) C 92) B 130) E 184) C


1) 0 47) B 93) A 139) A 185) B
2) E 48) C 94) B 140) E 186) B
3) D 49) A 95) D 141) C 187) E
4) E 50) D 96) E 142) c 188) B
5) E 51) E 97) D 143) D 189} A
6) B 52) C 98) A 144) D 190) D
7) E 53) 0 99) D 145) A 191) C
8) A 54) c 100) A 146) E 192) B
9) D 55) E 101) E 147) D 193) C
10) C 56) B 102) A 140) C 194) E
11) C 57) A 103) D 149) A 195) B
12) B 58) C 104) B 150) C 196) C
13) A 59) 8 105) C 151) A 197) B
14) B 60) E 106) A 152) C 198) D
15) □ 61) E 107) E 153) D 199) C
16) C 62) D 108) B 154) E 200) E
17) B 63) 0 109) C 155) A
18) D 64) D 110) C 156) E
19) C 65) A 111) D 157} A
20) B 66) A 112) A 158) B
21) D 67) C 113) E 159) B
22) A 68) B 114) 8 160) B
23) D 69) E 115) D 161) A
24) C 7D) E 116) D 162) C
25) B 71) E 117) D 163) D
26) C 72) A 118) A 164) A
27) A 73) E 119) A 165) E
20) B 74) B 120) E 166} A
29) A 75) C 121) E 167) A
30) D 76) E 122) C 168) B
31) D 77) B 123) C 169) E
32) C 78) C 124) D 170) E
33) C 79} B 125) E 171) C
34) B 80) B 126) C 172) D
35) C 81) D 127) E 173) B
36) C 82) B 128) A 174) A
37) C 83) D 129) C 175) D
30) A 84) A 130) B 176) D
39) D 85) C 131) B 177) D
40) B 86) A 132) E 178) B
41) E 07) D 133) D 179) D
42) E 88) D 134) D 180) D
43) A 09) E 135) D 131) A
44) E 90) C 136) A 182} E
45) B 91) E 137) Ê 183} B

353
RAZÕES TRIGONOMÉTRICAS
CDS
_s_
o
ser
0,000 0
tg
0,000
cotg
1,000 0_ _ 90
1 0.017 5 0,017,5 57,29 0,999 8 _ 89
0,034 9 0.034 9 28,64 . 0,999 4 88
2 87
3 0^052 3" 0.052 4 19,08 0,998 6 .
4 0,069 8_ 0,069 9 14,30 0,997 6 88
5 0,087 2_ 0,087 5 11,43 . 0,996.2 85
0,104 5 0,105,1 9,514 0,994 5 . 84
6
0,121 9 0,1228. 6,144 0,992 5 83
_ 7
0,139 2_ 0,140 5 7,115 0,990.3 82
8 6,314 81
0,156 4- 0,158 4 0,9877
_9_ 5,671 0,984 8 80
10 0,173 6 0.176 3
_ 0,190 8 0tl94 4 5,145 0,981 6 79
11
12 0,207 9 0,2126 4,705 \ 0,978 1 78
0,225 0 0,230 9 4,331 0,974 4 . 77
13
0.2,49 3 _ 4,011. 0,970 3 76 _
14 0,241 9
15 0,256.8 0,267 9 — 3,732 _ 0,96.5 9 75
0,275 6 0,286 7 3,467 0961 3 74___
16
0,292 4 0,305 7 3,271 0,956 3 73
17
0,309 0 0,324 9 ■ 3,078 _ 0,951 1 72
18
19 .. 0.325 6. 0,344 3 2,904 0,945 5 71
_ 20 0.342 0 0,364 0 2,74_7 0,939 7 ~7o—
_ 21 0,358 4 0,383 9 2,605 0,933 6 69 .
22 0,374 6. 0,404 0 2,475 0.927 2 68
23 0,390 7. _ 0,424 5 2J356 0,920 5 67_
24 0,406 7 0,445 2 2,246 0,913 5 66
25 0,422 6 0,466 3 2,145 0,906 3 65
26 0,438 4. 0.487 7 2,050 0,898 8 64
27 0,454 Q_ 0,509 5 1,963 0,891 0 _ 63___
23 0,469 5 0,531 7 1,881 0,882 9 62 _
29 0,484 8 0,554 3 1,804 0,874 6 61___ _
30 0,500 0 0,577 4 1,732 0,866 0 60
31 0,515 0 0,600 9 1,664 0,857 2 59
32 0,529 9 0,624 9 1,600 _0,84~8 0_ 58
33 0,544.6 0.649 4 1,540 0,838 7 57
34 ' Q.559T 0,674 5 1.483 0,829 0 _56
35 0.573 6 0,700 2 1,428 0,819 2 55
36 0.587 8 0,726 5 1,376 0,809 0_ 54____
37. \ .0,601 8 0,753 6 1,327 0,798 6 53___
38 0,615 7 _078i 3 1.280 0,788 0 52___
39 0,629 3 0,809 8 1,235 _0,77? 1 .51.
40 0,642 8 0,839 1 1,192 0,7660 50___
41 _ 0,656 1 0,869 3 . 1,1 SÓ- 0,754 7 49
42 0.669 1 0,900 4 1,111 0,743 1 48
43 0.682 0 0,932 5 1,072 0731 4 47..
44 0,694 7 0,965 7 1,036 0,719 3 46___
45 0,707.1 1,000 0 1,000 0,707 1_ 45.
COS cotg tg sen □lZZ
354
A Editora VestSeller tem 0 prazer de reeditar a Coleção Noções
de Matemática, uma obra prima composta por 8 volumes, que
trata de todo o conteúdo da Matemática do Ensino Médio de
forma primorosa.

Cada volume contém teoria completa e detalhada, com


excelente profundidade, incluindo demonstrações de
propriedades e de teoremas relevantes para o sólido
aprendizado do estudante.

Adicionalmente, a grande quantidade de exercícios resolvidos


e propostos, em cada capítulo, fornece ao leitor os meios
necessários para uma excelente fixação do conteúdo. Todas as
respostas são fornecidas ao final de cada livro.

Os seguintes volumes integram essa coleção:

Volume 1: Conjuntos e Funções.


Volume 2: Pro g ressõ es e Logaritmos.
Volume 3: Trigonometria.
Volume 4: Combinatoria, Matrizes e Determinantes.
Volume 5: Geometria Flana e Espacial.
Volume 6: Geometria Analítica.
Volume7: Números Complexos e Polinômios.
Volume 8: Introdução ao Cálculo Diferencial e Integral.

Essa coleção é indicada pana professores e estudantes de


ensino médio, em especial para aqueles que buscam um
aprendizado de nível médio a alto, compatível com os exames
dos vestibulares mais concorridos do Brasil, dentre eles as
escolas militares IME e ITA.

ISBN 978356005311-9

r/ f / EblTÚRA

www.vestseller.com.br
X

I1IMI
9 II? a a 5 6 0 «lê s 3 1 i 9 U

Você também pode gostar